You are on page 1of 186

More ebooks at https://telegram.

me/unacademyplusdiscounts
The Most Accepted
CRASH COURSE
PROGRAMME

JEE Main in
40 DAYS
MATHEMATICS

ARIHANT PRAKASHAN (Series), MEERUT


Arihant Prakashan (Series), Meerut
All Rights Reserved

© PUBLISHERS
No part of this publication may be re-produced, stored in a retrieval system or distributed
in any form or by any means, electronic, mechanical, photocopying, recording, scanning,
web or otherwise without the written permission of the publisher. Arihant has obtained all
the information in this book from the sources believed to be reliable and true. However,
Arihant or its editors or authors or illustrators don’t take any responsibility for the absolute
accuracy of any information published and the damages or loss suffered there upon.
All disputes subject to Meerut (UP) jurisdiction only.

ADMINISTRATIVE & PRODUCTION OFFICES


Regd. Office
‘Ramchhaya’ 4577/15, Agarwal Road, Darya Ganj, New Delhi -110002
Tele: 011- 47630600, 43518550; Fax: 011- 23280316
Head Office
Kalindi, TP Nagar, Meerut (UP) - 250002
Tele: 0121-2401479, 2512970, 4004199; Fax: 0121-2401648

SALES & SUPPORT OFFICES


Agra, Ahmedabad, Bengaluru, Bareilly, Chennai, Delhi, Guwahati,
Hyderabad, Jaipur, Jhansi, Kolkata, Lucknow, Meerut, Nagpur & Pune

ISBN : 978-93-13199-32-8

Published by Arihant Publications (India) Ltd.


For further information about the books published by Arihant
log on to www.arihantbooks.com or email to info@arihantbooks.com

/arihantpub /@arihantpub Arihant Publications /arihantpub


PREFACE
It is a fact that nearly 10 lacs students would be in the race with you in JEE Main, the gateway
to some of the prestigious engineering and technology institutions in the country, requires
that you take it seriously and head-on. A slight underestimation or wrong guidance will ruin
all your prospects. You have to earmark the topics in the syllabus and have to master them in
concept-driven-problem-solving ways, considering the thrust of the questions being asked in
JEE Main.

The book 40 Days JEE Main Mathematics serves the above cited purpose in perfect manner.
At whatever level of preparation you are before the exam, this book gives you an accelerated
way to master the whole JEE Main Physics Syllabus. It has been conceived keeping in mind the
latest trend of questions, and the level of different types of students.

The whole syllabus of Physics has been divided into day-wise-learning modules with clear
groundings into concepts and sufficient practice with solved and unsolved questions on that
day. After every few days you get a Unit Test based upon the topics covered before that day.
On last three days you get three full-length Mock Tests, making you ready to face the test. It is
not necessary that you start working with this book in 40 days just before the exam. You may
start and finish your preparation of JEE Main much in advance before the exam date. This will
only keep you in good frame of mind and relaxed, vital for success at this level.

Salient Features
Ÿ Concepts discussed clearly and directly without being superfluous. Only the required
material for JEE Main being described comprehensively to keep the students focussed.
Ÿ Exercises for each day give you the collection of only the Best Questions of the concept,
giving you the perfect practice in less time.
Ÿ Each day has two Exercises; Foundation Questions Exercise having Topically Arranged
Questions & Progressive Question Exercise having higher Difficulty Level Questions.
Ÿ All types of Objective Questions included in Daily Exercises (Single Option Correct,
Assertion & Reason, etc).
Ÿ Along with Daywise Exercises, there above also the Unit Tests & Full Length Mock Tests.
Ÿ At the end, there are all Online Solved Papers of JEE Main 2019; January & April attempts.

We are sure that 40 Days Physics for JEE Main will give you a fast way to prepare for Physics
without any other support or guidance.

Publisher
CONTENTS
Preparing JEE Main 2020 Mathematics in 40 Days !
Day 1. Sets, Relations and Functions 1-9
Day 2. Complex Numbers 10-19
Day 3. Sequences and Series 20-30
Day 4. Quadratic Equation and Inequalities 31-44
Day 5. Matrices 45-54
Day 6. Determinants 55-67
Day 7. Binomial Theorem and Mathematical Induction 68-77
Day 8. Permutations and Combinations 78-86
Day 9. Unit Test 1 (Algebra) 87-94
Day 10. Real Function 95-103
Day 11. Limits, Continuity and Differentiability 104-116
Day 12. Differentiation 117-126
Day 13. Applications of Derivatives 127-137
Day 14. Maxima and Minima 138-149
Day 15. Indefinite Integrals 150-162
Day 16. Definite Integrals 163-175
Day 17. Area Bounded by the Curves 176-187
Day 18. Differential Equations 188-198
Day 19. Unit Test 2 (Calculus) 199-208
Day 20. Trigonometric Functions and Equations 209-221
Day 21. Properties of Triangle, Height and Distances 222-232
Day 22. Inverse Trigonometric Function 233-242
Day 23. Unit Test 3 (Trigonometry) 243-250
Day 24. Cartesian System of Rectangular Coordinates 251-262
Day 25. Straight Line 263-274
Day 26. The Circle 275-288
Day 27. Parabola 289-300
Day 28. Ellipse 301-313
Day 29. Hyperbola 314-325
Day 30. Unit Test 4 (Coordinate Geometry) 326-335
Day 31. Vector Algebra 336-350
Day 32. Three Dimensional Geometry 351-366
Day 33. Unit Test 5 (Vector & 3D Geometry) 367-374
Day 34. Statistics 375-383
Day 35. Probability 384-394
Day 36. Mathematical Reasoning 395-405
Day 37. Unit Test 6 (Statistics, Probability & Mathematical Reasoning) 406-411

Day 38. Mock Test 1 412-416


Day 39. Mock Test 2 417-423
Day 40. Mock Test 3 424-430

Online JEE Main Solved Papers 2019 1-32


SYLLABUS
MATHEMATICS
UNIT 1 Sets, Relations and Functions
Sets and their representation; Union, intersection and complement of sets and their algebraic
properties; Power set; Relation, Types of relations, equivalence relations, functions;. one-one, into
and onto functions, composition of functions.

UNIT 2 Complex Numbers and Quadratic Equations


Complex numbers as ordered pairs of reals, Representation of complex numbers in the form a+ib
and their representation in a plane, Argand diagram, algebra of complex numbers, modulus and
argument
(or amplitude) of a complex number, square root of a complex number, triangle inequality,
Quadratic equations in real and complex number system and their solutions. Relation between
roots and
co-efficients, nature of roots, formation of quadratic equations with given roots.

UNIT 3 Matrices and Determinants


Matrices, algebra of matrices, types of matrices, determinants and matrices of order two and
three. Properties of determinants, evaluation of deter-minants, area of triangles using
determinants. Adjoint and evaluation of inverse of a square matrix using determinants and
elementary transformations, Test of consistency and solution of simultaneous linear equations in
two or three variables using determinants and matrices.

UNIT 4 Permutations and Combinations


Fundamental principle of counting, permutation as an arrangement and combination as
selection, Meaning of P (n,r) and C (n,r), simple applications.

UNIT 5 Mathematical Induction


Principle of Mathematical Induction and its simple applications.

UNIT 6 Binomial Theorem and its Simple Applications


Binomial theorem for a positive integral index, general term and middle term, properties of
Binomial coefficients and simple applications.

UNIT 7 Sequences and Series


Arithmetic and Geometric progressions, insertion of arithmetic, geometric means between two
given numbers. Relation between AM and GM Sum upto n terms of special series: ∑ n, ∑ n2, ∑ n3.
Arithmetico - Geometric progression.
UNIT 8 Limit, Continuity and Differentiability
Real valued functions, algebra of functions, polynomials, rational, trigonometric, logarithmic and
exponential functions, inverse functions. Graphs of simple functions. Limits, continuity and
differentiability.
Differentiation of the sum, difference, product and quotient of two functions. Differentiation of
trigonometric, inverse trigonometric, logarithmic exponential, composite and implicit functions
derivatives of order upto two. Rolle's and Lagrange's Mean Value Theorems. Applications of
derivatives: Rate of change of quantities, monotonic - increasing and decreasing functions,
Maxima and minima of functions of one variable, tangents and normals.
UNIT 9 Integral Calculus
Integral as an anti - derivative. Fundamental integrals involving algebraic, trigonometric,
exponential and
logarithmic functions. Integration by substitution, by parts and by partial fractions. Integration
using trigonometric identities.

Evaluation of simple integrals of the type


dx , dx , dx , dx ,
x2 ± a2 2
x ± a 2 a2 – x2 a 2 – x 2
dx dx , (px + q) dx ,
,
ax 2 + bx + c ax 2 + bx + c ax 2 + bx + c

(px + q) dx ,
ax 2 + bx + c a 2 ± x 2 dx and x 2 – a 2 dx

Integral as limit of a sum. Fundamental Theorem of Calculus. Properties of definite integrals.


Evaluation of definite integrals, determining areas of the regions bounded by simple curves in
standard form.

UNIT 10 Differential Equations


Ordinary differential equations, their order and degree. Formation of differential equations.
Solution of differential equations by the method of separation of variables, solution of
homogeneous and linear differential equations of the type dy +p (x) y = q(x)
dx

UNIT 11 Coordinate Geometry


Cartesian system of rectangular coordinates in a plane, distance formula, section formula, locus
and its equation, translation of axes, slope of a line, parallel and perpendicular lines, intercepts of
a line on the coordinate axes.

Ÿ Straight Lines
Various forms of equations of a line, intersection of lines, angles between two lines,
conditions for concurrence of three lines, distance of a point from a line, equations of
internal and external bisectors of angles between two lines, coordinates of centroid,
orthocentre and circumcentre of a triangle, equation of family of lines passing through the
point of intersection of two lines.
Ÿ Circles, Conic Sections
Standard form of equation of a circle, general form of the equation of a circle, its radius
and centre, equation of a circle when the end points of a diameter are given, points of
intersection of a line and a circle with the centre at the origin and condition for a line to be
tangent to a circle, equation of the tangent. Sections of cones, equations of conic sections
(parabola, ellipse and hyperbola) in standard forms, condition for y = mx + c to be a tangent
and point (s) of tangency.

UNIT 12 Three Dimensional Geometry


Coordinates of a point in space, distance between two points, section formula, direction ratios
and direction cosines, angle between two intersecting lines. Skew lines, the shortest distance
between them and its equation. Equations of a line and a plane in different forms, intersection of
a line and a plane, coplanar lines.

UNIT 13 Vector Algebra


Vectors and scalars, addition of vectors, components of a vector in two dimensions and three
dimensional space, scalar and vector products, scalar and vector triple product.

UNIT 14 Statistics and Probability


Measures of Dispersion Calculation of mean, median, mode of grouped and ungrouped data.
Calculation of standard deviation, variance and mean deviation for grouped and ungrouped data.
Probability Probability of an event, addition and multiplication theorems of probability, Baye's
theorem, probability distribution of a random variate, Bernoulli trials and Binomial distribution.

UNIT 15 Trigonometry
Trigonometrical identities and equations. Trigonometrical functions. Inverse trigonometrical
functions and their properties. Heights and Distances.

UNIT 16 Mathematical Reasoning


Statements, logical operations and implies, implied by, if and only if. Understanding of
tautology, contradiction, converse and contra positive.
HOW THIS BOOK IS
USEFUL FOR YOU ?
As the name suggest, this is the perfect book for your recapitulation of the whole syllabus, as it
provides you a capsule course on the subject covering the syllabi of JEE Main, with the smartest
possible tactics as outlined below:

1. REVISION PLAN
The book provides you with a practical and sound revision plan.
The chapters of the book have been designed day-wise to guide the students in a planned
manner through day-by-day, during those precious 35-40 days. Every day you complete a
chapter/a topic, also take an exercise on the chapter. So that you can check & correct your
mistakes, answers with hints & solutions also have been provided. By 37th day from the date
you start using this book, entire syllabus gets revisited.
Again, as per your convenience/preparation strategy, you can also divide the available 30-35
days into two time frames, first time slot of 3 weeks and last slot of 1 & 1/2 week. Utilize first
time slot for studies and last one for revising the formulas and important points. Now fill the
time slots with subjects/topics and set key milestones. Keep all the formulas, key points on a
couple of A4 size sheets as ready-reckner on your table and go over them time and again. If you
are done with notes, prepare more detailed inside notes and go over them once again. Study
all the 3 subjects every day. Concentrate on the topics that have more weightage in the exam
that you are targeting.

2. MOCK TESTS
Once you finish your revision on 37th day, the book provides you with full length mock tests
for day 38th, 39th, & 40th, thereby ensures your total & full proof preparation for the final
show.
The importance of solving previous years' papers and 10-15 mock tests cannot be
overemphasized. Identify your weaknesses and strengths. Work towards your strengths i.e.,
devote more time to your strengths to be 100% sure and confident. In the last time frame of 1
& 1/2 week, don't take-up anything new, just revise what you have studied before. Be exam-
ready with quality mock tests in between to implement your winning strategy.

3. FOCUS TOPICS
Based on past years question paper trends, there are few topics in each subject which have
more questions in exam than other. So far Mathematics is concerned it may be summed up as
below:
Calculus, Trigonometry, Algebra, Coordinate Geometry & Vector 3D.
More than 80% of questions are normally asked from these topics.
However, be prepared to find a completely changed pattern for the exam than noted above as
examiners keep trying to weed out 'learn by rot practice'. One should not panic by witnessing a
new pattern , rather should be tension free as no one will have any upper hand in the exam.
4. IMPROVES STRIKE RATE AND ACCURACY
The book even helps to improve your strike rate & accuracy. When solving practice tests or mock
tests, try to analyze where you are making mistakes-where are you wasting your time; which
section you are doing best. Whatever mistakes you make in the first mock test, try to improve that
in second. In this way, you can make the optimum use of the book for giving perfection to your
preparation.
What most students do is that they revise whole of the syllabus but never attempt a mock and
thus they always make mistake in main exam and lose the track.

5. LOG OF LESSONS
During your preparations, make a log of Lesson's Learnt. It is specific to each individual as to where
the person is being most efficient and least efficient. Three things are important - what is working,
what's not working and how would you like to do in your next mock test.

6. TIME MANAGEMENT
Most candidates who don't make it to good medical colleges are not good in one area- Time
Management. And, probably here lies the most important value addition that's the book
provides in an aspirant's preparation. Once the students go through the content of the book
precisely as given/directed, he/she learns the tactics of time management in the exam.
Realization and strengthening of what you are good at is very helpful, rather than what one
doesn't know. Your greatest motto in the exam should be, how to maximize your scoring with
the given level of preparation. You have to get about 200 plus marks out of a total of about 400
marks for admission to a good NIT (though for a good branch one needs to do much better
than that). Remember that one would be doomed if s/he tries to score 400 in about 3 hours.

7. ART OF PROBLEM SOLVING


The book also let you to master the art of problem solving. The key to problem solving does not lie
in understanding the solution to the problem but to find out what clues in the problem leads you
to the right solution. And, that's the reason Hints & Solutions are provided with the exercises after
each chapter of the book. Try to find out the reason by analyzing the level of problem & practice
similar kind of problems so that you can master the tricks involved. Remember that directly going
though the solutions is not going to help you at all.

8. POSITIVE PERCEPTION
The book put forth for its readers a 'Simple and Straightforward' concept of studies, which is the
best possible, time-tested perception for 11th hour revision / preparation.
The content of the book has been presented in such a lucid way so that you can enjoy what you are
reading, keeping a note of your already stressed mind & time span.
Cracking JEE Main is not a matter of life and death. Do not allow panic and pressure to create
confusion. Do some yoga and prayers. Enjoy this time with studies as it will never come back.
DAY ONE

Sets, Relations
and Functions
Learning & Revision for the Day
u Sets u Law of Algebra of Sets u Composition of Relations
u Venn Diagram u Cartesian Product of Sets u Functions or Mapping
u Operations on Sets u Relations u Composition of Functions

Sets

A set is a well-defined class or collection of the objects.

Sets are usually denoted by the symbol A, B, C, ... and its elements are denoted by a, b , c,
… etc.

If a is an element of a set A, then we write a ∈ A and if not then we write a ∉ A.

Representations of Sets
There are two methods of representing a set :

In roster method, a set is described by listing elements, separated by commas, within
curly braces{≠}. e.g. A set of vowels of English alphabet may be described as {a, e, i, o, u}.

In set-builder method, a set is described by a property P ( x), which is possessed by all its
PRED
elements x. In such a case the set is written as { x : P ( x) holds} or { x| P ( x) holds}, which MIRROR
Your Personal Preparation Indicator
is read as the set of all x such that P( x) holds. e.g. The set P = {0, 1, 4 , 9, 16,...} can be
written as P = { x2 | x ∈ Z }. u No. of Questions in Exercises (x)—
u No. of Questions Attempted (y)—
Types of Sets u No. of Correct Questions (z)—

The set which contains no element at all is called the null set (empty set or void (Without referring Explanations)
set) and it is denoted by the symbol ‘φ ’ or ‘{}’ and if it contains a single element, then it is
u Accuracy Level (z / y × 100)—
called singleton set.
u Prep Level (z / x × 100)—

A set in which the process of counting of elements definitely comes to an end, is called a
finite set, otherwise it is an infinite set. In order to expect good rank in JEE,
your Accuracy Level should be

Two sets A and B are said to be equal set iff every element of A is an element of B and above 85 & Prep Level should be
also every element of B is an element of A. i.e. A = B, if x ∈ A ⇔ x ∈ B. above 75.
2 40 ONE


Equivalent sets have the same number of elements but not i.e. A ∩ B = { x : x ∈ A and x ∈ B}.
exactly the same elements.
A B U

A set that contains all sets in a given context is called
universal set (U).

Let A and B be two sets. If every element of A is an element
of B, then A is called a subset of B, i.e. A ⊆ B.

If A is a subset of B and A ≠ B, then A is a proper subset of A∩B
B. i.e. A ⊂ B.

If A ∩ B = φ, then A and B are called disjoint sets.

The null set φ is a subset of every set and every set is a
subset of itself i.e. φ ⊂ A and A ⊆ A for every set A. They

Let U be an universal set and A be a set such that A ⊂ U.
are called improper subsets of A. Then, complement of A with respect to U is denoted by A′
or Ac or A or U − A. It is defined as the set of all those

If S is any set, then the set of all the subsets of S is called elements of U which are not in A.
the power set of S and it is denoted by P(S ). Power set of a
given set is always non-empty. If A has n elements, then A′ U
P( A) has 2 n elements.
A
NOTE • The set { φ} is not a null set. It is a set containing one
element φ.
• Whenever we have to show that two sets A and B are equal
show that A ⊆ B and B ⊆ A. ●
The difference A − B is the set of all those elements of A
• If a set A has m elements, then the number m is called which does not belong to B.
cardinal number of set A and it is denoted by n( A). Thus, i.e. A − B = { x : x ∈ A and x ∉ B}
n( A) = m. and B − A = { x : x ∈ B and x ∉ A}.
U U
Venn Diagram A B A B
The combination of rectangles and circles is called Venn Euler
diagram or Venn diagram. In Venn diagram, the universal set
is represented by a rectangular region and a set is represented
by circle on some closed geometrical figure. Where, A is the
set and U is the universal set. A–B B–A

The symmetric difference of sets A and B is the set
U ( A − B) ∪ (B − A) and is denoted by A ∆ B.
A i.e. A ∆ B = ( A − B) ∪ ( B − A)

A B U

Operations on Sets

The union of sets A and B is the set of all elements which A∆B
are in set A or in B or in both A and B.
i.e. A ∪ B = { x : x ∈ A or x ∈ B}
Law of Algebra of Sets
A B U If A, B and C are any three sets, then
1. Idempotent Laws
(i) A ∪ A = A (ii) A ∩ A = A
2. Identity Laws
A∪B
(i) A ∪ φ = A (ii) A ∩ U = A
3. Distributive Laws

The intersection of A and B is the set of all those elements
(i) A ∪ (B ∩ C) = ( A ∪ B) ∩ ( A ∪ C)
that belong to both A and B. (ii) A ∩ (B ∪ C) = ( A ∩ B) ∪ ( A ∩ C)
DAY 3

4. De-Morgan’s Laws
Relations
(i) ( A ∪ B)′ = A′ ∩ B ′

Let A and B be two non-empty sets, then relation R from A to B
(ii) ( A ∩ B)′ = A′ ∪ B ′
is a subset of A × B, i.e. R ⊆ A × B.
(iii) A − (B ∩ C) = ( A − B) ∪ ( A − C )
(iv) A − (B ∪ C) = ( A − B) ∩ ( A − C)

If (a, b ) ∈ R, then we say a is related to b by the relation R and we
write it as aRb.
5. Associative Laws ●
Domain of R = {a :(a, b ) ∈ R} and range of R = {b : (a, b ) ∈ R}.
(i) ( A ∪ B) ∪ C = A ∪ (B ∪ C) ●
If n( A) = p and n(B) = q , then the total number of relations from A
(ii) A ∩ (B ∩ C) = ( A ∩ B) ∩ C
to B is 2 pq .
6. Commutative Laws
(i) A ∪ B = B ∪ A (ii) A ∩ B = B ∩ A Types of Relations
(iii) A ∆ B = B ∆ A ●
Let A be any non-empty set and R be a relation on A. Then,
Important Results on Operation of Sets (i) R is said to be reflexive iff ( a, a) ∈ R, ∀ a ∈ A.
1. A − B = A ∩ B ′ (ii) R is said to be symmetric iff
2. B − A = B ∩ A ′ (a, b ) ∈ R
3. A − B = A ⇔ A ∩ B = φ ⇒ (b , a) ∈ R, ∀ a, b ∈ A
4. ( A − B) ∪ B = A ∪ B (iii) R is said to be a transitive iff ( a, b) ∈ R and (b, c) ∈ R
5. ( A − B) ∩ B = φ ⇒ ( a, c) ∈ R, ∀ a, b, c ∈ A
6. A ⊆ B ⇔ B ′ ⊆ A ′ i.e. aRb and bRc ⇒ aRc, ∀ a, b, c ∈ A.
7. ( A − B) ∪ ( B − A) = ( A ∪ B) − ( A ∩ B) ●
The relation I A = {(a, a) : a ∈ A} on A is called the identity
8. n ( A ∪ B) = n ( A) + n ( B) − n ( A ∩ B) relation on A.
9. n ( A ∪ B) = n ( A) + n ( B) ●
R is said to be an equivalence relation iff
⇔ A and B are disjoint sets. (i) it is reflexive i.e. ( a, a) ∈ R, ∀ a ∈ A.
10. n ( A − B) = n ( A) − n ( A ∩ B) (ii) it is symmetric i.e. ( a, b) ∈ R ⇒ (b, a) ∈ R, ∀ a, b ∈ A
11. n ( A ∆ B) = n ( A) + n ( B) − 2n ( A ∩ B) (iii) it is transitive
12. n ( A ∪ B ∪ C ) = n ( A) + n ( B) + n (C ) − n ( A ∩ B) i.e. ( a, b) ∈ R and (b, c) ∈ R
− n ( B ∩ C) − n ( A ∩ C) + n ( A ∩ B ∩ C) ⇒ ( a, c) ∈ R, ∀ a, b, c ∈ A
13. n ( A ′ ∪ B ′ ) = n ( A ∩ B) ′ = n (U ) − n ( A ∩ B)
Inverse Relation
14. n ( A ′ ∩ B ′ ) = n ( A ∪ B) ′ = n (U ) − n ( A ∪ B)
Let R be a relation from set A to set B, then the inverse of R, denoted
by R −1 , is defined by
Cartesian Product of Sets R −1 = {(b , a) : (a, b ) ∈ R}. Clearly, (a, b ) ∈ R ⇔ (b , a) ∈ R −1 .
Let A and B be any two non-empty sets. Then the NOTE • The intersection of two equivalence relations on a set is an
cartesian product A × B, is defined as set of all ordered equivalence relation on the set.
pairs (a, b ) such that a ∈ A and b ∈ B. • The union of two equivalence relations on a set is not necessarily
i.e. an equivalence relation on the set.

A × B = {(a, b ) : a ∈ A and b ∈ B} • If R is an equivalence relation on a set A, then R −1 is also an

B × A = {(b , a) : b ∈ B and a ∈ A} equivalence relation A.
and A × A = {(a, b ) : a, b ∈ A}.

A × B = φ, if either A or B is an empty set.

If n ( A) = p and n (B) = q , then Composition of Relations
n ( A × B) = n( A) ⋅ n(B) = pq . Let R and S be two relations from set A to B and B to C respectively,

A × (B ∪ C) = ( A × B) ∪ ( A × C) then we can define a relation SoR from A to C such that

A × (B ∩ C) = ( A × B) ∩ ( A × C) (a, c) ∈ SoR ⇔ ∃ b ∈ B such that (a, b ) ∈ R and (b , c) ∈ S . This relation

A × (B − C) = ( A × B) − ( A × C) is called the composition of R and S.

( A × B) ∩ (C × D) = ( A ∩ C) × (B ∩ D). RoS ≠ SoR
4 40 ONE

i.e. f : A → B is a many-one function, if it is not a one-one


Functions or Mapping function.

If A and B are two non-empty sets, then a rule f which ●
f is said to be onto function or surjective function, if each
associates each x ∈ A, to a unique member y ∈ B, is called a element of B has its pre-image in A.
function from A to B and it is denoted by f : A → B.
A B

The set A is called the domain of f (D f ) and set B is called f
a1 b1
the codomain of f (C f ).
a2 b2

The set consisting of all the f -images of the elements of the
a3 b3
domain A, called the range of f (R f ).

NOTE • A relation will be a function, if no two distinct ordered pairs Method to Check Onto Function
have the same first element.
Find the range of f ( x) and show that range of
• Every function is a relation but every relation is not
necessarily a function. f ( x) = codomain of f ( x).
• The number of functions from a finite set A into finite set ●
Any polynomial function of odd degree is always onto.
B is { n(B )}n ( A). ●
The number of onto functions that can be defined from a
finite set A containing n elements onto a finite set B
Different Types of Functions containing 2 elements = 2 n − 2 .

If n ( A) ≥ n (B), then number of onto function is 0.
Let f be a function from A to B, i.e. f : A → B. Then, ●
If A has m elements and B has n elements, where m < n,
f is said to be one-one function or injective function, if then number of onto functions from A to B is
different elements of A have different images in B. nm − nC1 (n − 1)m + nC2 (n − 2)m − ..., m < n.
A B ●
f is said to be an into function, if there exists atleast one
a1 f b1 element in B having no pre-image in A. i.e. f : A → B is an
a2 b2 into function, if it is not an onto function.
a3 b3 A B
a1 f b1
a4 b4
a2 b2
a3 b3
Methods to Check One-One Function a4 b4
Method I If f ( x) = f ( y ) ⇒ x = y , then f is one-one. a5 b5

Method II A function is one-one iff no line parallel to


X-axis meets the graph of function at more

f is said to be a bijective function, if it is one-one as well as
than one point. onto.

The number of one-one function that can be defined from a NOTE • If f : A → B is a bijective, then A and B have the same
 n( B ) number of elements.
Pn( A ) , if n (B) ≥ n ( A)
finite set A into finite set B is  . • If n ( A) = n (B ) = m, then number of bijective map from A to
0, otherwise B is m!.

f is said to be a many-one function, if two or more


Composition of Functions

elements of set A have the same image in B.


Let f : A → B and g : B → C are two functions. Then, the
A B composition of f and g, denoted by
a1
f
b1 gof : A → C, is defined as,
a2 b2 gof ( x) = g[ f ( x)], ∀ x ∈ A.
a3 b3
a4 b4 NOTE • gof is defined only if f ( x ) is an element of domain of g.
a5 b5 • Generally, gof ≠ fog.
DAY PRACTICE SESSION 1

FOUNDATION QUESTIONS EXERCISE


 1  (a) reflexive but neither symmetric nor transitive
1 If Q = x : x = , where y ∈ N , then
 y  (b) symmetric and transitive
2 (c) reflexive and symmetric
(a) 0 ∈Q (b) 1∈Q (c) 2∈Q (d) ∈Q (d) reflexive and transitive
3
2 If P ( A ) denotes the power set of A and A is the void set, 12 If g ( x ) = 1 + x and f {g ( x )} = 3 + 2 x + x , then f ( x ) is
then what is number of elements in P {P {P {P ( A )}}}? equal to
(a) 0 (b) 1 (c) 4 (d) 16 (a) 1 + 2 x 2 (b) 2 + x 2
(c) 1 + x (d) 2 + x
3 If X = {4n − 3n − 1: n ∈ N} and Y = {9 (n − 1): n ∈ N}; where N
is the set of natural numbers,then X ∪ Y is equal to 13 Let f ( x ) = ax + b and g ( x ) = cx + d , a ≠ 0, c ≠ 0. Assume
j JEE Mains 2014 a = 1, b = 2 , if ( fog ) ( x ) = ( gof ) ( x ) for all x. What can you
(a) N (b) Y-X (c) X (d) Y
say about c and d?
(a) c and d both arbitrary (b) c = 1and d is arbitrary
4 If A, B and C are three sets such that A ∩ B = A ∩ C and
(c) c is arbitrary and d = 1 (d) c = 1, d = 1
A ∪ B = A ∪ C, then
14 If R is relation from {11, 12 , 13 } to {8 , 10 , 12} defined by
(a) A = C (b) B = C (c) A ∩ B = φ (d) A = B
y = x − 3. Then, R −1 is
5 Suppose A1, A2,… , A 30 are thirty sets each having (a) {(8 , 11), (10, 13)} (b) {(11, 18), (13 , 10)}
5 elements and B1, B2,… , Bn are n sets each having (c) {(10, 13), (8 , 11)} (d) None of these
30 n
3 elements. Let ∪ A i = ∪ Bj = S and each element of S 15 Let R be a relation defined by R = {(4, 5), (1, 4), (4, 6),
i =1 j =1
(7, 6), (3, 7)}, then R −1 OR is
belongs to exactly 10 of Ai ’s and exactly 9 of Bj ’ s. The
(a) {(1, 1), (4, 4), (4, 7), (7, 4), (7, 7), (3, 3)}
value of n is equal to j NCERT Exemplar
(b) {(1, 1), (4, 4), (7, 7), (3, 3)}
(a) 15 (b) 3 (c) {(1, 5), (1, 6), (3, 6)}
(c) 45 (d) None of these (d) None of the above
6 If A and B are two sets and A ∪ B ∪ C = U. Then, 16 Let A be a non-empty set of real numbers and f : A → A
{( A − B ) ∪ (B − C ) ∪ (C − A )}′ is equal to be such that f (f ( x )) = x , ∀ x ∈ R. Then, f ( x ) is
(a) A ∪ B ∪ C (b) A ∪ (B ∩ C) (a) a bijection (b) one-one but not onto
(c) A ∩ B ∩ C (d) A ∩ (B ∪ C) (c) onto but not one-one (d) neither one-one nor onto
7 Let X be the universal set for sets A and B, if 17 The function f satisfies the functional equation
n( A ) = 200, n(B ) = 300 and n( A ∩ B ) = 100, then  x + 59
n ( A′ ∩ B′) is equal to 300 provided n( X ) is equal to 3f ( x ) + 2f   = 10x + 30 for all real x ≠ 1. The value
 x −1 
(a) 600 (b) 700 (c) 800 (d) 900
of f (7) is
8 If n( A ) = 1000, n(B ) = 500, n( A ∩ B ) ≥ 1 and n( A ∪ B ) = P , (a) 8 (b) 4 (c) − 8 (d) 11
then
18 The number of onto mapping from the set A = {1, 2, ...100}
(a) 500 ≤ P ≤ 1000 (b) 1001 ≤ P ≤ 1498
to set B = {1, 2} is
(c) 1000 ≤ P ≤ 1498 (d) 1000 ≤ P ≤ 1499
(a) 2100 − 2 (b) 2100 (c) 2 99 − 2 (d) 2 99
9 If n( A ) = 4, n(B ) = 3, n( A × B × C ) = 24, then n(C ) is equal to x −m
19 Let f : R − {n} → R be a function defined by f ( x ) = ,
(a) 2 (b) 288 (c) 12 (d) 1 x −n
where m ≠ n . Then,
10 If R = {( 3 , 3) ,( 6, 6), ( 9, 9), (12 , 12), ( 6 , 12), ( 3 , 9), (3,12),
(a) f is one-one onto (b) f is one-one into
(3, 6)} is a relation on the set A = {3 , 6 , 9 , 12}. (c) f is many-one onto (d) f is many-one into
The relation is
20 A function f from the set of natural numbers to integers
(a) an equivalence relation
n − 1
(b) reflexive and symmetric , when n is odd

(c) reflexive and transitive defined by f (n ) =  2 is
n
(d) only reflexive  − , when n is even
 2
11 Let R = {( x , y ) : x , y ∈ N and x 2 − 4xy + 3y 2 = 0}, where
(a) one-one but not onto (b) onto but not one-one
N is the set of all natural numbers. Then, the relation R is
(c) both one-one and onto (d) neither one-one nor onto
j
JEE Mains 2013
6 40 ONE

21 Let f : N → N defined by f ( x ) = x 2 + x + 1 , x ∈ N, then f is (c) S is an equivalence relation but R is not an equivalence


(a) one-one onto (b) many-one onto relation
(c) one-one but not onto (d) None of these (d) R and S both are equivalence relations
22 Let R be the real line. Consider the following subsets of 2 + x , x ≥ 0
24 If f ( x ) =  , then f (f ( x )) is given by
the plane R × R . 4 − x , x < 0
S = {( x , y ): y = x + 1and 0 < x < 2}  4 + x, x≥0  4 + x, x ≥ 0
(a) f (f (x)) =  (b) f (f (x)) = 
and T = {( x , y ): x − y is an integer}  6 − x, x<0  x, x<0
 4 − x, x≥0  4 − 2 x, x ≥ 0
Which one of the following is true? (c) f (f (x)) =  (d) f (f (x)) = 
 x, x<0  4 + 2 x, x < 0
(a) T is an equivalence relation on R but S is not
(b) Neither S nor T is an equivalence relation on R 25 Statement I A relation is defined by
(c) Both S and T are equivalence relations on R  x 2, 0 ≤ x ≤ 3
(d) S is an equivalence relation on R but T is not f (x ) =  is a function.
23 Consider the following relations 2 x , 3 ≤ x ≤ 9
R = {(x , y) | x and y are real numbers and x = wy for some Statement II In a function, every member must have a
rational number w}; unique image.
 m p 
S =  ,  m, n , p and q are integers such that n, q ≠ 0 (a) Statement I is true, Statement II is true; Statement II is a
 n q  correct explanation for Statement I
and qm = pn }. Then, (b) Statement I is true, Statement II is true; Statement II is
(a) R is an equivalence relation but S is not an equivalence not a correct explanation for Statement I
relation (c) Statement I is true; Statement II is false
(b) neither R nor S is an equivalence relation (d) Statement I is false; Statement II is true

DAY PRACTICE SESSION 2

PROGRESSIVE QUESTIONS EXERCISE


 1 6 On the set N of all natural numbers define the relation R
1 If f ( x ) + 2f   = 3x , x ≠ 0 and S = {x ∈ R : f ( x ) = f ( − x )};
x by aRb iff the g.c.d. of a and b is 2, then R is
then S j
JEE Mains 2016 (a) reflexive but not symmetric (b) symmetric only
(a) is an empty set (c) reflexive and transitive (d) equivalence relation
(b) contains exactly one element 7 Suppose f is a function satisfying f ( x + f ( x )) = 4f ( x ) and
(c) contains exactly two elements f (1) = 4. The value of f ( 21) is
(d) contains more than two elements
(a) 16 (b) 64 (c) 4 (d) 44
 2x − 1 
2 x ∈ R : 3 ∈R  is equal to  1 1
x + 4x + 3x
2 8 Let f  x +  = x 2 + 2 , x ≠ 0, then f ( x ) is equal to
   x  x
(a) R − {0} (b) R − {0, 1, 3 } (a) x 2 (b) x 2 − 1
(d) R − 0, − 1, − 3, 
1 (c) x 2 − 2 (d) x 2 + 1
(c) R − {0, − 1, − 3 }
 2 x
9 Let f ( x ) = , the fofofo K of ( x ) is
3 Given the relation R = {(1, 2) ( 2, 3)} on the set A = {1, 2, 3}, 1+ x2 1442443
x times
the minimum number of ordered pairs which when
x x
added to R make it an equivalence relation is (a) (b)
 n   n 
(a) 5 (b) 7 (c) 6 (d) 8 1+  Σ r  x2 1 +  Σ 1 x 2
r = 1  r = 1 
4 The set ( A ∪ B ∪ C ) ∩ ( A ∩ B ′∩ C ′ )′ ∩ C ′ is equal to  
x
x nx
j
NCERT Exemplar (c)   (d)
 1+ x2  1 + nx 2
(a) B ∩ C ′ (b) A ∩ C  
(c) B ′∩ C ′ (d) None of these 10 If two sets A and B are having 99 elements in common,
5 Let A = {1, 2, 3, 4}, B = {2, 4, 6}. Then the number of sets then the number of elements common to each of the sets
C such that A ∩ B ⊆ C ⊆ A ∪ B is A × B and B × A are
(a) 6 (b) 9 (c) 8 (d) 10 (a) 2 99 (b) 992 (c) 100 (d) 18
DAY 7

ANSWERS
SESSION 1 1. (b) 2. (d) 3. (d) 4. (b) 5. (c) 6. (c) 7. (b) 8. (d) 9. (a) 10. (c)
11. (a) 12. (b) 13. (b) 14. (a) 15. (a) 16. (a) 17. (b) 18. (a) 19. (b) 20. (c)
21. (c) 22. (a) 23. (c) 24. (a) 25. (d)

SESSION 2 1. (c) 2. (c) 3. (b) 4. (a) 5. (c) 6. (b) 7. (b) 8. (c) 9. (b) 10. (b)

Hints and Explanations


SESSION 1 6 From Venn Euler’s diagram, 11 Qa2 − 4 a ⋅ a + 3 a2 = 4 a2 − 4 a2 = 0
1 Clearly, 1 ≠ 0, 2 and 2 [Q y ∈ N ] ∴ (a, a) ∈ R, ∀a ∈ N ⇒ R is reflexive.
C U
y 3 Now, as a2 − 4 ab + 3b 2 = 0
A∩B∩C
1 C–A but b 2 − 4ba + 3 a2 ≠ 0
∴ can be 1.
y
∴ R is not symmetric.
⇒ x = 1 ∈Q Also, (a, b ) ∈ R and (b, c ) ∈ R
A–B B–C ⇒/ (a, c ) ∈ N
2 The number of elements in power set of B
A is 1. So, R is not transitive.
A
∴ P {P ( A )} = 2 = 2
1
12 Given, g ( x ) = 1 + x
⇒ P {P {P ( A )}} = 22 = 4 It is clear that,
{( A − B ) ∪ (B − C ) ∪ (C − A )} ′ and f { g ( x )} = 3 + 2 x + x …(i)
⇒ P {P {P {P ( A )}}} = 24 = 16 = A ∩ B ∩C ⇒ f (1 + x) = 3 + 2 x + x
3 We have, 7 Qn ( A ∪ B ) = n ( A ) + n (B ) − n ( A ∩ B ) Put 1 + x = y ⇒ x = ( y − 1)2
X = {4n − 3 n − 1: n ∈ N } ∴ n ( A ∪ B ) = 200 + 300 − 100 = 400 ∴ f ( y ) = 3 + 2 ( y − 1) + ( y − 1)2
∴ n ( A ′ ∩ B ′ ) = n ( A ∪ B )′ = n ( X ) = 2 + y2
X = {0, 9, 54, 243,...}
[put n = 1,2,3,...] − n( A ∪ B ) ∴ f ( x ) = 2 + x2
Y = {9(n − 1): n ∈ N } ⇒ 300 = n ( X ) − 400
⇒ n ( X ) = 700
13 Here, ( fog )( x ) = f {g ( x )} = a(cx + d ) + b
Y = {0, 9,18,27,...} and (gof ) ( x ) = g { f ( x )} = c (ax + b ) + d
[put n = 1,2,3,...] 8 We know, Since, cx + d + 2 = cx + 2c + d
It is clear that X ⊂ Y . n ( A ∪ B ) = n( A ) + n(B ) − n( A ∩ B )
[Q a = 1, b = 2]
∴ X ∪Y = Y ∴ P = 1500 − n ( A ∩ B )
⇒ n( A ∩ B ) = 1500 − P Hence, c = 1 and d is arbitrary.
4 Clearly, A ∩ B = A ∩ C and Clearly, 1 ≤ n( A ∩ B ) ≤ 500
A ∪ B = A ∪ C possible if 14 R is a relation from {11, 12, 13} to
[Qmaximum number of elements
B =C common in A and B = 500] {8, 10, 12} defined by
y =x−3⇒ x− y =3
⇒ 1 ≤ 1500 − P ≤ 500
5 Number of elements in ∴ R = {(11, 8), (13, 10)}
A 1 ∪ A 2 ∪ A 3 ∪…∪ A 30 is 30 × 5 but ⇒ − 1499 ≤ − P ≤ − 1000
⇒ 1000 ≤ P ≤ 1499 Hence, R −1 = {( 8, 11), (10, 13)}
each element is used 10 times, so
30 × 5 9 We know, 15 Clearly, R −1 = {(5, 4), (4, 1), (6, 4), (6, 7),
n(S ) = = 15 …(i)
10 n ( A × B × C ) = n ( A ) × n(B ) × n(C ) (7, 3)}
Similarly, number of elements in ∴ n(C ) =
24
=2 Now, as (4, 5) ∈ R and (5, 4) ∈ R −1 ,
4×3 therefore (4, 4) ∈ R −1OR
B1 ∪ B2 ... ∪ B n is 3 n but each element
is repeated 9 times, so 10 Since for each a ∈ A, (a, a)∈ R. R is Similarly, (1, 4) ∈ R and (4, 1) ∈ R −1
3n reflexive relation. ⇒ (1, 1) ∈ R −1OR
n(S ) =
9 (4, 6) ∈ R and (6, 7) ∈ R −1
Now, (6, 12) ∈ R but (12, 6) ∉ R. So, it is
3n not a symmetric relation. ⇒ (4, 7) ∈ R −1OR
⇒ 15 = [from Eq. (i)]
9 (7, 6) ∈ R and (6, 7) ∈ R −1
Also, (3, 6), (6, 12) ∈ R ⇒ (3, 12) ∈ R
⇒ n = 45 ⇒ (7, 7) ∈ R −1OR
⇒ R is transitive.
(7, 6) ∈ R and (6, 4) ∈ R −1
8 40 ONE

⇒ (7, 4) ∈ R −1OR integer is an image of odd natural ⇒ mq ⋅ ps = np ⋅ rq ⇒ ms = nr


and (3, 7) ∈ R and (7, 3) ∈ R −1 number. So, f is onto.

m r
=
⇒ (3, 3) ∈ R −1OR 21 Let x, y ∈ N such that f ( x ) = f ( y ) n s
m r
Hence, R −1OR = {(1, 1), (4, 4), (4, 7), ⇒ x2 + x + 1 = y 2 + y + 1 ⇒ S
n s
(7, 7), (7, 4), (3, 3)} ⇒ ( x2 − y 2 ) = y − x So, the relation S is transitive.
16 Let x, y ∈ A such that f ( x ) = f ( y ), then ⇒ ( x − y ) ( x + y + 1) = 0 Hence, the relation S is
f ( f ( x )) = f ( f ( y )) ⇒ x = y or x = − y − 1 ∉ N equivalence relation.
⇒ x= y ⇒ x= y 24 Clearly,
⇒ f is one-one. ⇒ f is one-one. 2 + f ( x ), f ( x) ≥ 0
Also,for any a∈ A, we have But f is not onto, as 1∈N does not have f ( f ( x )) = 
 4 − f ( x ), f ( x) < 0
f ( f (a)) = a any pre-image.
2 + (2 + x ), x ≥ 0
⇒ f (b ) = a , where b = f (a) ∈ A ∴ f is one-one but not onto. = 
Thus, for each a ∈ A (codomain) there 2 + (4 − x ), x < 0
exists b = f (a) ∈ A such that f (b ) = a 22 Since, (1, 2) ∈ S but (2, 1) ∉S 4 + x, x ≥ 0
= 
∴ f is onto. Thus S is not symmetric.
 6 − x, x < 0
Hence f is a bijective function. Hence, S is not an equivalence relation.
 2
 x + 59 
Given, T = {( x, y ) : ( x − y )∈ I } 25 Statement I f ( x ) =  x , 0 ≤ x ≤ 3
17 We have, 3 f ( x ) + 2 f   Now, x − x = 0∈ I , it is reflexive 2 x, 3 ≤ x ≤ 9
 x− 1 
relation. Now, f (3) = 9
= 10 x + 30 … (i) Again, now ( x − y )∈ I
x + 59 Also, f (3) = 2 × 3 = 6
On replacing x by , we get ⇒ y − x ∈ I , it is symmetric relation. Here, we see that for one value of x,
x−1
Let x − y = I1 there are two different values of f ( x ).
 x + 59 
3f   + 2 f ( x) and y − z = I2 Hence, it is not a function but
 x−1  Then, x − z = ( x − y ) + ( y − z) Statement II is true.
40 x + 560
= …(ii) = I1 + I2 ∈ I
x−1 So, T is also transitive. Hence, T is an SESSION 2
On solving Eqs. (i) and (ii), we get equivalence relation. 1 We have, f ( x ) + 2 f  1  = 3 x,
6 x − 4 x − 242
2  x
f ( x) = 23 Since, the relation R is defined as
x−1 R = {( x , y )| x , y are real numbers and x ≠ 0 … (i)
f   + 2 f ( x ) =
6 × 49 − 4 × 7 − 242 x = wy for some rational number w}. 1 3
∴ f (7) = =4 ∴ … (ii)
6 (a) Reflexive xRx as x = 1 x  x x

replacing x by 
Here, w = 1 ∈ Rational number 1
18 We know that if n( A ) = n and n(B ) = 2, 
So, the relation R is reflexive.  x 
the number of onto relations from A to
B = 2n − 2 (b) Symmetric xRy ⇒
/ yRx as 0R1 but On multiplying Eq. (ii) by 2 and then
subtracting it from Eq. (i), we get
∴ Required number of relations 1 R/ 0
6
So, the relation R is not − 3 f ( x) = 3 x −
= 2100 − 2 x
symmetric. 2
19 Suppose for any x, y ∈ R, Thus, R is not equivalence ⇒ f ( x) = − x
x
f ( x) = f ( y ) relation. Now, consider f ( x ) = f (− x )
x−m y −m
⇒ = Now, for the relation S, defined as,
2 2 4
x−n y −n  m p  ⇒ − x=− + x ⇒ = 2x
⇒ x= y S =   ,  m , n, p and q ∈ integers x x x
So, f is one-one.  n q 
Let α ∈ R be such that f ( x ) = α such that n , q ≠ 0 and qm = pn} ⇒ x2 = 2 ⇒ x = ± 2
x−m m − nα m m
∴ =α ⇒ x= (a) Reflexive S ⇒ mn = mn Thus, x contains exactly two elements.
x−n 1−α n n 2x −1
[true] 2 Clearly, 3 ∈ R only when
Clearly, x ∉ R for α = 1 Hence, the relation S is reflexive. x + 4 x2 + 3 x
So, f is not onto. m p x3 + 4 x2 + 3 x ≠ 0
(b) Symmetric S ⇒ mq = np
20 Let x, y ∈ N and both be even. n q Consider x3 + 4 x2 + 3 x = 0
x y p m
Then, f ( x ) = f ( y ) ⇒ − = − ⇒ np = mq ⇒ S ⇒ x ( x2 + 4 x + 3) = 0
2 2 q n
⇒ x=y ⇒ x ( x + 1) ( x + 3) = 0
Again, x, y ∈ N and both are odd. Hence, the relation S is ⇒ x = 0, − 1, − 3
Then, f ( x ) = f ( y ) ⇒ x = y symmetric.  2x − 1 
∴ x ∈ R : 3 ∈ R
So, f is one-one
m p
(c) Transitive S and
p r
S  x + 4 x2
+ 3 x 
n q q s = R − {0, − 1, − 3}
Since, each negative integer is an image
of even natural number and positive ⇒ mq = np and ps = rq
DAY 9

3. For R to be an equivalence relation, R Thus, aRb ⇒ bRa x


must be reflexive, symmetric and Hence, R is symmetric. 1 + x2
transitive. According to given option, R is =
x2
R will be reflexive if it contains (1, 1), symmetric only. 1+
(2, 2) and (3, 3) 1 + x2
7 We have, x
R will be symmetric if it contains =
f ( x + f ( x )) = 4 f ( x ) and f (1) = 4
(2, 1) and (3, 2) 1 + 2 x2
On putting x = 1, we get
R will be transitive if it contains (1, 3) x
and (3, 1) f (1 + f (1)) = 4 f (1) Similarly, f ( f ( f ( x ))) =
Hence, minimum number of ordered ⇒ f (1 + f (1)) = 16 1 + 3 x2
pairs = 7 ⇒ f (1 + 4) = 16 M M
x
4 ( A ∪ B ∪ C ) ∩ ( A ∩ B ′∩ C ′ )′∩ C ′ ⇒ f (5) = 16 fofo K of of ( x ) =
1442443 1 + nx2
= ( A ∪ B ∪ C ) ∩ ( A ′∪ B ∪ C ) ∩ C ′ On putting, x = 5, we get n times
= (φ ∪ B ∪ C ) ∩ C ′ f (5 + f (5)) = 4 f (5) x
=
= (B ∪ C ) ∩ C ′ ⇒ f (5 + 16) = 4 × 16  n 
= (B ∩ C ′ ) ∪ φ = B ∩ C ′ 1 +  Σ 1 x2
⇒ f (21) = 64 r =1 
5 Here, A ∩ B = {2, 4}
8 We have, 10 We know,
and A ∪ B = {1, 2, 3, 4, 6}
( A × B ) ∩ (C × D ) = ( A ∩ C )
f  x +  = x2 + 2
1 1
Q A ∩ B ⊆ C ⊆ A∪B
 x x × (B ∩ D )
∴ C can be {2, 4}, {1, 2, 4}, {3, 2, 4},
2 ∴ ( A × B ) ∩ (B × A ) = ( A ∩ B )
=  x +  − 2
{6, 2, 4}, {1, 6, 2, 4}, {6, 3, 2, 4}, 1
× (B ∩ A )
{1, 3, 2, 4}, {1, 2, 3, 4, 6}  x
Thus, number of elements common to
Thus, number of set C which satisfy the ⇒ f ( x ) = x2 − 2 A × B and B × A
given condition is 8.
x = n (( A × B ) ∩ (B × A ))
6 Clearly, g.c.d (a, a) = a, ∀ a ∈ N 9 We have, f ( x ) =
1 + x2 = n (( A ∩ B ) × (B ∩ A ))
∴R is not reflexive. = n( A ∩ B ) × n (B ∩ A )
f ( x)
If g.c.d (a, b ) = 2, then g.c.d (b, a) is ⇒ f ( f ( x )) = = 99 × 99 = 992
also 2. 1 + ( f ( x ))2
DAY TWO

Complex
Numbers
Learning & Revision for the Day

u Complex Numbers and Its u Argument or Amplitude of a u De-Moivre’s Theorem


Representation Complex Number u Cube Roots of Unity
u Algebra and Equality of u Different forms of a Complex u nth Roots of Unity
Complex Numbers Number
u Applications of Complex
u Conjugate and Modulus of a u Concept of Rotation
Numbers in Geometry
Complex Number u Square Root of a Complex
Number

Complex Numbers and Its Representation



A number in the form of z = x + iy, where x, y ∈ R and Y
P (x, y)
i = −1, is called a complex number. The real numbers
Imaginary axis

x and y are respectively called real and imaginary parts PRED


of complex number z.
i.e. x = Re (z), y = Im (z) and the symbol i is called iota.
y
MIRROR
Your Personal Preparation Indicator
X′ X

A complex number z = x + iy is said to be purely real if O x Real
y = 0 and purely imaginary if x = 0. axis u No. of Questions in Exercises (x)—
u No. of Questions Attempted (y)—

Integral power of iota (i)
Y′ u No. of Correct Questions (z)—
(i) i = −1, i 2 = − 1, i 3 = − i and i 4 = 1 Argand diagram (Without referring Explanations)
+1 +2
(ii) If n is an integer, then i 4 n = 1, i 4 n = i, i 4 n = −1
Accuracy Level (z / y × 100)—
and i 4 n + 3 = − i
u

n +1 n +2 n +3
u Prep Level (z / x × 100)—
(iii) i + i
n
+i +i =0

The complex number z = x + iy can be represented by a point P in a plane called In order to expect good rank in JEE,
your Accuracy Level should be
argand plane or Gaussian plane or complex plane. The coordinates of P are referred above 85 & Prep Level should be
to the rectangular axes XOX ′ and YOY′ which are called real and imaginary axes,
respectively.
DAY 11

Algebra and Equality of Argument or Amplitude of


Complex Numbers a Complex Number
If z1 = x1 + i y1 and z2 = x2 + i y2 are two complex numbers, then Let z = x + iy be a complex number, represented by a point
(i) z1 + z2 = ( x1 + x2 ) + i ( y1 + y2 ) P( x, y) in the argand plane. Then, the angle θ which OP makes
(ii) z1 − z2 = ( x1 − x2 ) + i ( y1 − y2 ) with the positive direction of Real axis (X -axis) is called
(iii) z1 z2 = ( x1 x2 − y1 y2 ) + i( x1 y2 + x2 y1 ) the argument or amplitude of z and it is denoted by arg ( z ) or
z1 ( x1 x2 + y1 y2 ) + i ( x2 y1 − x1 y2 ) amp ( z ).
(iv) =  y
z2 x22 + y22 The argument of z, is given by θ = tan −1  
 x
(v) z1 and z2 are said to be equal if x1 = x2 and y1 = y2 .

The value of argument θ which satisfies the inequality
NOTE • Complex numbers does not possess any inequality, − π < θ < π, is called principal value of argument.
e.g. 3 + 2i > 1 + 2i does not make any sense. ●
The principal value of arg(z) is θ, π − θ, −π + θ or − θ
according as z lies in the 1st, 2nd, 3rd or 4th quadrants
Conjugate and Modulus respectively, where θ = tan −1
y
.
of a Complex Number x

If z = x + iy is a complex number, then conjugate of z is P (x, y)

Imaginary axis
denoted by z and is obtained by replacing i by −i.
i.e. z = x − iy
y

If z = x + iy, then modulus or magnitude of z is denoted by
| z| and is given by| z| = x + y
2 2
θ
x Real axis
Results on Conjugate and Modulus ●
Argument of z is not unique. General value of argument of z
(i) (z) = z is 2nπ + θ.
(ii) z + z = 2 Re (z), z − z = 2 i Im(z)
(iii) z = z ⇔ z is purely real.
Results on Argument
(iv) z + z = 0 ⇔ z is purely imaginary. If z, z1 and z2 are complex numbers, then
(v) z1 ± z2 = z1 ± z2 (i) arg (z) = − arg (z)
(vi) z1 z2 = z1 z2 (ii) arg (z1 z2 ) = arg (z1 ) + arg (z2 )
 z1 
 z1  z1 (iii) arg   = arg (z1 ) − arg (z2 )
(vii)   = , if z2 ≠ 0  z2 
 z2  z2
a1 a2 a3 a1 a2 a3 (iv) The general value of arg (z) is 2nπ − arg (z).
π
(viii) If z = b1 b2 b3 , then z = b1 b2 b3 (v) If z is purely imaginary then arg (z) = ± .
2
c1 c2 c3 c1 c2 c3 (vi) If z is purely real then arg (z) = 0 or π.
where ai , b i , c i ; (i = 1, 2 , 3) are complex numbers. (vii) If| z1 + z2| = | z1 − z2|, then
| z| = 0 ⇔ z = 0
(ix)
 z1  π
| z| = | z| = |−z| = |−z|
(x) arg   or arg (z1 ) − arg (z2 ) =
− | z| ≤ Re(z), Im(z) ≤ | z|
(xi)  2
z 2
| z1 z2| = | z1|| z2|
(xii) (viii) If| z1 + z2| = | z1| + | z2|, then arg (z1 ) = arg (z2 )
z1 | z1|
(xiii) = , if| z2| ≠ 0
z2 | z2|
Different forms of a
(xiv) | z1 ± z2|2 = | z1|2 + | z2|2 ± z1 z2 ± z1 z2
= | z1|2 + | z2|2 ± 2 Re (z1 z2 ) Complex Number
(xv) | z | = | z| , n ∈ N
n n ●
Polar or Trigonometrical Form of z = x + iy is
(xvi) Reciprocal of a complex number For non-zero complex z = r (cos θ + i sin θ), where r =| z| and θ = arg (z).
number z = x + iy, the reciprocal is given by If we use the general value of the argument θ, then the
1 z
z −1 = = 2 . polar form of z is z = r [cos (2nπ + θ) + i sin (2nπ + θ)], where
z | z| n is an integer.
(xvii) Triangle Inequality ●
Euler’s form of z = x + iy is z = re iθ , where r =| z|,θ = arg (z)
(a)| z1 + z2 | ≤ | z1 | + | z 2 | (b) | z1 + z2 | ≥ || z1 | − | z 2 ||
(c)| z1 − z 2 | ≤ | z1 | + | z 2 | (d) | z1 − z2 | ≥ || z1 | − | z 2 || and e iθ = cos θ + i sin θ.
12 40

(i) The nth roots of unity are 1, α , α 2 ,... , α n − 1 , where


Concept of Rotation i2 π

α =e n
Let z1 , z2 , z3 be the vertices of ∆ABC as shown in figure, then
z −z  z − z |z − z | (ii) 1 + α + α 2 + α 3 + ...+ α n −1 = 0
α = arg  3 1  and 3 1 = 3 1 e iα
 z2 − z1  z2 − z1 | z2 − z1 | (iii) 1 ⋅ α ⋅ α 2 ... α n −1 = [−1]n −1
C(z3)
Applications of Complex
Numbers in Geometry
1. Distance between A(z1 ) and B (z2 ) is given by
a AB = | z2 − z1|.
A(z1) B(z2) 2. Let point P (z) divides the line segment joining A (z1 ) and
B (z2 ) in the ratio m : n . Then,
NOTE • Always mark the direction of arrow in anti-clockwise sense mz2 + nz1
and keep that complex number in the numerator on which (i) for internal division, z =
the arrow goes. m+n
mz2 − nz1
(ii) for external division, z =
Square Root of a Complex Number m−n

If z = a + ib , then 3. Let ABC be a triangle with vertices A (z1 ), B (z2 ) and C(z3 ),

z = a + ib = ±
1
[ | z| + a + i z − a] then centroid G (z) of the ∆ ABC is given by z
2 1
= (z1 + z2 + z3 )
1 3

If z = a − ib , then z = a − ib = ± [ | z| + a − i | z| − a]
2
z1 z1 1
1
Area of ∆ ABC is given by ∆ = z2 z2 1
De-Moivre’s Theorem 2
z3 z3 1

If n is any integer, then (cos θ + i sin θ)n = cos nθ + i sin nθ

If n is any rational number, then one of the values of 4. For an equilateral triangle ABC with vertices A(z1 ), B(z2 )
(cos θ + i sin θ)n is cos nθ + i sin nθ. and C(z3 ), z12 + z22 + z23 = z 2 z3 + z 3 z1 + z1 z 2

If n is any positive integer, then 5. The general equation of a straight line is az + az + b = 0,
 2 kπ + θ   2 kπ + θ  where a is a complex number and b is a real number.
(cos θ + i sin θ)1 / n = cos   + i sin  
 n   n  6. (i) An equation of the circle with centre at z0
where, k = 0, 1, 2,... n − 1 and radius r, is | z − z0| = r
(ii) | z − z0| < r represents the interior of circle and
Cube Root of Unity | z − z0| > r represents the exterior of circle.
Cube roots of unity are 1, ω, ω2 (iii) General equation of a circle is zz + az + az + b = 0,
−1 + 3 i −1 − 3 i where b is real number, with centre is − a and radius
where, ω = and ω2 = is aa − b .
2 2
7. If z1 and z2 are two fixed points and k > 0, k ≠ 1 is a real
Properties of Cube Roots of Unity | z − z1|
number, then = k represents a circle.
(i) 1 + ω + ω2 = 0 | z − z2|
(ii) ω3 = 1
For k = 1, it represents perpendicular bisector of the
0 if n ≠ 3 m, m ∈ N
(iii) 1 + ω n + ω2 n =  segment joining A(z1 ) and B (z2 ).
3 if n = 3 m, m ∈ N 8. If end points of diameter of a circle are A(z1 ) and B(z 2) and
P(z) be any point on the circle, then equation of circle in
nth Roots of Unity diameter form is
By nth root of unity we mean any complex number z which (z − z1 ) (z − z2 ) + (z − z 2) (z − z1 ) = 0
satisfies the equation zn = 1.
DAY 13

DAY PRACTICE SESSION 1

FOUNDATION QUESTIONS EXERCISE


1 4
1 Real part of is 11 If z − = 2, then the maximum value of | z | is
1 − cos θ + i sin θ z j AIEEE 2009

1 1 1
(a) − (b) (c) tanθ / 2 (d) 2 (a) 3 +1 (b) 5 +1 (c) 2 (d) 2 + 2
2 2 2
12 If z is a complex number such that z ≥ 2, then the
2 + 3i sin θ 1
2 A value of θ, for which is purely imaginary, is minimum value of z +
1 − 2i sin θ 2 j JEE Mains 2014
π π
(d) sin 
−1 3 −1 1 
(a) (b) (c) sin  (a) is equal to 5/2
3 6 4  3 (b) lies in the interval (1, 2)
13 (c) is strictly greater than 5/2
3 ∑ (i
n =1
n
+ i n + 1 ) is equal to (d) is strictly greater than 3/2 but less than 5/2
13 If | z1| = 2, | z 2 | = 3 then z1 + z 2 + 5 + 12i is less than or
(a) i (b) i − 1 (c) −i (d) 0
equal to
z −1
4 If is a purely imaginary number ( where, z ≠ −1), then (a) 8 (b) 18 (c) 10 (d) 5
z +1
14 If z < 3 − 1, then z + 2z cos α is
2
the value of | z | is
(a) less than 2 (b) 3 + 1
(a) −1 (b) 1 (c) 2 (d) −2
(c) 3 − 1 (d) None of these
z2
5 If z1 ≠ 0 and z 2 are two complex numbers such that is 15 The number of complex numbers z such that
z1
z − 1 = z + 1 = z − i , is
2z1 + 3z 2
a purely imaginary number, then is equal to (a) 0 (b) 1 (c) 2 (d) ∞
2z1 − 3z 2
2
j
JEE Mains 2013
16 Number of solutions of the equation z + 7z = 0 is/are
(a) 2 (b) 5 (c) 3 (d) 1 (a) 1 (b) 2 (c) 4 (d) 6
7−z 17 If z z + ( 3 − 4i )z + ( 3 + 4i )z = 0 represent a circle, the area
6 If f ( z ) = , where z = 1 + 2i , then | f ( z )| is equal to
1− z 2 of the circle in square units is
|z | (a) 5 π (b) 10π (c) 25 π 2 (d) 25 π
(a) (b) | z |
2
 π  π
(c) 2| z | (d) None of these 18 If z = 1 + cos   + i sin  , then {sin (arg( z ))} is equal to
 5  5
7 If 8 iz 3 + 12z 2 − 18z + 27i = 0, then the value of | z | is
10 − 2 5 5 −1
(a) 3 / 2 (b) 2 / 3 (c) 1 (d) 3 / 4 (a) (b)
4 4
8 If a complex number z satisfies the equation 5+1
(c) (d) None of these
z + 2 z + 1 + i = 0, then z is equal to j
JEE Mains 2013 4
(a) 2 (b) 3 (c) 5 (d) 1 19 If z is a complex number of unit modulus and argument
9 If α and β are two different complex numbers such that 1 + z 
θ, then arg   equals to
β −α 1 + z  JEE Mains 2013
| α | = 1, | β | = 1, then the expression
j
is equal to
1 − αβ π
(a) −θ (b) −θ (c) θ (d) π − θ
1 2
(a) (b) 1
2 20 Let z and ω are two non-zero complex numbers such that
(c) 2 (d) None of these z = ω and arg z + arg ω = π, then z equals
z −1 (a) ω (b) ω
10 If | z | = 1 and ω = (where z ≠ −1), then Re(ω ) is
z +1 (c) − ω (d) − ω
1 21 If z − 1 = 1, then arg ( z ) is equal to
(a) 0 (b) − 2
z+1 1 1
(a) arg (z) (b) arg (z + 1)
2 2 3
(c) 2
(d) None of these 1
z+1 (c) arg (z − 1) (d) None of these
2
14 40

15
 1
22 Let z = cos θ + i sin θ. Then the value of ∑ Im ( z 2 m −1 ) at, 33 If Re   = 3 , then z lies on
m =1
z
θ = 2°, is (a) circle with centre onY-axis
1 1 1 1 (b) circle with centre on X-axis not passing through origin
(a) (b) (c) (d)
sin 2 ° 3 sin 2 ° 2 sin 2 ° 4 sin 2 ° (c) circle with centre on X-axis passing through origin
(i )
23 If z = (i )(i ) , where i = −1, then | z | is equal to (d) None of the above

(a) 1 (b) e − π / 2 (c) 0 (d) e π / 2 34 If the imaginary part of ( 2z + 1) / (iz + 1) is −2, then the
8 locus of the point representing z in the complex plane is
 π π
1 + i sin + cos  (a) a circle (b) a straight line
24  8 8  equals to
(c) a parabola (d) None of these
 1 − i sin π + cos π 
 8 8 z
35 If | z | = 1 and z ≠ ± 1, then all the values of lie on
(a) 2 8 (b) 0 (c) −1 (d) 1
1− z 2
(a) a line not passing through the origin
25 If 1, α 1, α 2 , K , α n − 1 are the nth roots of unity, then (b) |z | = 2
(c) the X-axis
( 2 − α 1 )( 2 − α 2 ) K ( 2 − α n −1 ) is equal to
(d) theY-axis
(a) n (b) 2 n (c) 2 n + 1 (d) 2 n − 1
z
26 If ω( ≠ 1) is a cube root of unity and (1 + ω )7 = A + Bω. 36 If ω = and | ω | = 1, then z lies on
i
z−
Then, ( A, B ) is equal to 3
(a) (11
,) (b) (1, 0) (c) (−1, 1) (d) (0, 1) (a) a circle (b) an ellipse
27 If α , β ∈C are the distinct roots of the equation (c) a parabola (d) a straight line
x 2 − x + 1 = 0, then α 101 + β107 is equal to j JEE Mains 2018 37 If z 1 and z 2 are two complex numbers such that
(a) −1 (b) 0 (c) 1 (d) 2 z1 z 2
2
+ = 1, then

25
1 z 2 z1
28 If x 2 + x + 1 = 0, then ∑  x r +  is equal to
r=1
 xr  (a) z 1, z 2 are collinear
(b) z 1, z 2 and the origin form a right angled triangle
(a) 25 (b) 25 ω
(c) z 1, z 2 and the origin form an equilateral triangle
(c) 25 ω2 (d) None of these
(d) None of the above
29 Let ω be a complex number such that 2ω + 1 = z,
38 A complex number z is said to be unimodular, if z = 1.
1 1 1
Suppose z1 and z 2 are complex numbers such that
where z = −3. If 1 −ω − 1 ω 2 = 3k, then k is equal to
2
z1 − 2z 2
is unimodular and z 2 is not unimodular.
1 ω2 ω7 j
JEE Mains 2017 2 − z 1z 2
(a) −z (b) z (c) −1 (d) 1
Then, the point z1 lies on a j
JEE Mains 2015
1+ ω ω2 1 + ω2
(a) straight line parallel to X −axis
30 The value − ω − (1 + ω 2 ) (1 + ω) , where ω is cube (b) straight line parallel toY −axis
− 1 − (1 + ω 2 ) 1 + ω (c) circle of radius 2
(d) circle of radius 2
root of unity, is equal to
(a) 2 ω (b) 3 ω2 (c) − 3 ω2 (d) 3ω 39 If | z 2 − 1 | = | z |2 +1, then z lies on
31 If a , b and c are integers not all equal and ω is a cube (a) a real axis (b) an ellipse
root of unity (where, ω ≠ 1), then minimum value of (c) a circle (d) imaginary axis
| a + bω + cω 2 | is equal to
3 1 40 Let z satisfy z = 1 and z = 1 − z j
JEE Mains 2013
(a) 0 (b) 1 (c) (d)
2 2 Statement I z is a real number.
iπ /3
32 Let ω = e , and a, b, c, x , y , z be non-zero complex Statement II Principal argument of z is π /3.
numbers such that: (a) Statement I is true, Statement II is true; Statement II is a
a + b + c = x ; a + b ω + cω = y ; a + b ω + c ω = z
2 2 correct explanation for statement I
2 2 2
(b) Statement I is true, Statement II is true; Statement II is
x + y + z not a correct explanation for Statement I
Then the value of 2 2 2
is:
a + b + c (c) Statement I is true, Statement II is false
(d) Statement I is false, Statement II is true
(a) 1 (b) 2 (c) 3 (d) 4
DAY PRACTICE SESSION 2

PROGRESSIVE QUESTIONS EXERCISE


1 For positive integers n1 and n 2 the value of the expression 8 If a complex number z lies in the interior or on the
(1 + i )n1 + (1 + i 3 )n1 + (1 + i 5 )n 2 + (1 + i 7 )n 2 boundary of a circle of radius 3 and centre at ( − 4 , 0),
where i = −1, is a real number iff then the greatest and least value of | z + 1| are
(a) n1 = n2 (b) n2 = n2 − 1 (c) n1 = n2 + 1 (d) ∀n1 and n2 (a) 5, 0 (b) 6, 1 (c) 6, 0 (d) None of these
9 If z is any complex number satisfying z − 3 − 2i ≤ 2,
2
z
2 If z ≠ 1 and is real, then the point represented by the
z −1 then the minimum value of 2z − 6 + 5i is
complex number z lies (a) 2 (b) 3 (c) 5 (d) 6
(a) on the imaginary axis 10 A man walks a distance of 3 units from the origin towards
(b) either on the real axis or on a circle passing through the the North-East (N 45° E) direction. From there, he walks a
origin
distance of 4 units towards the North-West (N 45° W)
(c) on a circle with centre at the origin
direction to reach a point P. Then the position of P in the
(d) either on the real axis or on a circle not passing through
Argand plane is
the origin
(a) 3eiπ / 4 + 4i (b) (3 − 4i )eiπ / 4
2π 2π
3 Let ω be the complex number cos + i sin . Then the (c) (4 + 3i )eiπ / 4 (d) (3 + 4i )eiπ / 4
3 3
number of distinct complex numbers z satisfying 11 If 1, a1, a 2 ... a n −1 are n th roots of unity, then
1 1 1
z +1 ω ω2 + + ... + equals to
1 − a1 1 − a 2 1 − a n −1
ω z + ω2 1 = 0 is equal to
ω2 1 z +ω 2n − 1 n −1 n
(a) (b) (c) (d) None of these
n 2 n −1
(a) 0 (b) 1 (c) 2 (d) 4 2 2
12 For z ,ω ∈ C, if z ω − ω z = z − ω, then z is equal to
4 The locus of z = x + iy which satisfying the inequality
log1/ 2 z − 1 > log1/ 2 z − i is given by (a) ω or ω (b) ω or ω /|ω | 2
(c) ω or ω /|ω | 2 (d) None of these
(a) x + y < 0 (b) x − y > 0 (c) x − y < 0 (d) x + y > 0
10
 2kπ 2kπ 
5 Let z1 = 10 + 6i , z 2 = 4 + 6i . If z is any complex number 13 The value of ∑  sin + i cos  is
 11 11 
such that arg ( z − z1 ) / ( z − z 2 ) = π / 4, then z − 7 − 9i is k =1

equal to (a) 1 (b) −1 (c) − i (d) i


(a) 18 (b) 3 2 (c) 3 / 2 (d) None of these 14 Let z1 and z 2 be roots of the equation z + pz + q = 0, 2

6 Let z = x + iy be a complex number where x and y are p, q ∈ C. Let A and B represents z1 and z 2 in the complex
integers. Then the area of the rectangle whose vertices plane. If ∠AOB = α ≠ 0 and OA = OB; O is the origin, then
are the roots of the equation zz 3 + zz 3 = 350 is p 2 / 4q is equal to
(a) 48 (b) 32 (c) 40 (d) 80 (a) sin2 (α / 2) (b) tan2 (α / 2) (c) cos2 (α / 2) (d) None of these

7 If α + i β = cot −1( z ), where z = x + iy and α is a constant, 15 If 1, ω and ω 2 are the three cube roots of unity α , β, γ are
then the locus of z is the cube roots of p, q < 0, then for any x , y , z the
(a) x2 + y2 − x cot 2 α − 1 = 0  x α + y β + z γ
expression   is equal to
(b) x2 + y2 − 2 x cot α − 1 = 0  x β + y γ + z α
(c) x2 + y2 − 2 x cot 2 α + 1 = 0
(d) x2 + y2 − 2 x cot 2 α − 1 = 0 (a) 1 (b) ω (c) ω 2 (d) None of these

ANSWERS
SESSION 1 1. (b) 2. (d) 3. (b) 4. (b) 5. (d) 6. (a) 7. (a) 8. (c) 9. (b) 10. (a)
11. (b) 12. (b) 13. (b) 14. (a) 15. (b) 16. (b) 17. (d) 18. (b) 19. (c) 20. (c)
21. (c) 22. (d) 23. (a) 24. (c) 25. (d) 26. (a) 27. (c) 28. (d) 29. (a) 30. (c)
31. (b) 32. (c) 33. (c) 34. (b) 35. (d) 36. (d) 37. (c) 38. (c) 39. (d) 40. (d)

SESSION 2 1. (d) 2. (b) 3. (b) 4. (b) 5. (b) 6. (a) 7. (d) 8. (c) 9. (c) 10. (d)
11. (b) 12. (b) 13. (c) 14. (c) 15. (c)
16 40

Hints and Explanations


SESSION 1 ⇒ x2 + y 2 = 1 10 Given,|z | = 1
1 ⇒ |z |2 = 1 ⇒ |z |= 1
1 Let z = ⇒ zz = 1
1 − cos θ + i sin θ 5 Given, z2 is a purely imaginary z−1 z−1
1 Now, 2Re(ω ) = ω + ω = +
= z1 z+1 z+1
2sin2 (θ / 2) + 2i sin(θ / 2)cos(θ / 2) Let z = ni. Then, (z − 1)(z + 1) + (z − 1)(z + 1)
1 1 =
= z2 z+1
2
2+ 3⋅
2i sin(θ / 2) [cos(θ / 2) − i sin(θ / 2)] 2z1 + 3z2 z1 2 + 3ni
= = 2zz − 2
cos(θ / 2) + i sin(θ / 2) 1 1 2z1 − 3z2 z2 2 − 3ni = =0 [Q zz = 1]
= = + cot(θ / 2) 2 − 3⋅ z+1
2
2i sin(θ / 2) 2 2i z1
1 1
= − i ⋅ cot θ / 2 ∴ Re(ω ) = 0.
4 + 9n2
2 2 = =1
4 + 9n2   z − 4  + 4
11 |z | =  
 
2 Let z = 2 + 3i sin θ is purely imaginary 7− z

  z z
1 − 2i sin θ 6 Given, f (z ) = and z = 1 + 2i
|z | ≤  z −  +
1 − z2 4 4
then we have ⇒
Re (z ) = 0 7 − (1 + 2i )  z  |z |
∴ f (z ) =
2 + 3i sin θ 1 − (1 + 2i )2 4
Consider, z = ⇒ |z | ≤ 2 +
1 − 2i sin θ 6 − 2i 6 − 2i |z|
= =
(2 + 3i sin θ)(1 + 2i sin θ) 1 − (1 − 4 + 4i ) 4 − 4i |z |2 − 2 |z | − 4
= ⇒ ≤0
(1 − 2i sin θ)(1 + 2i sin θ) 6 − 2i 1+ i 6 + 4i + 2 |z |
= × =
(2 − 6sin2 θ) + (4sin θ + 3sin θ )i 4(1 − i ) (1 + i ) 4(12 − i 2 ) Since,|z | > 0
=
1 + 4sin2 θ 8 + 4i 1 ⇒ |z |2 − 2 |z | − 4 ≤ 0
= = (2 + i )
Q Re(z ) = 0 4(2) 2 ⇒ [|z | − ( 5 + 1)] [|z |− (1 − 5)] ≤ 0
2 − 6sin2 θ
∴ =0 4+ 1 5 |z | ⇒ 1− 5 ≤ |z | ≤ 5+ 1
1 + 4sin2 θ Now,| f (z )| = = =
2 2 2

1
sin2 θ = ⇒ sinθ = ±
1 12 z ≥ 2 is the region on or outside circle
[Qz = 1 + 2i , given ⇒|z |= 5]
3 3 whose centre is (0,0) and radius is 2.
1
−1  1  7 Given, 8 iz 3 + 12z 2 − 18z + 27i = 0 Minimum z + is distance of z, which
⇒ θ = ± sin   2
 3 ⇒ 4z 2 (2iz + 3) + 9 i (2iz + 3) = 0
lie on circle z = 2 from  − , 0 .
1
13 13 ⇒ (2iz + 3) (4z2 + 9 i ) = 0
 2 
∑ (i + i n +1 ) = (1 + i ) ∑i
n n
3 ⇒ 2iz + 3 = 0 or 4z2 + 9 i = 0
n =1 n =1 Y
3
i (1 − i 13 ) ∴ |z | =
= (1 + i ) 2
1− i
8 We have, ( x + iy ) + 2 x + iy + 1 + i = 0
= i − 1 [Q i 13 = i , i 2 = −1]
[put z = x + iy ]
4 Let z = x + iy A
z−1 x + iy − 1 ( x − 1) + iy
⇒ ( x + iy ) + 2 ( x + 1)2 + y 2 + i = 0 X′
(–2, 0) (– 1 ,0
( (0, 0) (2, 0)
X
= = ⇒ x + 2 ( x + 1) + y = 02 2 2
z + 1 x + iy + 1 ( x + 1) + iy
( x + 1) − iy and y+1= 0
×
( x + 1) − iy ⇒ x + 2 ( x + 1) + (−1)2 = 0
2

( x − 1)( x + 1) − iy ( x − 1) + iy and y = −1 Y′
( x + 1) − i 2 y 2 ⇒ x2 = 2[( x + 1)2 + 1]
= ∴ Minimum z +
1
( x + 1)2 − i 2 y 2 ⇒ x2 = 2 x2 + 4 x + 4
2
x2 − 1 + iy ( x + 1 − x + 1) + y 2 ⇒ x + 4 x + 4 = 0 ⇒ ( x + 2)2 = 0
2

=  1 
= Distance of  − , 0 from (−2, 0)
⇒ x = −2
( x + 1)2 + y 2  2 
∴ z = −2 − i ⇒ z = 4 + 1 = 5
z − 1 ( x2 + y 2 − 1) i (2 y ) 2
⇒ = + β −α β −α =  −2 + 1  + 0 = 3
z + 1 ( x + 1)2 + y 2 ( x + 1)2 + y 2 9 =  
 2 2
z−1 1 − αβ β ⋅ β − αβ
Since, is purely imaginary. Alternate Method
z+1  Q |β | = 1 
We know,|z1 + z2 |≥||z1 |−|z2||
z − 1 and |β|2 = ββ = 1
∴ Re   =0  
 z + 1 ∴ z+
1
≥ |z| −
1
= |z|−
1
x + y −1
2 2 β −α 1 |β − α| |β − α| 2 2 2
⇒ =0 = = = =1
( x + 1)2 + y 2 β (β − α ) |β| | β − α| |β − α| 1 3
≥ z− =
⇒ x + y −1 = 0
2 2 [Q|z|=|z|] 2 2
DAY 17

∴ z+
1

3 20 Let z = ω = r and let arg ω = θ ⇒ − ω2 = A + Bω ⇒ 1 + ω = Α + Βω
2 2 [Qω14 = ω12 ⋅ ω2 = ω2 ]
Then, ω = r (cos θ + i sin θ) = re i θ
1 3 On comparing both sides, we get
∴ Minimum value of z + is ⋅ and arg z = π − θ
2 2 A = 1, B = 1
Hence, z = r (cos( π − θ) + i sin( π − θ))
13 Fact: z1 + z2 + ... + z n = r (− cos θ + i sin θ) 27 α,β are the roots of x2 − x + 1 = 0
= − r (cos θ − i sin θ) Q Roots of x2 − x + 1 = 0 are − ω, − ω 2
≤ z1 + z2 + ... + z n z = −ω
∴ z1 + z2 + (5 + 12i ) ∴ Let α = −ω and β = − ω 2
21 Given,|z − 1| = 1 ⇒ z − 1 = e i θ ,
≤ z1 + z2 + 5 + 12i ⇒ α101 + β107 = (− ω )101 + (− ω2 )107
= 2 + 3 + 13 = 18 where arg(z − 1) = θ …(i)
= − (ω101 + ω214 ) = − (ω2 + ω )
2 ⇒ z = eiθ + 1
14 Consider z2 + 2z cos α ≤ z + 2 z [Qω3 n + 2 = ω2 and ω3 n+1 = ω]
⇒ z = 1 + cos θ + i sin θ
cos α ≤ z + 2 z
2 = −(−1) = 1 [1 + ω + ω2 = 0]
[Qe i θ = cos θ + i sin θ]
2θ θ θ 28 x2 + x + 1 = 0
< ( 3 − 1)2 + 2( 3 − 1) = 2 cos + 2i sin ⋅ cos
2 2 2 ⇒ x = ω , ω2
= 3+ 1−2 3 + 2 3 −2= 2 θ 1
⇒ arg (z ) = = arg(z − 1) [from Eq. (i)] 1 1
∴ z + 2z cos α < 2
2 So, x + r = ω r + r = − 1
r
2 2 x ω
15 Let z = x + iy 22 Given that z = cos θ + i sin θ = e iθ or 2 according as r is not divisible by 3
15 15
|z − 1| = |z + 1|
∑ lm(z ∑ lm(e or divisible by 3.
2 m −1 iθ 2 m −1
∴ )= )
Re z = 0 ⇒ x = 0 m=1 m=1 ∴ Required sum
|z − 1| = |z − i| ⇒ x = y
15
= 17(−1) 2 + 8 ⋅ 2 2 = 49
∑ lme
i(2 m −1 )θ
=
|z + 1| = |z − i| ⇒ y = − x m=1
29 Given, z = 2ω + 1
Since, only (0, 0) will satisfy all = sin θ + sin 3θ + sin 5θ + ... + sin 29θ
−1 + z −1 + 3i
14 ⋅ 2θ   15⋅ 2θ  ⇒ω = ⇒ω =
sin  θ +
conditions.
∴ Number of complex number z = 1.  sin   2 2
 2   2  [Qz = −3]
=

sin  
2
16 Given z + 7z = 0 ⇒ ω is complex cube root of unity
2 1 1 1
⇒ z z + 7z = 0 ⇒ z (z + 7) = 0
sin(15θ)sin(15θ) 1 1 −ω2 − 1 ω2 = 3k
Case (i) : z = 0, ∴ z = 0 = 0 + i0 = = [Qθ = 2° ] Now,
sin θ 4sin 2° ω2 ω7
Case (ii) : z = −7 ∴ z = −7 + 0i 1
Hence, there is only two solutions. 23 Clearly, i = cos π + i sin π = e iπ /2 1 1 1
2 2
z = 0 and z = −7 i2 ⋅
π ⇒ 1 ω ω2 = 3k
∴ (i )i = (e iπ /2 )i = e 2
= e − π /2
17 Given zz + (3 − 4i )z + (3 + 4i )z = 0 ( i )(i ) e − π /2 − π /2
1 ω2 ω
Let z = x + iy Now, (i ) = (i ) ⇒ z = (i )e
e − π /2
Q1 + ω + ω2 = 0
Then, zz = x2 + y 2 ⇒ |z|=|i | =1  
 ω7 = ω 
∴ x2 + y 2 + (3 − 4i )( x + iy ) 24 Let z = cos + i sin π
π
+ (3 + 4i )( x − iy ) = 0 8 8 Applying R1 → R1 + R2 + R3 , we get
1 π π 3 1 + ω + ω2 1 + ω + ω2
⇒ x2 + y 2 + 6 x + 8 y = 0 Then, = cos − i sin
z 8 8 1 ω ω2 = 3k
⇒ ( x + 6 x) + ( y + 8 y ) = 0
2 2
8
 1 + cos π + i sin π  1 ω2
ω
⇒ ( x + 3)2 + ( y + 4)2 = 32 + 42   8

Now,  8 8  =  1 + z 
⇒ [ x − (−3)]2 + [ y − (−4)]2 = 52 π
 1 + cos − i sin  π 1 + z 
−1 3 0 0
So, area of circle be π R = 25π2  8 8 ⇒ 1 ω ω2 = 3k
8 8
[QR = radius = 5]  (1 + z )z   π π 1 ω2 ω
=  = z =  cos + i sin 
8

 (1 + z )   8
18 If z = 1 + cos θ + i sin θ, then arg(z ) = θ
8 ⇒ 3(ω2 − ω 4 ) = 3k
π π
2 = cos 8 ⋅ + i sin 8 ⋅ ⇒ k = ω2 − ω ⇒ k = −1 − 2ω
π/5 π 8 8
⇒ k = −(1 + 2ω ) ⇒ k = − z
∴ arg(z ) = =
2 10 [using De-moivre’s theorem]
⇒ sin(arg z ) = cos π = −1(Q sin π = 0) 30 Using 1 + ω + ω2 = 0, we get
π 5−1 1 + ω ω2 −ω
= sin   = sin18° = 25 Clearly, ( x − 1)( x − α1 )( x − α2 ) …
 10  4 ( x − α n −1 ) = x – 1 n ∆ = 1 + ω2 ω − ω2
Putting x = 2, we get ω2 + ω ω − ω2
19 Given, z = 1 and arg z = θ ( 2 − α1 )( 2 − α2 )K( 2 − α n − 1 ) = 2n − 1 Applying C1 → C1 + C2 ,
1
∴ z = e iθ and z = 26 We have, (1 + ω )7 = A + Bω 0 ω2 −ω
z
  We know that 1 + ω + ω = 0 2
∆ = 0 ω − ω2
 1+ z 1+ z
∴ 1 + ω = −ω2 ω2 + 2ω ω − ω2
Now, arg   = arg  
1 + z 1 + 1  ⇒ (− ω2 ) 7 = A + Bω
 z = (ω2 + 2ω )(− ω + ω2 ) = − 3ω2
⇒ − ω14 = A + Bω
= arg (z ) = θ
18 40

n π n π
31 |a + bω + c ω2|2 = (a + bω + c ω2 ) 38 Given, z2 is not unimodular i.e. z2 ≠ 1 + ( 2 )n1 cos 1 − i sin i 1 
 4 4 
(a + b ω + c ω ) 2 z − 2z2
and 1 is unimodular. n π π
= (a + bω + c ω2 )(a + bω2 + c ω ) 2 − z1 z2 + ( 2 )n2 cos 2 − i sin n2 
 4 4 
[Q ω = ω2 and ω 2 = ω] z1 − 2z2
⇒ =1 π π
+ ( 2 )n2 cos 2 + i sin 2 
 n n
= a + b + c 2 − ab – bc − ca
2 2
2 − z1 z2
 4 4 
1 ⇒ z1 − 2z2
2
= 2 − z1 z2
2
= [(a − b ) 2 + (b − c ) 2 + (c − a) 2 ] n1  n1 π  n π
2 = ( 2 ) 2cos + ( 2 )n2 2cos 2 
⇒ (z1 − 2z2 )(z1 − 2z2 ) = (2 − z1 z2 )  4   4 
So, it has minimum value 1 for a = b = 1 2
(2 − z1 z2 ) [Qzz = z ] which is purely real ∀ n1 , n2 .
and c = 2. 2 2
2 2
32 Clearly, x + y + z = x x + y y + zz
2 ⇒ z1
2 2
+ 4 z2 − 2z1 z2 − 2z1 z2 2 Clearly, z = z
2 2
z−1 z −1
= (a + b + c )(a + b + c ) = 4 + z1 z2 − 2z1 z2 − 2z1 z2 ⇒ zzz − z2 = zzz − z2
2
2 2
⇒ ( z2 − 1)( z1 − 4) = 0
2 ⇒ z (z − z ) − (z − z )(z + z ) = 0
+ (a + bω + cω2 )(a + bω + c ω ) 2
2 z2 ≠1 ⇒ (z − z ) ( z − (z + z )) = 0
+ (a + bω + cω )(a + bω + c ω )
2 Q
2 2 2 ∴ z1 =2 Either z = z ⇒ real axis
= 3( a + b + c )
Let z1 = x + iy ⇒ x2 + y 2 = (2)2 or
2
z = z + z ⇒ zz − z − z = 0
2 2 2
x + y + z Point z1 lies on a circle of radius 2.
⇒ 2 2 2
=3 i.e. ( x2 + y 2 = 2 x )
a + b + c 39 Let z = re i θ represents a circle passing through origin.
  Then, |r 2e 2 iθ − 1| = r 2 + 1 3 ω = e 2 πi / 3 = imaginary cube root of unity
33 Given, Re  1  = 3 ⇒ Re  z 2  = 3 ⇒ (r 2 cos 2θ − 1) 2 + (r 2 sin 2θ) 2
 z  |z|  ∴ 1 + ω + ω2 = 0
= (r 2 + 1) 2
 1 z  ⇒ r 4 − 2r 2 cos 2θ + 1 = r 4 + 2 r 2 + 1 z+1 ω ω2
Q z = |z |2  π Now, ∆ = ω z + ω2
  ⇒ cos 2θ = − 1 ⇒ θ =
1
x 2 ω2 1 z+ω
⇒ 2 = 3 ⇒ 3 x2 + 3 y 2 − x = 0
x + y2 π π
⇒ z = r  cos + i sin  = ir z z z
So, it is a circle whose centre is on  2 2
= ω z + ω2 1
X-axis and passes through the origin.
40 Let z = x + iy ω2 1 z+ω
2z + 1 (2 x + 1) + 2iy
34 = Then, x2 + y 2 = 1
iz + 1 (1 − y ) + ix (applying R1 → R1 + R2 + R3 )
and x + iy = 1 − ( x − iy )
[(2 x + 1) + 2iy ] ⋅ [(1 − y ) − ix] 1 1 1 1
= ⇒ x2 + y 2 = 1 and 2 x = 1 ⇒ x = = z ω z + ω2 1
(1 − y )2 − i 2 x2 2
(2 x − y + 1) − (2 x2 + 2 y 2 + x − 2 y )i 3 ω2 1 z+ω
= and y =±
1 + x2 + y 2 − 2 y 2 = z {[(z + ω2 )(z + ω ) − 1]
1 3
∴ Imaginary part ∴ z=
± i + [ω2 − ω( z + ω)] + [ω − ω2 (z + ω2 )]}
− (2 x2 + 2 y 2 + x − 2 y ) 2 2
= = −2 = z {z2 + z(ω + ω2 ) + ω3
1 3
1 + x2 + y 2 − 2 y Now, take, z = + i −1 − ωz − ω2 z} = z3
2 2
⇒ x + 2 y − 2 = 0, which is a straight line.  3 / 2 π ∴ ∆ = 0 ⇒ z = 0 ⇒ z = 0 is the only
3

∴ θ = tan −1   = solution.
z z 1  1/2  3
35 Clearly, = = , which
1 − z2 zz − z2 z − z 4 In the problem, base = 1 / 2∈(0,1)
is always imaginary. SESSION 2 ∴ z−1 < z− i ⇒
2
z−1 < z− i
2

36 |ω | = 1 ⇒|z | = z − i
n1
1 Clearly, (1 + i ) + (1 + i 3 )n1 ⇒ (z − 1)(z − 1) < (z − i )( z + i )
3
+ (1 + i 5 )n2 + (1 + i 7 )n2 [Q z
2
= z z]
It is the perpendicular bisector of the = (1 + i ) + (1 − i ) + (1 + i )n2 + (1 − i )n2
n1 n1

n1 ⇒ (1 + i )z + (1 − i )z > 0
line segment joining (0, 0) to  0,  i.e. π π 
1 
= 2  cos + i sin  ⇒ (z + z ) + i (z − z ) > 0
 3   4 4  
 z + z  z − z
1 n1 ⇒   + i >0
the line y = ⋅   π π 
6 +  2  cos  −  + i sin  −     2   2 
   4  4
⇒  z + z −  z − z > 0
   
37 Given, z1 + z2 = 1 ⇒ z12 + z22 = z1 z2   π π 
n2
 2   2i 
z2 z1 +  2  cos  −  + i sin  −   
   4  4 ⇒ Re(z ) − Im(z ) > 0 ⇒ x − y > 0
⇒ z12 + z22 + z32 = z1 z2 + z1 z3 + z2 z3 ,
5 arg z − (10 + 6i ) = π
n2
  π π 
where z3 = 0 2 cos + i sin 
+
  4 4    z − (4 + 6i )  4
−1 y − 6 y−6 π
So, z1 , z2 and the origin form an
π π
= ( 2 )n1 cos 1 + i sin 1  ⇒ tan − tan −1 =
n n
equilateral triangle. x − 10 x−4
 4 
4 4
[take z = x + iy ]
DAY 19

y−6 y−6 2k π
10
2k π 
− 2z − 6 + 5i = 2 z −  3 − i 
5
13 We have, ∑  sin + i cos 
⇒ x − 10 x − 4 = 1  2   11
k =1 11 
( y − 6)( y − 6) 2kπ 2kπ 
 = i ∑  cos
10
1+ 5 
( x − 10)( x − 4) = 2 Distance of z from  3, −  − i sin 
  2   k =1  11 11 

⇒ x2 + y 2 − 14 x − 18 y + 112 = 0 10 − i 2 kπ 10
where z lies on circle (i).
⇒ ( x − 7)2 + ( y − 9)2 = 18 = (3 2 )2 = i ∑e 11 =i ∑α
k

∴ min 2z − 6 + 5i = 2PA = 2  − 2 = 5


9 k =1 k =1
⇒ z − (7 + 9i ) = 3 2 2 
where α = e − i 2 π /11
iπ / 4 = i [α + α2 + α3 + ... + α10 ]
6 We have, zz = (z + z ) = 350 10 Clearly, 0 − 3e iπ / 4 = 3 e iπ /2
2 2

⇒ 2( x2 + y 2 )( x2 − y 2 ) = 350 z − 3e 4 α(1 − α10 ) (α − α11 )


=i =i
⇒ ( x2 + y 2 )( x2 − y 2 ) = 175 −3e − iπ / 4 3 1−α 1−α
∴ = i
Since x, y ∈ I , the only possible case z − 3e iπ / 4 4 (α − 1)
=i [Qα = cos 2 π − sin 2 π = 1]
11

which gives integral solutions, is N (1 − α )


x2 + y 2 = 25 …(i) P(z)
= −i
x2 − y 2 = 7 …(ii)
From Eqs. (i) and (ii) x2 = 16; y 2 = 9 14 Given, z2 + pz + q = 0
⇒ x = ±4; y = ±3 ⇒ Area = 48 3eiπ/4 ⇒ z1 + z2 = − p and z1 z2 = q
−1 Q OA = OB
7 We have, α + iβ = cot (z )
⇒ z1 = z2
⇒ cot(α + iβ ) = x + iy E z2
and cot(α − iβ ) = x − iy O ∴ = e iα = cos α + i sin α
z1
Now, consider z − 3e iπ / 4 = 4ie iπ / 4
Y
cot 2α = cot [(α + iβ ) + (α − iβ )] z = (3 + 4i )e iπ / 4
cot(α + iβ ) ⋅ cot(α − iβ ) − 1
= 11 Given z n = 1, where B(z2)
cot(α + iβ ) + cot(α − iβ )
z = 1, a1 , a2 ,... an −1 …(i)
( x + y − 1)
2 2
1 1
= Let α = , then z = 1 − A(z1)
2x 1− z α
∴ x2 + y 2 − 2 x cot 2α − 1 = 0  1
n

∴ 1 −  = 1 [by (i)] α
8 Given,|z + 4 | ≤ 3  α
X
Now,|z + 1 | = |z + 4 − 3 | ⇒ (α − 1) − α = 0
n n O
≤ |z + 4 | + | 3 | ≤ 3 + 3 = 6 ⇒ − C1 α n −1 + C2α n −2 + ... + (−1)n = 0 z1 + z2
1 1 1 ⇒ = 1 + cos α + i sin α
Hence, greatest value of|z + 1 | = 6 where, α = , ..., z1
1 − a1 1 − a2 1 − an −1 α α α
Since, least value of the modulus of a = 2cos  cos + i sin 
complex number is 0. 1 1 1 2 2 2
⇒ + + ...+
Consider,|z + 1 | = 0 ⇒ z = − 1 1 − a1 1 − a2 1 − an −1 (z1 + z2 )2 2 α iα
⇒ = 4cos e
Now, |z + 4 | = |− 1 + 4 | = 3 C2 n(n − 1) (n − 1) z12 2
= = =
⇒| z + 4 | ≤ 3 is satisfied by z = − 1. C1 2/ n 2 z
= 4cos 2 α / 2 ⋅ 2
∴ Least value of|z + 1| = 0 z1
2 2
12 z ω − ω z = z − ω ... (i) α
9 z − 3 − 2i ≤ 2 ⇒ (z1 + z2 )2 = 4cos 2 z1 z2
2 2 2
⇒ ( z + 1)ω = ( ω + 1)z
⇒ p2 = 4 q cos 2 α / 2
z +1
2 p2 α
z ∴ = cos 2
⇒ = = real 4q 2
(3, 2) ω 2
ω +1
z zω 15 Q p < 0, take p = − q 3 (q > 0)
⇒ =z ⇒ = zω ... (ii)
ω ω ∴ p1 /3 = q(−1)1 /3 = − q , − qω, − qω2
A
Also, from Eq. (i), zz ω − ωω z = z − ω Now, take α = −q , β = − qω, γ = − qω2
P(3, – 5/2) ⇒ zz ω − ωz ω − z + ω = 0 Then, given expression
⇒ (z ω − 1)(z − ω ) = 0 ⇒ z = ω or zω = 1
x + yω + zω2
⇒ z lies on or inside the circle i.e. zω = 1 = = ω2
1 2
xω + yω2 + z
( x − 3)2 + ( y − 2)2 = 22 = 4 ... (i) ⇒ z = ω or z = = ω/ ω
ω
DAY THREE

Sequence
and Series
Learning & Revision for the Day
u Definition u Geometric Progression (GP) u Sum of Special Series
u Arithmetic Progression (AP) u Geometric Mean (GM) u Summation of Series by the
u Arithmetic Mean (AM) u Arithmetico-Geometric Difference Method
Progression (AGP)

Definition

By a sequence we mean a list of numbers, arranged according to some definite rule.
or
We define a sequence as a function whose domain is the set of natural numbers or some
subsets of type {1, 2, 3, ... k}.

If a1, a2 , a3 ,..., an,.... is a sequence, then the expression a1 + a2 + a3 + ... + an+... is called the
series.

If the terms of a sequence follow a certain pattern, then it is called a progression.

PRED
Arithmetic Progression (AP) MIRROR

It is a sequence in which the difference between any two consecutive terms is always Your Personal Preparation Indicator
same.

An AP can be represented as a , a + d, a + 2 d, a + 3 d, … where, a is the first term, d is the
u No. of Questions in Exercises (x)—
common difference. u No. of Questions Attempted (y)—
u No. of Correct Questions (z)—

The nth term, t n = a + (n − 1)d (Without referring Explanations)

Common difference d = t n − t n−1
Accuracy Level (z / y × 100)—
The nth term from end, t n = l − (n − 1)d, where l is the last term.
u

u Prep Level (z / x × 100)—


n n

Sum of first n terms, S n = [2 a + (n − 1)d ] = [a + l ], where l is the last term.
2 2 In order to expect good rank in JEE,
1 your Accuracy Level should be

If sum of n terms is S n, then nth term is t n = S n − S n−1 , t n = [t n− k + t n+ k ], where k < n above 85 & Prep Level should be
2 above 75.
DAY 21

NOTE • Any three numbers in AP can be taken as a − d , a, a + d. • Three non-zero numbers a, b, c are in GP iff b2 = ac.
• Any four numbers in AP can be taken as • If a, b and c are in AP as well as GP, then a = b = c.
a − 3d , a − d , a + d , a + 3d. • If a > 0 and r > 1 or a < 0 and 0 < r < 1, then the GP will be
an increasing GP.
• Any five numbers in AP can be taken as
a − 2d , a − d , a, a + d , a + 2d. • If a > 0 and 0 < r < 1 or a < 0 and r > 1, then the GP will be a
decreasing GP.
• Three numbers a, b, c are in AP iff 2b = a + c.

Important Results on GP
An Important Result of AP ●
If a1 , a2 , a3 ,K , an is a GP of positive terms, then

In a finite AP, a1 , ..., an, the sum of the terms equidistant log a1 , log a2 ,K , log an is an AP and vice-versa.
from the beginning and end is always same and equal to ●
In a finite GP, a1 , a2 , ..., an, the product of the terms
the sum of first and last term
i.e. a1 + an = ak + an –( k − 1 ), ∀ k = 1, 2, 3,..., n − 1. equidistant from the beginning and the end is always
same and is equal to the product of the first and the last
term.
Arithmetic Mean (AM) i.e. a1 an = ak ⋅ an − ( k − 1 ), ∀ k = 1, 2, 3, ..., n − 1.
a+b

If a, A and b are in AP, then A = is the arithmetic
2
mean of a and b.
Geometric Mean (GM)

If a, A1 , A2 , …, An , b are in AP, then A1, A2,..., An are

If a, G and b are in GP, then G = ab is the geometric mean
the n arithmetic means between a and b.
of a and b.

The n arithmetic means, A1, A2 , ..., An, between a and b are
r (b − a) ●
If a, G1, G2,K , Gn, b are in GP, then G1, G2 ,K , Gn are the n
given by the formula, Ar = a + ∀ r = 1, 2, ... n geometric means between a and b.
n+1

The n GM’s, G1, G2 , ..., Gn, inserted between a and b, are

Sum of n AM’s inserted between a and b is n A i.e. r
 a + b b n + 1
A1 + A2 + A3 + K + An = n   given by the formula, Gr = a   .
 2   a

NOTE

Product of n GM’s, inserted between a and b, is the nth
• The AM of n numbers a1 , a2 , ... , an is given by
power of the single GM between a and b,
( a1 + a2 + a3 + K + an )
AM = i.e. G1 ⋅ G2 ⋅ ... ⋅ Gn = G n = (ab )n/2 .
n

NOTE • If a and b are of opposite signs, then their GM can not exist.
Geometric Progression (GP)
• If A and G are respectively the AM and GM between two

It is a sequence in which the ratio of any two consecutive
terms is always same. numbers a and b, then a, b are given by

A GP can be represented as a, ar , ar 2 , … [ A ± ( A + G)( A − G) ] .
where, a is the first term and r is the common ratio. • If a1 , a2 , a3 ,... , an are positive numbers, then their GM

The nth term, t n = ar n –1 = ( a1 a2 a3 ... an ) 1 / n .
l

The nth term from end, t n′ = n − 1 , where l is the last term.
r Arithmetico-Geometric
 1 – r n
a   , r ≠1 Progression (AGP)

Sum of first n terms, S n =   1 – r  ●
A progression in which every term is a product of a term

 na, r =1 of AP and corresponding term of GP, is known as
a arithmetico-geometric progression.

If| r | < 1, then the sum of infinite GP is S ∞ =
1−r ●
If the series of AGP be a + (a + d)r + (a + 2 d)r 2 + ...
a + {a + (n − 1)d}r n−1 + ..., then
NOTE • Any three numbers in GP can be taken as , a, ar.
r a dr (1 − r n−1 ) {a + (n − 1) d }r n
(i) S n = + − ,r ≠1
a a
• Any four numbers in GP can be taken as 3 , , ar, ar 3 . 1−r (1 − r )2 1−r
r r a dr
a a (ii) S ∞ = + ,| r | < 1
• Any five numbers in GP can be taken as 2 , , a, ar, ar 2 . 1 − r (1 − r )2
r r
22 40

Method to find the Sum of n-terms of ●


Sum of squares of first n natural numbers,
n (n + 1)(2 n + 1)
Arithmetic Geometric Progression 12 + 22 + K + n2 = Σ n 2 =
6
Usually, we do not use the above formula to find the sum of n ●
Sum of cubes of first n natural numbers,
terms. 2
 n (n + 1) 
13 + 23 + 33 + ... + n3 = Σ n3 =
Infact we use the mechanism by which we derived the  2 
formula, shown below:
(i) Sum of first n even natural numbers
Let, S n = a + (a + d) r + (a + 2d) r 2 2 + 4 + 6 + K+ 2 n = n (n + 1)
+ K + (a + (n − 1) d) r n − 1 …(i) (ii) Sum of first n odd natural numbers
Step I Multiply each term by r (Common ratio of GP) and 1 + 3 + 5 + K + (2 n − 1) = n2
obtain a new series
⇒ r S n = ar + (a + d) r 2 + K + Summation of Series by
(a + (n − 2) d) r n −1
+ (a + (n − 1) d) r …(ii)
n
the Difference Method
Step II Subtract the new series from the original series by If nth term of a series cannot be determined by the methods
shifting the terms of new series by one term discussed so far. Then, nth term can be determined by the
⇒ (1 − r ) S n= a + [dr + dr 2 +K + dr n−1 ] − (a + (n − 1) d) r n method of difference, if the difference between successive
terms of series are either in AP or in GP, as shown below:
1 − rn −1
⇒ S n(1 − r ) = a + dr   − (a + (n − 1) d) r n Let T1 + T2 + T3 + ... be a given infinite series.
 1−r  If T2 − T1 , T3 − T2 ,K are in AP or GP, then Tn can be found by
a  1 − r n − 1  (a + (n − 1) d) n following procedure.
⇒ Sn = + dr   − r
1−r  (1 − r )2  1−r Clearly, S n = T1 + T2 + T3 + K + Tn …(i)
Again, S n = T1 + T2 + K + Tn−1 + Tn …(ii)

Sum of Special Series ∴ S n − S n = T1 + (T2 − T1 ) + (T3 − T2 ) + ... + (Tn − Tn−1 ) − Tn


⇒ Tn = T1 + (T2 − T1 ) + (T3 − T2 ) + K + (Tn − Tn−1 )

Sum of first n natural numbers,
n (n + 1) ⇒ Tn = T1 + t 1 + t 2 + t 3 + t n−1
1 + 2 + ... + n = Σ n = n
2 where, t 1, t 2 , t 3 ,K are terms of the new series ⇒ S n = ∑ Tr
r =1

DAY PRACTICE SESSION 1

FOUNDATION QUESTIONS EXERCISE


 7 n −1 1 n+1 n
1 If log3 2, log3 ( 2x − 5) and log3  2 x −  are in AP, then x (a) (b) (c) (d)
 2 a1 an + 1 a1 an + 1 a1 an + 1 a1 an +1

is equal to 5 A man arranges to pay off a debt of ` 3600 by 40 annual


(a) 2 (b) 3 (c) 4 (d) 2, 3 instalments which are in AP. When 30 of the instalments
2 The number of numbers lying between 100 and 500 that are paid, he dies leaving one-third of the debt unpaid.
are divisible by 7 but not by 21 is The value of the 8th instalment is
(a) 57 (b) 19 (c) 38 (d) None of these (a) ` 35 (b) ` 50
(c) ` 65 (d) None of these
3 If 100 times the 100th term of an AP with non-zero
common difference equals the 50 times its 50th term, 6 Let a1, a 2, a 3, ... be an AP, such that
then the 150th term of this AP is a1 + a 2 + ... + a p p3 a
= ; p ≠ q , then 6 is equal to
(a) − 150 (b) 150 times its 50th term a1 + a 2 + a 3 + ... + aq q3 a 21
(c) 150 (d) zero j
JEE Mains 2013
4 If a1, a 2, ..., an + 1 are in AP, then 41 121
(a) (b)
1 1 1 11 1681
+ +K+ is 11 121
a1a 2 a 2a 3 anan + 1 (c) (d)
41 1861
DAY 23

7 A person is to count 4500 currency notes. 19 If | a | < 1 and | b | < 1, then the sum of the series
Let an denotes the number of notes he counts in the n th 1 + (1 + a ) b + (1 + a + a 2 ) b 2 + (1 + a + a 2 + a 3 ) b 3 + K is
minute. If a1 = a 2 = ... = a10 = 150 and a10, a11 ,... are in AP 1 1
(a) (b)
with common difference – 2, then the time taken by him (1 − a) (1 − b) (1 − a) (1 − ab)
to count all notes, is (c)
1
(d)
1
(a) 24 min (b) 34 min (c) 125 min (d) 135 min (1 − b) (1 − ab) (1 − a) (1 − b) (1 − ab)
8 If log 3
a + log(3)1/ 3 a + log31/ 4 a + K upto 8th term
2 2 2
20 A man saves ` 200 in each of the first three months of his
= 44, then the value of a is service. In each of the subsequent months his saving
1 increases by ` 40 more than the saving of immediately
(a) ± 3 (b) 2 2 (c) (d) None of these
2 previous month. His total saving from the start of service
9 n arithmetic means are inserted between 7 and 49 and will be ` 11040 after j AIEEE 2011

their sum is found to be 364, then n is (a) 19 months (b) 20 months


(c) 21 months (d) 18 months
(a) 11 (b) 12 (c) 13 (d) 14
21 If one GM, g and two AM’s, p and q are inserted between
10 If x = 111K1 (20 digits), y = 333K3 (10 digits) and
x −y 2 two numbers a and b, then ( 2p − q )( p − 2 q ) is equal to
z = 222K 2 (10 digits), then is equal to (a) g 2 (b) −g 2 (c) 2g (d) 3 g 2
z
(a) 1 (b) 2 (c) 1 / 2 (d) 3 2
22 If five GM’s are inserted between 486 and , then fourth
3
11 If the 2nd, 5th and 9th terms of a non-constant AP are in
GM will be
GP, then the common ratio of this GP is j JEE Mains 2016
8 4 7 (a) 4 (b) 6 (c) 12 (d) − 6
(a) (b) (c) 1 (d)
5 3 4 23 The sum to 50 terms of the series
12 Three positive numbers form an increasing GP. If the 2
 1  1
1 + 2 1 +  + 3 1 +  + K , is given by
middle term in this GP is doubled,then new numbers are  50  50
in AP. Then, the common ratio of the GP is j JEE Mains 2014
(a) 2500 (b) 2550
(a) 2+ 3 (b) 3 + 2 (c) 2 − 3 (d) 2 + 3 (c) 2450 (d) None of these

13 A GP consists of an even number of terms. If the sum of 24 If (10)9 + 2(11)1(10)8 + 3(11)2(10)7 + ... + 10 (11)9 = k (10)9 ,
all the terms is 5 times the sum of terms occupying odd then k is equal to j
JEE Mains 2014
places,then its common ratio is 121 441
(a) (b) (c) 100 (d) 110
10 100
(a) 2 (b) 3 (c) 4 (d) 5
14 The sum of first 20 terms of the sequence 0.7, 0.77, 25 The sum of the infinity of the series
2 6 10 14
0.777, …, is j
JEE Mains 2013 1 + + 2 + 3 + 4 + ... is
7 7 − 20
3 3 3 3
(a) [179 − 1020 ] (b) [99 − 10 ]
81 9 (a) 3 (b) 4 (c) 6 (d) 2
7 7
(c) [179 + 10 − 20 ] (d) [99 + 10 − 20 ] 26 The sum of the series1 + 3 + 5 + K upto 20 terms is
3 3 3
81 9
(a) 319600 (b) 321760
15 If x , y and z are distinct prime numbers, then
(c) 306000 (d) 347500
(a) x, y and z may be in AP but not in GP
12
(b) x, y and z may be in GP but not in AP
(c) x, y and z can neither be in AP nor in GP
27 Let a1, a 2, a 3, ..., a 49 be in AP such that ∑ a 4k + 1 = 416
k =0
(d) None of the above
and a 9 + a 43 = 66. If a12 + a 22 + K + a17
2
= 140 m, then m is
16 Let n ( > 1) be a positive integer, then the largest integer m
equal to j
JEE Mains 2018
such that (nm + 1) divides (1 + n + n 2 + K + n127 ), is
(a) 66 (b) 68 (c) 34 (d) 33
(a) 32 (b) 8 (c) 64 (d) 16
28 Let a, b, c ∈ R . if f ( x ) = ax + bx + c be such that
2
17 An infinite GP has first term x and sum 5, then x belongs
a + b + c = 3 and f ( x + y ) = f ( x ) + f ( y ) + xy , ∀ x , y ∈ R ,
(a) x < − 10 (b) −10 < x < 0 (c) 0 < x < 10 (d) x > 10 10
18 The length of a side of a square is a metre. A second then ∑ f (n ) is equal to
n =1 j
JEE Mains 2017
square is formed by joining the mid-points of these
squares. Then, a third square is formed by joining the (a) 330 (b) 165 (c) 190 (d) 255
mid-points of the second square and so on. Then, sum of 29 The sum of the series ( 2)2 + 2( 4)2 + 3 ( 6)2 + ... upto 10
the area of the squares which carried upto infinity is terms is j
JEE Mains 2013
2 2 2 2 2 2 2 2
(a) a m (b) 2a m (c) 3 a m (d) 4 a m (a) 11300 (b) 11200 (c) 12100 (d) 12300
24 40

30 Let A be the sum of the first 20 terms and B be the sum 3 5 7


32 The sum + + + ... upto 11 terms is
of the first 40 terms of the series 12 12 + 22 12 + 22 + 32 j JEE Mains 2013

7 11 11 60
12 + 2 ⋅ 22 + 32 + 2 ⋅ 42 + 52 + 2 ⋅ 62 + K (a) (b) (c) (d)
2 4 2 11
If B − 2A = 100λ, then λ is equal to j JEE Mains 2018 1 1
(a) 232 (b) 248 (c) 464 (d) 496 33 The sum of the series1 + + + ...
1+ 2 1+ 2 + 3
31 The sum of first 9 terms of the series upto 10 terms is j JEE Mains 2013
13 13 + 23 13 + 23 + 33 18 22 20 16
+ + +... is (a) (b) (c) (d)
1 1+ 3 1+ 3 + 5 j JEE Mains 2015 11 13 11 9
(a) 71 (b) 96 (c) 142 (d) 192

DAY PRACTICE SESSION 2

PROGRESSIVE QUESTIONS EXERCISE


1 The value of12 + 32 + 52 + ... + 252 is j JEE Mains 2013 8 If Sn is the sum of first n terms of a GP : {an } and Sn′ is the
(a) 2925 (b) 1469 (c) 1728 (d) 1456 sum of another GP : {1 / an }, then Sn equals
2 If the function f satisfies the relation f ( x + y ) = f ( x ) ⋅ f ( y ) Sn′ a1 ′ an ′
(a) (b) a1an Sn′ (c) Sn (d) Sn
for all natural numbers x , y , f (1) = 2 and a1an an a1
n

∑ f (a + r ) = 16 ( 2n −1), then the natural number a is 9 If the sum of the first ten terms of the series
r =1
2 2 2 2
(a) 2 (b) 3 (c) 4 (d) 5  3  2  1  4 16
1  +  2  +  3  + 4 +  4  + ..., is
2
m,
7  5  5  5  5 5
3 If the sum of an infinite GP is and sum of the squares
2 then m is equal to j JEE Mains 2016
147
of its terms is , then the sum of the cubes of its terms (a) 102 (b) 101 (c) 100 (d) 99
16
is 10 If m is the AM of two distinct real numbers I and n (l, n > 1)
315 700 985 1029 and G1, G 2 and G 3 are three geometric means between I
(a) (b) (c) (d)
19 39 13 38 and n, then G14 + 2G 24 + G 34 equals j
JEE Mains 2015
5 55 555 (a) 4l 2mn (b) 4 lm 2n
4 The sum of the infinite series + 2 + + K is
13 13 133 (c) 4 lmn 2 (d) 4l 2m 2n 2
31 65 65 75 11 The sum of the series ( 2 + 1) + 1 + ( 2 − 1) + K ∞ is
(a) (b) (c) (d)
18 32 36 36
4+ 3 2
(a) 2 (b) 2 + 3 2 (c) 2 − 3 2 (d)
5 Given sum of the first n terms of an AP is 2n + 3 n 2. 2
Another AP is formed with the same first term and double
12 The largest term common to the sequences 1, 11, 21, 31,
of the common difference, the sum of n terms of the new
... to 100 terms and 31, 36, 41, 46, ... to 100 terms is
AP is
(a) 531 (b) 471 (c) 281 (d) 521
(a) n + 4n 2 (b) 6n 2 − n (c) n 2 + 4n (d) 3 n + 2n 2
∞ ∞ 13 If a, b, c are in GP and x is the AM between a and b, y the
π
6 For 0 < θ < , if x = ∑ cos 2n θ , y = ∑ sin2n θ and AM between b and c, then
2 n =0 n =0 a c a c
∞ (a) + =1 (b) + =2
x y x y
z = ∑ cos 2n θ sin2n θ, then xyz is equal to (c)
a c
+ =3 (d) None of these
n =0
x y
(a) xz + y (b) x + y + z (c) yz + x (d) x + y − z
14 Suppose a, b and c are in AP and a 2, b 2 and c 2 are in
1 1 1 π4 1 1 1
7 If 4 + 4 + 4 + K to ∞ = , then 4 + 4 + 4 + K to 3
1 2 3 90 1 3 5 GP. If a < b < c and a + b + c = , then the value of a is
2
∞ is equal to 1 1 1 1 1 1
π4 π4 89 π 4 π4 (a) (b) (c) − (d) −
(a) (b) (c) (d) 2 2 2 3 2 3 2 2
96 45 90 90
DAY 25

15 For any three positive real numbers a, b and c, if 17 Statement I The sum of the series
9 ( 25a + b ) + 25 (c − 3ac ) = 15b ( 3a + c ), then
2 2 2 1 + (1 + 2 + 4) + ( 4 + 6 + 9) + ( 9 + 12 + 16) + K
(a) b, c and a are in GP j JEE Mains 2017 + ( 361 + 380 + 400) is 8000.
(b) b, c and a are in AP n
(c) a, b and c are in AP Statement II ∑ [k 3 − (k − 1)3 ] = n 3, for any natural
(d) a, b and c are in GP k =1

16 If S1, S 2, S 3,K are the sum of infinite geometric series number n.


whose first terms are 1, 2 , 3,K and whose common ratios (a) Statement I is true, Statement II is true; Statement II is a
1 1 1 correct explanation for Statement I
, , ,... respectively, then
2 3 4 (b) Statement I is true, Statement II is ture; Statement II is
S12 + S 22 + S 32 + K + S10
2
is equal to not a correct explanation for Statement I
(c) Statement I is true; Statement II is false
(a) 485 (b) 495
(c) 500 (d) 505 (d) Statement I is false; Statement II is true

ANSWERS
SESSION 1 1 (b) 2 (c) 3 (d) 4 (d) 5 (c) 6 (b) 7 (b) 8 (a) 9 (c) 10 (a)
11 (b) 12 (d) 13 (c) 14 (c) 15 (a) 16 (c) 17 (c) 18 (b) 19 (c) 20 (c)
21 (b) 22 (b) 23 (a) 24 (c) 25 (a) 26 (a) 27 (c) 28 (a) 29 (c) 30 (b)
31 (b) 32 (c) 33 (c)

SESSION 2 1 (a) 2 (b) 3 (d) 4 (c) 5 (b) 6 (b) 7 (a) 8 (b) 9 (b) 10 (b)
11 (d) 12 (d) 13 (b) 14 (d) 15 (b) 16 (d) 17 (a)

Hints and Explanations


SESSION 1 The numbers between 100 and 500 4 Let d be the common difference of
that are divisible by 21 are 105,
1 Q2 log 3 (2 − 5) = log 3 2 + log 3  2x − 7 
x given AP and let
 126, 147, …, 483. 1 1 1
2 S = + +K+ . Then,
Let such numbers be m. a1 a2 a2 a3 an an
⇒ 
(2 − 5) = 2  2 − 
x 2 x 7 +1
∴ 483 = 105 + (m − 1) × 21
 2 1  d d d 
⇒ 18 = m − 1 ⇒ m = 19 S =  + +K+ 
⇒ t 2 + 25 − 10 t = 2t − 7 d a a a a a a
∴ Required number  1 2 2 3 n n +1 
[put 2x = t ] = n − m = 57 − 19 = 38  a2 − a1 + a3 − a2 
⇒ t − 12t + 32 = 0
2
 a1 a2 a2 a3 
3 Let a be the first term and d (d ≠ 0) 1
=  
⇒ (t − 8) (t − 4) = 0 be the common difference of a given d  a n +1 − an
⇒ 2x = 8 or 2x = 4 +K+
AP, then  an an + 1 

∴ x = 3 or x = 2 T100 = a + (100 − 1) d = a + 99 d  1 1  1 1 
At, x = 2, log 3 (2x − 5) is not defined.
T 50 = a + (50 − 1) d = a + 49 d   a − a  +  a − a  
=  
1 1 2 2 3
Hence, x = 3 is the only solution.
T150 = a + (150 − 1) d = a + 149 d d   1 1 
2 The numbers between 100 and 500 Now, according to the given condition,  +K+  − 
a an + 1  
that are divisible by 7 are 105, 112,   n 
100 × T100 = 50 × T 50
119, 126, …, 490, 497. 1 1  1  1  a n +1 − a1
⇒ 100(a + 99 d ) = 50(a + 49 d ) =  − =  
Let such numbers be n. d  a1 an + 1  d  a1 an + 1 
∴ t n = an + (n − 1) d ⇒ 2(a + 99 d ) = (a + 49 d )
⇒ 497 = 105 + (n − 1) × 7 ⇒ 2a + 198 d = a + 49 d 1  a + nd − a1  n
=  1 =
⇒ n − 1 = 56 ⇒ a + 149 d = 0 d  a1 an + 1  a 1 an +1
⇒ n = 57
∴ T150 = 0
26 40

5 Given, 3600 = 40 [ 2a + (40 − 1) d ]  1 1


+ +
1  Given, sum of all terms = 5 × sum of
2 2  1 log 3
1 1  terms occupying odd places, i.e.
8 S n = log a  log 3 log 3
⇒ 3600 = 20 (2a + 39d )

2 3 4
 a + ar + ar 2 + ... + a r 2 n −1
⇒ 180 = 2a + 39d …(i)  + K upto 8th term 
= 5 × (a + a r 2 + a r 4 + ... + a r 2 n −2 )
After 30 instalments one-third of the log a2
⇒ [2 + 3 + 4 + K + 9] = 44 a (r 2 n − 1) 5a[(r 2 )n − 1]
debt is unpaid. log 3 ⇒ =
3600 r −1 r2 − 1
Hence, = 1200 is unpaid and 2400 [given]
3 ⇒ 44 log a2 = 44 log 3  a(r n − 1)
is paid. QS n = r − 1 
30 ∴ a= ± 3  
Now, 2400 = {2a + (30 − 1) d }
2 9 We know that, r2n − 1 5(r 2 n − 1)
⇒ =
∴ 160 = 2a + 29d …(ii) A1 + A2 + K + A n = nA, where r −1 (r − 1)(r + 1)
On solving Eqs. (i) and (ii), we get a+ b
A= 5
a = 51, d = 2 ⇒ 1= ⇒ r + 1 = 5⇒ r = 4
2 r+1
Now, the value of 8th instalment 7 + 49 
= a + (8 − 1) d ∴ 364 =   n
 2  14 Let S = 0.7 + 0.77 + 0.777 + … upto 20
= 51 + 7 ⋅ 2 = ` 65
364 × 2 terms
a1 + a2 + K + a p ⇒ n= = 13
p3 56 7 77 777
6 Given that, = = + + + … upto 20 terms
a1 + a2 + K + a q q3 10 102 103
10 Given, x = 1 (999K 9) = 1 (1020 − 1)
p
[ 2a1 + ( p − 1 )d ] 9 9  1 + 11 + 111 
2 p3 1 1 = 7  10 102 103 
⇒ = 3 y = (999K 9) = (1010 − 1)
q q 3 3  + … upto 20 terms 
[ 2a2 + (q − 1 )d ] 
2 2 2
and z = (999K 9) = (1010 − 1)  9 + 99 + 999 
where, d is a common difference of an 7
9 9 =  10 100 1000 
9
+ … upto 20 terms 
AP.
x − y 2 1020 − 1 − (1010 − 1)2 
2a 1 + ( p − 1)d p2 ∴ =
⇒ = 2 z 2(1010 − 1)  1  1   1 
2a 2 + (q − 1)d q 7 1 −  + 1 − 2  + 1 − 3  
1010 + 1 − (1010 − 1) =  10   10   10 
d = =1 9 
a1 + ( p − 1) 2 + … upto 20 terms
2 = p
2
 

d q 2
11 Let a be the first term and d be the 7
a2 + (q − 1) = (1 + 1 + … upto 20 terms)
2 common difference. 9 
On putting p = 11 and q = 41 , we get Then, we have a + d , a + 4d , a + 8 d in  1 + 1 + 1 
a1 + (11 − 1)
d GP, −  10 102 103 
2 = (11)
2
i.e. (a + 4d )2 = (a + d ) (a + 8d )  
 + … upto 20 terms  
a2 + (41 − 1)
d (41)2 ⇒ a2 + 16d 2 + 8ad = a2 + 8ad
2  1   1  
20

a1 + 5d 121 + ad + 8d 2  1 −    
⇒ = 7 10   10 
⇒ 8d 2 = ad = 20 − 
a2 + 20d 1681  1 
9 1−
a6 121 ⇒ 8d = a [Qd ≠ 0]  
⇒ =  10 
a21 1681
Now, common ratio,  
a + 4d 8d + 4d 12d 4
r = = = =  Qsum of n terms of GP, 
7 Number of notes that the person counts a+ d 8d + d 9d 3  a(1 − r n ) 
in 10 min  Sn = , where r < 1
12 Let a, ar , ar 2 be in GP ( where, r > 1).  1− r 
= 10 × 150 = 1500
7 1  1  
20
On multiplying middle term by 2, we
= 20 − 1 −    
Since, a10, a11 , a12 ,... are in AP with get that a, 2ar , ar 2 are in AP. 9 9  10  
common difference − 2.  
⇒ 4 ar = a + ar 2 ⇒ r 2 − 4r + 1 = 0
7  179 1  1  
20
7
Let n be the time taken to count
4 ± 16 − 4 =  +   = [179 + 10−20]
remaining 3000 notes. ⇒ r = = 2± 3 9 9 9  10   81
n 2
Then, [2 × 148 + (n − 1) × −2] = 3000
2 ∴ r = 2 + 3 [QAP is increasing] 15 x, y , z are in GP
⇒ n2 − 149 n + 3000 = 0 ⇔ y 2 = xz
2 3
13 Let the GP be a, a r , a r , a r , ⇔ x is factor of y. Which is not
⇒ (n − 24)(n − 125) = 0 ar 2 n −2 , ar 2 n −1 . possible, as y is a prime number.
∴ n = 24 and 125 If x = 3, y = 5and z = 7, then they are in
where, a, ar 2 , ar 4 , ar 6, ... occupy odd
Then, the total time taken by the person AP.
places and ar , ar 3 , ar 5, ar 7 ,... occupy Thus, x, y and z may be in AP but not
to count all notes
= 10 + 24 even places. in GP.
= 34 min
DAY 27

− 1
⇒ S 50 
16 Clearly, n
⇒ 600 + {40(12 + n − 1)} = 11040  = − 50 + 50 x − 50 x
50 50

n128 − 1 2  50 
1+ n+ n + K+ n 2 127
=
n−1 ⇒ 600 + 20 n (n + 11) = 11040 ⇒ S 50 = 2500.
 a (r n − 1) ⇒ 30 + n2 + 11 n = 552 24 Given, k ⋅ 109 = 109 + 2(11)1 (10)8
QS n = r − 1 
  ⇒ n + 11 n − 522 = 0
2 + 3(11)2 (10)7 + ... + 10(11)9
(n 64
− 1) (n64
+ 1) 2
k = 1 + 2  + 3  
= 11 11
⇒ n2 + 29 n − 18 n − 522 = 0
n−1  10   10 
= (1 + n + n2 + K + n 63 ) (n 64 + 1) ⇒ n (n + 29) − 18(n + 29) = 0 9
+ ... + 10 
11
⇒ (n − 18)(n + 29) = 0 ...(i)
Thus, the largest value of m for which  10 
n m + 1 divides ∴ n = 18 2
[neglecting n = − 29 ]  11  k = 1  11  + 2  11 
1 + n + n2 + K + n127 is 64.      
∴ Total time = (n + 3) = 21 months  10   10   10 
5− x
17 Since, S ∞ = x = 5 ⇒ r = 9 10
1− r 5 21 Since, g = ab . Also, a, p , q and b are + ... + 9   + 10  
11 11
...(ii)
For infinite GP,|r | < 1 in AP.  10   10 
5− x b −a
⇒ −1 < < 1 ⇒ −10 < − x < 0 So, common difference d is . On subtracting Eq.(ii) from Eq.(i),
5 3 we get
∴ 0 < x < 10 b − a 2a + b 2
∴ p = a+ d = a+ =
k  1 −  = 1 +
11 11  11 
3 3 + 
18 Sum of the area of the squares which  10  10  10 
b − a a + 2b
carried upto infinity q =b −d =b − = 9 10
+ ... + 9   − 10  
3 3 11 11
a2 a2
= a2 + + + ... Now, (2 p − q )( p − 2q )  10   10 
2 4
2
(4a + 2b − a − 2b ) (2a + b − 2a − 4b )  11 10 
1    − 1
a = ⋅
= = 2a2 m2
1 3 3   10
1− ⇒ k 
10 − 11   10  −10  11 
2 =  
= − ab = − g 2  10   11 − 1  10 
 
19 Clearly, 1 + (1 + a)b + (1 + a + a )b 2 2  10 
+ (1 + a + a2 + a3 ) b 3 + K ∞ 22 Here, a = 486 and b = 2  a( r n − 1) 
3
∞ Q in GP,sum of n terms = r − 1 ,
∑ (1 + a + a2 + K + an − 1 ) b n − 1
r
=  
We know that, G r = a  
b
 when r > 1
n +1
n =1 
 a
∞  1 − an  n − 1  10
11 
10
= ∑ 
 1− a
b
∴ G 4 = 486  ⋅
2 1 
4/6
[Q here, n = 5]
⇒ − k = 10 10   − 10 − 10   
11
 10   10 
n =1 
 3 486   
1  ∞ ∞  ∴ k = 100
∑ b − ∑ an b n − 1 
n −1 4/6
=
= 486 
1 
1 − a n = 1 
n =1   3 ⋅ 243  25 Let S = 1 + 2 + 62 + 10 14
+ 4 +K
1  ∞ ∞  4/6 3 3 33 3
= 486 
1 
∑ b − a ∑ (ab )
n −1 n −1 1
=   = 486 ⋅ =6 2
1 + + 2 + 3 + K
3 5 7
1 − a n = 1 n =1   729  34 =1+
3  3 3 3 
1 a
= [1 + b + b + K ∞] −
2
23 Let x = 1 + 1 and S 50 be the sum of 2 1 2⋅1 / 3 
1− a 1− a =1+ +
3  1 − 1 / 3 (1 − 1 / 3)2 

50
[1 + ab + (ab )2 + K ] first 50 terms of the given series.
1 1 a 1 Then, S 50 = 1 + 2 x + 3 x2 Qsum of infinite AGP, is 
= ⋅ − ⋅ 
1 − a 1 − b 1 − a 1 − ab + ... + 50 x 49 …(i) a dr 
 S∞ = + 
[Q|b | < 1 and |ab | = |a||b | < 1]  1− r (1 − r )2 
⇒ x S 50 = x + 2 x2
1 − ab − a (1 − b ) 2 3 2 9  2 3
= + K + 49 x 49 + 50 x 50 …(ii) =1+ + ⋅ = 1 + ⋅2⋅ = 3
(1 − a) (1 − b ) (1 − ab ) 3  2 3 4  3 2
1 − ab − a + ab ⇒ (1 − x )S 50 = 1 + x + x2 + x3
= 26 13 + 33 + K + 393 = 13 + 23 + 33
(1 − a) (1 − b ) (1 − ab ) + K + x 49 − 50 x 50
1 [subtracting Eq. (ii) from Eq. (i)] + K + 403 – (23 + 43 + 63 + ... + 403 )
= 2
(1 − b ) (1 − ab ) 1 − x 50 40 × 41 
⇒ S 50 (1 − x ) = − 50 x 50 =   − 8(1 + 2
3 3
1− x  2 
20 Let the time taken to save
` 11040 be (n+3) months.  − 1 1 − x
50
+ 33 + K + 203 )
⇒ S 50   = − 50 x 50
For first 3 months he saves ` 200 each  50   − 1 20 × 21 
2
 
 50  = (20 × 41)2 − 8  
month.  2 
n Q x = 1 + 1  = 202 [412 − 2(21)2 ]
In (n+3) months, 3 × 200 + {2(240) 
2 50 
= 319600
+ (n − 1) × 40} = 11040
28 40

27 Let a1 = a and d = common difference 30 We have, 2n + 1


32 T n =
Q a1 + a5 + a9 + L + a49 = 416 12 + 2 ⋅ 22 + 32 + 2 ⋅ 42 + 52 + 2 ⋅ 62 + … (12 + 22 + ... + n2 )
∴ a + (a + 4d ) + (a + 8d ) A = sum of first 20 terms 2n + 1 6
= =
+ … (a + 48d ) = 416 B = sum of first 40 terms n (n + 1 ) (2n + 1 ) n (n + 1 )


13
(2a + 48d ) = 416 ∴ A = 12 + 2 ⋅ 22 + 32 + 2 ⋅ 42 + 52 6
2 1 1 
+ 2 ⋅ 62 + … + 2 ⋅ 202 = 6 − 
⇒ a + 24d = 32 …(i)  n ( n + 1 )
A = (1 + 2 + 3 + … + 202 ) + (22 + 42
2 2 2
Also, we have a9 + a43 = 66  1 1
T1 = 6  −  , T2 = 6  −  , …
1 1
∴ a + 8d + a + 42d = 66 + 62 + … + 202 )
 1 2  2 3 
⇒ 2a + 50d = 66 A = (12 + 22 + 32 + … + 202 )
T11 = 6 
1 1
⇒ a + 25d = 33 …(ii) + 4(12 + 22 + 32 + …+102 ) −
Solving Eqs. (i) and (ii), we get  11 12 
20 × 21 × 41 4 × 10 × 11 × 21 1
a = 8 and d = 1 A= + 1  6 × 11 11
6 6 ∴S = 6  −  = 12 = 2
Now, a12 + a22 + a32 +L+ 2
a17 = 140m  1 12 
20 × 21 20 × 21 × 63
A= (41 + 22) =
8 + 9 + 10 + … + 24 = 140m
2 2 2 2 6 6 33 n th term of the series is
⇒ (12 + 22 + 32 + … + 242 ) − (12 + 22 Similarly, 1 2
Tn = =
B = (12 + 22 + 32 + … + 402 ) + 4(12 n (n + 1 ) n (n + 1 )
+ 32 + … + 72 ) = 140m
+ 22 +… + 202 ) 2
24 × 25 × 49 7 × 8 × 15
⇒ − = 140m 1 1 
6 6 40 × 41 × 81 4 × 20 × 21 × 41 ⇒ Tn = 2 − 
B = +  n n + 1
3×7× 8× 5 6 6
⇒ (7 × 5 − 1) = 140m  1 1
40 × 41 40 × 41 × 123 ⇒ T1 = 2  −  , T2 = 2  −  ,
6 1 1
B = (81 + 42) =
⇒ 7 × 4 × 5 × 34 = 140m 6 6  1 2  2 3
⇒ 140 × 34 = 140m Now, B − 2 A = 100λ
T3 = 2  −  , ..., T10 = 2 
1 1 1 1
− 
⇒ m = 34 40 × 41 × 123  3 4  10 11 

28 We have, f ( x ) = ax2 + bx + c 6 ∴ S 10 = T1 + T2 + K + T20
2 × 20 × 21 × 63
Now, f ( x + y ) = f ( x ) + f ( y ) + xy − = 100λ 1 − 1 + 1 − 1 + 1 − 1 
6 
Put y = 0 ⇒ f ( x ) = f ( x ) + f (0) + 0 40 = 2 2 2 3 3 4 
⇒ f (0) = 0
⇒ (5043 − 1323) = 100λ 1 1
6  +K+ − 
⇒ c =0 40  10 11 
⇒ × 3720 = 100λ
= 2  1 −
Again, put y = − x 6 1

∴ f (0) = f ( x ) + f (− x ) − x2 ⇒ 40 × 620 = 100λ  11 
40 × 620 10 20
⇒ 0 = ax2 + bx + ax2 − bx − x2 ⇒ λ= = 248 = 2⋅ =
100 11 11
1
⇒ 2ax2 − x2 = 0 ⇒ a =
2 31 Write the nth term of the given series
Also, a + b + c = 3
SESSION 2
and simplify it to get its lowest

1
+ b + 0 = 3⇒ b =
5 form.Then, apply, S n = ΣT n . 1 Let S = 12 + 32 + 52 + K + 252
2 2 Given series is = (12 + 22 + 32 + 42 + K + 252 )
x2 + 5x 13 13 + 23 13 + 23 + 33 − (22 + 42 + 62 + K + 242 )
∴ f ( x) = + + + K∞
2 1 1+ 3 1+ 3+ 5 = (1 + 2 + 3 + 42 + K + 252 )
2 2 2
n2 + 5n 1 2 5
Now, f (n ) = = n + n Let T n be the nth term of the given − 22 (1 + 22 + 32 + K + 122 )
2 2 2 series.
10
1 10 2 5 10 25(25 + 1) (2 × 25 + 1)
=
∴ ∑ f (n ) = 2 ∑ n + 2 ∑ n ∴ Tn =
13 + 23 + 33 + ... + n3 6
n =1 n =1 n =1 12 (12 + 1) (2 × 12 + 1)
1 + 3 + 5 + K upto n terms − 4×
1 10 × 11 × 21 5 10 × 11
⋅= + × 2 6
2 6 2 2  n(n + 1) 25 × 26 × 51 4 × 12 × 13 × 25
  = −
 = (n + 1)
2
=
385 275 660 2
= + = = 330 6 6
2
2 2 2 n 4 = 25 × 13 × 17 − 4 × 2 × 13 × 25
29 Series (2)2 + 2(4)2 + 3(6)2 + K 9
(n + 1)2 1 = 5525 − 2600 = 2925
Now, S 9 = ∑ =
= 4 {1 ⋅ 12 + 2 ⋅ 22 + 3 ⋅ 32 + K } n =1 4 4 2 Now, f (2) = f (1 + 1)
∴ Tn = 4n ⋅ n 2 [(22 + 32 + ... + 102 ) + 12 − 12 ] = f (1) ⋅ f (1) = 22 and f (3) = 23
1  10(10 + 1)(20 + 1) Similarly, f (n ) = 2n
− 1
2
n(n + 1)
and S n = ΣT n = 4Σ n3 = 4  =
 2  4  6  n n
∑ f (a + ∑2
a+r
∴ 16(2n − 1) = r )=
Now, S 10 = [10 ⋅ (10 + 1)] 2 384 r =1 r =1
= = 96
4 = 2 (2 + 22 + K + 2n )
a
= (110)2 = 12100
DAY 29

 2n − 1  1 2 2 2 2
= 1 − cos 2 θ sin2 θ =   +   +   + 42 +  
and 8 12 16 24
= 2a ⋅ 2   [GP series]
 2−1 z  5  5  5  5
1 xy − 1
= 2a + 1 (2n − 1) =1− = + ... to 10 terms
xy xy 1
⇒ 2a + 1 = 16 = 24 = (8 + 12 + 16 + 202 + 242
2 2 2
⇒ x y = x yz − z 52
∴ a= 3 ∴ x yz = x y + z = x + y + z + ... to 10 terms)
42
7 Let S = 14 + 14 + 14 + K + to ∞ = (2 + 3 + 4 + 5
2 2 2 2
3 Let GP be a, ar , ar ,K,|r | < 1.
2
1 3 5 52
According to the question, + ... to 10 terms)
1 1 1 π4
a 7 a2 147 Since, 4 + 4 + 4 + K to ∞ = 42
= , = 1 2 3 90 = (22 + 32 + 42 + 52 + ... + 112 )
1 − r 2 1 − r2 16 52
 1 1 1
∴  4 + 4 + 4 + K to ∞   16 2
On eliminating a, we get 1 3 5  = ((1 + 22 + ... + 112 ) − 12 )
2 25
(1 − r 2 ) =   (1 − r ) 2 π4
147 7
+  4 + 4 + 4 + K to ∞  = 16  11 ⋅ (11 + 1) (2 ⋅ 11 + 1)
1 1 1
16  2 2  =  − 1
4 6 90 25  6 
1  1 1 1 
⇒ 3(1 + r ) = 4 (1 − r ) ⇒ r = ,a = 3 ⇒  4 + 4 + 4 + K to ∞  =
16
(506 − 1) =
16
× 505
7 1 3 5  25 25
∴ Sum of cubes 1  1 π4 16 16
+ 4 + 4 + K + to ∞  =
1 1 ⇒ m= × 505
a3 (3)3 1029 +4 
= = = 2 1 4
2 3  90 5 25
1 − r3 3
38 ⇒ m = 101
1 −  
1 1 π4 π4
 7 ⇒S + ⋅ =
16 90 90 10 Given, m is the AM of l and n.
 1 π4 
4 Let S = 5 + 552 + 555 +… …(i) 1 1
Q 4 + 4 + 4 + K to ∞ =  ∴ l + n = 2m
13 13 133  1 2 3 90 
and G1 , G2 , G3 are geometric means
S 5 55
and = + 3 + … ...(ii) π4  1 15 π 4 π4 between l and n.
13 132 13 ⇒ S = 1 −  = =
90  16  16 × 90 96 So, l ,G1 ,G2 ,G3 , n are in GP.
On subtracting Eq. (ii) from Eq. (i), we
get n −1 Let r be the common ratio of this GP.
8 Let an = a r .
12 5 50 500
S = + + +… a (1 − r n ) ∴ G1 = lr , G2 = lr 2 , G3 = lr 3 ;
13 13 132 133 Then, S n = 1/4
1− r
n = lr 4 ⇒ r =  
10 n
which is a GP with common ratio . l
 1  1 −  1 
n
13
      Now, G14 + 2G24 + G34 = (lr )4
13  5  10   65  a r
∴ S = × ÷ 1 −  = and S n′ =  
12  13  13   36 1 + 2(lr 2 )4 + (lr 3 )4
1−
 a  r = l 4 × r 4 (1 + 2r 4 + r 8 ) = l 4 × r 4 ( r 4 + 1)2
QS ∞ = 1 − r   1 
  1 nn+ l
2
Q first term of  a  is a  = l4 ×  = ln × 4 m = 4 l m n
2 2

5 Here, T1 = S1 = 2(1 ) + 3(1 )2 = 5   n  l  l 
 1 [Q n + l = 2m]
T2 = S 2 − S 1 = 16 − 5 = 11 
and common ratio is
r 
[QS 2 = 2(2) + 3(2) = 16] 2 11 Given series is a geometric series with
 1  (r n − 1)
T3 = S 3 − S 2 = 33 − 16 = 17   a = 2 + 1 and r = 2 − 1.
 a
= ⋅r ∴ Required sum
[Q S 3 = 2(3) + 3 (3)2 = 33 ] r n (r − 1)
a 2+1 2+1
Hence, sequence is 5, 11, 17. 1− r 1 n = = =
∴ a = 5and d = 6 = ⋅ 1 − r 1 − ( 2 − 1) 2 − 2
1 − r a⋅ r n − 1
For new AP, A = 5, D = 2 × 6 = 12 ( 2 + 1) (2 + 2)
1 − rn 1 a (1 − r n ) 1 =
n = ⋅ = ⋅ (2 − 2 ) (2 + 2)
∴ S ′ n = [2 × 5 + (n − 1 )12] 1 − r an 1− r a an
2 2 2+ 2+ 2+ 2 4+ 3 2
1 = =
= 6 n2 − n = Sn ⋅ 4−2 2
a1 an
6 Sum of three infinite GP’s are
1 1 ⇒ S n = a1 an S n′ 12 Clearly, the common terms of the given
x= = , sequences are
1 − cos 2 θ sin2 θ 9 Let S10 be the sum of first ten terms of 31, 41, 51, ...
Similarly, the series. Now, 100th term of 1, 11, 21, 31, ... is
1 1 Then, we have 1 + 99 × 10 = 991
y = and z = 2 2 2 and 100th term of 31, 36, 41, 46, ... is
cos 2 θ 1 − cos 2 θ sin2 θ S 10 =  1  +  2  +  3 
3 2 1
31 + 99 × 5 = 526 .
1 1  5  5  5
Now, + =1 [Q sin2 θ + cos 2 θ = 1] 2
Let the largest common term be 526.
x y
+ 42 +  4  + ... to 10 terms Then, 526 = 31 + (n − 1) 10
4
⇒ x + y = xy  5 ⇒ (n − 1) 10 = 495
30 40

⇒ n − 1 = 49.5 In this case, 17 Statement I


⇒ n = 50.5 (a − c )2 = (a + c )2 − 4ac = 0 Let S=(1)+(1+2+4)+(4+6+9)
But n is an integer, n = 50. ⇒ a=c +(9+12+16)+... +(361+380+400)
Hence, the largest common term is But a ≠ c , as a < c . = (0+0+1)+(1+2+4)+(4+6+9)
31 + (50 − 1) 10 = 521 . 1
Case II When a + c = 1 and ac = − +(9+12+16) +... +(361+380+400)
13 Since, a, b, c are in GP. 4
Now, we can clearly observe the
∴ b 2 = ac …(i) In this case, (a − c )2 = 1 + 1 = 2
elements in each bracket.
Also, as x is A between a and b ⇒ a−c = ± 2 The general term of the series is
a+ b But a < c , a − c = − 2 T r = (r − 1)2 + (r − 1) r + (r 2 )
∴ x= …(ii)
2 On solving a + c = 1 Now, the sum to n terms of the series is
b +c and a − c = − 2, we get
Similarly, y = …(iii) n
Sn = ∑ [(r − 1) + (r − 1)r + (r )2 ]
2
2 1 1
a c 2a 2c a= − . r =1
Now, consider + = + 2 2
x y a+ b b + c n  r 3 − (r − 1)3 
15 We have, 225a2 + 9b2 + 25c 2 = ∑ 
[using Eqs. (ii) and (iii)] r =1  r − ( r − 1) 
− 75ac − 45ab − 15bc = 0
 ab + ac + ac + bc  [Q(a3 − b 3 ) = (a − b )(a2 + ab + b 2 )]
=2 ⇒ (15a)2 + (3b )2 + (5c )2

 ab + ac + b + bc 
2
− (15a)(5c ) − (15a)(3b ) − (3b )(5c ) = 0 n
= ∑ [r − (r − 1)3 ]
3
 ab + bc + 2ac  1
=2 ⇒ [(15a − 3b )2 + (3b − 5 c )2 r =1

 ab + bc + 2ac  2 + (5c − 15a)2 ] = 0 = (13 − 03 ) + (23 − 13 ) + (33 − 23 )
[using Eq. (i)] ⇒ 15a = 3b , 3b = 5 c and 5 c = 15a
=2 + ... + [n3 − (n − 1)3 ]
∴ 15a = 3b = 5c
14 Since, a, b, c are in AP a b c Rearranging the terms, we get
⇒ = = = λ (say)
∴ 2b = a + c …(i) 1 5 3 S n = − 03 + (13 − 13 ) + (23 − 23 )
Also, as a2 , b 2 and c 2 are in GP ⇒ a = λ, b = 5λ, c = 3λ + (33 − 33 ) + ... + [(n − 1)3 − ( n − 1)3 ] + n3
∴ b 4 = a2c 2 …(ii)
Hence, b, c and a are in AP. = n3
3 16 Here, S r is sum of an infinite GP, r is ⇒ S 20 = 8000
Q a+ b + c =
2 1 Hence, Statement I is correct.
first term and is common ratio
3 r +1
∴ 3b = [using Eq. (i)] Statement II We have, already proved
2 r
Sr = =r +1 in the Statement I that
1 1
⇒ b = 1− n
Sn = ∑ (r − (r − 1)3 ) = n3
3
2 r +1
r =1
⇒ a+ c =1 [using Eq. (i)] 10
⇒ ∑ Sr = 22 + 32 + K + 112
2
1 1 Hence, Statement II is also correct and
and ac = or − [using Eq. (ii)] r =1
4 4 = 12 + 22 + 32 + K + 112 − 1 it is a correct explanation for
1 11 × 12 × 23 Statement I.
Case I When a + c = 1 and ac = = − 1 = 505
4 6
DAY FOUR

Quadratic
Equation and
Inequalities
Learning & Revision for the Day
u Quadratic Equation u Maximum and Minimum u Arithmetic-Geometric-
u Relation between Roots and Value of ax 2 + bx + c Harmonic Mean
Coefficients u Sign of Quadratic Expression Inequality
u Formation of an Equation u Position of Roots u Logarithm Inequality
u Transformation of Equations u Inequalities

Quadratic Equation

An equation of the form ax2 + bx + c = 0, where a ≠ 0, a, b and c, x ∈ R, is called a real
quadratic equation. Here a, b and c are called the coefficients of the equation. PRED

The quantity D = b – 4ac is known as the discriminant of the equation ax + bx + c = 0
2

−b ± D
2
MIRROR
Your Personal Preparation Indicator
and its roots are given by x =
2a
u No. of Questions in Exercises (x)—

An equation of the form az2 + bz + c = 0, where a ≠ 0, a, b and c, z ∈C (complex) is called a
No. of Questions Attempted (y)—
−b ± D
u

complex quadratic equation and its roots are given by z = . u No. of Correct Questions (z)—
2a (Without referring Explanations)

Nature of Roots of Quadratic Equation u Accuracy Level (z / y × 100)—


u Prep Level (z / x × 100)—
Let a, b , c ∈ R and a ≠ 0, then the equation ax + bx + c = 0
2

(i) has real and distinct roots if and only if D > 0. In order to expect good rank in JEE,
your Accuracy Level should be
(ii) has real and equal roots if and only if D = 0. above 85 & Prep Level should be
above 75.
(iii) has complex roots with non-zero imaginary parts if and only if D < 0.
32 40

Some Important Results Formation of an Equation


(i) If p + iq (where, p, q ∈ R, q ≠ 0) is one root of
ax2 + bx + c = 0, then second root will be p − iq Quadratic Equation
(ii) If a, b , c ∈ Q and p + q is an irrational root of If the roots of a quadratic equation are α and β, then the
equation will be of the form x2 − (α + β) x + αβ = 0.
ax + bx + c = 0, then other root will be p − q .
2

(iii) If a, b , c ∈Q and D is a perfect square, Cubic Equation


then ax2 + bx + c = 0 has rational roots. If α , β and γ are the roots of the cubic equation, then the
equation will be form of
(iv) If a = 1, b , c ∈ I and roots of ax2 + bx + c = 0 are
x3 − (α + β + γ) x2 + (αβ + βγ + γα ) x − αβγ = 0.
rational numbers, then these roots must be integers.
(v) If the roots of ax2 + bx + c = 0 are both positive, then
the signs of a and c should be like and opposite to the Transformation of Equations
sign of b. Let the given equation be
(vi) If the roots of ax2 + bx + c = 0 are both negative, then a0 x n + a1 x n −1 + ... + an −1 x + an = 0 …(A)
signs of a, b and c should be like. Then, the equation
(vii) If the roots of ax2 + bx + c = 0 are reciprocal to each (i) whose roots are k (≠ 0) times roots of the Eq. (A),
other, then c = a. x
is obtained by replacing x by in Eq. (A).
(viii) In the equation ax2 + bx + c = 0 (a, b , c ∈ R), k
(ii) whose roots are the negatives of the roots of Eq. (A),
if a + b + c = 0, then the roots are 1, c and if is obtained by replacing x by − x in Eq. (A).
a
c (iii) whose roots are k more than the roots of Eq. (A),
a − b + c = 0, then the roots are −1 and − . is obtained by replacing x by ( x − k ) in Eq. (A).
a
(iv) whose roots are reciprocals of the roots of Eq. (A),
is obtained by replacing x by 1/x in Eq. (A) and then
Relation between Roots multiply both the sides by x n .
and Coefficients
Maximum and Minimum
Quadratic Roots
If α and β are the roots of quadratic equation ax2 + bx + c = 0;
Value of ax2 + bx + c
b (i) When a > 0, then minimum value of ax2 + bx + c is
a ≠ 0, then sum of roots = α + β = −
a −D 4ac − b 2 −b
c or at x =
and product of roots = αβ = . 4a 4a 2a
a
And, also ax2 + bx + c = a( x − α )( x − β) (ii) When a < 0, then maximum value of ax2 + bx + c is
−D 4ac − b 2 −b
Cubic Roots or at x =
4a 4a 2a
If α , β and γ are the roots of cubic equation
b
ax3 + bx2 + cx + d = 0; a ≠ 0, then α + β + γ = −
c
a Sign of Quadratic Expression
βγ + γα + αβ = Let f ( x) = ax2 + bx + c, where a, b and c ∈ R and a ≠ 0.
a
d (i) If a > 0 and D < 0, then f ( x) > 0, ∀x ∈ R.
and αβγ = −
a (ii) If a < 0 and D < 0, then f ( x) < 0, ∀x ∈ R.

Common Roots (Conditions) (iii) If a > 0 and D = 0, then f ( x) ≥ 0, ∀x ∈ R.


(iv) If a < 0 and D = 0, then f ( x) ≤ 0, ∀x ∈ R.
Suppose that the quadratic equations are ax + bx + c = 0 and
2

a′ x2 + b ′ x + c′ = 0. (v) If a > 0, D > 0 and f ( x) = 0 have two real roots α and


β, where (α < β), then f ( x) > 0, ∀ x ∈(− ∞, α ) ∪ (β, ∞) and
(i) When one root is common, then the condition is
f ( x) < 0, ∀ x ∈(α , β).
(a′ c − ac′ )2 = (bc′ − b ′ c)(ab ′ − a′ b ).
(vi) If a < 0, D > 0 and f ( x) = 0 have two real roots α and β,
(ii) When both roots are common, then the condition is
a b c where (α < β), then f ( x) < 0, ∀ x ∈ (−∞, α ) ∪ (β, ∞) and
= = . f ( x) > 0, ∀ x ∈(α , β).
a′ b ′ c′
DAY 33

Position of Roots Arithmetic-Geometric-


Let ax + bx + c = 0 has roots α and β. Then, we have the
2
Harmonic Mean Inequality
following conditions:
The Arithmetic-Geometric-Harmonic Mean of positive real
(i) with respect to one real number (k ). numbers is defined as follows
Situation Required conditions Arithmetic Mean ≥ Geometric Mean ≥ Harmonic Mean
α<β< k D ≥ 0, af (k ) > 0, k > − b / 2a a+b 2
(i) If a, b > 0 then ≥ ab ≥
k<α<β D ≥ 0, af (k ) > 0, k < − b /2a 2 1 1
+
a b
α< k<β D > 0, af (k ) < 0
(ii) If ai > 0 , where i = 1, 2, 3, K , n, then
(ii) with respect to two real numbers k1 and k2 .
a 1 + a 2 + ... + an n
Situation Required conditions ≥ (a1 ⋅ a2 ... an )1 / n ≥
n 1 1 1
+ + ... +
k1 < α < β < k2 D ≥ 0, af (k1 ) > 0, a1 a2 an
af (k2 ) > 0, k1 < − b /2a < k2
α < k1 < k2 < β D > 0, af (k1 ) < 0, af (k2 ) < 0
k1 < α < k2 < β D > 0, f (k1 ) f (k2 ) < 0
Logarithm Inequality
If a is a positive real number other than 1 and ax = m, then x
is called the logarithm of m to the base a, written as log a m. In
Inequalities log a m, m should always be positive.
Let a and b be two real numbers. If a − b is negative, we say (i) If m ≤ 0, then log a m will be meaningless.
that a is less than b ( a < b ) and if a − b is positive, then a is (ii) log a m exists, if m , a > 0 and a ≠ 1.
greater than b (a > b ). This shows the inequalities concept.

Important Results on Inequalities Important Results on Logarithm


(i) If a > b , then a ± c > b ± c, ∀ c ∈ R. (i) alog a x = x ; a > 0, ≠ 1, x > 0
=x ; a , b > 0, ≠ 1, x > 0
log b x log b a
(ii) If a > b , then (ii) a
a b (iii) log a a = 1, a > 0, ≠ 1
(a) for m > 0, am > bm, >
m m 1
a b (iv) log a x = ; x , a > 0, ≠ 1
(b) for m < 0, am < bm, < log x a
m m
(iii) (a) If a > b > 0 , then (v) log a x = log a b logb x =
logb x
; a, b > 0, ≠ 1, x > 0
1 1 logb a
•a >b
2 2
•| a| > |b | • <
a b
(b) If a < b < 0, then (vi) For x > 0; a > 0, ≠ 1
1 1 1
• a2 > b 2 • | a| > |b | • > (a) log an ( x) = log a x
a b n
(iv) If a < 0 < b , then  m
(b) log an x m =   log a x
(a) a2 > b 2 , if| a| > |b | (b) a2 < b 2 , if| a| < |b |  n
(v) If a < x < b and a, b are positive real numbers, then
(vii) For x > y > 0
a2 < x2 < b 2 .
(vi) If a < x < b and a is negative number and b is positive (a) log a x > log a y, if a > 1
number, then (b) log a x < log a y, if 0 < a < 1
(a) 0 ≤ x2 < b 2 , if |b | >| a | (b) 0 ≤ x2 < a2 , if | a | >|b |
(viii) If a > 1 and x > 0, then
(vii) If ai > b i > 0 , where i = 1, 2, 3, ..., n, then
(a) log a x > p ⇒ x > a p
a1 a2 a3 ... an > b1b2b3 ... b n .
(b) 0 < log a x < p ⇒ 0 < x < a p
(viii) If ai > b i , where i = 1, 2, 3, ..., n, then
a1 + a2 + a3 + ... + an > b1 + b2 + ... + b n . (ix) If 0 < a < 1 , then
(ix) If| x | < a, then (a) log a x > p ⇒ 0 < x < a p
(a) for a > 0, − a < x < a. (b) 0 < log a x < p ⇒ a p < x < 1
(b) for a < 0, x ∈ φ .
34 40

DAY PRACTICE SESSION 1

FOUNDATION QUESTIONS EXERCISE


1 If 3x 2 − 7x − 30 + 2x 2 − 7x − 5 = x + 5, then x is equal to 14 If P ( x ) = ax 2 + bx + c and Q ( x ) = − ax 2 + dx + c = 0 ;
(a) 2 (b) 3 (c) 6 (d) 5 ac ≠ 0, then the equation P ( x ) ⋅ Q ( x ) = 0 has
(a) four real roots (b) exactly two real roots
2 The number of solutions for equation x − 5 x + 6 = 0 is 2
(c) either two or four real roots (d) atmost two real roots
(a) 4 (b) 3 (c) 2 (d) 1 1
15 The rational values of a in ax 2 + bx + 1 = 0 if is a
4+ 3
2
3 The roots of the equation 2x − 1 − 3 2x − 1 + 2 = 0 are root, are
(a) − , 0,  (b) − , 0,  (c) − , , 0,1 (d) − , 0,1, 
1 1 1 3 3 1 1 3 (a) a = 13, b = − 8 (b) a = − 13, b = 8
 2 2  2 2  2 2   2 2 (c) a = 13, b = 8 (d) a = − 13, b = − 8
4 The product of all the values of x satisfying the equation 16 If 1 − i , is a root of the equation x 2 + ax + b = 0, where
(5 + 2 6) x2 − 3
+ (5 − 2 6) x2 − 3
= 10 is a, b ∈ R , then the values of a and b are
(a) 1,−1 (b) 2,− 2
(a) 4 (b) 6 (c) 8 (d) 19
(c) 3 ,−3 (d) None of these
5 The root of the equation 2(1 + i )x − 4( 2 − i )x − 5 − 3i = 0,
2

where i = −1, which has greater modulus, is 17 The values of p for which one root of the equation
3 − 5i 5 − 3i 3+i 3i + 1 x 2 − 30x + p = 0 is the square of the other, is/are
(a) (b) (c) (d)
2 2 2 2 (a) Only 125 (b) 125 and − 216
(c) 125 and 215 (d) Only 216
6 x 2 + x + 1 + 2k ( x 2 − x − 1) = 0 is perfect square for how
x −m x +n
many value of k 18 If the roots of the quadratic equation = are
mx + 1 nx + 1
(a) 2 (b) 0 (c) 1 (d) 3
reciprocal to each other, then
7 If the roots of (a + b )x − 2(bc + ad )x + c + d 2 = 0 are
2 2 2 2
(a) n = 0 (b) m = n (c) m + n = 1 (d) m 2 + n 2 = 1
equal, then
a c a b a b 19 Let α and α 2 be the roots of x 2 + x + 1 = 0, then the
(a) = (b) + = 0 (c) = (d) a + b = c + d
b d c d d c equation whose roots are α 31 and α 62 , is
8 The least value of α for which tan θ and cot θ are roots of (a) x 2 − x + 1 = 0 (b) x 2 + x − 1 = 0
(c) x 2 + x + 1 = 0 (d) x 60 + x 30 + 1 = 0
the equation x 2 + ax + 1 = 0, is
(a) 2 (b) 1 (c) 1/2 (d) 0 20 If α and β are the roots of x 2 − a( x − 1) + b = 0, then the
1 1 2
9 If one root of the equation x 2 − λx + 12 = 0 is even prime value of + 2 + is
α − aα β − aβ a + b
2
while x 2 + λx + µ = 0 has equal roots, then µ is equal to
4 1
(a) 8 (b) 16 (c) 24 (d) 32 (a) (b) (c) 0 (d) −1
a+b a+b
10 If a + b + c = 0, then the roots of the equation
4ax 2 + 3bx + 2c = 0, where a , b, c ∈ R are 21 The value of a for which the sum of the squares of the
(a) real and distinct (b) imaginary roots of the equation x 2 − (a − 2)x − a − 1 = 0 assume the
(c) real and equal (d) infinite least value is j
AIEEE 2005
11 The equation (cos β − 1) x 2 + (cos β)x + sin β = 0 in the (a) 2 (b) 3 (c) 0 (d) 1
variable x has real roots, then β lies in the interval 22 If α and β be the roots of the equation
(a) (0, 2 π) (b) (− π, 0)
π π
(c)  − ,  (d) (0, π)
2x 2 + 2(a + b )x + a 2 + b 2 = 0, then the equation whose
 2 2 roots are (α + β )2 and (α − β )2 , is
3c (a) x 2 − 2abx − (a 2 − b 2 )2 = 0 (b) x 2 − 4abx − (a 2 − b 2 )2 = 0
12 If ax 2 + 2bx − 3 c = 0 has no real root and < a + b,
4 (c) x 2 − 4abx + (a 2 − b 2 )2 = 0(d) None of these
then the range of c is
(a) (−1, 1 ) (b) (0, 1) 23 Let α , β be the roots of x 2 − 2x cos φ + 1 = 0, then the
(c) (0, ∞) (d) (− ∞, 0) equation whose roots are α n and β n , is
13 If a , b and c are real numbers in AP, then the roots of (a) x 2 − 2 x cosnφ − 1 = 0 (b) x 2 − 2 x cosnφ + 1 = 0
ax 2 + bx + c = 0 are real for (c) x 2 − 2 x sinnφ + 1 = 0 (d) x 2 + 2 x sinnφ − 1 = 0

(a) all a and c (b) no a and c 24 The harmonic mean of the roots of the equation
c a ( 5 + 2 )x 2 − ( 4 + 5 )x + 8 + 2 5 = 0 is
(c) −7 ≥ 4 3 (d) +7 ≥2 3
a c (a) 2 (b) 4 (c) 6 (d) 8
DAY 35

25 If the ratio of the roots of λx 2 + µx + ν = 0 is equal to the 35 If the equations x 2 + 2x + 3 = 0 and ax 2 + bx + c = 0,


ratio of the roots of x + x + 1 = 0, then λ , µ and ν are in
2
a, b, c ∈ R , have a common root, then a : b : c is
(a) AP (b) GP j AIEEE 2012
(c) HP (d) None of these (a) 1 : 2 : 3 (b) 3 : 2 : 1 (c) 1 : 3 : 2 (d) 3 : 1 : 2
1 1 1
26 If the roots of the equation + = are equal in 36 The equation formed by decreasing each root of
x +p x +q r ax 2 + bx + c = 0 by 1 is 2x 2 + 8x + 2 = 0, then
magnitude but opposite in sign, then the product of the (a) a = − b (b) b = − c (c) c = − a (d) b = a + c
roots is
37 If f ( x ) = x + 2bx + 2c and g ( x ) = − x − 2cx + b 2 such
2 2 2

(a) −2 (p 2 + q 2 ) (b) − (p 2 + q 2 ) that min f ( x ) > max g ( x ), then the relation between b and
− (p 2 + q 2 )
(c) (d) −pq c is
2
(a) | c| < | b| 2 (b) 0 < c < b 2
27 If the roots of the equation ax 2 + bx + c = 0 of the form (c) | c| < | b| 2 (d) | c | > | b | 2
k +1 k+2
and , then (a + b + c )2 is equal to 38 If a ∈ R and a1, a 2 , a 3 … , a n ∈ R , then
k k +1
( x − a1 )2 + ( x − a 2 )2 +… + ( x − a n )2 assumes its least
(a) 2b 2 − ac (b) ∑a 2 (c) b 2 − 4ac (d) b 2 − 2ac value at x =
28 If α and β are the roots of the equation ax 2 + bx + c = 0 (a) a1 + a2 + … + an (b) 2 (a1 + a2 + a3 + ... + an )
such that β < α < 0, then the quadratic equation whose (c) n (a1 + a2 + … + an ) (d) None of these
roots are α , β , is given by 39 If the roots of the equation bx 2 + cx + a = 0 is imaginary,
(a) a x + b x + c = 0
2
(b) ax − b x + c = 0
2
then for all real values of x, the expression
(c) a x − b x + c = 0
2
(d) a x
2
+ b| x + c = 0 3b 2 x 2 + 6bcx + 2c 2 is j AIEEE 2009
(a) greater than 4ab (b) less than 4ab
29 If α and β be the roots of x 2 + px + q = 0, then (c) greater than −4ab (d) less than −4ab
(ωα + ω 2β)(ω 2α + ωβ)
is equal to 40 If x 2 + 2ax + 10 − 3a > 0 for all x ∈ R , then
α 2 β2
+ (a) − 5 < a < 2 (b) a < − 5
β α
(c) a > 5 (d) 2 < a < 5
q p
(a) − (b) αβ (c) − (d) ω  1
p q 41 If the expression ax − 1 +  is non-negative for all
 x
p
30 If α and β are roots of the equation x 2 + px + 3. = 0, positive real x, then the minimum value of a must be
4 1
such that | α − β | = 10, then p belongs j
JEE Mains 2013 (a) 0
2
(a) {2, − 5 } (b) {− 3, 2 } (c) {− 2, 5 } (d) {3, − 5 } 1
(c) (d) None of these
4
31 Sachin and Rahul attemped to solve a quadratic
equation. Sachin made a mistake in writing down the 42 The number of real solutions of the equation
x
constant term and ended up in roots ( 4, 3). Rahul made a  9
  = − 3 + x − x is
2

mistake in writing down coefficient of x to get roots ( 3, 2). 10


The correct roots of equation are j
AIEEE 2011 (a) 0 (b) 1
(a) −4, − 3 (b) 6, 1 (c) 4, 3 (d) −6, − 1 (c)2 (d) None of these
32 If α and β are the roots of x 2 + x + 2 = 0 and γ,δ are the 43 If α and β be the roots of the quadratic equation
roots of x 2 + 3x + 4 = 0, then (α + γ )(α + δ )(β + γ ) ax 2 + bx + c = 0 and k be a real number, then the
(β + δ ) is equal to condition, so that α < k < β is given by
(a) −18 (b) 18 (c) 24 (d) 44 (a) ac > 0 (b) ak 2 + bk + c = 0
π P Q (c) ac < 0 (d) a 2 k 2 + abk + ac < 0
33 In ∆PQR , R = . If tan and tan are the roots of the
2 2 2 44 2x − 3 < x + 5 , then x belongs to
equation ax 2 + bx + c = 0, then
(c)  − , 8  (d)  −8, 
2 2
(a) (−3,5) (b) (5, 9)
(a) a = b + c (b) b = c + a  3   3
(c) c = a + b (d) b = c x −5
45 The least integral value α of x such that > 0,
34 If the equation k ( 6x 2 + 3) + rx + 2x 2 − 1 = 0 and x 2 + 5x − 14
6 k ( 2x 2 + 1) + px + 4x 2 − 2 = 0 have both roots common, satisfies j
JEE Mains 2013
then 2r − p is equal to (a) α + 3 α − 4 = 0
2
(b) α − 5 α + 4 = 0
2

(a) 2 (b) 1 (c) 0 (d) k (c) α 2 − 7α + 6 = 0 (d) α 2 + 5 α − 6 = 0


36 40

| x − 2 | −1 51 If a, b, c are positive real numbers such that a + b + c = 1,


46 The solution set of ≤ 0 is
|x −2| −2 then the greatest value of (1 − a ) (1 − b ) (1 − c ), is
(a) [0, 1] ∪ (3, 4) (b) [0, 1] ∪ [3, 4] 1 8 4
(a) (b) (c) (d) 9
27 27 27
(c) [−1, 1) ∪ (3, 4] (d) None of these
x +2 1 52 If a, b, c, d are positive real numbers such that
47 Number of integral solutions of > is a + b + c + d = 2, then M = (a + b ) (c + d ) satisfies the
x2 +1 2
relation
(a) 0 (b) 1 (c) 2 (d) 3
(a) 0 ≤ M ≤ 1 (b) 1 ≤ M ≤ 2 (c) 2 ≤ M ≤ 3 (d) 3 ≤ M ≤ 4
48 If the product of n positive numbers is 1, then their sum is
53 log2 ( x − 3x + 18) < 4, then x belongs to
2

(a) a positive integer (b) divisible by n


1 (a) (1, 2) (b) (2, 16)
(c) equation to n + (d) greater than or equal to n
n (c) (1, 16) (d) None of these

49 The minimum value of P = bcx + cay + abz , when 54 If log0. 3 ( x − 1) > log0. 09 ( x − 1), then x lies in
xyz = abc, is (a) (1, 2) (b) (− ∞, 1)
(a) 3abc (b) 6abc (c) (2, ∞) (d) None of these
(c) abc (d) 4abc 55 What is the solution set of the following inequality?
50 If a, b and c are distinct three positive real numbers, then  x + 5
logx   >0
 1 1 1 1 − 3x 
(a + b + c )  + +  is
a b c 1
(a) 0 < x < (b) x ≥ 3
(a) > 1 (b) > 9 3
1
(c) < 9 (d) None of these (c) < x<1 (d) None of these
3

DAY PRACTICE SESSION 2

PROGRESSIVE QUESTIONS EXERCISE


1 Let S = {x ∈ R : x ≥ 0 and 2 x − 3 + x ( x − 6) + 6 =.0 5 If a < b < c < d , then the roots of the equation
( x − a )( x − c ) + 2 ( x − b )( x − d ) = 0 are
Then, S j
JEE Mains 2018
(a) real and distinct (b) real and equal
(a) is an empty set
(c) imaginary (d) None of these
(b) contains exactly one element
(c) contains exactly two elements 6 Let α and β be the roots of equation px 2 + qx + r = 0,
(d) contains exactly four elements 1 1
p ≠ 0. If p, q and r are in AP and + = 4, then the value
2 The roots of the equation 2 x + 2
⋅3 3 x / ( x − 1)
= 9 are given by α β
of α − β is
(b) log2   , 1
2 j
JEE Mains 2014
(a) 1 − log2 3, 2
 3 61 2 17 34 2 13
(log 3) (a) (b) (c) (d)
(c) 2 , − 2 (d) − 2 , 1 − 9 9 9 9
(log 2)
7 If α and β are the roots of the equation ax 2 + bx + c = 0
3 Let α and β be the roots equation x 2 − 6x − 2 = 0. If (a ≠ 0, a, b, c being different), then (1 + α + α 2 ) (1 + β + β 2 ) is
a − 2a 8
a n = α − β for n ≥ 1, then the value of 10
n n
is equal (a) zero (b) positive
2a 9 (c) negative (d) None of these
to j
JEE Mains 2015 8 The minimum value of the sum of real numbers
(a) 6 (b) − 6 (c) 3 (d) − 3 a −5 , a −4 , 3a −3 , 1, a 8 and a10 a > 0 is
x 10 + 1 (a) 9 (b) 8 (c) 2 (d) 1
4 If x 2 − 5x + 1 = 0, then is equal to
x5 9 For a > 0, the roots of the equation
(a) 2424 (b) 3232 logax a + logx a 2 + loga 2 x a 3 = 0 is given by
(c) 2525 (d) None of these (a) a −4 / 3 (b) a −3 / 4 (c) a1 / 2 (d) a −1
DAY 37

10 If a , b and c are in AP and if the equations 19 If α and β are the roots of the equation ax 2 + bx + c = 0,
(b − c ) x + (c − a )x + (a − b ) = 0 and 2 (c + a ) x
2 2 α
then the quadratic equation whose roots are and
+ (b + c ) x = 0 have a common root, then 1+ α
β
(a) a 2 , b 2 and c 2 are in AP (b) a 2 , c 2 and b 2 are in AP is
(c) c 2 , a 2 and b 2 are in AP (d) None of these 1+ β
(a) ax 2 − b (1 − x) + c (1 − x)2 = 0
11 If the equations x 2 + ax + 12 = 0, x 2 + bx + 15 = 0 and (b) ax 2 − b (x − 1) + c (x − 1)2 = 0
x 2 + (a + b )x + 36 = 0 have a common positive root, then (c) ax 2 + b (1 − x) + c (1 − x)2 = 0
the ordered pair (a , b ) is (d) ax 2 + b (x + 1) + c (1 + x)2 = 0
(a) (− 6, − 7) (b) (− 7, − 8) 20 If both the roots of the quadratic equation
(c) (− 6, − 8) (d) (− 8, − 7)
x 2 − 2kx + k 2 + k − 5 = 0 are less than 5, then k lies in the
12 If x is real, then the maximum and minimum value of the interval j AIEEE 2005

x 2 − 3x + 4 (a) [4,5] (b) (−∞, 4) (c) (6, ∞) (d) (5, 6)


expression will be
x 2 + 3x + 4
21 All the values of m for which both roots of the equation
1
(a) 2, 1 (b) 5, x 2 − 2mx + m 2 − 1 = 0 are greater than −2 but less than 4
5
1 lie in the interval
(c) 7, (d) None of these
7 (a) m > 3 (b) −1 < m < 3
(c) 1 < m < 4 (d) −2 < m < 0
13 If a ∈ R and the equation
−3( x − [ x ])2 + 2( x − [ x ]) + a 2 = 0 (where, [ x ] denotes the 22 Let α and β be real and z be a complex number. If
greatest integer ≤ x) has no integral solution, then all z 2 + αz + β = 0 has two distinct roots on the line
Re ( z ) = 1, then it is necessary that j AIEEE 2011
possible values of a lie in the interval j JEE Mains 2014

(a) β ∈ (− 1, 0) (b) | β | = 1 (c) β ∈ [1, ∞) (d) β ∈ (0, 1)


(a) (−1, 0) ∪ (0,1) (b) (1, 2)
− sin x
(c) (−2, − 1) (d) (−∞, − 2) ∪ (2, ∞) 23 The equation e sin x
−e − 4 = 0 has j AIEEE 2012
2π 2π (a) infinite number of real roots
14 If a = cos + i sin , then the quadratic equation
7 7 (b) no real root
whose roots are α = a + a 2 + a 4 and β = a 3 + a 5 + a 6 , is (c) exactly one real root
(a) x 2 − x + 2 = 0 (b) x 2 + 2 x + 2 = 0 (d) exactly four real roots
(c) x 2 + x + 2 = 0 (d) x 2 + x − 2 = 0 24 Ifa, b, c, d are positive real numbers such that
15 If α and β are roots of 375 x 2 − 25x − 2 = 0 and 1 1 1 1
a+ = 4, b + = 1, c + = 4 and d + = 1, then
n b c d a
S n = α n + β n , then lim
n→ ∞ ∑S
r =1
r is equal to
(a) a = c and b = d (b) b = d but a ≠ c
(c) ab = 1and cd ≠ 1 (d) cd = 1and ab ≠ 1
7 1
(a) (b)
116 12 (ω and ω2 are complex cube roots of unity)
29
(c) (d) None of these
358 25 Let f : R → R be a continuous function defined by
1
16 If S = {a ∈ N, 1 ≤ a ≤ 100} and [tan x ] − tan x − a = 0 has
2 f (x ) =
e x + 2e − x
real roots, where [. ] denotes the greatest integer
function, then number of elements in set S equals 1
Statement I f (c ) = , for some c ∈ R .
(a) 2 (b) 5 (c) 6 (d) 9 3
17 The sum of all real values of x satisfying the equation 1
2
+ 4 x − 60 Statement II 0 < f ( x ) ≤ , ∀ x ∈R .
( x 2 − 5 x + 5 )x = 1 is j
JEE Mains 2016 2 2 j
AIEEE 2010
(a) 3 (b) −4 (c) 6 (d) 5 (a) Statement I is false, Statement II is true
18 If λ is an integer and α , β are the roots of (b) Statement I is true, Statement II is true.
λ Statement II is a correct explanation of Statement I
4x 2 − 16x + = 0 such that 1 < α < 2 and 2 < β < 3, then
4 (c) Statement I is true, Statement II is true;
the possible values of λ are Statement II is not a correct explanation for Statement I
(a) {60, 64, 68} (b) {61, 62, 63} (d) Statement I is true, Statement II is false
(c) {49, 50,…, 62, 63} (d) {62, 65, 68, 71, 75}
38 40

ANSWERS
1. (c) 2. (a) 3. (d) 4. (c) 5. (a) 6. (a) 7. (c) 8. (a) 9. (b) 10. (a)
SESSION 1
11. (d) 12. (d) 13. (c) 14. (c) 15. (a) 16. (d) 17. (b) 18. (a) 19. (c) 20. (c)
21. (d) 22. (b) 23. (b) 24. (b) 25. (b) 26. (c) 27. (c) 28. (c) 29. (a) 30. (c)
31. (b) 32. (d) 33. (c) 34. (c) 35. (a) 36. (b) 37. (d) 38. (d) 39. (c) 40. (a)
41. (c) 42. (a) 43. (d) 44. (c) 45. (d) 46. (b) 47. (d) 48. (d) 49. (a) 50. (b)
51. (b) 52. (a) 53. (a) 54. (a) 55. (d)

SESSION 2 1. (c) 2. (d) 3. (c) 4. (c) 5. (a) 6. (d) 7. (b) 8. (b) 9. (a) 10. (b)
11. (b) 12. (c) 13. (a) 14. (c) 15. (b) 16. (d) 17. (a) 18. (c) 19. (c) 20. (b)
21. (b) 22. (c) 23. (b) 24. (a) 25. (b)

Hints and Explanations


SESSION 1 4 Q 5− 2 6 = 1 If equation is a perfect square then root
5+ 2 6 are equal
1 We have,
1 i.e. (1 − 2k )2 − 4(1 + 2k ) (1 − 2k ) = 0
3 x2 − 7 x − 30 + 2 x2 − 7 x − 5 = x + 5 ∴ t + = 10, 1 −3
t i.e. k = , .
⇒ 3 x2 − 7 x − 30 = ( x + 5) − 2 x2 − 7 x − 5 2 10
2
−3
where t = (5 + 2 6 )x …(i) Hence, total number of values = 2.
On squaring both sides, we get
⇒ t − 10t + 1 = 0
2
7 Since, roots are real.
3 x2 − 7 x − 30
⇒ t = 5± 2 6 ∴ {2(bc + ad )}2 = 4(a2 + b 2 )(c 2 + d 2 )
= x2 + 25 + 10 x + (2 x2 − 7 x − 5)
t = (5 + 2 6 )± 1 ⇒ 4b 2c 2 + 4a2d 2 + 8abcd = 4a2c 2 + 4a2d 2
−2( x + 5) 2 x2 − 7 x − 5 or …(ii)
+4b 2c 2 + 4b 2d 2
⇒ −10 x − 50 = −2( x + 5) 2 x2 − 7 x − 5 From Eqs. (i) and (ii), ⇒ 4a c + 4b d − 8abcd = 0
2 2 2 2

x + 5 ≠ 0, 2 x2 − 7 x − 5 = 5 x2 − 3 = ± 1 ⇒ 4(ac − bd )2 = 0
[Q x = −5does not satisfy the given ⇒ x2 = 2, 4 ⇒ ac = bd
equation] a b
⇒ x = − 2 , 2 , − 2, 2 ⇒ =
⇒ 2 x2 − 7 x − 30 = 0 d c
∴ x=6 ∴ Required product = 8
8 Given equation is x2 + ax + 1 = 0
2 Given equation is x2 − 5 x + 6 = 0 5 The given equation is Since, roots are real
When x ≥ 0, x2 − 5x + 6 = 0 2(1 + i )x2 − 4(2 − i )x − 5 − 3i = 0 ∴ a2 − 4 ≥ 0 ⇒ a ≥ 2
and when x < 0, x2 + 5x + 6 = 0 16(2 − i )2 Thus, the least value of a is 2.
4(2 − i ) ±
⇒ x2 − 3 x − 2 x + 6 = 0; x ≥ 0 + 8(1 + i )(5 + 3i ) 9 We know that only even prime is 2,
⇒x=
and x2 + 3 x + 2 x + 6 = 0; x < 0 4(1 + i ) ∴ (2)2 − λ(2) + 12 = 0.
⇒ ( x − 3)( x − 2) = 0, x ≥ 0 i 4− i −1 − i 3 − 5i ⇒ λ=8 ...(i)
=− or = or
and ( x + 3)⋅ ( x + 2) = 0, x < 0 1+ i 1+ i 2 2 Qx2 + λx + µ = 0 has equal roots.
∴ x = 3, x = 2 and x = −3, x = −2 −1 − i 1 1 1 ∴ λ2 − 4µ = 0 [QD = 0]
Now, = + =
There are four solutions of this equation. 2 4 4 2 ⇒ (8)2 − 4µ = 0 ⇒ µ = 16
3 Given equation is 3 − 5i 9 25 17 10 Here, D = (3b )2 − 4 (4 a)(2c )
and = + =
2
2 x − 1 − 32 x − 1 + 2 = 0 2 4 4 2 = 9b 2 − 32ac = 9 (− a − c )2 − 32ac
Let 2 x − 1 = t , then Also,
17
>
1 = 9 a2 − 14 ac + 9c 2
2 2  a 2 14 a 
t 2 − 3t + 2 = 0
3 − 5i = 9c 2    − ⋅ + 1
⇒ (t − 1)(t − 2) = 0 ⇒ t = 1,2 Hence, required root is .   c  9 c 
⇒ 2 x − 1 = 1 and 2 x − 1 = 2 2
  a 7 
2
49 
⇒ 2 x − 1 = ±1 and 2 x − 1 = ±2 6 Given equation = 9c 2   −  − + 1 > 0
  c 9  81 
3 1 (1 + 2k ) x2 + (1 − 2k )x + (1 − 2k ) = 0
⇒ x = 1, 0 and x = , −
2 2 Hence, the roots are real and distinct.
DAY 39

11 For real roots, discriminant, [since, non-real complex roots occur in a−1 = 0
conjugate pairs] ⇒ a=1
D = b − 4 ac ≥ 0
2

= cos 2 β − 4(cos β − 1)sin β ≥ 0 Product of roots, 22 Since, α and β are the roots of the
b
= cos 2 β + 4(1 − cos β )sin β ≥ 0 = (1 − i )(1 + i ) ⇒ b = 2 equation
So, sin β should be > 0. 1 2 x2 + 2(a + b )x + a2 + b 2 = 0
[Q cos 2 β ≥ 0,1 − cos β ≥ 0] 17 Let roots be α and α2 . a2 + b 2
∴ (α + β )2 = (a + b )2 and αβ =
⇒ β ∈ (0, π ) Then, α + α2 = 30 and α3 = p 2
⇒ α2 + α − 30 = 0 Now, (α − β )2 = (α + β )2 − 4αβ
12 Here, D = 4b + 12ca < 0
2
 a2 + b 2 
⇒ b 2 + 3ca < 0 …(i) ⇒ (α + 6)(α − 5) = 0 = (a + b )2 − 4 
 2 
⇒ ca < 0 ∴ α= −6, 5
= − (a − b )2
If c > 0, then a < 0 ⇒ p= α3 = (−6)3 = −216
Now, the required equation whose
3c and p= (5)3 = 125
Also, < a+ b roots are (α + β )2 and (α − β )2 is
4 ∴ p= 125and −216
x2 − {(α + β )2 + (α − β )2 } x
⇒ 3ca > 4 a2 + 4 ab x−m x+ n + (α + β )2 (α − β )2 = 0
18 Given, =
mx + 1 nx + 1 ⇒ x2 − {(a + b )2 − (a − b )2 } x
⇒ b + 3ca > 4 a2 + 4 ab + b 2
2

= (2a + b )2 ≥ 0 …(ii) ⇒ x2 (m − n ) + 2mnx + (m + n ) = 0 − (a + b )2 (a − b )2 = 0


From (i) and (ii), c > 0, is not true.
1
Roots are α, respectively, then ⇒ x − 4abx − (a2 − b 2 )2 = 0
2

α
∴ c<0 23 The given equation is
1 m+ n
α⋅ = x2 − 2 x cos φ + 1 = 0
13 Since, D ≥ 0 α m−n
2cos φ ± 4cos 2 φ − 4
∴ b 2 − 4 ac ≥ 0 ⇒ m − n = m + n ⇒ n = 0. ∴ x=
2 2
c + a
⇒   − 4 ac ≥ 0 [Q 2b = a + c ] 19 Since, α,α2 be the roots of the equation = cos φ ± i sin φ
 2 
x2 + x + 1 = 0 Let α = cos φ + i sin φ, then
⇒ c 2 − 14ca + a2 ≥ 0 ∴ α + α 2 = −1 ... (i) β = cos φ − i sin φ
2
and α = 1
3
∴ α n + β n = (cos φ + i sin φ)n
⇒   − 14   + 1≥ 0
c c ... (ii)
 a  a Now, α 31 + α 62 = α31 (1 + α31 ) + (cos φ − i sin φ)n
⇒ α + α 62 = α30 ⋅ α(1 + α 30 ⋅ α)
31 = 2cos nφ
2

⇒  c − 7 ≥ 48 and α nβ n = (cos nφ + i sin nφ)


  ⇒ α31 + α 62 = (α3 )10 ⋅ α{1 + (α3 )10 ⋅ α}
a  ⋅ (cos nφ − i sin nφ)
⇒ α31 + α 62 = α(1 + α) [from Eq. (ii)]
c = cos 2 nφ + sin2 nφ = 1
⇒ −7 ≥ 4 3 ⇒ α31 + α 62 = −1 [from Eq. (i)] ∴ Required equation is
a
Again, α31 ⋅ α 62 = α 93 x2 − 2 x cos nφ + 1 = 0
14 Let D1 and D2 be the discriminants of ⇒ α31 ⋅ α 62 = [α3 ]31 = 1
24 Given equation is
given equation, respectively. Then ∴ Required equation is
(5 + 2 )x2 − (4 + 5)x + 8 + 2 5 = 0
D1 + D2 = b 2 − 4 ac + d 2 + 4 ac x2 − (α31 + α 62 )x + α31 ⋅ α 62 = 0
Let x1 and x2 are the roots of the
⇒ x2 + x + 1 = 0
= b2 + d 2 > 0 equation, then
So, either D1 and D2 are positive or 20 Since, α and β are the roots of
4+ 5
x2 − ax + a + b = 0, then x1 + x2 = ... (i)
atleast one D’s is positive. 5+ 2
a+ β = a
Therefore, P ( x )⋅ Q ( x ) = 0 has either two αβ = a + b 8 + 2 5 2(4 + 5)
and and x1 x2 = =
or four real roots. 5+ 2 5+ 2
⇒ a2 + αβ = aα
1 4− 3 4− 3 ⇒ α2 − aα = − (a + b ) = 2( x1 + x2 ) ... (ii)
15 One root = × =
4+ 3 4− 3 13 and αβ + β2 = aβ Clearly, harmonic mean
4+ 3 ⇒ β2 − aβ = − (a + b ) 2 x1 x2 4( x1 + x2 )
= = =4
∴ Other root = 1 1 2 x1 + x2 ( x1 + x2 )
13 ∴ + +
∴The quadratic equation is α2 − aα β2 − aβ a+ b [from Eq. (ii)]
4+ 3 4 − 3 1 1 2
x2 −  + = + + =0 25 Let α, β and α ′, β ′ be the roots of the
 x − (a + b ) − (a + b ) (a + b )
 13 13  given equations, respectively.
4+ 3 4− 3 21 Let α and β be the roots of equation µ ν
+ ⋅ =0 ∴ α + β = − , αβ = …(i)
13 13 x2 − (a − 2)x − a − 1 = 0 λ λ
or 13 x2 − 8 x + 1 = 0 Then, α + β = a − 2 and αβ = − a − 1 and α ′ = ω and β ′ = ω2
α α′
This equation must be identical with Now, α2 + β2 = (α + β )2 − 2αβ Q = [given]
ax2 + bx + 1 = 0; β β′
⇒ α2 + β2 = (a − 2)2 + 2(a + 1)
∴ a = 13 and b = − 8. α ω
⇒ α2 + β2 = a2 − 2a + 6 ⇒ = ⇒ β = αω
16 Sum of roots β ω2
−a ⇒ α2 + β2 = (a − 1)2 + 5
From Eq. (i),
= (1 − i ) + (1 + i ) ⇒ a = − 2. The value of α2 + β2 will be least, if
1
40 40

µ 2 ν = α2 + β2 − αβ (Qω + ω2 = −1) 34 Given, (6 k + 2)x2 + rx + 3 k − 1 = 0


α + αω = − ,α ω =
λ λ = (α + β ) − 3αβ = p − 3q
2 2
and (12k + 4)x2 + px + 6 k − 2 = 0
µ ν α 2 β2 α 3 + β3
⇒ − αω2 = − , α2ω = Also, + = For both common roots,
λ λ β α αβ 6k + 2 r 3k − 1
[Q− ω2 = 1 + ω] = =
(α + β )3 − 3αβ(α + β) p(3q − p2 ) 12 k + 4 p 6 k − 2
µ2 ν = =
⇒ = ⇒ µ 2 = λν αβ q r 1
λ2 λ ⇒ = ⇒ 2r − p = 0
∴ The given expression p 2
26 Simplified form of given equation is ( p2 − 3q ) q
(2x + p + q ) r = ( x + p )( x + q ) = =− 35 Given equations are
p(3q − p2 ) p
⇒ x + ( p + q − 2r )x
2 x2 + 2 x + 3 = 0 ... (i)
q
− ( p + q ) r + pq = 0 and ax2 + bx + c = 0 ... (ii)
3p Since, Eq. (i) has imaginary roots.
Since, sum of roots = 0 30 Clearly, α + β = − p and αβ =
⇒ − ( p + q − 2r ) = 0 4 So, Eq. (ii) will also have both roots
p+q Also, (α − β )2 = 10 same as Eq. (i)
⇒ r = a b c
2 ⇒ (α + β )2 − 4αβ = 10 Thus, = =
and product of roots ⇒ p2 − 3 p = 10 1 2 3
= − ( p + q ) r + pq ⇒ ( p + 2)( p − 5) = 0 Hence, a:b :c is 1:2:3.
( p + q )2 ∴ p = −2, 5 36 α, β be the roots of ax2 + bx + c = 0
=− + pq
2 31 Let the quadratic equation be ∴ α + β = − b / a, αβ = c / a
1
= − ( p2 + q 2 ) ax2 + bx + c = 0 and its roots are α and Now roots are α − 1, β − 1
2 β. Their sum, α + β − 2 = (− b / a) − 2
Sachin made a mistake in writing down = − 8 /2 = − 4
27 Clearly, sum of roots,
constant term. Their product,
k+1 k+2 b
+ =− ... (i) ∴ Sum of roots is correct i.e. α + β = 7 (α − 1) (β − 1) = αβ − (α + β ) + 1
k k+1 a Rahul made mistake in writing down
and product of roots, = c /a + b/a + 1 = 1
coefficient of x.
k+1 k+2 c ∴ b / a = 2 i.e. b = 2a
× = ∴ Product of roots is correct.
k k+1 a also c + b = 0 ⇒ b = − c .
i.e. α ⋅ β = 6
k+2 c 4b 2 − 8c 2
⇒ = ⇒ Correct quadratic equation is 37 min f ( x ) = − D = −
k a 4a 4
x2 − (α + β )x + αβ = 0.
2 c c−a 2a = − (b 2 − 2c 2 )
⇒ = −1 = ⇒k = ⇒ x2 − 7 x + 6 = 0 having roots 1
k a a c−a (upward parabola)
and 6.
On putting the value of k in Eq. (i), we D 4c 2 + 4b 2
get 32 Since, α + β = − 1,αβ = 2, max g ( x ) = − =
c+a 2c b 4a 4
+ =− γ + δ = − 3, and γδ = 4 = b2 + c 2
2a c+a a
⇒ (c + a)2 + 4ac = −2b(a + c ) ∴ (α + γ )(α + δ )(β + γ )(β + δ ) (downward parabola)
= (α2 − 3α + 4)(β2 − 3β + 4) Now, 2c 2 − b 2 > b 2 + c 2
⇒ (a + c )2 + 2b(a + c ) = −4ac
⇒ c 2 > 2b 2 ⇒ |c| > 2 |b|
⇒ (a + c )2 + 2b(a + c ) + b 2 = b 2 − 4ac = 4 − 3αβ2 + 4β2 − 3α2β + 9αβ
⇒ (a + b + c )2 = b 2 − 4ac − 12β + 4α2 − 12α + 16 38 We have,
28 Since, α and β are the roots of the = 4 − 3(2)β + 4β2 + 4α2 ( x − a1 )2 + ( x − a2 )2 + … + ( x − an )2
equation ax + bx + c = 0
2
−3( 2)α + 9( 2) − 12 ( β + α ) + 16 = n x2 − 2 x(a1 + a2 + … + an )
b c + (a12 + a22 + …+ a2n )
∴ α + β = − and αβ = = 4 − 6β + 4(α2 + β2 )
a a So, it attains its minimum value at
− 6α + 18 + 12 + 16 2(a1 + a2 + … + an )
Now, sum of roots = α + β x=
= 50 + 6 + 4[(α + β )2 − 2αβ] 2n
= − α −β (Qβ < α < 0)
 b b = 56 − 12 = 44 a + a2 + … + an  using : x = −b 
= − −  = (Qα + β > 0) = 1
 a n  2a 
a
33 Given, R = π ⇒ P + Q = π
c 2 2 39 Given bx2 + cx + a = 0 has imaginary
and product of roots = α β =
a P Q π roots.
⇒ + =
Hence, required eqution is 2 2 4 ⇒ c 2 − 4ab < 0 ⇒ c 2 < 4ab …(i)
b c Q Let f ( x ) = 3b 2 x2 + 6bcx + 2c 2
x2 − x + =0 P
tan + tan
a a π 2 2 Here, 3b 2 > 0
⇒ 1 = tan =
⇒ a x2 − b x + c = 0 4 1 − tan P tan Q So, the given expression has a
2 minimum value.
29 Since, α and β are the roots of the 2
b −D
− ∴ Minimum value =
equation x2 + px + q = 0 therefore a 4a
b
α + β = − p and αβ = q ⇒ 1= = ⇒a+ b =c 4ac − b 2 4(3b 2 )(2c 2 ) − 36b 2c 2
1−
c c −a = =
Now, (ωα + ω2β )(ω2α + ωβ ) 4a 4(3b 2 )
a
= α2 + β2 + (ω 4 + ω2 )αβ (Qω3 = 1)
DAY 41

12b 2c 2 | x − 2| − 1 51 Using GM ≤ AM, we have


=− = − c 2 > −4ab 46 Given, ≤0
12b 2 | x − 2| − 2 {(1 − a) (1 − b ) (1 − c )1 /3 }
[from Eq. (i)] Let | x − 2 |= k 1− a+ 1−b + 1−c

40 According to given condition, Then, given equation, 3
k −1 (k − 1) (k − 2) 8
4a2 − 4(10 − 3a) < 0 ≤0 ⇒ ≤0 ⇒ (1 − a) (1 − b ) (1 − c ) ≤
⇒ a2 + 3a − 10 < 0 k −2 (k − 2)2 27
8
⇒ (a + 5) (a − 2) < 0 ⇒ (k − 1) (k − 2) ≤ 0 ⇒ 1 ≤ k ≤ 2 Hence, the greatest value is .
27
⇒ − 5 < a < 2. ⇒ 1 ≤ | x − 2|≤ 2
Case I When 1 ≤ | x − 2| 52 Using AM ≥ GM, we have
41 We have, ax − 1 + 1 ≥ 0
x (a + b ) + (c + d )
⇒ | x − 2|≥ 1 ≥ {(a + b ) (c + d )}1 /2
ax2 − x + 1 2
⇒ ≥0 ⇒ x − 2 ≥ 1 or x − 2 ≤ −1
x 2
⇒ x ≥ 3 and x ≤ 1 …(i) ⇒ ≥ M 1 /2 ⇒ M ≤ 1
⇒ ax − x + 1 ≤ 0 as x > 0
2
2
Case II When | x − 2|≤ 2 As a, b, c , d > 0.
It will hold if a > 0 and D ≤ 0
1 ⇒ −2 ≤ x − 2 ≤ 2 Therefore, M = (a + b ) (c + d ) > 0.
a > 0 and 1 − 4a ≤ 0 ⇒ a ≥
4 ⇒ −2 + 2 ≤ x ≤ 2 + 2 Hence, 0 ≤ M ≤ 1.
1 ⇒ 0≤ x ≤ 4 ...(ii)
Therefore, the minimum value of a is . 53 log 2 ( x2 − 3 x + 18) < 4
4 From (i) and (ii), x ∈ [0, 1] ∪ [3, 4]
⇒ x2 − 3 x + 18 < 16
42 Let f ( x ) = − 3 + x − x2 . x+2 1 (Qlog a b < c ⇔ b < ac , if a > 1)
47 − >0
Then, f ( x ) < 0 for all x, because coeff. of x2 + 1 2 ⇒ x2 − 3 x + 2 < 0
x2 < 0 and disc. < 0. − x2 − 1 + 2 x + 4 ⇒ ( x − 1)( x − 2) < 0 ⇒ x ∈ (1,2)
⇒ >0
Thus, LHS of the given equation is 2( x2 + 1)
always positive whereas the RHS is 54. Clearly, x − 1 > 0 ⇒ x > 1
3 + 2 x − x2
always less than zero. Hence, the given ⇒ >0 and log 0 . 3 ( x − 1) > log ( 0 . 3 )2 ( x − 1)
equation has no solution. 2( x2 + 1)
1
Since, denominator is positive ⇒ log 0 . 3 ( x − 1) > log 0 . 3 ( x − 1)
43 Let f ( x ) = ax2 + bx + c . 2
∴3 + 2 x − x2 > 0 ⇒ log 0 . 3 ( x − 1) > 0 ⇒ x < 2
Then, k lies betweenα andβ, if a f (k ) < 0 ⇒ − 1< x < 3
⇒ a (ak 2 + bk + c ) < 0 Hence, x ∈ (1, 2)
⇒ x = 0, 1, 2
⇒ a k 2 + abk + ac < 0.
2
55 By definition of log x, x > 0 and
48 Let a1 , a2 , … , an be n positive integers
44 We have, 2 x − 3 < x + 5  x + 5
such that a1 a2 … an = 1.  >0
⇒ 2x − 3 − x + 5 < 0 Using AM ≥ GM, we have  1 − 3x 
 a + a2 + … + an ( x + 5)(1 − 3 x )
⇒ 1 ≥ (a1 a2 … an )1 / n ⇒ >0
 3 − 2 x + x + 5 < 0, x ≤ − 5 n (1 − 3 x )2
 3
⇒ 3 − 2 x − x − 5 < 0, − 5 < x ≤ ⇒ a1 + a2 + … an ≥ n.
 2 ⇒ ( x + 5)(1 − 3 x ) > 0
 2 x − 3 − x − 5 < 0, x >
3 49 Since, AM ≥ GM
 2 bcx + cay + abz ⇒ ( x + 5)(3 x − 1) < 0
∴ ≥ (a2b 2c 2 ⋅ xyz )1 /3
 3 ⇒ − 5 < x < 1/3
 x > 8, x ≤ −5 ⇒ bcx + cay + abz ≥ 3abc
 2 3 As x > 0, 0 < x < 1 / 3
⇒  x > − ,− 5< x ≤ [Qxyz = abc ]
 3 2 x+ 5
3 50 We know that, AM > GM ∴ < 1⇒ x < − 1
 x < 8, x > 1 − 3x
 2 a+ b + c
∴ > (abc )1 /3 ... (i) This does not satisfy 0 < x < 1 / 3.
⇒  2 3  3 
x ∈  − , ∪  , 8
3
Hence, there is no solution.
 3 2   2  1 1 1
+ + 1 /3
a b c >  1 
x ∈  − , 8
2 and
⇒ 3  abc  SESSION 2
 3 
 1 + 1 + 1 > 3 1 We have
x−5 ⇒   ... (ii)
45 >0  a b c  (abc )1 /3 2 x −3 + x ( x − 6) + 6 = 0
x + 5x − 14
2

From (i) and (ii), we get


( x + 7 )( x − 2) ( x − 5) Let x −3 = y
⇒ >0
( x − 2)2 ( x + 7 )2  a + b + c   1 + 1 + 1 ⇒ x = y+3
  
 3  a b c 
3 ∴ 2 y + ( y + 3)( y − 3) + 6 = 0
> ⋅ (abc )1 /3 ⇒ 2 y + y2 − 3 = 0
⇒ x ∈ (−7, 2) ∪ (5, ∞ ) (abc )1 /3 2
⇒ y + 2 y −3 = 0
⇒ (a + b + c ) + +  > 9
So, the least integral value α of x is − 6, 1 1 1
which satisfy the equation a b c ⇒ ( y + 3)( y − 1) = 0
α2 + 5α − 6 = 0 ⇒ y ≠ −3 ⇒ y = 1
42 40

⇒ y = ±1 5 Given equation can be rewritten as 9 We have,


⇒ x − 3 = ±1 3 x2 − (a + c + 2b + 2d )x + ac + 2bd = 0 log a a 2 log a a
+
⇒ x = 4,2 Now, discriminant, D log a a + log a x log a x
= (a + c + 2b + 2d )2 − 4⋅ 3(ac + 2bd )
⇒ x = 16, 4 +
3 log a a
=0
= {(a + 2d ) + (c + 2b )}2 − 12(ac + 2bd )
x +2 2 log a a + log a x
2 We have, 2 ⋅ 33 x / ( x − 1 ) = 9 = {(a + 2d ) − (c + 2b )}2 + 4(a + 2d )
(c + 2b ) − 12(ac + 2bd ) 1 2 3
Taking log on both sides, we get ⇒ + + =0
3x = {(a + 2d ) − (c + 2b )}2 1+ t t 2+ t
( x + 2)log 2 + log 3 = 2 log 3
( x − 1) −8ac + 8ab + 8dc − 8bd ( let log a x = t )
= {(a + 2d ) − (c + 2b )}2 + 8(c − b )(d − a) 2t + t 2 + 2t 2 + 6t + 4 + 3t 2 + 3t
⇒ ( x2 + x − 2)log 2 + 3 x log 3 ⇒ =0
Which is + ve, since a < b < c < d . t (1 + t ) (2 + t )
= 2( x − 1)log 3 Hence, roots are real and distinct.
⇒ 6t 2 + 11t + 4 = 0
⇒ x2 log 2 + (log 2 + log 3)x 6 Since, α and β are roots of ⇒ 6t + 8t + 3t + 4 = 0
2

− 2 log 2 + 2 log 3 = 0 px2 + qx + r = 0, p ≠ 0


⇒ (2t + 1) (3t + 4) = 0
− (log 2 + log 3) ± −q r
∴ α+β = , αβ = ⇒
1
t = − ,−
4
{(log 2 + log 3)2 − 4 log 2 p p 2 3
(− 2 log 2 + 2 log 3)} Since, p,q and r are in AP. 1 4
∴ x= ⇒ log a x = − , −
∴ 2q = p + r 2 3
2 log 2
1 1 α+β ∴ x = a−1 /2 , a−4 /3
Also, + =4 ⇒ =4
− (log 2 + log 3) α β αβ
−q
10 Since, x = 1 is a root of first equation. If
{(3 log 2)2 − 6 log 2 log 3 ⇒ α + β = 4αβ ⇒ =
4r
± α is another root of first equation, then
+ (log 3)2 } p p a−b
= α × 1= α =
2 log 2 Q 2q = p + r b −c
⇒ 2(−4r ) = p + r ⇒ p = −9r
− (log 2 + log 3) ± (3 log 2 − log 3) −q
(Product of roots)
= α+β = =
4r
=
4r
=−
4
2 log 2 Q
−9r 2a − 2b 2a − (a + c )
p p 9 = = =1
log 3 r r 1 2b − 2c a + c − 2c
∴ x = − 2, 1 − and αβ = = =
log 2 p −9r −9 So, the roots of first equation are 1 and
1.
3 Given, α and β are the roots of the Now, consider
(α − β )2 = (α + β )2 − 4αβ Since, the equations have a common root,
equation x2 − 6 x − 2 = 0 1 is also a root of second equation.
16 4 16 + 36
Qan = α n − β n , n ≥ 1 ⇒ a10 = α10 − β10 = + = ⇒ 2(c + a) + b + c = 0
81 9 81
a8 = α 8 − β 8 and a9 = α 9 − β 9 52 ⇒ 2(2b ) + b + c = 0
⇒ (α − β ) =
2

Now, consider 81 [since, a, b and c are in AP]


2 ⇒ c = − 5b
a10 − 2a8 [Qα and β are the ⇒ α −β = 13
9 Also, a + c = 2b ⇒ a = 2b − c
2a9 roots of = 2b + 5 b = 7b
7 α + β = −b / a and αβ = c / a ∴ a2 = 49b 2 , c 2 = 25 b 2
α10 − β10 − 2(α 8 − β 8 ) x2 − 6 x − 2 = 0
= Now, (1 + α + α2 )(1 + β + β2 ) Hence, a2 , c 2 and b 2 are in AP.
2(α 9 − β 9 ) or x2 = 6 x + 2 = 1 + (α + β ) + (α2 + β2 )
α 8(α2 − 2) − β 8(β2 − 2) ⇒ α2 = 6α + 2 + αβ + αβ(α + β) + (αβ) 2 11 On adding first and second equations,
= we get
2(α 9 − β 9 ) ⇒ α2 − 2 = 6α = 1 + (α + β ) + (α + β )2
2 x2 + (a + b )x + 27 = 0
and β2 = 6β + 2 − αβ + αβ(α + β) + (αβ) 2
α 8 ⋅ 6α − β 8 ⋅ 6β On subtracting above equation from
=
= 1 − + 2 − +    −  + 2
b b2 c c b c2
2(α 9 − β 9 ) ⇒ β2 − 2 = 6β] given third equation, we get
a a a  a  a a
x2 − 9 = 0 ⇒ x = 3, − 3
6α 9 − 6β 9 6 (a2 + b 2 + c 2 − ab − bc − ca)
= = =3 = Thus, common positive root is 3.
2(α 9 − β 9 ) 2 a2
Now, (3)2 + 3 a + 12 = 0 ⇒ a = − 7
= [(a − b ) + (b − c )2 + (c − a)2 ] / 2a2
2
and 9 + 3b + 15 = 0 ⇒ b = − 8
4 We have, x2 − 5x + 1 = 0 which is positive.
Hence, the order pair (a, b ) is (− 7, − 8).
1 1 8 Using AM ≥ GM
x+ = 5 ⇒ x2 + 2 = 52 − 2 = 23
12 Let y = x2 − 3 x + 4
2
x x −5 −4 −3 −3 −3
1 a + a + a + a + a x + 3x + 4
and x3 + 3 = 53 − 3 × 5 = 110 ⇒ ( y − 1)x2 + 3( y + 1)x + 4( y − 1) = 0
x + 1 + a8 + a10
∴ Q x is real.
Now,  x2 + 2   x3 + 3 
1 1 8
 x   x  ≥ (a−5 ⋅ a−4 ⋅ a−3 ⋅ a−3 ⋅ a−3 ⋅ 1 ⋅ a8 ⋅ a10 )1 / 8 ∴ D≥0
⇒ a−5 + a−4 + 3a−3 + 1 + a8 + a10 ⇒ 9( y + 1)2 − 16( y − 1)2 ≥ 0
= 23 × 110 = 2530 ⇒ −7 y 2 + 50 y − 7 ≥ 0
≥ 8⋅1
= 2530 −  x +  = 2525 ⇒ 7 y 2 − 50 y + 7 ≤ 0
1 1
⇒ x5 + Hence, minimum value is 8.
x5  x ⇒ ( y − 7)(7 y − 1) ≤ 0
DAY 43

⇒ y ≤ 7 and y ≥
1 Now, consider When x = 3,
7 n n x2 + 4 x − 60 = 9 + 12 − 60 = −39, which
1 lim ΣS r = lim Σ (α r
+β ) r
is not an even integer.
⇒ ≤ y≤7 n→ ∞ r = 1 n→ ∞ r = 1
7 Thus, in this case, we get x = 2
= (α + α2 + α3 + K ∞ )
Hence, maximum value is 7 and Hence, the sum of all real values of
+ (β + β2 + β3 + K ∞ )
1
minimum value is . α β α − αβ + β − αβ x = −10 + 6 + 4 + 1 + 2 = 3
7 = + =
1−α 1−β (1 − α )(1 − β ) 18 Given, 4 x2 − 16 x + λ = 0
13 Here, a ∈ R and equation is α + β − 2αβ 4
−3{ x − [ x]}2 + 2{ x − [ x]} + a2 = 0 = 16 ± (256 − 4λ )
1 − (α + β ) + αβ ∴ x=
Let t = x − [ x], then 8
1 4
−3t 2 + 2t + a2 = 0 + 8 ± (64 − λ )
= 15 375 =
1 ± 1 + 3a2 4
⇒ t = 1−
1

2
3 15 375 (64 − λ )
Q t = x − [ x] = { x} [fractional part] ⇒ α, β = 2 ±
25 + 4 4
∴ 0≤ t ≤ 1 =
375 − 25 − 2 Here, 64 − λ > 0
1 ± 1 + 3a2
⇒ 0≤ <1 29 1 ∴ λ < 64
3 = =
348 12 Also, 1 < α < 2 and 2 < β < 3
[Q 0 ≤ { x} < 1]
But 1 − 1 + 3a2 < 0 therefore 16 Here, [tan2 x] = integer 64 − λ
∴ 1< 2 − <2
and a = integer 4
1 + 1 + 3a 2
0≤ <1 So, tan x is also an integer.
3 64 − λ
Then, tan2 x − tan x − a = 0 and 2< 2 + <3
⇒ 1 + 3a < 2
2
⇒ a = tan x (tan x − 1) = I (I − 1) 4
⇒ 1 + 3a2 < 4 ⇒ a2 − 1 < 0 = Product of two 64 − λ
⇒ (a + 1)(a − 1) < 0 ⇒ − 1< − <0
consecutive integers 4
∴ a = 2, 6, 12, 20, 30, 42, 56, 72, 90 (64 − λ )
Hence, set S has 9 elements. and 0< <1
4
x2 + 4 x − 60
⇒ a ∈ (−1,1) 17 Given, ( x − 5x + 5)
2
=1 (64 − λ )
⇒ 1> >0
For no integral solution we consider the Clearly, this is possible when 4
interval (−1, 0) ∪ (0,1).
I. x2 + 4 x − 60 = 0 and (64 − λ )
0< <1
14 Given, a = cos 2 π + i sin 2 π
and
x2 − 5 x + 5 ≠ 0 4
7 7
or (64 − λ )
∴ a7 = cos 2 π + i sin 2 π = 1 i.e. 0< <1
[Qe iθ = cos θ + i sin θ] II. x2 − 5 x + 5 = 1 4
Also, α = a + a2 + a4 , or ⇒ 0 < (64 − λ ) < 4
β = a3 + a5 + a6 III. x − 5 x + 5 = −1 and
2 ⇒ 0 < 64 − λ < 16 ⇒ λ > 48
Then, the sum of roots, x 2 + 4 x − 60 = Even integer or 48 < λ < 64
S = α + β = a + a2 + a3 + a4 ∴ λ = {49, 50, 51, 52, ... , 63}
Case I When x2 + 4 x − 60 = 0
+ a5 + a6 19 Since, roots of ax2 + bx + c = 0 are α and
⇒ x2 + 10 x − 6 x − 60 = 0
a(1 − a ) a − a
6 7
β. Hence, roots of cx2 + bx + a = 0, will
⇒ S = = ⇒ x( x + 10) − 6( x + 10) = 0
1− a 1− a 1 1
⇒ ( x + 10)( x − 6) = 0 be and . Now, if we replace x by
a−1 α β
= = −1 [Q a7 = 1] ⇒ x = −10 or x = 6
1− a Note that, for these two values of x − 1, then roots of
1
and product of the roots, x, x2 − 5x + 5 ≠ 0 c ( x − 1)2 + b( x − 1) + a = 0 will be 1 +
α
P = αβ = (a + a2 + a4 ) (a3 + a5 + a6 ) Case II When x2 − 5x + 5 = 1 1 1
= a4 + a5 + 1 + a6 + 1 + a2 + 1 and 1 + . Now, again replace x by ,
⇒ x2 − 5x + 4 = 0 β x
+ a + a3 [Q a7 = 1] ⇒ x2 − 4 x − x + 4 = 0 we will get c (1 − x )2 + b(1 − x ) + ax2 = 0,
= 3 + (a + a2 + a3 + a4 + a5 + a6 ) ⇒ ( x − 4) ( x − 1) = 0 α β
=3−1=2 whose roots are and .
⇒ x = 4 or x = 1 1+ α 1+ β
Hence, the required quadratic equation
Case III When x2 − 5x + 5 = −1
is x2 + x + 2 = 0 20 Let f ( x ) = x2 − 2kx + k 2 + k − 5
⇒ x2 − 5x + 6 = 0
15 Since, α and β are the roots of ⇒ x2 − 2 x − 3 x + 6 = 0 Since, both roots are less than 5.
b
375 x − 25x − 2 = 0.
2
⇒ x( x − 2) − 3( x − 2) = 0 ∴ D ≥ 0, − < 5and f (5) > 0
2a
25 1 ⇒ ( x − 2)( x − 3) = 0
∴ α +β= = Here, D = 4k − 4(k 2 + k − 5)
2

375 15 ⇒ x = 2 or x = 3
= −4k + 20 ≥ 0
2 Now, when x = 2,
and αβ = − x2 + 4 x − 60 = 4 + 8 − 60 = −48, which ⇒ k≤5 ... (i)
375
is an even integer.
44 40


b
<5⇒ k<5 ... (ii) (m + 3) (m + 1) > 0 But,  a +   b +   c + 
1 1 1
2a ⇒ −∞ < m < −3 and  b  c  d
and f (5) > 0 −1 < m < ∞ ... (iv)  d + 1  = 4 × 1 × 4 × 1 = 16
⇒ 25 − 10k + k 2 + k − 5 > 0  
From (i), (ii), (iii) and (iv), we get  a
⇒ k 2 − 9k + 20 > 0 m lie between −1 and 3. 1 1 1 1
∴ a = ,b = ,c and d =
⇒ (k − 5)(k − 4) > 0 b c d a
22 Let z = x + iy , given Re(z ) = 1
⇒ k < 4 and k > 5 …(iii) 1 1 1 1
∴ x = 1 ⇒ z = 1 + iy ⇒ a = = 2, b = = , c = = 2
From (i), (ii) and (iii), we get b c 2 d
k<4 Since, the complex roots are conjugate
1 1
to each other. and d = =
21 Since, both roots of equation a 2
So, z = 1 + iy and 1 − iy are two roots of
1 1
x2 − 2mx + m2 − 1 = 0 are greater than −2 z2 + α z + β = 0. ⇒ a = 2, b = ,c = 2 and d =
but less than 4. 2 2
b Q Product of roots = β ⇒ a = c and b = d
∴ D ≥ 0, −2 < − < 4, ⇒ (1 + iy )(1 − iy ) = β
2a 1
∴ β = 1 + y 2 ≥ 1 ⇒ β ∈ [1, ∞ ) 25 We have, f ( x ) =
f (4) > 0 and f (−2) > 0 e + x
x 2
Now, D≥0 23 Given equation is e
⇒ 4m2 − 4m2 + 4 ≥ 0 e sin x − e − sin x = 4 ⇒ e sin x −
1
=4 Using AM ≥ GM , we get
⇒ 4> 0⇒m ∈R ... (i) e sin x 2
1
Let e sin x = t , then t −
=4 ex + x 1 /2

t e ≥  e x ⋅ 2  , as e x > 0
2  e 
x
⇒ t − 1 − 4t = 0 ⇒ t − 4t − 1 = 0
2 2

2
4 ± 16 + 4 ⇒ e + x ≥2 2
x
⇒ t = e
2
1 1
t = 2 ± 5 ⇒ e sin x = 2 ± 5 ⇒ 0< ≤
b 2
−2 < − <4 But −1 ≤ sin x ≤ 1 ⇒ e −1 ≤ e sin x ≤ e 1 ex + x 2 2
2a e
⇒ e sin x ∈  ,e 
1 1
 2m  ∴ 0 < f ( x) ≤ ,∀x∈R
⇒ −2 <  <4  e 
 2⋅ 1  2 2
Also, 0 < e < 2 + 5 Statement II is true and Statement I is
⇒ −2 < m < 4 ... (ii)
Hence, given equation has no solution. also true as for some ‘c’.
f (4) > 0 1
⇒ 16 − 8m + m2 − 1 > 0 24 Using AM > GM, we have ⇒ f (c ) = [for c = 0]
3
⇒ m2 − 8m + 15 > 0 a+
1
>2
a
, b + >2
1 b 1
⇒ (m − 3)(m − 5) > 0 b b c c which lies betwen 0 and .
2 2
⇒ −∞ < m < 3 and 5 < m < ∞ ... (iii) 1 c 1 d
c + >2 and d + > 2 So, Statement II is correct explanation
and f (−2) > 0 d d a a of Statement I.
⇒ 4 + 4m + m2 − 1 > 0  a + 1   b + 1   c + 1   d + 1  > 16
⇒ m2 + 4m + 3 > 0       
 b  c  d  a
DAY FIVE

Matrices
Learning & Revision for the Day

u Matrix u Algebra of Matrices u Trace of a Matrix


u Types of Matrices u Transpose of a Matrix u Equivalent Matrices
u Equality of Matrices u Some Special Matrices u Invertible Matrices

Matrix

A matrix is an arrangement of numbers in rows and columns.

A matrix having m rows and n columns is called a matrix of order m × n and the
number of elements in this matrix will be mn.
 a11 a12 a13 ... a1 n 
a a22 a23 ... a2 n 

A matrix of order m × n is of the form A =  21 
 ... ... ... ... ... 
a 
 m1 am2 am3 ... amn 

Some important terms related to matrices



The element in the ith row and jth column is denoted by aij.

The elements a11 , a22 , a33 , ...... are called diagonal elements.

The line along which the diagonal elements lie is called the principal diagonal or PRED
simply the diagonal of the matrix.
MIRROR
Your Personal Preparation Indicator

Types of Matrices u No. of Questions in Exercises (x)—



If all elements of a matrix are zero, then it is called a null or zero matrix and it is u No. of Questions Attempted (y)—
denoted by O. u No. of Correct Questions (z)—

A matrix which has only one row and any number of columns is called a row matrix (Without referring Explanations)
and if it has only one column and any number of rows, then it is called a column
matrix. u Accuracy Level (z / y × 100)—
u Prep Level (z / x × 100)—

If in a matrix, the number of rows and columns are equal, then it is called a square
matrix. If A = [aij]n × n , then it is known as square matrix of order n.
In order to expect good rank in JEE,

If in a matrix, the number of rows is less/greater than the number of columns, then it your Accuracy Level should be
above 85 & Prep Level should be
is called rectangular matrix. above 75.

If in a square matrix, all the non-diagonal elements are zero, it is called a diagonal
matrix.
46 40 FIVE

If in a square matrix, all non-diagonal elements are zero


Transpose of a Matrix

and diagonal elements are equal, then it is called a


scalar matrix. Let A be m × n matrix, then the matrix obtained by interchanging

If in a square matrix, all non-diagonal elements are zero the rows and columns of A is called the transpose of A and is
and diagonal elements are unity, then it is called an unit denoted by A′ or AC or A T .
(identity) matrix. We denote the identity matrix of order If A be m × n matrix, A′ will be n × m matrix.
n by I n and when order is clear from context then we
simply write it as I.
Important Results

In a square matrix, if aij = 0, ∀ i > j, then it is called an
(i) If A and B are two matrices of order m × n, then
upper triangular matrix and if aij = 0, ∀i < j , then it is ( A ± B)′ = A′ ± B ′
called a lower triangular matrix.
(ii) If k is a scalar, then (k A)′ = k A′
NOTE • The diagonal elements of diagonal matrix may or may (iii) ( A′ )′ = A
not be zero. (iv) ( AB)′ = B′ A′
(v) ( A n )′ = ( A′ )n
Equality of Matrices
Two matrices A and B are said to be equal, if they are of Some Special Matrices
same order and all the corresponding elements are equal.

A square matrix A is called an idempotent matrix,
if it satisfies the relation A2 = A.
Algebra of Matrices ●
A square matrix A is called nilpotent matrix of order k,

If A = [aij]m × n and B = [b ij] m × n be two matrices of same if it satisfies the relation A k = O, for some k ∈ N .
order, then A + B = [aij + b ij]m × n and A − B = [aij − b ij]m × n , ●
The least value of k is called the index of the nilpotent
where i = 1, 2, ..., m, j = 1, 2, ..., n. matrix A.

A square matrix A is called an involutary matrix,

If A = [aij] be an m × n matrix and k be any scalar, then, if it satisfies the relation A2 = I .
kA = [kaij]m × n . ●
A square matrix A is called an orthogonal matrix,

If A = [aij]m × n and B = [b ij]n × p be any two matrices such if it satisfies the relation AA′ = I or A′ A = I .
that number of columns of A is equal to the number of ●
A square matrix A is called symmetric matrix,
rows of B, then the product matrix AB = [c ij], of order if it satisfies the relation A′ = A.
n
m × p, where c ij = ∑a b kj . A square matrix A is called skew-symmetric matrix,

ik
k =1 if it satisfies the relation A′ = − A.

NOTE • If A and B are idempotent matrices, then A + B is idempotent


Some Important Properties iff AB = − BA.

A + B = B + A (Commutativity of addition)  a1 a2 a3 
• If A =  b1 b2 b3  is orthogonal, then

( A + B) + C = A + (B + C) (Associativity of addition)  
 c1 c 2 c 3 

α ( A + B) = αA + αB, where α is any scalar.
Σ ai2 = Σ bi2 = Σ c i2 = 1 and Σ ai bi = Σ bi c i = Σ ai c i = 0

(α + β) A = αA + βA, where α and β are any scalars.
• If A B are symmetric matrices of the same order, then

α (βA) = (αβ) A, where α and β are any scalars.
(i) AB is symmetric if and only if AB = BA .

( AB) C = A (BC) (Associativity of multiplication)
(ii) A ± B , AB + BA are also symmetric matrices.

AI = A = IA • If A and B are two skew-symmetric matrices, then

A (B + C) = AB + AC (Distributive property) (i) A ± B, AB − BA are skew-symmetric matrices.
NOTE • A2 = A ⋅ A, A3 = A ⋅ A ⋅ A = A2 ⋅ A1 , K (ii) AB + BA is a symmetric matrix.
• Every square matrix can be uniquely expressed as the sum of
• If the product AB is possible, then it is not necessary that symmetric and skew-symmetric matrices.
the product BA is also possible. Also, it is not necessary
1 1 1 1
that AB = BA. i.e. A = ( A + A′ ) + ( A − A′ ), where ( A + A′ ) and ( A − A′ )
2 2 2 2
• The product of two non-zero matrices can be a zero
are symmetric and skew-symmetric respectively.
matrix.
DAY 47

(iii) Addition of constant multiple of the elements of any row


Trace of a Matrix (column) to the corresponding elements of any other row
The sum of the diagonal elements of a square matrix A is (column), indicated as
called the trace of A and is denoted by tr( A). Ri → Ri + kR j (Ci → Ci + kC j).
(i) tr( λA) = λ tr( A) (ii) tr( A ) = tr( A ′ )
(iii) tr( AB) = tr( BA) Invertible Matrices

A square matrix A of order n is said to be invertible if there
exists another square matrix B of order n such that
Equivalent Matrices AB = BA = I .
Two matrices A and B are said to be equivalent, if one is ●
The matrix B is called the inverse of matrix A and it is
obtained from the other by one or more elementary operations denoted by A −1 .
and we write A ~ B.
Following types of operations are called elementary
operations. Some Important Results

Inverse of a square matrix, if it exists, is unique.
(i) Interchanging any two rows (columns).
This transformation is indicated by

AA −1 = I = A −1 A
Ri ↔ R j (Ci ↔ C j)

If A and B are invertible, then ( AB)−1 = B −1 A −1
(ii) Multiplication of the elements of any row (column) by a ●
( A − 1 )T = ( A T )− 1
non-zero scalar quantity, indicated as ●
If A is symmetric, then A −1 will also be symmetric matrix.
Ri → kRi (Ci → kCi ) ●
Every orthogonal matrix is invertible.

DAY PRACTICE SESSION 1

FOUNDATION QUESTIONS EXERCISE


0 1 0 −1 2 3
1 If A =   and B =  , then which of the following is  1 −2 3 
1 1 1 0  5 If A =   
 and B = 4 5, then
 −4 2 5 
correct? j
NCERT Exemplar 2 1 
(a) (A + B) ⋅ (A − B) = A + B
2 2
(b) (A + B) ⋅ (A − B) = A 2 − B 2 (a) AB, BA exist and are equal
(c) (A + B) ⋅ (A − B) = I (d) None of these (b) AB, BA exist and are not equal
(c) AB exists and BA does not exist
2 If p, q , r are 3 real numbers satisfying the matrix (d) AB does not exist and BA exists
3 4 1 
6 If ω ≠ 1 is the complex cube root of unity and matrix
equation, [ p q r ] 3 2 3 = [ 3 0 1 ], then 2p + q − r is
  ω 0 
2 0 2 H= 70
, then H is equal to
0 ω
equal to j
JEE Mains 2013
(a) H (b) 0 (c) −H (d) H 2
(a) − 3 (b) − 1 (c) 4 (d) 2
7 If A and B are 3 × 3 matrices such that AB = A and
3 In a upper triangular matrix n × n, minimum number of
BA = B, then
zeroes is
n (n − 1) n (n + 1) (a) A 2 = A and B 2 ≠ B (b) A 2 ≠ A and B 2 = B
(a) (b) (c) A 2 = A and B 2 = B (d) A 2 ≠ A and B 2 ≠ B
2 2
2n (n − 1) 8 For each real number x such that − 1 < x < 1, let
(c) (d) None of these
2
 1 −x 
1 2 a 0 1 − x 1− x x +y
4 Let A =  and B =  ; a , b ∈ N. Then, A( x ) =  and z =
 1 
. Then,
3 4 0 b  −x 1 + xy
 
(a) there exists more than one but finite number of B’s such 1 − x 1 − x 
that AB = BA (a) A (z) = A(x) + A(y)
(b) there exists exactly one B such that AB = BA (b) A(z) = A(x) [A(y)]−1
(c) there exist infinitely many B’s such that AB = BA (c) A (z) = A(x) ⋅ A(y)
(d) there cannot exist any B such that AB = BA (d) A(z) = A(x) − A(y)
48 40 FIVE

cos α − sin α 0 1 2 2
9 If A(α ) =  sin α cos α 0, then A(α ) A(β ) is equal to 18 If A =  2 1 − 2 and AT A = AAT = I , then xy is
   
 0 0 1  x 2 y 
(a) A(αβ) (b) A(α + β) (c) A(α − β) (d) None equal to
10 If A is 3 × 4 matrix and B is a matrix such that A′ B and (a) −1 (b) 1 (c) 2 (d) − 2
BA′ are both defined, then B is of the type 19 If A and B are symmetric matrices of the same order and
(a) 4 × 3 (b) 3 × 4 (c) 3 × 3 (d) 4 × 4 X = AB + BA and Y = AB − BA, then ( XY )T is equal to
1 2 2 (a) XY (b)YX
(c) − YX
11 If A = 2 1 −2 is a matrix satisfying the equation
 (d) None of these
  1 0 0 1 0 0
a 2 b 
20 Let A = 0 1 1 , I = 0 1 0 and
 
AAT = 9I , where I is 3 × 3 identity matrix, then the ordered    
pair (a,b) is equal to j JEE Mains 2015 0 −2 4 0 0 1 
(a) (2, − 1) (b) (−2, 1) (d) (−2, − 1) 1 2 
(c) (2, 1) A −1 = ( A + cA + dI ) . The values of c and d are
 6 
0 0 1  1 0 0
12 If E = 0 0 1  and F = 0 1 0, then E 2 F + F 2E (a) (− 6, − 11) (b) (6, 11)
    (c) (− 6, 11) (d) (6, − 11)
0 0 0 0 0 1 
21 Elements of a matrix A of order 9 × 9 are defined as
(a) F (b) E (c) 0 (d) None
aij = ωi + j (where ω is cube root of unity), then trace ( A ) of
13 If A and B are two invertible matrices and both are the matrix is
symmetric and commute each other, then (a) 0 (b) 1 (c) ω (d) ω2
−1 −1 −1
(a) both A B and A B are symmetric
 1 −1 1   4 2 2
(b) neither A −1B nor A −1B −1 are symmetric 1 
(c) A −1B is symmetric but A −1B −1 is not symmetric 22 If A =  2 1 − 3  and A −1 = −5 0 α , then
  10  
(d) A −1B −1 is symmetric but A −1B is not symmetric  1 1 1   1 − 2 3 
14 If neither α nor β are multiples of π /2 and the product AB α is equal to
of matrices (a) − 2 (b) 5 (c) 2 (d) − 1
 cos 2 α sin α cos α  −1
A= 23 If A is skew-symmetric and B = (I − A ) (I + A ) , then B is
 cos α sin α sin2 α  (a) symmetric
 cos 2 β cos β sin β  (b) skew-symmetric
and B=
 cos β sin β sin2 β  (c) orthogonal
(d) None of the above
is null matrix, then α − β is
24 Let A be a square matrix satisfying A 2 + 5A + 5I = O .
(a) 0 (b) multiple of π
(c) an odd multiple of π/ 2 (d) None of these The inverse of A + 2I is equal to
(a) A − 2I (b) A + 3I
 1 2 3
(c) A − 3I (d) does not exist
15 The matrix  1 2 3 is
   1 0
−1 −2 −3 25 Let A =  48
. Then A is
1 / 3 1 
(a) idempotent (b) nilpotent
 1 0  1 0
(c) involutary (d) orthogonal (a)  (b)  3 1 − 1  1
48
1  2 
cos θ − sin θ   (1 / 3) 3 48  
16 If A =  , then
 sin θ cos θ   1 0
(c)  (d) None of these

16 1
(a) A is skew-symmetric (b) symmetric
(c) idempotent (d) orthogonal 26 If X is any matrix of order n × p and I is an identity matrix
 a a −1 −2  2 of order n × n, then the matrix M = I − X ( X ′ X )−1 X ′ is

17 If A = a + 1 1 a + 4 is symmetric, then a is
2 I. Idempotent matrix
  II. MX = O
 −2 4a 5 
(a) Only I is correct (b) Only II is correct
(a) −2 (b) 2 (c) −1 (d) None (c) Both I and II are correct (d) None of them is correct
DAY 49

27 Let A and B be two symmetric matrices of order 3. 28 Consider the following relation R on the set of real square
Statement I A (BA) and ( AB ) A are symmetric matrices of order 3.
matrices. R = {( A, B ) : A = P −1BP for some invertible matrix P}
Statement I R is an equivalence relation.
Statement II AB is symmetric matrix, if matrix
Statement II For any two invertible 3 × 3 matrices M
multiplication of A with B is commutative.
and N,(MN )−1 = N −1M −1.
(a) Statement I is true, Statement II is true; Statement II is a
correct explanation for Statement I (a) Statement I is false, Statement II is true
(b) Statement I is true, Statement II is true; Statement II is
(b) Statement I is true, Statement II is true; Statement II is
correct explanation of Statement I
not a correct explanation for Statement I
(c) Statement I is true, Statement II is true; Statement II is
(c) Statement I is true; Statement II is false
not a correct explanation of Statement I
(d) Statement I is false; Statement II is true (d) Statement I is true, Statement II is false

DAY PRACTICE SESSION 2

PROGRESSIVE QUESTIONS EXERCISE


2 1 
6 If both A −
1 1
I and A + I are orthogonal matrices, then
1 If A =   , then I + 2A + 3A + ... ∞ is equal to
2

− 4 − 2  2 2
(a) A is orthogonal
 4 1  3 1  5 2  5 2 (b) A is skew-symmetric matrix
(a)   (b)   (c)  − 8 − 3  (d)  − 3 – 8 
 − 4 0  − 4 − 1     (c) A is symmetric matrix
(d) None of the above
 1 2
2 The matrix A that commute with the matrix   is  −1 + i 3 −1 − i 3 
 3 4
 2i 2i 
1  2a 2b  1  2b 2a  7 If A =  , i = −1 and f ( x ) = x 2 + 2,
(a) A =   (b) A =    1 + i 3 1−i 3 
2  3b 2a + 3b  2  3a 2a + 3b 
 
1  2a + 3b 2a   2 i 2i 
(c) A =   (d) None of these
3  3a 2a + 3b  then f ( A ) is equal to

3 The total number of matrices that can be formed using 5 5 − i 3   1 0  3 − i 3   1 0


(a)   (b)   
different letters such that no letter is repeated in any  2   0 1  2   0 1
matrix, is 1 0  1 0
(c)  (d) (2 + i 3 ) 
(a) 5! (b) 2 × 5 5 0 1 
 0 1
(c) 2 × (5!) (d) None of these
1 0
4 If A is symmetric and B is a skew-symmetric matrix, then 8 If A =  n
 , then A is equal to
for n ∈ N, which of the following is not correct? 1 1 
(a) A n is symmetric (a) 2 n − 1 A − (n − 1) I (b) nA − (n − 1) I
(b) B n is symmetric if n is even (c) 2 n − 1 A + (n − 1) I (d) nA + (n − 1) I
(c) A n is symmetric if n is odd only  cos θ sin θ 
(d) B n is skew-symmetric if n is odd 9 Let A =  . Let A = [bij ]2 × 2 . Define
n

− sin θ cos θ 
2 1 5 4  An 
5 Consider three matrices X =  , Y = 6 5 and lim A n = lim [bij ]2 × 2 . Then lim   is
4 1   n→ ∞ n→ ∞ n→ ∞  n 

 5 − 4
Z = . Then, the value of the sum
(a) zero matrix (b) unit matrix
− 6 5   0 1
(c)  (d) limit does not exist

 X (YZ )2   X (YZ )3   −1 0
 XYZ 
tr ( X ) + tr   + tr   + tr   + K to ∞ is
 2   4   8  10 If B is skew-symmetric matrix of order n and A is n × 1
column matrix and AT BA = [ p ], then
(a) 6 (b) 9
(c) 12 (d) None of these (a) p < 0 (b) p = 0
(c) p > 0 (d) Nothing can be said
50 40 FIVE

11 If A, B and A + B are idempotent matrices, then AB is 14 If A1, A3 , ..., A2 n − 1 are n skew-symmetric matrices of
n
equal to
(a) BA (b) − BA (c) I (d) O
same order, then B = ∑ ( 2r − 1) ( A
r =1
2r − 1 )2 r −1
will be

 3 1  (a) symmetric
 2  , A = 1 1 and Q = PAP T , then P T Q 2019P (b) skew-symmetric
12 If P =  2  0 1
  (c) neither symmetric nor skew-symmetric
− 1 3
 2 (d) data not adequate
2 
is equal to a b c 
 1 2019 15 Let matrix A = b c a , where a, b, c are real positive
(a)   
0 1  c a b 
 4 + 2019 3 6057  numbers with abc = 1. If AT A = I , then a 3 + b 3 + c 3 is
(b)  
 2019 4 − 2019 3  (a) 3 (b) 4
(c) 2 (d) None of these
1 2 + 3 1 
(c)   16 If A is an 3 × 3 non-singular matrix such that AA′ = A′ A
4 −1 2 − 3
and B = A −1A′, then BB ′ equals j JEE Mains 2014
1  2019 2 − 3 −1
(d)   (a) (B )′ (b) I + B
4 2 + 3 2019  (c) I (d) B −1
13 Which of the following is an orthogonal matrix? 17 A is a 3 × 3 matrix with entries from the set {−1, 0, 1} . The
6 2 −3  6 2 3  probability that A is neither symmetric nor
(a)
1 2 3 6 (b)
1 2 −3 6  skew-symmetric is
7   7  
 3 −6 2   3 6 − 2  39 − 36 − 33 + 1 39 − 36 − 33
(a) (b)
 −6 −2 −3  6 −2 3 39 39
(c)
1 2 3 6 (d)
1 2 2 −3  39 − 36 + 1 39 − 33 + 1
7   7   (c) (d)
 −3 6 2   −6 2 3  39 39

ANSWERS
SESSION 1 1. (d) 2. (a) 3. (a) 4. (c) 5. (b) 6. (a) 7. (c) 8. (c) 9. (b) 10. (b)
11. (d) 12. (b) 13. (a) 14. (c) 15. (b) 16. (d) 17. (b) 18. (c) 19. (c) 20. (c)
21. (a) 22. (b) 23. (c) 24. (b) 25. (c) 26. (c) 27. (b) 28. (c)

SESSION 2 1. (c) 2. (a) 3. (c) 4. (c) 5. (a) 6. (b) 7. (d) 8. (b) 9. (a) 10. (b)
11. (b) 12. (a) 13. (a) 14. (b) 15. (d) 16. (c) 17. (a)
DAY 51

Hints and Explanations


SESSION 1 5 Here, A is 2 × 3 matrix and B is 3 × 2 A ( x ) ⋅ A( y ) =
1

matrix. (1 − x ) (1 − y )
1 Here,
0 1   0 −1  0 0 ∴Both AB and BA exist, and AB is a 2 × 2  1 − x  1 −y
A+ B = + = matrix, while BA is 3 × 3 matrix. − x 1  − y 1 
1 1 1 0  2 1  
 ∴ AB ≠ BA.
1   0 −1  0 1
0 2 = ⋅
A−B = − = 6 Clearly, (1 − x ) (1 − y )
1 1 1 0   0 1

ω 0 ω 0  ω2 0   1 + xy − ( x + y )
0 1 0 1  H2 = = …(iv)
A2 = A ⋅ A =  0 ω 0 ω   0 ω2   − ( x + y ) 1 + xy 
1 1 1 1       
  
ω 2
0   ω 0   ω 3
0 From Eqs. (iii) and (iv), we get
 0 + 1 0 + 1 1 1 H3 =  =
= = 2   0 ω 3 A(z ) = A( x ) ⋅ A( y ).
 0 + 1 1 + 1  1 2  0 ω    0 ω 
  
ω70 0  ω 69 ⋅ ω 0  cos α − sin α 0
 0 − 1  0 −1 ∴ H =
70
=
and B 2 = B ⋅ B = ⋅ 70  9 A(α ) A(β ) =  sin α cos α 0
1 0  1
   0   0 ω   0 ω 69 ⋅ ω   
(ω3 )23 ⋅ ω   0 0 1
 −1 0  0
= = cos β − sin β 0
 0 −1
   0 (ω ) ⋅ ω 
3 23
×  sin β cos β 0
1 1  −1 0   0 1 ω 0   
∴ A2 + B 2 = + = =
 0 ω
=H [Qω3 = 1]  0 0 1
1 2  0 −1 1 1  
      cos(α + β ) − sin (α + β ) 0
1 1  −1 0  2 1 7 Since, AB = A =  sin(α + β ) cos(α + β ) 0
A2 − B 2 = − =  
1 2  0 −1 1 3
      ∴ B =I ⇒ B2 = B  0 0 1
 0 0  0 2 = A(α + β )
and ( A + B )( A − B ) = Similarly, BA = B
2 1   0 1
   10 Clearly, order of A′ is 4 × 3.
⇒ A=I
 0 + 0 0 + 0  0 0 Now, for A ′ B to be defined, order of B
= = ⇒ A2 = A
 0 + 0 4 + 1   0 5 should be 3 × m and for BA′ to be
   
defined, order of B should be n × 4.
Clearly, ( A + B )( A − B ) ≠ A2 − B 2 Hence, A2 = A and B 2 = B
Thus, for both A ′ B and BA′ to be
≠ A + B ≠ I.
2 2
defined, order of B should be 3 × 4.
8 We have,
2 [3 p + 3q + 2r , 4 p + 2q + 0, 1  1 − x 1 2 2
A ( x) = …(i)
p + 3q + 2r ] = [3 0 1] 1 − x  − x 1  11 Given, A =  2 1 −2
 
⇒ 3 p + 3q + 2r = 3, 4 p + 2q = 0, 1  1 −y a 2 b
∴ A( y ) = …(ii)
p + 3q + 2r = 1 1 − y  − y 1  1 2
a
⇒ p = 1,q = −2, r = 3
1 ⇒ A T = 2 1
2
∴ 2 p + q − r = 2 − 2 − 3 = −3 and A (z ) = 
1−
(x + y) 2 −2
b
3 We know that, a square matrix A = [aij] 1 + xy  1 2 2 1 2 a
is said to be an upper triangular matrix  ( x + y ) Now, AA T =  2 1 −2 2 1 2
if aij = 0, ∀ i > j. 1 −    
 1 + xy 
Consider, an upper triangular matrix  (x + y)  a 2 b  2 −2 b 
− 1   9 0 a + 4 + 2b 
1 2 3  1 + xy
  = 2a + 2 − 2b 
A =  0 5 6  ( x + y ) 
0 9

  −  a + 4 + 2b 2a + 2 − 2b a + 4 + b 
1 2 2
 0 0 7 3 × 3 1 + xy  1 + xy 
=  
1 + xy − x − y − ( x + y ) It is given that, AA T = 9I
3(3 − 1) 1 
Here, number of zeroes = 3 = 1 + xy  9 0 a + 4 + 2b 
2  
∴ Minimum number of zeroes  ( x + y ) ⇒ 0 9 2a + 2 − 2b 
1 −  
n (n − 1) 1 + xy  1 + xy   a + 4 + 2b 2a + 2 − 2b a + 4 + b 
2 2
= =  
2 (1 − x ) (1 − y ) − ( x + y ) 1 
1 2   a 0  a 2b  1 + xy  9 0 0
4 Clearly, AB =  =  
    1 =  0 9 0
3 4  0 b  3a 4 b  =  
 a 0 1 2   a 2 a  (1 − x ) (1 − y )  0 0 9
and BA = =  1 + xy − ( x + y )
 0 b  3 4 3b 4 b  On comparing, we get
      − ( x + y ) 1 + xy 
…(iii)
If AB = BA, then a = b .   a + 4 + 2b = 0
Hence, AB = BA is possible for Now, consider ⇒ a + 2b = −4 …(i)
infinitely many values of B’s. 2a + 2 − 2b = 0
52 40 FIVE

⇒ a − b = −1 …(ii)  cos θ sin θ  1 0 1 + c + d − 6 0


= =I
a2 + 4 + b 2 = 9  − sin θ cos θ  0 1  ⇒ 0 − 11 − c + d − 6
and …(iii)     
On solving Eqs. (i) and (ii), we get ∴ A is orthogonal.  0 − 38 − 10c − 2d
a = −2,b = −1 0 
17 A is symmetric
This satisfies Eq. (iii) also. 19 + 5c + d 
Hence, (a,b ) ≡ (−2,−1) ⇒ a − 1 = a + 1, a + 4 = 4a
2 2

46 + 14c + 4 d − 6
⇒ a2 − a − 2 = 0, a2 − 4a + 4 = 0
12 F is unit matrix ⇒ F 2 = F  0 0 0
⇒ a = 2.
and E2 F+ F 2 E = E2 + E =  0 0 0
0 0 1  0 0 1 18 Since, A is orthogonal, each row is  
orthogonal to the other rows.  0 0 0
Also, E 2 =  0 0 1 ×  0 0 1
    ⇒ R1 ⋅ R3 = 0 ⇒ 1 + c + d − 6 = 0;
0 0 0  0 0 0
⇒ x + 4 + 2y = 0 − 11 − c + d − 6 = 0
 0 0 0 Also, R2 ⋅ R3 = 0 ⇒ c + d = 5; − c + d = 17
=  0 0 0 On solving, we get c = − 6, d = 11.
⇒ 2x + 2 − 2y = 0
  These value also satisfy other equations.
 0 0 0 On solving, we get x = − 2, y = − 1
∴ E 2 + E = E. ∴ xy = 2 21 Clearly, tr ( A ) = a11 + a22 + a33 + a44
19 Since, A and B are symmetric matrices + a55 + a66 + a77 + a88 + a99
13 Consider, ( A −1 B )T = B T ( A −1 )T = ω2 + ω 4 + ω 6 + ω 8 + ω10 + ω12
∴ X = AB + BA
= B T ( A T )−1 = B A −1 will be a symmetric matrix and + ω14 + ω16 + ω18
[Q A T = A and B T = B ] Y = AB − BA will be a skew-symmetric = (ω + ω + 1) + (ω + ω + 1)
2 2

= A −1 B matrix.
+ (ω2 + ω + 1) [Qω3 n = 1, n ∈ N ]
−1 −1 Thus, we get X T = X and Y T = − Y
[Q AB = BA ⇒ A ( AB )A = 0+ 0+ 0 [Q1 + ω + ω2 = 0]
Now, consider ( XY )T = Y T X T
= A −1 (BA ) A −1 ⇒ BA −1 = A −1 B] =0
= (− Y )( X ) = − YX
⇒ A −1 B is symmetric.
22 Clearly, AA −1 = I
Now, consider 20 Clearly, 6 A −1 = A2 + cA + dI
Now, if R1 of A is multiplied by C3 of
( A −1 B −1 )T = ((BA )−1 )T ⇒ (6 A −1 )A = ( A2 + cA + dI ) A
T A −1 , we get 2 − α + 3 = 0 ⇒ α = 5
= (( AB )−1 ) [Q AB = BA] [Q Post multiply both sides by A]
= (B −1
A −1 T
) = (A −1 T
) (B −1 T
) ⇒ 6( A −1 A ) = A3 + cA2 + dIA 23 Consider,
T −1 T −1 −1 −1 ⇒ 6I = A + cA + dA
3 2 BB T = (I − A )−1 (I + A ) (I + A )T [(I − A )−1 ] T
= (A ) (B ) = A B
[Q A −1 A = I and IA = A ] = (I − A )−1 (I + A ) (I − A ) (I + A )−1
⇒ A −1 B −1 is also symmetric.
⇒ A + cA + dA − 6I = O
3 2
…(i) = (I − A )−1 (I − A ) (I + A ) (I + A )−1
 
14 AB =  cos α cos α2sin α 
2

1 0 0 = I⋅I = I
cos α sin α
 sin α 
Here, A2 = A ⋅ A =  0 1 1 × Hence, B is an orthogonal matrix.
 cos 2 β cos β sin β   
× 0 −2 4 24 We have, A2 + 5A + 5I = O
cos β sin β sin2 β 
1 0 0 1 0 0 ⇒ A2 + 5A + 6I = I
cos α cos β cos(α − β) 0 1
=
 sin α cos β cos(α − β ) 1 =  0 − 1 5  ⇒ ( A + 2I ) ( A + 3I ) = I
    
cos α sin β cos(α − β )  0 −2 4  0 −10 14 ⇒ A + 2I and A + 3I are inverse of
1 0 0 each other.
sin α sin β cos(α − β ) 
 0 0 and A3 = A2 ⋅ A =  0 −1 5  ×
25 If A =    
1 0 1 0
=   , then A2 =
 0 0  2a 1 
   0 −10 14  a 1   
⇒ cos(α − β ) = 0 1 0 0 1 0 0  1 0  1 0
⇒ α − β = (2n + 1) π / 2 A3 = , …, A n =
 0 1 1  =  0 −11 19  3a 1   na 1 
       
 1 2 3
15 Let A =  1 2 3   0 −2 4  0 −38 46 Here, a = 1 / 3,
  Now, from Eq. (i), we get  1 0
 − 1 −2 − 3 ∴ A 48 =
16 1 
1 0 0 1 0 0  
 0 0 0  0 −11 19  + c  0 −1 5 
    26 We have, M = I − X ( X ′ X )−1 X ′
Then, A2 =  0 0 0
   0 −38 46  0 −10 14 = I − X ( X −1 ( X ′ )−1 )X ′
 0 0 0 1 0 0 [Q( AB )−1 = B −1 A −1 ]
Hence, A is nilpotent matrix of index 2. + d 0 1 1 = I − ( XX −1 ) (( X ′ )−1 X ′ )
 
cos θ sin θ 
16 A ′ =  
≠ A or − A. 0 −2 4 [by associative property]
 − sin θ cos θ 1 0 0  0 0 0 =I −I ×I [Q AA −1 = I = A −1 A]
cos θ − sin θ − 6  0 1 0 =  0 0 0 =I −I [Q I 2 = I ]
A A′ =    
 sin θ cos θ 
   0 0 1  0 0 0 =O
DAY 53

2 Let A = 
a b
Clearly, M 2 = O = M ⇒  A ′ − I   A − I  = I
1 1
 be a matrix that …(i)
So, M is an idempotent matrix. Also, c d   2   2 
MX = O .  1 2 1 ′
commute with   . Then, and  A + I   A + 1 I = I
 
27 Given, A = A and B = B
T T  3 4  2   2 
Statement I [ A (BA )] T = (BA )T ⋅ A T  a b   1 2  1 2  a b   A′ + 1 I   A + 1 I  = I
    =    ⇒     …(ii)
T T T  c d   3 4  3 4  c d   2   2 
= (A B )A
 a + 3b 2a + 4 b  From Eq. (i), we get
= (AB) A = A (BA) ⇒  
 c + 3d 2c + 4 d  1 1 1
So, A(BA ) is symmetric matrix. A ′ A − IA ′ − IA + I = I
Similarly, ( AB ) A is symmetric matrix.  a + 2c b + 2d  2 2 4
= 
Hence, Statement I is true. Also,  3a + 4c 3b + 4d  1 1 1
⇒ A ′ A − A ′− A + I = I …(iii)
Statement II is true but not a correct On equating the corresponding 2 2 4
explanation of Statement I. elements, we get Similarly, from Eq. (ii), we get
28 Given, R = {( A, B ) : A = P −1 BP for a + 3b = a + 2c ⇒ 3b = 2c …(i) 1 1 1
2a + 4b = b + 2d ⇒ 2a + 3b = 2d …(ii) A ′ A + A ′ + A + I = I …(iv)
some invertible matrix P} 2 2 4
For Statement I c + 3d = 3a + 4c ⇒ a + c = d …(iii)
On subtracting Eq. (iii) from Eq. (iv), we
(i) Reflexive ARA 2c + 4d = 3b + 4d ⇒ 3b = 2c …(iv)
get
Thus, A can be taken as
⇒ A = P −1 AP A + A′ = O
 a b  1  2a 2b 
which is true only, if P = I .  3b 3  = A′ = − A
 a + b  2  3b 2a + 3b  or
Thus, A = P −1 AP for some  2 2  Hence, A is a skew-symmetric matrix.
invertible matrix P.
3 Clearly, matrix having five elements is  ω ω2 
So, R is Reflexive. of order 5 × 1 or 1 × 5.  i  =ω 1 ω
(ii) Symmetric ∴Total number of such matrices = 2 × 5!.
7 We have, A =  i 2   − ω − 1
−ω
ARB ⇒ A = P −1 BP − ω  i  
4 ( A n )′ = ( A AL A )′ = ( A ′ A ′L A ′ )  i i 
⇒ PAP −1 = P (P −1 BP ) P −1 1 − ω2 0 
= ( A ′ )n = A n for all n ∴ A2 = − ω2 
⇒ PAP −1 = ( PP −1 ) B( PP −1 )
∴ A is symmetric for all n ∈ N .
n  0 1 − ω2 
∴ B = PAP −1 Also, B is skew-symmetric  − ω2 + ω 4 0 
−1
= 4
Now, let Q = P ⇒ B ′ = − B.  0 − ω 2
+ ω 
Then, B = Q −1 AQ ⇒ BRA ∴ (B n )′ = (B B L B )′ = (B ′ B ′ L B ′ )  − ω2 + ω 
0
=
⇒ R is symmetric. = (B ′ )n
 0 − ω2 + ω 
(iii) Transitive ARB and BRC = (− B )n = (−1)n B n . Q f ( x) = x + 2
2
[given]
⇒ A = P −1 BP ⇒ B n is symmetric if n is even and is ∴ f ( A ) = A2 + 2I
and B = Q −1CQ skew-symmetric if n is odd.
 − ω2 + ω 0  2 0
5 4  5 − 4 1 0 =
⇒ A = P −1 (Q −1CQ ) P 5 Here, YZ =  = − ω 2
+ ω  +  0 2
 6 5  − 6 5   0 1   0   
= (P −1Q −1 )C (QP )
 X (YZ )2  1 0
= (− ω + ω + 2)
2
∴ tr ( X ) + tr 
= (QP )−1 C (QP ) XYZ   0 1
 + tr    
 2   4 
So, ARC.  1 0
 X (YZ )3  = (3 + 2ω )
⇒ R is transitive + tr   0 1
 +K  
So, R is an equivalence relation.  8 
1 0
= (2 + i 3 )
= tr ( X ) + tr   + tr   + K
For Statement II It is always true X X  0 1
that (MN )−1 = N −1 M −1  
2  4
8 A2 =     
Hence, both statements are true but 1 1 1 0 1 0 1 0
= tr ( X ) + tr ( X ) + tr ( X ) + K =
   
second is not the correct 2 4 1 1  1 1  2 1 
explanation of first.
= tr ( X ) 1 + + 2 + K
 1 1
  1 0 1 0 1 0
2 2 A3 = =
SESSION 2 2 1  1 1  3 1 
1     
= tr ( X )
1 Clearly, A2 =   
2 1 2 1 1 ....................
1−
 
 − 4 − 2  − 4 − 2 2 ....................
 0 0 = 2 tr ( X ) = 2 (2 + 1) = 6  1 0
==O An =
 0 0  n 1
  6 Since, both A − 1 I and A + 1 I are  
2 2
∴ I + 2 A + 3 A2 + ... = I + 2 A  n 0  n − 1 0 
orthogonal, therefore, we have = −
 A − 1 I ′  A − 1 I = I  n n  0 n − 1
1 0  4 2  5 2   
= + =    
 0 1   − 8 − 4  − 8 − 3  2   2  = nA − (n − 1) I
     
54 40 FIVE

cos θ sin θ cos θ sin θ


9 A2 =  



13 We know that a matrix  a2 + b 2 + c 2 ab + bc + ca
 − sin θ cos θ  − sin θ cos θ  a1 a2 a3  =  ab + bc + ca b 2 + c 2 + a2

 cos 2θ sin 2θ  A = b1 b2 b3  will be orthogonal if  ab + bc + ca ab + bc + ca
=   
 − sin 2θ cos 2θ ac + ab + bc 
  c 1 c 2 c 3 
 cos 3θ sin 3θ  AA ′ = I , which implies ab + bc + ca 
Similarly, A =3
etc 
 − sin 3θ cos 3θ Σ a2i = Σb 2i = Σc 2i = 1 a2 + b 2 + c 2 
 
 cos nθ sin nθ  and Σ aib i = Σb ic i = Σc i ai = 0 A T A = I ⇒ a2 + b 2 + c 2 = 1
∴ An =
 − sin nθ cos nθ Now, from the given options, only
  and ab + bc + ca = 0
 6 2 −3
b11 b12  1 Since a, b, c > 0,
=
b  2 3 6  satisfies these conditions. ∴ ab + bc + ca ≠ 0 and hence no real
 21 b22  7 
 3 − 6 2  value of a3 + b 3 + c 3 exists.
A n  0 0 b ij
Now, lim = as lim =0
n→ ∞  0 0 n→∞ n  6 2 −3 16 AA ′ = A ′ A, B = A −1 A ′.
n   1
Hence, 2 3 6  is an orthogonal
7  BB ′ = ( A −1 A ′ ) ( A −1 ⋅ A ′ )′
10 A T B A = [ p] ⇒ ( A T BA )T = [ p] T = [ p]  3 −6 2 
matrix. = ( A −1 A ′ ) [( A ′ )′ ( A −1 )′ ]
⇒ A T B T A = A T (− B ) A = [ p] = ( A −1 A ′ ) [ A( A ′ )−1 ]
14 We have,
⇒ [− p] = [ p] ⇒ p = 0. [Q ( A −1 )′ = ( A ′ )−1 ]
B = A1 + 3 A33 + K + (2n − 1) A 22 nn −− 11
11 Since, A, B and A + B are idempotent = A −1
(A′ A) (A′ ) −1
matrix Now, B T = ( A1 + 3 A33
= A −1 ( AA ′ ) ( A ′ )−1 [Q A ′ A = AA ′ ]
∴ A2 = A; B 2 = B and ( A + B )2 = A + B + K + (2n − 1) A22nn −− 11 )T
= ( A −1 A ) [ A ′ ( A ′ )−1 ]
Now, consider ( A + B )2 = A + B
= A1T + (3 A33 )T + K + ((2n − 1) A22nn −− 11 )T =I⋅I =I
⇒ A2 + B 2 + AB + BA = A + B
⇒ A + B + AB + BA = A + B = A1T + 3( A3T )3 17 Total number of matrices = 39. A is
⇒ AB = − BA + K + (2n − 1) ( A 2T n − 1 )2 n − 1 symmetric, then aij = a ji .
Now, 6 places (3 diagonal, 3
12 P is orthogonal matrix as P P = I T =− A− 3 A33 − K − (2n − 1) A22nn −− 11
non-diagonal), can be filled from any of
Q 2019 = (PAP T ) ( PAP T ) [Q A1 , A3 , K , A 2 n − 1 are skew- −1, 0, 1 in 36 ways. A is skew-symmetric,
...(PAP T ) = PA2019P T symmetric matrices then diagonal entries are ‘o’ and
∴ ( A i )T = − A i ∀ i = 1, 3, 5, .... 2n − 1 ] a12 , a13 , a23 can be filled from any of
∴ P Q T 2019
P = P ⋅ PA
T 2019
P ⋅P = A
T 2019
−1, 0, 1 in 33 ways. Zero matrix is
= − [ A + 3 A33 + K + (2n − 1) A22nn −− 11 ]
1 1 1 1 1 2 common.
Now, A = 2
= =−B
 0 1  0 1  0 1 ∴Favourable matrices are
    
Hence, B is a skew-symmetric matrix. 39 − 36 − 33 + 1.
1 2 1 1 1 3 a b c  a b c 
A3 = = Hence, required probability
 0 1  0 1  0 1
     15 A T A = b c a × b c a 39 − 36 − 33 + 1
    =
1 2019 c a b c a b 39
⇒ A2019 =
0 1 

DAY SIX

Determinants
Learning & Revision for the Day

u Determinants u Area of Triangle by using u Inverse of a Matrix


u Properties of Determinants Determinants u Solution of System of Linear
u Cyclic Determinants u Minors and Cofactors Equations in Two and Three
u Adjoint of a Matrix Variables

Determinants
Every square matrix A can be associated with a number or an expression which is called its
determinant and it is denoted by det (A) or |A| or ∆ .
 a11 a12 ... a1 n  a11 a12 ... a1 n
a a22 ... a2 n  a a22 ... a2 n
If A =  21  , then det ( A) = 21
 M M M M  M M M M
a 
 n1 an2 ... a nn  an1 an2 ... ann

a b  a b

If A =   , then | A| = = ad − bc
c d  c d
a b c  a b c
PRED

If A =  p q r  , then | A| = p q r
 
MIRROR
Your Personal Preparation Indicator
u v w  u v w
q r p r p q u No. of Questions in Exercises (x)—
=a −b +c [expanding along R1 ] u No. of Questions Attempted (y)—
v w u w u v
u No. of Correct Questions (z)—
= a (qw − vr ) − b ( pw − ur ) + c ( pv − uq ) (Without referring Explanations)
There are six ways of expanding a determinant of order 3 corresponding to each of three rows
(R1 , R2 , R3 ) and three columns (C1 , C2 , C3 ). u Accuracy Level (z / y × 100)—
u Prep Level (z / x × 100)—
NOTE • Rule to put + or − sign in the expansion of determinant + − +
In order to expect good rank in JEE,
of order 3. − + − your Accuracy Level should be
above 85 & Prep Level should be
• A square matrix A is said to be singular, if | A| = 0 + − + above 75.
and non-singular, if | A| ≠ 0.
56 40 SIX

1 x x3
Properties of Determinants
(ii) 1 y y3 = ( x − y) ( y − z) (z − x)( x + y + z)
(i) If each element of a row (column) is zero, then ∆ = 0.
1 z z3
(ii) If two rows (columns) are proportional, then ∆ = 0.
(iii) | A T| = | A|, where A T is a transpose of a matrix. 1 x2 x3
(iv) If any two rows (columns) are interchanged, then ∆ (iii) 1 y2 y3 = ( x − y) ( y − z) (z − x)( xy + yz + zx)
becomes −∆. 1 z2 z3
(v) If each element of a row (column) of a determinant is a b c
multiplied by a constant k, then the value of the new
determinant is k times the value of the original (iv) b c a = − (a + b + c)(a 2 + b 2 + c2 – ab – bc – ca)
determinant c a b
= − (a3 + b 3 + c3 − 3 abc)
(vi) det (kA) = k n det( A), if A is of order n × n.
a bc abc a a2 a3
(vii) If each element of a row (column) of a determinant is
written as the sum of two or more terms, then the (v) b ca abc = b b 2 b 3 = abc (a − b )(b − c)(c − a)
determinant can be written as the sum of two or more c ab abc c c2 c3
determinants i.e.
a1 + a2 b c a1 b c a2 b c
p1 + p2 q r = p1 q r + p2 q r Area of Triangle by
u1 + u2 v w u1 v w u2 v w using Determinants
(viii) If a scalar multiple of any row (column) is added to If A( x1 , y1 ), B( x2 , y2 ) and C( x3 , y3 ) are vertices of ∆ABC, then
another row (column), then ∆ is unchanged
x1 y1 1
a b c a b c 1
Area of ∆ABC = x2 y2 1
i.e. p q r = p + ka q + kb r + kc , which is 2
x3 y3 1
u v w u v w
obtained by the operation R2 → R2 + kR1 1
=[ x1 ( y2 − y3 ) + x2 ( y3 − y1 ) + x3 ( y1 − y2 )]
2
x1 y1 1
Product of Determinants If these three points are collinear, then x2 y2 1 = 0
a1 b1 c1 α1 β1 γ1 and vice-versa. x3 y3 1
If| A| = a2 b2 c2 and| B| = α 2 β2 γ2 , then
a3 b3 c3 α3 β3 γ3 Minors and Cofactors
a1α 1 + b1β1 + c1 γ1 a1α 2 + b1β2 + c1 γ2 The minor M ij of the element aij is the determinant
| A| × | B| = a2α 1 + b2β1 + c2 γ1 a2α 2 + b2β2 + c2 γ2
obtained by deleting the ith row and jth column of ∆.
a3α 1 + b3β1 + c3 γ1 a3α 2 + b3β2 + c3 γ2 a11 a12 a13
a1α 3 + b1β3 + c1 γ3
If ∆ = a21 a22 a23 ,
a2α 3 + b2β3 + c2 γ3 =| AB|
a31 a32 a33
a3α 3 + b3β3 + c3 γ3 a a23 a21 a23
then M11 = 22 , M12 = etc.
[multiplying row by row] a32 a33 a31 a33
We can multiply rows by columns or columns by rows or The cofactor Cij of the element aij is (−1)i + j M ij.
columns by columns
a11 a12 a13
a a23 a a23
NOTE • | AB | = | A| |B | = |BA| = | AT B | =| ABT | = | AT BT | If ∆ = a21 a22 a23 , then C11 = 22 , C12 = − 21 etc.
a32 a33 a31 a33
• | An | = | A| n , n ∈ Z + a31 a32 a33
The sum of product of the elements of any row (or column)
Cyclic Determinants with their corresponding cofactors is equal to the value of
determinant.
In a cyclic determinant, the elements of row (or column) are
arranged in a systematic order and the value of a determinant i.e. ∆ = a 11 C 11 + a 12 C 12 + a 13 C 13
is also in systematic order. = a 21 C21 + a 22 C 22 + a 23 C 23
1 x x2 = a 31 C 31 + a 32 C 32 + a 33 C 33
(i) 1 y y2 = ( x − y) ( y − z) (z − x) But if elements of a row (or column) are multiplied with
1 z z2 cofactors of any other row (or column), then their sum is zero.
DAY 57

Adjoint of a Matrix 1. Matrix Method


[ ]
If A = aij
nxn
, then adjoint of A, denoted by adj ( A), is defined
In this method we first write the above system of equations in
matrix form as shown below.
as [Cij]Tnxn , where Cij is the cofactor of aij.  a1 b1 c1   x  d1 
T  a b c   y  = d  or AX = B
a11 a12 a13  C11 c12 C13   2 2 2
    2
If A = a21 a22 a23 , then adj ( A) = C21 C22 C23   3
a b c3  z d3 
   
3

a31 a32 a33  C31 C32 C33   a1 b1 c1  x d1 


− b where, A =  a2 b2 c2  , X =  y  and B = d 
NOTE • If A = 
a b
, then adj ( A) = 
d
.
     2
 c d   − c a   a3 b3 c3  z d3 
Case I When system of equations is non-homogeneous
Properties of Adjoint of a Matrix (i.e. when B ≠ 0).
Let A be a square matrix of order n, then ●
If| A| ≠ 0, then the system of equations is consistent and has
(i) (adj A) A = A (adj A) = | A| ⋅ I n a unique solution given by X = A −1 B.
(ii) |adj A | = | A|n − 1 , if| A| ≠ 0 ●
If| A| = 0 and (adj A), B ≠ 0, then the system of equations is
(iii) adj (AB) = (adj B) (adj A) inconsistent and has no solution.

If| A| = 0 and (adj A) ⋅ B = O, then the system of equations
(iv) adj ( A T ) = (adj A)T
may be either consistent or inconsistent according as the
(v) adj (adj A) = | A|n – 2 A, if| A| ≠ 0 system have infinitely many solutions or no solution.
2
(vi) |adj (adj A)| = | A |( n – 1 ) , if| A| ≠ 0 Case II When system of equations is homogeneous
(i.e. when B = 0).
Inverse of a Matrix ●
If| A| ≠ 0, then system of equations has only trivial solution,
Let A be any non-singular (i.e.| A| ≠ 0) square matrix, then namely x = 0, y = 0 and z = 0.
inverse of A can be obtained by following two ways. ●
If| A| = 0, then system of equations has non-trivial solution,
which will be infinite in numbers.
1. Using determinants
1 2. Cramer’s Rule Method
In this, A −1 = adj ( A)
| A| In this method we first determine
a1 b1 c1 d1 b1 c1
2. Using Elementary operations
D = a2 b2 c2 , D1 = d2 b2 c2 ,
In this, first write A = IA (for applying row operations) or
a3 b3 c3 d3 b3 c3
A = AI (for applying column operations) and then reduce A of
LHS to I, by applying elementary operations simultaneously a1 d1 c1 a1 b1 d1
on A of LHS and I of RHS. If it reduces to I = PA or I = AP, D2 = a2 d2 c2 and D3 = a2 b2 d2
then P = A −1 . a3 d3 c3 a3 b3 d3

Properties of Inverse of a Matrix Case I When system of equations is non-homogeneous



If D ≠ 0, then it is consistent with unique solution given by
(i) A square matrix is invertible if and only if it is D D D
non-singular. x = 1 , y = 2 ,z = 3.
D D D
(ii) If A = diag (λ 1 , λ 2 ,..., λ n ), ●
If D = 0 and atleast one of D1 , D2 and D3 is non-zero, then it
then A −1 = diag (λ−11 , λ−12 ,..., λ−1n ) provided is inconsistent (no solution).
λ i ≠ 0 ∀, i = 1, 2, … n. ●
If D = D1 = D2 = D3 = 0, then it may be consistent or
inconsistent according as the system have infinitely many
solutions or no solution.
Solution of System of Linear
Case II When system of equations is homogeneous
Equations in Two and Three
If D ≠ 0, then x = y = z = 0 is the only solution, i.e. the
Variables

trivial solution.
Let system of linear equations in three variables be ●
If D = 0, then it has infinitely many solutions.
a1 x + b1 y + c1 z = d1 , a2 x + b2 y + c2 z = d2
Above methods can be used, in a similar way, for the solution
and a3 x + b3 y + c3 z = d3 . of system of linear equations in two variables.
Now, we have two methods to solve these equations.
40 SIX

DAY PRACTICE SESSION 1

FOUNDATION QUESTIONS EXERCISE


1 If x = cy + bz , y = az + cx and z = bx + ay , where x , y 9 If ω is an imaginary cube root of unity, then the value of
and z are not all zero, then a 2 + b 2 + c 2 is equal to a bω 2 aω
(a) 1 + 2abc (b) 1 − 2abc bω c bω2 is
(c) 1 + abc (d) abc −1
cω 2 aω c
2 Consider the set A of all determinants of order 3 with
entries 0 or 1 only. Let B be the subset of A consisting of (a) a 3 + b 3 + c 2 − 3 abc (b) a 2 b − b 2 c
(c) 0 (d) a 2 + b 2 + c 2
all determinants with value 1 and C be the subset of A
consisting of all determinants with value –1. Then, x − 4 2x 2x
(a) C is empty 10 If 2x x − 4 2x = ( A + Bx )( x − A )2 , then the
(b) B and C have the same number of elements 2x 2x x −4
(c) A = B ∪ C
(d) B has twice as many elements as C ordered pair ( A, B ) is equal to j JEE Mains 2018
1 logx y logx z (a) (−4,−5) (b) (−4, 3)
(c) (−4,5) (d) (4,5)
3 If x , y and z are positive, then logy x 1 logy z is
logz x logz y 1 11 If x, y, z are non-zero real numbers and
1+ x 1 1
equal to
1 + y 1 + 2y 1 = 0,
(a) 0 (b) 1 (c) −1 (d) None of these
4 If a , b and c are cube roots of unity, then 1 + z 1 + z 1 + 3z
ea e2a e3a −1 then x −1 + y −1 + z −1 is equal to
eb e 2b e 3b − 1 is equal to (a) 0 (b) −1
(c) −3 (d) −6
ec e2c e3c −1
b+c c+a a+b a b c
(a) 0 (b) e (c) e 2 (d) e 3
12 If c + a a + b b + c = k b c a , then k is equal to
5 If px 4 + qx . 3 + rx 2 + sx + t a+b b+c c+a c a b
x 2 + 3x x −1 x + 3
(a) 0 (b) 1 (c) 2 (d) 3
= x +1 −2x x − 4 , where p, q , r , s
x −3 x + 4 3x 13 Let a, b and c be such that (b + c ) ≠ 0. If
a a + 1 a −1 a +1 b +1 c −1
and t are constants, then t is equal to −b b + 1 b − 1 + a −1 b −1 c + 1 = 0,
(a) 0 (b) 1 (c) 2 (d) –1 c c −1 c + 1 ( −1)n + 2 a (−1)n +1b ( −1)n c
1 x x +1
then the value of n is
6 If f ( x ) = 2x x ( x − 1) ( x + 1)x , (a) zero (b) an even integer
3x ( x − 1) x ( x − 1)( x − 2) ( x + 1)x ( x −1) (c) an odd integer (d) an integer
then f ( 50) is equal to 14 If one of the roots of the equation
(a) 0 (b) 50 (c) 1 (d) −50 7 6 x 2 − 13
7 If α , β and γ are the roots of the equation x + px + q = 0 ,
3
2 x 2 − 13 2 = 0 is x = 2, then sum of all
α β γ x − 13
2
3 7
then the value of the determinant β γ α is
other five roots is
γ α β
(a) – 2 (b) 0
(a) 0 (b) −2 (c) 2 (d) 4 (c) 2 5 (d) 15
8 If ω is a cube root of unity, then a root of the following 15 If a, b and c are sides of a scalene triangle, then the value
x − ω − ω2 ω ω2 a b c
ω x − ω −1 1 = 0 is of b c a is
ω2 1 x − 1 − ω2 c a b JEE Mains 2013
j

(a) x = 0 (b) x = − 1 (a) non-negative (b) negative


(c) x = ω (d) None of these (c) positive (d) non-positive
DAY 59

16 If the system of linear equations 5a − b


24 If A =  and A adj A = AAT , then 5a + b is equal
x + ky + 3z = 0, 3x + ky − 2z = 0 3 2 
and 2x + 4y − 3z = 0 to j JEE Mains 2016
xz (a) − 1 (b) 5 (c) 4 (d) 13
has a non-zero solution ( x , y , z ), then 2 is equal to
y
a b c   q −b y 
25 If A = x y z , B = −p a −x  and if A is invertible,
 
j JEE Mains 2018
(a) −10 (b) 10    
(c) −30 (d) 30 p q r  r −c z 
then which of the following is not true?
17 If A = [aij ]nxn and aij = (i 2 + j 2 − ij )( j − i ), n is odd, then
which of the following is not the value of tr ( A ). (a) | A | = −| B | and | adj A| ≠| adj B|
(b) | A | = − | B | and | adj A | = | adj B |
(a) 0 (b) | A |
(c) A is invertible iff B is invertible
(c) 2 | A | (d) None of these
(b) None of the above
18 If the coordinates of the vertices of an equilateral triangle
26 If A an 3 × 3 non-singular matrix such that AA′ = A′ A and
with sides of length a are ( x1, y1 ),( x 2 , y 2 ) and ( x 3 , y 3 ),
2 B = A −1A′, then BB′ is equal to j JEE Mains 2014
x1 y1 1
(a) I + B (b) I (c) B −1 (d) (B −1)′
then x 2 y 2 1 is equal to −1
 1 − tan θ   1 tan θ  a −b 
x3 y3 1 27 If    = , then
1  b a 
j NCERT Exemplar
tan θ 1  − tan θ
a4 3a2 5a4 3a4
(a) (b) (c) (d) (a) a = 1, b = 1
4 4 4 4
(b) a = sin 2θ, b = cos 2θ
19 The points A (a, b + c ), B (b, c + a ) and C (c, a + b ) are (c) a = cos 2θ, b = sin2θ
(a) vertices of an isoscele triangle (d) None of the above
(b) vertices of an equilateral triangle
1 0 0
(c) collinear
28 If A = 2 1 0 and u1 , u 2 are column matrices such
(d) None of the above  
20 If A1 , B1,C1 , ......... are respectively the co-factors of the 3 2 1 
elements x1, y1, z1,.... of the determinant 1  0
that Au1 = 0 and Au 2 = 1 , then u1 + u 2 is equal to
x1 y1 z1    
B C2
∆ = x2 y2 z 2 , then 2 equals to 0 0
B3 C3
x3 y3 z3  −1  −1  −1  1
(a) x1 ∆ (b) x1 x 3 ∆ (a)  1 (b)  1 (c)  −1 (d)  −1
       
(c) (x1 + y1)∆ (d) None of these  0  −1  0  −1
2 −3  29 The number of values of k, for which the system of
21 If A =  , then adj ( 3A 2 + 12A ) is equal to
 −4 1  j
JEE Mains 2017 equations (k + 1)x + 8y = 4k
72 − 84  and k x + (k + 3)y = 3k − 1 has no solution, is
(a)  (b) 
51 63 
 − 63 51   84 72  (a) infinite (b) 1 j
JEE Mains 2013
72 − 63  (c) 2 (d) 3
(c) 
51 84 
(d) 
 63 72   − 84 51  30 The system of equations
αx + y + z = α − 1, x + αy + z = α − 1
22 Which of the following is/are incorrect?
and x + y + αz = α − 1
(i) Adjoint of a symmetric matrix is symmetric
has no solution, if α is
(ii) Adjoint of a unit matrix is a unit matrix
(iii) A(adj A ) = (adj A ) =| A| I (a) 1 (b) not –2
(iv) Adjoint of a diagonal matrix is a diagonal matrix . (c) either –2 or 1 (d) –2
(a) (i) (b) (ii) 31 If the system of linear equations j
JEE Mains 2013
(c) (iii) and (iv) (d) None of these
x1 + 2x 2 + 3x 3 = 6, x1 + 3x 2 + 5x 3 = 9
1 α 3 2x1 + 5x 2 + ax 3 = b
23 If P = 1 3 3 is the adjoint of a 3 × 3 matrix A and is consistent and has infinite number of solutions, then
 
2 4 4 (a) a = 8, b can be any real number
| A| = 4, then α is equal to j
JEE Mains 2013 (b) b = 15, a can be any real number
(c) a ∈ R − {8 } and b ∈ R − {15 }
(a) 4 (b) 11 (c) 5 (d) 0 (d) a = 8, b = 15
60 40 SIX

32 If the trivial solution is the only solution of the system of 34 The set of all values of λ for which the system of linear
equations equations 2x1 − 2x 2 + x 3 = λx1, 2x1 − 3x 2 + 2x 3 = λx 2 and
x − ky + z = 0, kx + 3y − kz = 0 – x1 + 2x 2 = λx 3 has a non-trivial solution.
and 3x + y − z = 0 (a) is an empty set j JEE Mains 2015
Then, the set of all values of k is (b) is a singleton set
(a) { 2 , − 3 } (b) R − { 2 , − 3 } (c) R − { 2 } (d) R − { − 3 } (c) contains two elements
(d) contains more than two elements
33 Let A, other than I or − I, be a 2 × 2 real matrix such that
A 2 = I , I being the unit matrix. Let tr ( A ) be the sum of 35 Statement I Determinant of a skew-symmetric matrix of
diagonal elements of A. j JEE Mains 2013 order 3 is zero.
Statement I tr ( A ) = 0 Statement II For any matrix A , det( AT ) = det( A ) and
Statement II det ( A ) = − 1 det( − A ) = − det( A ).
(a) Statement I is true, Statement II is true; Statement II is a Where, det ( A ) denotes the determinant of matrix A.
correct explanation for Statement I Then, j JEE Mains 2013

(b) Statement I is true, Statement II is true; Statement II is (a) Statement I is true and Statement II is false
not a correct explanation for Statement I (b) Both statements are true
(c) Statement I is true; Statement II is false (c) Both statements are false
(d) Statement I is false; Statement II is true (d) Statement I is false and Statement II is true

DAY PRACTICE SESSION 2

PROGRESSIVE QUESTIONS EXERCISE


1 If a 2 + b 2 + c 2 = −2 and 6 If the equations a ( y + z ) = x , b( z + x ) = y , c( x + y ) = z
1 + a x (1 + b )x (1 + c )x
2 2 2 1 1 1
have non-trivial solution, then + + is equal
1+ a 1+ b 1+ c
f ( x ) = (1 + a 2 )x 1 + b 2 x (1 + c 2 )x ,
to
(1 + a 2 )x (1 + b 2 )x 1 + c 2 x (a) 1 (b) 2 (c) – 1 (d) −2
then f ( x ) is a polynomial of degree. 7 Let a, b and c be positive real numbers. The following
(a) 2 (b) 3 (c) 0 (d) 1 system of equations in x , y and z.
2 If A is a square matrix of order 3 such that | A| = 2, then x2 y2 z2 x2 y2 z2 x2 y2 z2
2
+ 2 − 2 = 1, 2 − 2 + 2 = 1 and − 2 + 2 + 2 = 1
−1 −1 a b c a b c a b c
| (adj A ) | is
has
(a) 1 (b) 2 (c) 4 (d) 8
(a) no solution (b) unique solution
3 The equations (k − 1)x + ( 3k + 1)y + 2kz = 0, (c) infinitely many solutions (d) finitely many solutions
(k − 1)x + ( 4k − 2)y + (k + 3)z = 0 8 If S is the set of distinct values of b for which the following
and 2x + ( 3k + 1)y + 3(k − 1)z = 0 system of linear equations
gives non-trivial solution for some values of k, then the
ratio x : y : z when k has the smallest of these values. x + y + z = 1, x + ay + z = 1 and ax + by + z = 0
(a) 3 : 2 :1 (b) 3 : 3 : 2 (c) 1 : 3 : 1 (d) 1:1:1 has no solution, then S is j
JEE Mains 2017
(a) an infinite set
4 If x = 1 + 2 + 4 +.... upto k terms, y = 1 + 3 + 9+.... upto k (b) a finite set containing two or more elements
terms and c = 1 + 5 + 25+ ... upto k terms. Then, (c) singleton set
(d) a empty set
x 2y 4z
∆= 3 3 3 equals to 9 If M is a 3 × 3 matrix, where M T M = I and
det(M) = 1, then the value of det (M − I ) is
2k 3k 5k
(a) –1 (b) 1
(a) (20)k (b) 5 k (c) 0 (d) 2 k + 3 k + 5 k (c) 0 (d) None of these
1 + 2x 1 1− x 10 If a1, a 2 , . . ., a n , . . . are in GP, then the determinant
5 Product of roots of equation 2 − x 2+ x 3+ x = 0 log a n log a n + 1 log a n + 2
x 1+ x 1− x 2 ∆ = log a n + 3 log a n + 4 log a n + 5 is equal to
1 3 4 1 log a n + 6 log a n + 7 log a n + 8
(a) (b) (c) (d)
2 4 3 4 (a) 2 (b) 4 (c) 0 (d) 1
DAY 61

11 Let A be a square matrix of order 2 with | A| ≠ 0 such that 14 Area of triangle whose vertices are (a, a 2 ),(b, b 2 ),(c, c 2 ) is
| A + | A| adj ( A )| = 0, then the value of | A −| A| adj ( A )| is 1
, and the area of triangle whose vertices are
(a) 1 (b) 2 2
(c) 3 (d) 4 ( p, p 2 ),(q , q 2 ) and (r , r 2 ) is 4, then the value of
12 Let P and Q be 3 × 3 matrices P ≠ Q . If P 3 = Q 3 and (1 + ap )2 (1 + bp )2 (1 + cp )2
P 2Q = Q 2P , then the determinant of (P 2 + Q 2 ) is
(1 + aq )2 (1 + bq )2 (1 + cq )2 is
(a) − 2 (b) 1
(c) 0 (d) − 1 (1 + ar )2 (1 + br )2 (1 + cr )2

3 1 + f (1) 1 + f ( 2) (a) 2 (b) 4 (c) 8 (d) 16


13 If α , β ≠ 0, f (n ) = α + β and 1 + f (1) 1 + f ( 2) 1 + f ( 3)
n n l m n
1 + f ( 2) 1 + f ( 3) 1 + f ( 4) 15 Let det A = p q r and if (l − m )2 + ( p − q )2 = 9,
1 1 1
= K (1 − α )2 (1 − β )2 (α − β )2 , then K is equal to
j JEE Mains 2014 (m − n )2 + (q − r )2 = 16, (n − l )2 + (r − p )2 = 25, then the
(a) αβ (b)
1
(c) 1 (d) − 1 value of (det A )2 equals to
αβ (a) 36 (b) 100 (c) 144 (b) 169

ANSWERS
SESSION 1 1 (b) 2 (b) 3 (a) 4 (a) 5 (a) 6 (a) 7 (a) 8 (a) 9 (c) 10 (c)
11 (c) 12 (c) 13 (c) 14 (a) 15 (b) 16 (b) 17 (d) 18 (d) 19 (c) 20 (a)
21 (b) 22 (d) 23 (b) 24 (b) 25 (a) 26 (b) 27 (c) 28 (d) 29 (d) 30 (d)
31 (d) 32 (b) 33 (b) 34 (c) 35 (a)

SESSION 2 1 (a) 2 (c) 3 (d) 4 (c) 5 (a) 6 (b) 7 (d) 8 (c) 9 (c) 10 (c)
11 (d) 12 (c) 13 (c) 14 (d) 15 (c)
62 40 SIX

Hints and Explanations


SESSION 1 1 e a e 2a 1 e a e 2a − a b aω2
1 Given equation can be rewritteen as = 1 e b
e 2b
− 1 e b e 2b = ω ⋅ ω − b c bω2
2

x − cy − bz = 0
c
1 e e 2c
1 ec e2c − c a cω2
cx − y + az = 0 = 0 [Q a + b + c = 0 ⇒ e a + b + c = 1 ] a b a
bx + ay − z = 0 = − ω5 b c b = 0
5 Put x = 0 in the given equation, we get
Since, x, y and z are not all zero c a c
0 −1 3
∴The above system have non-trivial x − 4 2x 2x 
solution. t = 1 0 − 4 = − 12 + 12 = 0
10 Given,  2x x−4 2x 
1 −c −b −3 4 0  
 2x 2x x − 4
∴ c −1 a = 0 6 On taking common factors x from
C2 , ( x + 1) from C3 and ( x − 1) from = ( A + Bx )( x − A ) 2
b a −1
R3 , we get On applying C1 → C1 + C2 + C3 , we get
⇒ 1(1 − a2 ) + c (−c − ab ) − b( ac + b ) = 0 1 1 1  5x − 4 2x 2x 
⇒ 1 − a2 − c 2 − abc − abc − b2 = 0 f ( x ) = x ( x2 − 1) 2 x x − 1 x  5x − 4 x−4 2x 
⇒ a2 + b 2 + c 2 = 1 − 2abc  
3x x − 2 x  5x − 4 2x x − 4
2 If we interchange any two rows of a 1 0 0 = ( A + Bx )( x − A )2
determinant in the set B, its value
= x ( x − 1) 2 x − ( x + 1) 1
2 On taking common (5x − 4) from C1 , we
becomes −1 and hence it is in C.
3 x − 2( x + 1) 2 get
Likewise, for every determinant in C,
there is corresponding determinant in B. 1 2 x 2x 
C3 → C3 − C2 ,
So , B and C have the same number of =0 C → C − C  (5x − 4)1 x − 4 2 x 
 2 1   
x − 4
2
elements.
1 2 x
1 log x y log x z ∴ f (50) = 0
= ( A + Bx )( x − A )2
3 Let ∆ = log y x 1 log y z 7 Clearly, α + β + γ = 0 On applying R2 → R2 − R1
log z x log z y 1 On applying C1 → C1 + C2 + C3 in the and R3 → R3 − R1 , we get
log x log y log z given determinant, we get
1 2x 2x 
α +β+ γ β γ
log x log x log x ∴ (5x − 4) 0 − x − 4 0 
log x log y log z ∴∆ = α +β+ γ γ α  
=  0 0 − x − 4
log y log y log y α +β+ γ α β
log x log y log z 0 β γ = ( A + Bx )( x − A )2
log z log z log z = 0 γ α =0 On expanding along C1 , we get
By taking common factors from the 0 α β (5x − 4)( x + 4)2 = ( A + Bx )( x − A ) 2
rows, we get On comparing, we get
1 8 On applying C1 → C1 + C2 + C3 , we get A = − 4 and B = 5
∆ =
log x ⋅ log y ⋅ log z x ω ω2 1+ x 1 1
log x log y log z x x + ω2 1 =0 11 Let ∆ = 1 + y 1 + 2 y 1 =0
log x log y log z x 1 x+ω 1 + z 1 + z 1 + 3z
log x log y log z [Q1 + ω + ω2 = 0] On applying C1 → C1 − C3
Now, by taking common factor from the 1 ω ω2
and C2 → C2 − C3 , we get
1 1 1 ⇒ x 1 x + ω2 1 =0
x 0 1
columns, we get 1 1 1 = 0 1 1 x+ω
∆ = y 2y 1
1 1 1
∴ x=0 −2z −2z 1 + 3z
e a e 2a e 3a e a e 2a 1 a(1 + ω ) bω2 aω
4 Let ∆ = e b e 2b e 3b
− e b e 2b 1 On expanding along R1 , we get
9 ∆ = b(ω + ω2 ) c bω2
ec e2c e 3c
ec e2c 1 ∆ = x[2 y (1 + 3z ) + 2z]
c (ω2 + 1) aω c
= e a ⋅ eb ⋅ ec +1[−2 yz + 4 yz] = 0
[QC1 → C1 + C3 ] ⇒ 2[ xy + 3 xyz + xz] + 2 yz = 0
1 e a e 2a e a 1 e 2a
− aω2 bω2 aω
1 e b
e 2b
+ e b 1 e 2b ⇒ xy + yz + zx + 3 xyz = 0
= −b c bω2
1 e ec 2c
ec 1 e2c 1 1 1
− cω aω c ⇒ + + = −3
x y z
DAY 63

b + c c + a a+ b 1 1 1 18 If ( x1 , y 1 ),( x2 , y 2 ) and ( x3 , y 3 ) are the


12 Let ∆ = c + a a + b b + c ⇒ ( x2 − 4) 2 x2 − 13 2 = 0 vertices of a triangle, then Area
a+ b b + c c + a x2 − 13 3 7 x1 y 1 1
1
2 (a + b + c ) c + a a + b Now, on applying C2 → C2 − C1 = x2 y 2 1 …(i)
2
= 2 (a + b + c ) a + b b + c and C3 → C3 − C1 , we get x3 y 3 1
2 (a + b + c ) b + c c + a 1 0 0 Also, we know that, if a is the length of
( x2 − 4) 2 x2 − 15 0 =0 an equilateral triangle, then
[using C 1 → C 1 + C 2 + C 3 ]
x2 − 13 16 − x2 20 − x2 3 2
a+ b + c c + a a+ b Area = a …(ii)
On expanding along R1 , we get 4
=2 a+ b + c a+ b b + c
( x2 − 4)( x2 − 15)( x2 − 20) = 0 From Eqs. (i) and (ii), we get
a+ b + c b + c c + a
Thus, other five roots are x1 y1 1
[taking common 2 from C1 ] −2, ± 15, ± 2 5 3 2 1
a = x2 y2 1
a + b + c −b −c Hence, sum of other five roots is −2. 4 2
x3 y3 1
= 2 a + b + c −c − a a b c x1 y1 1
a + b + c − a −b 15 Let ∆ = b c a 3 2
⇒ a = x2 y2 1
c a b 2
[using C 2 → C2 − C1 , C3 → C3 − C1 ] x3 y3 1
a b c = a(bc − a2 ) − b (b2 − ac ) + c ( ab − c 2 )
On squaring both sides, we get
=2 b c a = abc − a3 − b 3 + abc + abc − c 3 2
x1 y 1 1
c a b = − (a3 + b 3 + c 3 − 3 abc ) 3 4
a = x2 y 2 1
= − (a + b + c ) (a2 + b 2 4
[using C1 → C1 + C2 + C3 ] + c 2 − ab − bc − ca) x3 y 3 1
1
On comparing, k = 2 = − (a + b + c ) 19 Clearly, area of
2
a a+ 1 a−1 {(a − b ) 2 + (b − c ) 2 + (c − a) 2 } a b +c 1
1
13 − b b + 1 b − 1 + (−1)n < 0, where a ≠ b ≠ c (∆ABC ) = b c + a 1
2
c c −1 c + 1 16 We have, c a+ b 1
a+ 1 b + 1 c −1 x + ky + 3z = 0; 3 x + ky − 2z = 0; a 1 1
1
a−1 b −1 c + 1 2 x + 4 y − 3z = 0 = (a + b + c ) b 1 1 =0
2
a −b c System of equation has non-zero c 1 1
a a+ 1 a−1
solution, if [QC2 → C2 + C1 ]
 1 k 3 x1 z1
= −b b + 1 b − 1  3 k −2 = 0 20 Clearly, B2 = = x1 z3 − x3 z1
x3 z3
c c −1 c + 1  
 2 4 −3 C2 = − 1
x y1
= −( x1 y 3 − x3 y 1 )
a+ 1 a−1 a x3 y 3
⇒ (−3k + 8) − k (−9 + 4) + 3(12 − 2k ) = 0
+ (−1)n b + 1 b − 1 −b x z1
⇒ −3k + 8 + 9k − 4k + 36 − 6k = 0 B3 = − 1 = −( x1 z2 − x2 z1 )
c −1 c + 1 c x2 z2
[Q| A ′|= | A|] ⇒ −4k + 44 = 0 ⇒ k = 11
x y1
a a+ 1 a−1 Let z = λ , then we get and C3 = 1 = x1 y 2 − y 1 x2
x2 y 2
= −b b + 1 b − 1 x + 11 y + 3λ = 0 …(i) B C2
c c −1 c + 1 3 x + 11 y − 2λ = 0 …(ii) ∴ 2
B3 C 3
a+ 1 a a−1 and 2 x + 4 y − 3λ = 0 …(iii)
x1 z3 − x3 z1 −( x1 y 3 − y 1 x3 )
+ (−1)n + 1 b + 1 − b b −1 On solving Eqs. (i) and (ii), we get =
−( x1 z2 − x2 z1 ) x1 y 2 − x2 y 1
c −1 c c +1 5λ −λ
x= ,y = ,z = λ x1 z3 − x1 y 3
a a+ 1 a−1 2 2 =
xz 5λ2 − x1 z2 + x2 z1 x1 y 2 − x2 y 1
= [1 + (− 1)n +2 ] − b b + 1 b − 1 ⇒ = = 10
− x3 z1
2 2
y λ y 1 x3
c c −1 c + 1 2 ×  −  +
 2 − x1 z2 + x2 z1 x1 y 2 − x2 y 1
This is equal to zero only, if n + 2 is an
odd i.e. n is an odd integer. x z − x1 y 3 x z − x1 y 3
17 We have, = 1 3 + 1 3
x2 − 13 − x1 z2 x1 y 2 x2 z1 − x2 y 1
7 6 aij = (i 2 + j2 − ij )( j − i )
− x3 z1 y 1 x3 − x3 z1 y 1 x3
14 2 x2 − 13 2 =0 ∴ a ji = (i 2 + j2 − ij )(i − j ) + +
= −(i 2 + j2 − ij )( j − i ) = −aij − x1 z2 x1 y 2 x2 z1 − x2 y 1
x2 − 13 3 7
⇒ A is a skew-symmetric matrix of = x12 (z3 y 2 − z2 y 3 ) − x1 x2 (z3 y 1 − z1 y 3 )
On applying R1 → R1 + R2 + R3 , we get
odd order. − x1 x3 (z1 y 2 − z2 y 1 ) + x2 x3 (z1 y 1 − z1 y 1 )
x2 − 4 x2 − 4 x2 − 4 ∴ tr ( A ) = 0 and| A|= 0 = x1 [ x1 (z3 y 2 − z2 y 3 ) − x2 (z3 y 1 − z1 y 3 )
2 x2 − 13 2 =0 + x3 (z2 y 1 − z1 y 2 )]
x2 − 13 3 7 = x1 ∆
64 40 SIX
− 3  a −b 
21 We have, A =  10   + 3b = 13
2 2b
=
− 4 1   15 b a 

∴ A2 = A ⋅ A 20b + 45b
⇒ = 13  1 − tan2 θ −2 tan θ 
 2 − 3  2 − 3  1 + tan2 θ 1 + tan2 θ 
15
=    a −b 
− 4 1  − 4 1  65b ⇒ = b a 
⇒ = 13 ⇒ b = 3  2 tan θ 1 − tan2 θ   
 4 + 12 − 6 − 3  16 − 9 15  1 + tan2 θ 2 
=  1 + tan θ 
=   Now, substituting the value of b in Eq.
 − 8 − 4 12 + 1   − 12 13 
(iii), we get 5a = 2 cos 2θ − sin 2θ  a −b 
 16 − 9 ⇒ =  
Now, 3 A2 + 12 A = 3   Hence, 5a + b = 2 + 3 = 5  sin 2θ cos 2θ  b a 
 − 12 13 
q −b y ⇒ a = cos 2θ and b = sin 2θ
 2 − 3 25 Clearly,|B|= − p a − x
+ 12   28 On adding Au1 and Au2 , we get
− 4 1  r −c z
1   0 1 + 0
 48 − 27  24 − 36 q −b y
= +   Au1 + Au2 =  0 + 1  =  0 + 1
 − 36 39   − 48 12  = − p −a x      
 0  0  0 + 0
 72 − 63 r −c z
=  1 
 − 84 51  (taken (–1) common from R2 )
q b y ⇒ A(u1 + u2 ) = 1 
 51 63  
∴adj (3 A2 + 12 A ) =   = (+ ) p a x  0
 84 72 
r c z Since, A is non-singular matrix, i.e.
22 All the given statements are true.
(taken (–1) common from C2 ) | A | ≠ 0, on multiplying both sides by
1 α 3  A −1 , we get
p a x
23 Given, adj A = P = 1 3 3 
  = − q b y (QR1 ↔ R2 ) 1 
2 4 4 A −1 A(u1 + u2 ) = A −1 1 
r c z
 
Find the determinant of P and apply the  0
formula a x p
−1
|adj A|=| A|n −1 =−b y q 1 0 0 1 
5a − b  c z r ⇒ u1 + u2 = 2 1 0 × 1  …(i)
24 Given, A =     
3 2  (QC1 ↔ C2 and then C2 ↔ C3 )
3 2 1   0
and A adj A = AA T = −| A T | = −| A| 1 0 0
Clearly, A(adj A ) = A I2 Now, |A | = 2 1 0
Thus,| A|= −|B|
[Q if A is square matrix of order n, then 3 2 1
Hence,| A| ≠ 0 iff|B|≠ 0
A(adj A ) = (adj A ) ⋅ A = A I n ] = 1 (1 − 0) − 0 + 0 = 1
5a − b ∴ A is invertible iff B is invertible
= I2 ∴ adj of A
3 2 Also,|adj A|= | A| = |B| = |adj B |
2 2
T
 1 −2 1   1 0 0
= (10a + 3b ) I2 26 Given, A is non-singular matrix =  0 1 −2 =  −2 1 0
⇒| A|≠ 0.    
1 0  0 0 1   1 −2 1 
= (10a + 3b )  
 0 1 Also we have, AA T = A T A and
 1 0 0
+ B = A −1 A T
=
 10 a 3b 0 
...(i) ∴ A −1 =  −2 1 0 [Q| A | = 1]
10a + 3b  Now, BB ′ = ( A −1 A T ) ( A −1 A T )T  
 0
 1 −2 1 
 5a − b   5a 3 = A −1 A T A ( A −1 )T [Q( A ′ )′ = A]
and AA T =  From Eq. (i),
2   − b 2
−1 −1 T
3 = A AA ( A ) [Q AA = A T A ]
T T

1 0 0 1 
25a2 + b 2 15a − 2b  = IA T ( A −1 )T = A T ( A −1 )T u1 + u2 = −2 1 0 × 1 
= ...(ii)    
 15a − 2b 13  = ( A −1 A )T [Q( AB )T = B T A T ]  1 −2 1   0
Q A(adj A ) = AA T = IT = I
 1 + 0 + 0  1 
10a + 3b 0  27 We have, ⇒ u1 + u2 =  −2 + 1 + 0 =  −1
∴ −1
   
 0 10a + 3b   1 − tan θ  1 tan θ
 tan θ   − tan θ  1 − 2 + 0  −1
 1   1 
25a2 + b 2 15a − 2b 
=  a −b 
 15a − 2b 13  = 
29 Given equations can be written in
b a  matrix form as AX = B
[using Eqs. (i) and (ii)]
 1 − tan θ 1 k + 1 8   x
⇒ ⋅ where, A =  , X =  y 
⇒ 15a − 2b = 0 1  1 + tan2 θ  k k + 3  
 tan θ
2b
⇒ a= ...(iii)  1 − tan θ  a −b  4 k 
15 = and B =  
 tan θ 1  b a  3 k − 1 
and 10a + 3b = 13 ...(iv) 
On substituting the value of ‘a’ from Eq. 1 1 − tan2 θ −2 tan θ  For no solution,| A |= 0

(iii) in Eq. (iv), we get 1 + tan θ  2 tan θ
2
1 − tan2 θ and (adj A) B ≠ 0
DAY 65

k+1 8 ⇒ (α + 2)(α − 1)2 = 0 2− λ −2 1


Now, | A |= =0
k k+3 ∴ α = 1, − 2 ∴ 2 −(3 + λ ) 2 = 0
But α = 1 makes given three equations –1 2 −λ
⇒ (k + 1) (k + 3) − 8 k = 0
same. So, the system of equations has
⇒ k2 + 4 k + 3 − 8 k = 0 ⇒ (2 − λ )(3λ + λ2 − 4) + 2(−2λ + 2)
infinite solution. Hence, answer is
α = − 2 for which the system of +1(4 − 3 − λ ) = 0
⇒ k2 − 4 k + 3 = 0
equations has no solution. ⇒ (2 − λ )(λ2 + 3λ − 4) + 4(1 − λ )
⇒ (k − 1) (k − 3) = 0 + (1 − λ ) = 0
31 For consistency| A | = 0 and
∴ k = 1, k = 3 ⇒ (2 − λ )( λ + 4)( λ − 1) + 5 (1 − λ ) = 0
(adj A ) B = O
 k + 3 −8  1 2 3 ⇒ (λ − 1)[(2 − λ )(λ + 4) − 5] = 0
Also, adj A =  
 − k k + 1 1 3 5 =0 ⇒ (λ − 1)(λ2 + 2λ − 3) = 0
⇒ (λ − 1)[(λ − 1)(λ + 3)] = 0
 k + 3 −8  4 k  2 5 a
∴(adj A) B =   3 k − 1 ⇒ (λ − 1)2 (λ + 3) = 0
 − k k + 1  ⇒ 1(3 a − 25) − 2(a − 10) + 3(5 − 6) = 0
⇒ λ = 1,1,−3
(k + 3) (4 k ) − 8 (3 k − 1) ⇒ a= 8
=  35 Determinant of skew-symmetric matrix
 −4 k + (k + 1) (3 k − 1) 
2 On solving, (adj A ) B = O , we get
of odd order is zero and of even order is
b = 15
 4 k 2 − 12k + 8 perfect square.
=  32 Since, x − ky + z = 0, Also, det( A T ) = det( A )
 − k + 2k − 1 
2

kx + 3 y − kz = 0 and and det(− A ) = (−1)n det( A )


Put k = 1, then 3 x + y − z = 0 has trivial solution. Hence, Statement II is false.
 4 − 12 + 8  0 1 −k
(adj A) B =   =  , not true. 1
 −1 + 2 − 1   0 ∴ k 3 −k ≠0 SESSION 2
Put k = 3, then (adj A) 3 1 −1 1 Given that,
36 − 36 + 8  8 
B =  =   ≠ 0, true. 1 + a2 x (1 + b 2 )x (1 + c 2 )x
 −9 + 6 − 1   −4 ⇒ 1 (− 3 + k ) + k (− k + 3k )
+ 1 (k − 9) ≠ 0 f ( x ) = (1 + a2 )x 1 + b 2 x (1 + c 2 )x
Hence, the required value of k is 3.
⇒ k − 3 + 2 k2 + k − 9 ≠ 0 (1 + a2 )x (1 + b 2 )x 1 + c 2 x
Alternate Method Condition for the
system of equations has no solution is ⇒ 2 k 2 + 2 k − 12 ≠ 0 On applying C1 → C1 + C2 + C3 , we get
a1 b c k+1 8 4k ⇒ k2 + k − 6 ≠ 0 1 + a2 x + x + b 2 x + x + c 2 x
= 1 ≠ 1 ⇒ = ≠
a2 b2 c2 k k + 3 3k − 1 ⇒ (k + 3) (k − 2) ≠ 0 f ( x ) = x + a2 x + 1 + b 2 x + x + c 2 x
k+1 8 ∴ k ≠ 2, − 3 x + a2 x + x + b 2 x + 1 + c 2 x
Take =
k k+3 k ∈ R − {2, − 3}
(1 + b 2 )x (1 + c 2 )x
⇒ k 2 + 4 k + 3 = 8k
33 Let A = 
a b 1 + b2 x (1 + c 2 )x
⇒ k2 − 4 k + 3 = 0  such that A ≠ I , − I
 d
c (1 + b 2 )x 1 + c2 x
⇒ (k − 1) (k − 3) = 0 2
∴ k = 1, 3 a b  1 0 1 + (a2 + b 2 + c 2 + 2)x (1 + b 2 )x
and   =  0 1
8 4⋅1 c d    = 1 + (a2 + b 2 + c 2 + 2)x (1 + b 2 )x
If k = 1, then ≠ , [false]
1+ 3 2  a2 + bc ab + bd  1 0 1 + (a2 + b 2 + c 2 + 2)x (1 + b 2 )x
4⋅3 ⇒ =
bc + d 2   0 1 
8
and k = 3, then ≠ , [true]  ac + cd (1 + c 2 )x
6 9−1
⇒ b(a + d ) = 0,c (a + d ) = 0 (1 + c 2 )x
∴ k =3 and a2 + bc = 1,bc + d 2 = 1 1 + c2 x
Hence, only one value of k exists. ⇒ a = 1,d = −1,b = c = 0 or 1 (1 + b 2 )x (1 + c 2 )x
30 The system of given equations has no a = −1,d = 1,b = c = 0 = 1 1 + b 2 x (1 + c 2 )x
solution, if 1 0  1 (1 + b 2 )x 1 + c 2 x
A=  , then
α 1 1  0 − 1
1 α 1 =0 [Qa2 + b 2 + c 2 = − 2, given]
det( A ) = −1 and tr ( A ) = 1 − 1 = 0
1 1 α On applying
34 Given system of linear equations are R1 → R1 − R3 , R2 → R2 − R3 , we get
On applying C1 → C1 + C2 + C3 , we get 2 x1 − 2 x2 + x3 = λx1 0 0 x −1
α+2 1 1 ⇒ (2 − λ) x1 − 2 x2 + x3 = 0 …(i) = 0 1− x x −1
α +2 α 1 =0 2 x1 − 3 x2 + 2 x3 = λx2 1 (1 + b 2 )x 1 + c 2 x
α +2 1 α ⇒ 2 x1 − (3 + λ )x2 + 2 x3 = 0 …(ii)
0 x −1
and − x1 + 2 x2 = λx3 = = ( x − 1) 2
On applying R2 → R2 − R1 , R3 → R3 − R1 , 1− x x −1
⇒ − x1 + 2 x2 − λx3 = 0 …(iii)
1 1 1
Since, the system has non-trivial Hence, f ( x ) is of degree 2.
(α + 2) 0 α − 1 0 =0
solution.
0 0 α −1
66 40 SIX

2 Clearly, adj A −1 = | A −1 |2 = 1 2 −1 a a For a = 1


| A| 6 Here, b −1 b = 0 ∆ = ∆1 = ∆2 = ∆3 = 0
c c −1 For b = 1 only
1 x + y + z = 1, x + y + z = 1
and |(adj A −1 )−1 |=
|(adj A −1 )| On applying, C2 → C2 − C1 and and x+ y + z=0
C3 → C3 − C1 , we get i.e. no solution (QRHS is not equal)
= | A|2 = 22 = 4
−1 a+ 1 a+ 1
Hence, for no solution, b = 1 only.
3 For non - trivial solution, b −(b + 1) 0 =0
9 Clearly,
k − 1 3k + 1 2k c 0 − (1 + c )
det(M − I ) = det(M − I ) ⋅ det(M )
k − 1 4k − 2 k + 3 = 0 R1 R2 [Qdet(M ) = 1]
On applying R1 → , R2 →
2 3k + 1 3(k − 1) a+ 1 b+1 = det(M − I )det(M T )
R3
⇒ k = 0 or 3 and R3 → , we get [Q det( A T ) = det( A )]
c+1
Now, when k = 0, then the equation 1 = det(MM T − M T )
− 1 1
becomes a+ 1 = det(I − M T ) [Q MM T = I ]
b
−x + y = 0 ...(i) −1 0 = 0 = − det(M − I )
T
b+1
− x − 2 y + 3z = 0 ...(ii) c = − det(M T − I )T
0 −1
and 2 x + y − 3z = 0 (iii) c+1
= − det(M − I )
From (i), we get x = y = λ (say), Then, 1 b c
⇒ − + + =0 ⇒ 2det(M − I ) = 0
from Eq. (ii), we get a+ 1 b + 1 c + 1
⇒ det(M − I ) = 0
1 1 c
− λ − 2λ + 3z = 0 ∴− + 1− + 1− =0
a+ 1 b+1 c+1 10 Since, a1 , a2 , . . . , an, . . . are in GP, then,
⇒ 3z = 3λ 1 1 1 log an , log an + 1 , log an + 2 , ...,
⇒ + + =2
⇒ z=λ a+ 1 b + 1 c + 1 log an + 8 , ...are in AP.
∴ x: y : z = 1:1:1
2
y2 2
7 Let x2 = X , 2 = Y , z2 = Z Given that,
a b c log an log an + 1 log an +2
4 Clearly, x = 2k − 1, y = 3 − 1
k

Then, X + Y − Z = 1, X − Y + Z = 1, ∆ = log an + 3 log an + 4 log an + 5


2
−X + Y + Z = 1 log an + 6 log an + 7 log an + 8
5k − 1
and z = Now, coefficient matrix is
4 a a+ d a + 2d
 1 1 −1
 A =  1 −1 1  ∴ ∆ = a + 3d a + 4d a + 5d
Q sum to n terms of a GP is  
 a + 6d a + 7d a + 8d
 −1 1 1 
a(r n − 1) where a and d are the first term and
given by Here,| A|≠ 0
r − 1  common difference of AP.
∴ It has unique solution, viz., X = 1,
2k − 1 3k − 1 5k − 1 Y = 1 and Z = 1 On applying R2 → 2R2 , we get
a a+ d a + 2d
Now, ∆ = 3 3 3 Hence , x = ± a; y = ± b and z = ± c . 1
∆ = 2a + 6d 2a + 8d 2a + 10d
2k 3k 5k 1 1 1 2
8 Here, ∆ = 1 a 1 a + 6d a + 7d a + 8d
2k 3k 5k 1 1 1
a b 1 On applying R2 → R2 − R3 , we get
= 3 3 3 −3 1 1 1
a a+ d a + 2d
2k 3k 5k 2k 3k 5k = 1 (a − b ) − 1 (1 − a) + 1 (b − a2 ) 1
∆ = a a+ d a + 2d = 0
= − (a − 1)2 2
= 0−3× 0= 0 a + 6d a + 7d a + 8d
1 1 1
1 + 2x 1 1− x ∆1 = 1 a 1 11 Let A =  
a b
5 Let p( x ) = 2 − x 2 + x 3 + x = 0 . Then,
0 b 1 c d 
x 1 + x 1 − x2 | A|= ad − bc = k (say)
= 1 (a − b ) − 1 (1) + 1 (b ) = (a − 1)
 d −b 
Clearly, product of roots 1 1 1 and adj ( A ) =  
=
Constant term  −c a 
∆2 = 1 1 1
Coefficient of x 4 Now,| A +| A| adj ( A )|= 0
a 0 1
Here, constant term is given by a + kd (1 − k )b
1 1 1 = 1 (1) − 1 (1 − a) + 1 (0 − a) = 0 ⇒ =0
(1 − k )c d + ka
P(0) = 2 2 3 = −1 1 1 1
⇒ (a + kd )(d + ka) − (1 − k )2 bc
0 1 1 and ∆ 3 = 1 a 1
⇒ ad + a2 k + k d 2 + k 2 ad
a b 0
and coefficient of x 4 is −2 −(1 + k 2 − 2k )bc
1 = 1 ( − b ) − 1 (− a) + 1 (b − a2 ) ⇒ (ad − bc ) + ( ad − bc )k 2 + k( a2 + d 2
∴ Product of roots is .
2 = − a(a − 1) + 2bc ) = 0
DAY 67

⇒ (ad − bc ) + ( ad − bc ) k 2 + k ( a2 + d 2 ) 3 1+ α+ β 1 p p2 1 2a a2
+2(ad − k )) = 0 ⇒ ∆ = 1 + α + β 1 + α 2 + β2 = 1 q q × 1 2b b 2
2

[Qbc = ad − k ] 1 + α 2 + β2 1 + α 3 + β3 1 r r2 1 2c c 2
⇒ (ad − bc ) + ( ad − bc )k 2 1 + α 2 + β2
1 + α 3 + β3  1 p p2 1 a a2 
+ k (a + d )2 − 2k 2 = 0  
⇒ (k + k − 2k 2 ) + k (a + d )2 = 0
3
1 + α4 + β4 = 2 1 q q 2 × 1 b b 2 
 1 r r2 1 c c2 
⇒ k [(k − 1)2 + (a + d )2 ] = 0 1⋅1 + 1⋅1 + 1⋅1 1⋅1 + 1⋅α + 1⋅β  
⇒ k = 1 and a + d = 0 = 1⋅1 + α ⋅1 + β ⋅1 1⋅1 + α ⋅α + α ⋅β = 2(2 A1 × 2 A2 ) = 2(8 × 1) = 16
Now,| A −| A|adj A| 1 ⋅ 1 + 1 ⋅ α 2 + 1 ⋅ β2 1 ⋅ 1 + α 2 ⋅ α + β2 ⋅ β
a − kd (1 + k )b a−d 2b 1 ⋅ 1 + 1 ⋅ α 2 + 1 ⋅ β2 15 According to given conditions we get a
= = right angled triangle whose vertices are
(1 + k )c d − ak 2c d−a 1 ⋅ 1 + α ⋅ α 2 + β ⋅ β2
(n, r ),(m,q ) and (l , p ) .
= −(a − d ) − 4bc
2 1 ⋅ 1 + α 2 ⋅ α 2 + β2 ⋅ β2
= −((a + d )2 − 4ad ) − 4bc 1 1 1 1 1 1 1 1 1
2
A(n, r)
= 4ad − 4bc = 4k = 4 = 1 α β 1 α β = 1 α β
12 On subtracting the given equation, we 1 α 2 β2 1 α 2 β2 1 α 2 β2
5
get 4
On expanding, we get
P 3 − P 2Q = Q 3 − Q 2 P
∆ = (1 − α )2 (1 − β )2 (α − β2 )
⇒ P 2 (P − Q ) = Q 2 (Q − P )
Hence, K (1 − α )2 (1 − β )2 (α − β )2 C(l, p)
⇒ (P − Q ) (P 2 + Q 2 ) = 0 ...(i) B(m, q) 3
= (1 − α )2 (1 − β )2 (α − β )2
Now, if |P 2 + Q 2 | ≠ 0
∴ K =1 l m n
(P 2 + Q 2 ) is invertible .
(1 + ap )2 (1 + bp )2 (1 + cp )2 Also, we have,| A|= p q r
On post multiply both sides by
(P 2 + Q 2 )−1 , we get 14 Let ∆ = (1 + aq )2 (1 + bq )2 (1 + cq )2 1 1 1
(1 + ar )2 (1 + br )2 (1 + cr )2 2
l p 1
P − Q = 0, which is a contradiction. 1 + 2ap + a p 1 + 2bp + b p
2 2 2 2
⇒| A| = m q 1 = [2ar (∆ABC )]
2 2

Hence,|P 2 + Q 2|= 0 = 1 + 2aq + a2q 2 1 + 2bq + b 2q 2 n r 1


3 1 + f (1) 1 + f (2) 1 + 2ar + a2 r 2 1 + 2br + b 2 r 2 2

= 2 × × 3 × 4 = 144
1 + 2cp + c 2 p2 1
13 Let ∆ = 1 + f (1) 1 + f (2) 1 + f (3)
1 + 2cq + c 2q 2  2 
1 + f (2) 1 + f (3) 1 + f (4)
1 + 2cr + c 2 r 2
DAY SEVEN

Binomial Theorem
and Mathematical
Induction
Learning & Revision for the Day
u Binomial Theorem u Properties of Binomial u Binomial Theorem for
u Binomial Theorem for Positive Coefficient Negative/Rational Index
Index u Applications of Binomial u Principle of Mathematical
Theorem Induction

Binomial Theorem
Binomial theorem describes the algebraic expansion of powers of a binomial. According to this
theorem, it is possible to expand ( x + y)n into a sum involving terms of the form axb yc , where
the exponents b and c are non-negative integers with b + c = n. The coefficient a of each term is
 n
a specific positive integer depending on n and b, is known as the binomial coefficient   .
b 

Binomial Theorem for Positive Index


An algebraic expression consisting of two terms with (+ ) ve or (−)ve sign between them, is called
binomial expression.
PRED
If n is any positive integer, MIRROR
Your Personal Preparation Indicator
then ( x + a)n = nC0 x n + nC1 x n − 1 a + L + n Cnan
n
= ∑ nCr ⋅ x n − r ar , where x and a are real (complex) numbers. u No. of Questions in Exercises (x)—
r=0 u No. of Questions Attempted (y)—
(i) The coefficient of terms equidistant from the beginning and the end, are equal. u No. of Correct Questions (z)—
n
(ii) ( x − a)n = nC0 x n − nC1 x n − 1 a + L + (− 1)n Cnan (Without referring Explanations)

(iii) (1 + x)n = nC0 + nC1 x + nC2 x2 + L + nCn x n u Accuracy Level (z / y × 100)—


(iv) Total number of terms in the expansion ( x + a)n is (n + 1). u Prep Level (z / x × 100)—
(n + 1) (n + 2)
(v) If n is a positive integer, then the number of terms in ( x + y + z)n is .
2 In order to expect good rank in JEE,
your Accuracy Level should be
above 85 & Prep Level should be
above 75.
DAY 69

(vi) The number of terms in the expansion of


Properties of Binomial
n + 2

( x + a) + ( x − a) =  2
n n
, if n is even Coefficients
n+1 In the binomial expansion of (1 + x)n,
 , if n is odd
 2 (1 + x)n = nC0 + nC1 ⋅ x + nC2 ⋅ x2 + ... + nCr ⋅ x r + ... nCn ⋅ x n,
(vii) The number of terms in the expansion of
where, nC0, nC1,...,n Cn are the coefficients of various powers of
 n
, if n is even x are called binomial coefficients and it is also written as

( x + a) − ( x − a) =  2
n n
n+1  n  n  n
 , if n is odd C0, C1, ..., Cn or   ,   , … ,  
 2  0  1  n

n
Cr = nCn− r ●
n
Cr1 = n Cr2 ⇒ r1 = r2 or r1 + r2 = n
General Term and Middle Term
(i) Let (r + 1)th term be the general term in the expansion of n+ 1
n
Cr n−r +1

n
Cr + n Cr −1 = Cr • n
=
( x + a)n. Cr −1 r
Tr + 1 = nCr x n − r ar n−1
n
Cr n+ 1
Cr + 1

r ⋅ nCr = n ⋅ Cr −1 • =
(ii) If expansion is ( x − a)n, then the general term is r +1 n+1
C0 + C1 + C2 + ...+ Cn = 2 n
( − 1) r ⋅ n C r x n − r a r .

(iii) The middle term in the expansion of (a + x)n. ●


C0 + C2 + C4 + ... = C1 + C3 + C5 +... = 2 n − 1
n  C0 − C1 + C2 − C3 + ... + (− 1)n ⋅ Cn = 0
(a) Case I If n is even, then  + 1 th term is middle ●

2 
(2 n)!
term. ●
C20 + C12 + C22 + ... + C2n = 2 nCn =
(n + 1) (n !)2
(b) Case II If n is odd, then th term and
2 (−1)n/2 ⋅n Cn/2 , if n is even
(n + 3) ●
C20 − C12 + C22 − C32 + .... = 
th terms are middle terms.  0, if n is odd
2
(iv) ( p + 1)th term from end = (n − p + 1)th term from ●
C0 ⋅ Cr + C1 ⋅ Cr + 1 + ... + Cn − r ⋅ Cn
beginning. (2 n)!
= 2 nCn− r =
(v) For making a term independent of x we put r = n in (n − r )!(n + r )!
general term of ( x + a)n, so we get nCnan, that is ●
C1 − 2C2 + 3C3 − .... = 0
independent of x. ●
C0 + 2C1 + 3C2 + ... + (n + 1) ⋅ Cn = (n + 2) 2 n − 1
NOTE If the coefficients of rth, ( r + 1)th, ( r + 2)th term of ( 1 + x ) are
n

in AP, then n2 − ( 4 r + 1) n + 4 r 2 = 2

C0 − C2 + C4 − C6 +K = 2 n ⋅ cos
4


C1 − C3 + C5 − C7 + ... = 2 ⋅ sin
n

Greatest Term 4
If Tr and Tr + 1 be the rth and (r + 1)th terms in the expansion of
(1 + x)n, then Applications of Binomial
Tr + 1 Cr ⋅ x
n r
n−r +1 Theorem
= r −1
= ⋅x
Tr Cr − 1 ⋅ x
n
r 1. R-f Factor Relation
Let numerically, Tr + 1 be the greatest term in the above Here, we are going to discuss problems involving
Tr + 1 ( A + B)n = I + f , where I and n are positive integers
expansion. Then, Tr + 1 ≥ Tr or ≥ 1.
Tr 0 ≤ f ≤ 1,| A − B2| = k and| A − B| < 1.
n−r +1 (n + 1)
∴ | x | ≥ 1 or r ≤ | x| …(i) 2. Divisibility Problem
r (1 + | x |)
In the expansion, (1 + α )n. We can conclude that,
(i) Now, substituting values of n and x in Eq. (i), we get
r ≤ m + f or r ≤ m, where m is a positive integer and f is (1 + α )n − 1 is divisible by α, i.e. it is a multiple of α.
a fraction such that 0 < f < 1.
3. Differentiability Problem
(ii) When r ≤ m + f , Tm + 1 is the greatest term, when r ≤ m,
Sometimes to generalise the result we use the
Tm and Tm +1 are the greatest terms and both are equal.
differentiation.
(iii) The coefficients of the middle terms in the expansion of
(a + x)n are called greatest coefficients. (1 + x)n = nC0 + nC1 x + nC2 x 2 + … + nCn x n
70 40

On differentiating w.r.t. x, we get


n(1 + x)n−1 = 0 + n C1 + 2 ⋅ x ⋅ nC2 + … + n ⋅ nCn ⋅ x n−1
Principle of Mathematical
Put x = 1, we get, n 2 n−1 = nC1 + 2 nC2 + … + n nCn
Induction
In algebra, there are certain results that are formulated in
terms of n, where n is a positive integer. Such results can be
Binomial Theorem for proved by a specific technique, which is known as the
principle of mathematical induction.
Negative/Rational Index
Let n be a rational number and x be a real number such that
n(n − 1) 2 n(n − 1)(n − 2) 3
First Principle of Mathematical Induction
x < 1, then (1 + x)n = 1 + nx + x + x + ... It consists of the following three steps
2! 3!
Step I Actual verification of the proposition for the starting

If n is a positive integer, then (1 + x)n contains (n + 1) terms
value of i.
i.e. a finite number of terms. When n is any negative integer
or rational number, then expansion of (1 + x)n contains Step II Assuming the proposition to be true for k, k ≥ i and
infinitely many terms. proving that it is true for the value (k + 1) which is
next higher integer.

When n is a positive integer, then expansion of (1 + x)n is
valid for all values of x. If n is any negative integer or Step III To combine the above two steps. Let p(n) be a
rational number, then expansion of (1 + x)n is valid for the statement involving the natural number n such that
values of x satisfying the condition| x| < 1. (i) p(1) is true i.e. p(n) is true for n = 1.
(i) (1 + x)−1 = 1 − x + x2 − x3 + ... (ii) p(m + 1) is true, whenever p(m) is true
(ii) (1 − x)−1 = 1 + x + x2 + x3 + K i.e. p(m) is true ⇒ p(m + 1) is true. Then, p(n) is true
(iii) (1 + x)−2 = 1 − 2 x + 3 x2 − 4 x3 + ... for all natural numbers n.
(iv) (1 − x)−2 = 1 + 2 x + 3 x2 + 4 x3 + ... Product of r consecutive integers is divisible by r !

DAY PRACTICE SESSION 1

FOUNDATION QUESTIONS EXERCISE


1 If (1 + ax )n = 1 + 8x + 24x 3 + ..., then the values of a and n 6 If the 7th term in the binomial expansion of
9
are  3 
3 + 3 ln x  , x > 0 is equal to 729, then x can be
(a) 2, 4 (b) 2, 3 (c) 3, 6 (d) 1, 2  84  j
JEE Mains 2013
2 The coefficient of x n in the expansion of (1 + x )2n and (a) e 2 (b) e (c) e / 2 (d) 2e
(1 + x )2n −1 are in the ratio j
NCERT Exemplar 7 If the number of terms in the expansion of
(a) 1 : 2 (b) 1 : 3 n
 2 4
(c) 3 : 1 (d) 2 : 1 1 − + 2  , x ≠ 0 is 28, then the sum of the coefficients
 x x 
3 The value of (1. 002)12 upto fourth place of decimal is of all the terms in this expansion, is j
JEE Mains 2016
(a) 1.0242 (b) 1.0245 (a) 64 (b) 2187 (c) 243 (d) 729
(c) 1.0004 (d) 1.0254
8 In the binomial expansion of (a − b ) , n ≥ 5, the sum of 5th
n
4 The coefficient of x 4 in the expansion of (1 + x + x 2 + x 3 )n a
is and 6th terms is zero, then is equal to
b
(a) n
C4 (b) n
C4 + n C2 5 6
(a) (b)
(c) n
C4 + n C2 + n C2 (d) n
C4 + n C2 + n C1 ⋅ n C2 n−4 n −5
10 n −5 n−4
1  7 (c) (d)
5 If the middle term of  + x sin x  is equal to 7 , then 6 5
x  8
the value of x is j
NCERT Exemplar
9 In the expansion of the following expression
π π 1 + (1 + x ) + (1 + x )2 + ... + (1 + x )n , the coefficient of
(a) 2nπ + (b) nπ + x 4( 0 ≤ k ≤ n ) is
6 6
π π (a) n +1
Ck + 1 (b) n Ck
(c) nπ + (− 1)n (d) nπ + (− 1) n
6 3 (c) Cn −k −1 (d) None of these
DAY 71

10 The coefficient of t 24 in the expansion of 21 If the sum of the coefficients in the expansion of
(1 + t 2 )12(1 + t 12 )(1 + t 24 ) is ( x − 2y + 3z )n is 128, then the greatest coefficient in the
(a) 12
C6 + 2 (b) 12
C5 (c) 12
C6 (d) 12
C7 expansion of (1 + x )n is
11 The coefficient of x 53
in the following expansion (a) 35 (b) 20 (c) 10 (d) None of these
100
22. If for positive integers r > 1, n > 2, the coefficient of the
∑ 100Cm ( x − 3)100−m ⋅ 2m is ( 3r )th and (r + 2)th powers of x in the expansion of
m =0
(1 + x )2n are equal, then
(a) 100
C47 (b) 100
C53 (c) − 100
C53 (d) − 100
C100
(a) n = 2r (b) n = 3r
12 If p is a real number and if the middle term in the (c) n = 2r + 1 (d) None of these
8
p 
expansion of  + 2 is 1120, then the value of p is n
1 n
r
2 
j NCERT Exemplar
23 If an = ∑ n
, then ∑ n C is equal to
r =0 Cr r =0 r
(a) ±3 (b) ±1
(c) ± 2 (d) None of these (a) (n − 1)an (b) nan
1
6
(c) nan (d) None of these
 2 2
13 The constant term in the expansion of 1 + x +  , is
 x n
1 + rx
(a) 479 (b) 517 (c) 569 (d) 581
24 ∑ ( −1)r (n Cr ) 1 + nx is equal to
r =0

14 If in the expansion of (1 + x )m (1 − x )n , the coefficient of x (a) 1 (b) −1 (c) n (d) 0


and x 2 are 3 and −6 respectively, them m is  30  30  30  30  30  30
25     −     + ... +     is equal to
(a) 6 (b) 9 (c) 12 (d) 24  0  10  1   11  20  30
15 If n is a positive integer, then ( 3 + 1)2n − ( 3 − 1)2n is (a) 30
C11 (b) 60
C10 (c) 30
C10 (d) 65
C55
(a) an irrational number j AIEEE 2012
26 The value of ( C1 − C1) + ( C2 − C2 )
21 10 21 10

(b) an odd positive integer + ( 21C3 −10 C3 ) + ( 21C4 −10 C4 ) + ... + ( 21C10 −10 C10 ) is
(c) an even positive integer j JEE Mains 2017
(d) a rational number other than positive integers
(a) 2 21 − 211 (b) 2 21 − 210 (c) 2 20 − 2 9 (d) 2 20 − 210
21
 a b  27 The sum of the series
16 If the (r + 1) th term in the expansion of  3 + 
 b 3
a 20
C0 − 20 C1 + 20 C2 − 20 C3 + ...+ 20C10 is j
AIEEE 2007
1
has the same power of a and b, then the value of r is (a) − C10
20
(b) 20C10 (c) 0 (d) 20
C10
2
(a) 9 (b) 10 (c) 8 (d) 6
n −3 28 If (1 + x )n = C0 + C1x + C2x 2 +...+ Cn X n , then the value of
 1
17 If x 2k
occurs in the expansion of  x + 2  , then C0 + 2C1 + 3C2 + ...+ (n + 1)Cn will be
 x 
(a) (n + 2)2n −1 (b) (n + 1)2n
(a) n − 2k is a multiple of 2 (b) n − 2k is a multiple of 3 (c) (n + 1)2n −1 (d) (n + 2)2n
(c) k = 0 (d) None of these
29 If n > ( 8 + 3 7 )10, n ∈ N, then the least value of n is
18 The ratio of the coefficient of x 15 to the term
15 (a) (8 + 3 7 )10 − (8 − 3 7 )10
 2
(8 + 3 7 )10 + (8 − 3
independent of x in the expansion of  x 2 +  , is (b) 7 )10
 x (c) (8 + 3 7 )10 − (8 − 3 7 )10 + 1
j
JEE Mains 2013 (d) (8 + 3 7 )10 − (8 − 3 7 )10 − 1
(a) 7 : 16 (b) 7 : 64 (c) 1 : 4 (d) 1 : 32 30 49n + 16n − 1 is divisible by
20
 1  (a) 3 (b) 19 (c) 64 (d) 29
19 The greatest term in the expansion of 3 1 +  is
 3 31 If A = 1000 1000
and B = (1001) 999
, then
 20 1  20 1
(a)   (b)   (a) A > B (b) A = B
 7  27  6  81
(c) A < B (d) None of these
1  20
(c)   (d) None of these
9  9 32 If n − 1Cr = (k 2 − 3) ⋅ nCr + 1, then k belongs to
(a) (− ∞, − 2 ] (b) [2 , ∞) (c) [ − 3 , 3 ] (d) ( 3 , 2 ]
20 The largest term in the expansion of ( 3 + 2x )50, where
1 33 The remainder left out when 82n − ( 62)2n + 1 is divided by
x = is
5 9, is
(a) 5th (b) 3th (c) 7th (d) 6th (a) 0 (b) 2 (c) 7 (d) 8
72 40

34 If x is positive, the first negative term in the expansion of 37 For each n ∈ N, 23n − 1 is divisible by
(1 + x ) 27 / 5
is j AIEEE 2003 (a) 8 (b) 16
(a) 7th term (b) 5th term (c) 8th term (d) 6th term (c) 32 (d) None of these
35 Let P (n ) : n 2 + n + 1 ( n ∈ N ) is an even integer. Therefore, 38 Let S (k ) = 1 + 3 + 5+...+( 2k − 1) = 3 + k 2.
P (n ) is true Then, which of the following is true? j AIEEE 2004
(a) for n > 1 (b) for all n (c) for n > 2 (d) None of these (a) S(1) is correct
36 For all n ∈ N, 1 × 1! + 2 × 2 ! + 3 × 3! + ...+ n × n ! is equal (b) S (k) ⇒ S (k + 1)
to j NCERT Exemplar (c) S (k) ⇒/ S (k + 1)
(a) (n + 1) ! − 2 (b) (n + 1)! (d) Principle of mathematical induction can be used to
(c) (n + 1) ! − 1 (d) (n + 1)! − 3 prove the formula

DAY PRACTICE SESSION 2

PROGRESSIVE QUESTIONS EXERCISE


1 The coefficient of x 2m + 1 in the expansion of 7 If a and d are two complex numbers, then the sum to
E =
1
, x < 1 is (n + 1) terms of the following series
(1 + x )(1 + x )(1 + x 4 )(1 + x 8 )...(1 + x 2m )
2
aC0 − (a + d )C1 + (a + 2d )C2 − ... + ... is
(a) 3 (b) 2 (c) 1 (d) 0 a
(a) (b) na
2n
C C C
2 C1 − 2 + 3 – ... + ( − 1)n − 1 n is equal to (c) 0 (d) None of these
2 3 n
(−1)n − 1
n n
 n  m
(a) 1 −
1
+
1
−... + (b) 1 +
1 1
+ + ... +
1 8 ∑ ∑     is equal to
m  p 
2 3 n 2 3 n p =1 m = p
1 1 1
(c) 1 + + + ... + (d) None of these (a) 3n (b) 2n
2 3 n −1 (c) 3n + 2n (d) 3n − 2n
10
 1  9 The sum of the series
3 If the coefficient of x 5 in ax 2 + is a times and equal
 bx 
n
1 3r 7r 15r 
10 ∑ ( −1)r nCr  2r + 22r + 23r + 24r + ... + m terms is
 1  r =0
to the coefficient of x −5 in ax − 2 2 , then the value
 b x  2mn − 1 2mn − 1
(a) (b)
of ab is 2 (2n − 1)
mn
2n − 1
(a) (b)−3 (b) − (b)6 (c) (b)−1 (d) None of these 2mn + 1
(c) n (d) None of these
4 The sum of coefficients of integral powers of x in the 2 +1
binomial expansion of (1 − 2 x )50, is j
JEE Mains 2015 10 The value of x, for which the 6th term in the expansion of
7
(a)
1 50
(3 + 1 ) (b)
1 50
(3 )  log2 9 x −1 + 7 1 
2 +  is 84, is equal to
2 2 (1/ 5 )log2 (3 x −1 + 1)
1 50 1 50  2 
(c) (3 − 1 ) (d) (2 + 1 )
2 2 (a) 4 (b) 3 (c) 2 (d) 5
n
5 The term independent of x in expansion of  1 
11 If the last term in the binomial expansion of  21/ 3 − 
 x +1 x −1   2
 2/ 3 −  is
x − x 1/ 3 + 1 x − x 1/ 2  log3 8
JEE Mains 2013  1 
j

is  5/ 3  , then the 5th term from the beginning is


(a) 4 (b) 120 (c) 210 (d) 310 3 
6 If (1 + x )n = C0 + C1x + C2x 2 + ... + Cn x n , then (a) 210 (b) 420
C02 + C12 + C22 + C32 + ... + Cn2 is equal to (c) 105 (d) None of these
n! (2n)! 12 The sum of the coefficients of all odd degree terms in the
(a) (b)
n !n ! n !n ! expansion of
(2n)!
(c) (d) None of these ( x + x 3 − 1)5 + ( x − x 3 − 1)5,( x > 1) is j
JEE Mains 2018
n!
(a) −1 (b) 0 (c) 1 (d) 2
DAY 73

13 The greatest value of the term independent of x, as α 15 If the ratio of the fifth term from the beginning to the fifth
20 n
 sin α   1 
varies over R, in the expansion of  x cos α +  is term from the end in the expansion of  4 2 + 4  is
 x   3
20 20 20 6 : 1, then
(a) C10 (b) C15 (c) C19 (d) None of these
14 Statement I For each natural number Statement I The value of n is 10.
n− 4
n,(n + 1)7 − n 7 − 1 is divisible by 7.
2 4 ⋅ 3 −1
Statement II For each natural number n, n 7 − n is divisible Statement II = 6 NCERT Exemplar
4 +n j

by 7. j AIEEE 2011
2⋅3 4
(a) Statement I is false, Statement II is true
(a) Statement I is true; Statement II is true; Statement II is a
(b) Statement I is true, Statement II is true, Statement II is correct explanation for Statement I
correct explanation of Statement I.
(b) Statement I is true; Statement II is true; Statement II is
(c) Statement I is true, Statement II is true; Statement II is not a correct explanation for Statement I
not a correct explanation of Statement I
(c) Statement I is true; Statement II is false
(d) Statement I is true, Statement II is false
(d) Statement I is false; Statement II is true

ANSWERS
SESSION 1 1 (a) 2 (d) 3 (a) 4 (d) 5 (c) 6 (b) 7 (d) 8 (d) 9 (a) 10 (a)
11 (c) 12 (c) 13 (d) 14 (c) 15 (a) 16 (a) 17 (b) 18 (d) 19 (a) 20 (c)
21 (a) 22 (c) 23 (c) 24 (d) 25 (c) 26 (d) 27 (b) 28 (a) 29 (b) 30 (c)
31 (a) 32 (d) 33 (b) 34 (c) 35 (d) 36 (c) 37 (d) 38 (b)

1 (c) 2 (b) 3 (b) 4 (a) 5 (c) 6 (b) 7 (c) 8 (d) 9 (a) 10 (c)
SESSION 2
11 (a) 12 (d) 13 (d) 14 (b) 15 (c)

Hints and Explanations


10
1 Given that,(1 + ax )n = 1 + 8 x + 24 x2 + ... 3 We have, (1. 002)12 or it can be 5  1 + x sin x 
n n(n − 1) 2 2 rewritten as (1 + 0. 002) 12
x 
⇒ 1+ ax + a x + ...
1 1⋅ 2 ⇒ (1. 002)12 = 1 + 12 C1 ( 0. 002) Here, n = 10 [even]
⇒ Middle term =  + 1 th = 6th
= 1 + 8 x + 24 x2 + ... + 12 C2 (0. 002)2 + 12 C3 (0. 002)3 + ... 10
On comparing the coefficients of x, x2 ,  2 
We want the answer upto 4 decimal 10− 5
C 5  
we get places and as such we have left further 1
T6 = 10
( x sin x )5
n(n − 1) 2 expansion.  x
na = 8, a = 24
1⋅ 2 ∴(1. 002)12 = 1 + 12( 0. 002) 7 63
⇒ 252(sin x )5 = 7 =
⇒ na(n − 1)a = 48 12⋅ 11 12⋅ 11⋅ 10 8 8
+ (0. 002)2 + (0. 002)3 + ... 1 1
⇒ 8(8 − a) = 48 ⇒ 8 − a = 6 1⋅ 2 1⋅ 2⋅ 3 ⇒ (sin x ) =
5
⇒ sin x =
⇒ a=2 ⇒ n= 4 32 2
= 1 + 0. 024 + 2. 64 × 10−4 +1.76 × 10−6 + ...
⇒ sin x = sin π/6
2 Coefficient of x in (1 + x ) =
n 2n 2n
Cn = 1. 0242 π
∴ x = nπ + (− 1)n
and coefficient of x n 4 (1 + x + x2 + x3 )n = {(1 + x )n (1 + x2 )n } 6
in (1 + x )2 n −1 =2 n −1 C n = (1 + nC1 x + nC2 x2 + nC3 x3 3

∴ Required ratio + nC 4 x 4 +… nC n x n )
6 T7 = 9C 6  3 3  ( 3 ln x )6 = 729
 84
(2n )!
2n
Cn (1 + nC1 x2 + nC2 x 4 + K + nC n x2 n ) 84 × 33
= = n ! n ! = 2 :1 ⇒ × 33 × (ln x )6 = 729
2 n −1
Cn (2n − 1)! Therefore, the coefficient of x 4 84
n !(n − 1)! = nC2 + nC2 nC1 + nC 4 = (ln x )6 = 1
= nC 4 + nC2 + nC1 nC2 ⇒ x=e
74 40

7 Clearly number of terms in the ⇒ p 4 = 16 17 The general term in the expansion of


n −3
expansion of ⇒ p=±2 x+ 1 
n   is given by
 1 − 2 + 4  is (n + 2)(n + 1) or n +2 C . 6  x2 
 2 13  1 + x + 2  = 1 +    x + 2 
2 6
 x x  2 r
 x  1  x T r +1 = n −3 C r ( x )n −3 − r  2 
1
1 1
[assuming and 2 distinct] 2 6
x 
x  6  6
+    x +  + ... +    x + 
x 2 2
(n + 2)(n + 1) = n −3 C r x n −3 −3 r
∴ = 28  2  x  6  x
2 As x2 k occurs in the expansion of
∴ Constant term n −3
⇒ (n + 2)(n + 1) = 56 = (6 + 1)(6 + 2)  6  2  6  4 2  x + 1  , we must have
= 1 +     21 +    2 +  
⇒ n=6    
2 1  4  2  x2 
Hence, sum of coefficients
 6  6 3 n − 3 − 3r = 2k for some non-negative
= (1 − 2 + 4)6 = 36 = 729    2 integer r.
 6  3 
⇒ 3(1 + r ) = n − 2k
8 Since, in a binomial expansion of = 1 + 60 + 360 + 160 = 581 ⇒ n − 2k is a multiple of 3.
(a − b )n , n ≥ 5, the sum of 5th and 6th
14 (1 + x )m (1 − x )n r
terms is equal to zero. 18 T r + 1 = 15C r ( x2 )15 − r ⋅  2 
∴ n C 4 an − 4 (−b )4 + n C 5an − 5(−b )5 = 0  m(m − 1)x  2  x
= 1 + mx + + ...
n! 2 ! 30 − 2 r
⋅ 2 ⋅ x−r
an − 4 ⋅ b 4   = 15 r
⇒ Cr x
(n − 4)! 4! 1 − nx + n(n − 1) x2 −... 30 − 3 r
= 15
Cr ⋅ x ⋅ 2r …(i)
n!
− an − 5b 5 = 0  2!  For coefficient of x , put 30 − 3 r = 15
15
(n − 5)! 5!
= 1 + (m − n )x
⇒ 3 r = 15 ⇒ r = 5
an − 5 ⋅ b 4  −  = 0
n! a b
⇒  n2 − n (m2 − m ) 2
+ − mn + ∴ Coefficient of x15 = C 5 ⋅ 25
15
(n − 5)! 4!  n − 4 5  x + ...
a n−4  2 2  For coefficient of independent of x
⇒ =
b 5 Given, m − n = 3 ⇒ n = m − 3 i.e. x 0 put 30 − 3r = 0
9 The given expression is n2 − n m2 − m
and − mn + = −6 ⇒ r = 10
1 + (1 + x ) + (1 + x )2 + ...+ (1 + x )n being in 2 2 ∴ Coefficient of x 0 = C10 ⋅ 210
15
GP. (m − 3)(m − 4)
⇒ − m(m − 3) Coefficient of x15
Let, S = 1 + (1 + x ) + (1 + x )2 + ...+ (1 + x )n 2 By condition ⇒
n +1 m2 − m Coefficient of x 0
(1 + x ) −1 + = −6
= = x −1 [(1 + x )n +1 − 1] 2 C 5 ⋅ 25
15 15
C ⋅ 25
(1 + x ) − 1 = 15 = 15 10 10 = 1 : 32
⇒ m2 − 7m + 12 − 2m2 + 6m C10 ⋅ 2 10
C10 ⋅ 2
∴ The coefficient of x k in S. + m2 − m + 12 = 0
= The coefficient of x k +1 in ⇒ −2m + 24 = 0 ⇒ m = 12 19 Greatest term in the expansion of
[(1 + x )n +1 − 1] (1 + x )n is T r
2 n −1 +1
= n +1
C k +1 15 ( 3 + 1) = 2n 2n
C 0( 3 ) 2n
+ 2n
C1 ( 3 )
+ 2n
C2 ( 3 )2 n −2
+ ...+ 2n
C2 n ( 3 ) 2 n −2 n  (n + 1)x 
where, r =  
10 We have, (1 + t 2 )12 (1 + t 12 )(1 + t 24 ) ( 3 − 1)2 n =2 n C 0( 3 )2 n (−1)0  1+ x 
= (1 + 12 C1t 2 + 12 C2t + ...+ 12 C 6t 12 1
+ 2 n C1 ( 3 )2 n −1 (−1)1 + 2 n C2 ( 3 )2 n −2 (−1)2 + ... Here, n = 20, x =
+ ...+ 12 C12t 24 + ... )(1 + t 12 + t 24 + t 36 ) 3
+ 2 n C2 n ( 3 )2 n −2 n (−1)2 n
∴Coefficient of t 24 in  21 
Adding both the binomial expansions ∴ r = 
(1 + t 2 )12 (1 + t 12 )(1 + t 24 )
above, we get  3 + 1
= 12
C6 + 12
C12 + 1 = 12
C6 + 2 ( 3 + 1)2 n − ( 3 − 1)2 n = 2[2 n C1 ( 3 )2 n −1 = [10.5 ( 3 − 1)] = (7.69) ≈ 7

11 The given sigma expansion + 2 n C3 ( 3 )2 n −3 + 2 n C 5( 3 )2 n − 5 Hence, greatest term is


7
100 + ... + 2 n C2 n −1 ( 3 )2 n −(2 n −1 )]  20  1   20 1

100
C m ( x − 3) 100− m
⋅ 2 can be written
m 3    =  .
m= 0 which is most certainly an irrational  7   3  7  27
as [( x − 3) + 2]100 = ( x − 1)100 = (1 − x )100 number because of odd powers of 3 in 50
(3 + 2 x )50 = 350  1 +
2x 
∴Coefficient of x 53 in each of the terms. 20 Q 
 3
100
(1 − x )100 = (−1)53 C 53 = −100C 53 16 ∴General term is r
T r +1 = 350 50C r  
21 − r
2x
r Here,
8  a   b   3
= 21C r  3   
12 Given expression is  + 2
p Tr +1
2   b  3
a r −1
T r = 350 50C r −1  
2x
and
Here, n = 8 [even] 7−
r 2r 7
−  3
= 21
Cr a 2 ⋅ b3 2
⇒ Middle term =  + 1 th term
8 1
But x = (given)
2  Q Power of a = Power of b [given] 5
50
= 5 th term r 2 7 ∴
T r +1 C 2 1
≥ 1 ⇒ 50 r ⋅ ≥ 1
⇒ 7− = r −
T 5 = 8C 4 ( p / 2)8− 4 (24 ) 2 3 2 Tr C r −1 3 5
8 × 7 × 6 × 5 p4 ∴ r =9 ⇒ 102 − 2r ≥ 15r ⇒ r ≤ 6
⇒ × × 24 = 1120
4 × 3 × 2 × 1 24
DAY 75

21 Sum of the coefficients in the expansion 26 (21 C1 −10 C1 ) + (21 C2 −10 C2 ) (1001)999 < (1000)1000
of ∴ B< A
+ (21 C3 −10 C3 )+ ...+ (21 C10 −10 C10 )
( x − 2 y + 3z )n is (1 − 2 + 3)n = 2n n −1 n n −1
(put x = y = z = 1) = (21 C1 + 21 C2 + ...+ 21 C10 ) 32 Since, C r = (k 2 − 3) Cr
r+1
∴ 2n = 128 ⇒ n = 7 −(10C1 + 10 C2 + ... + 10 C10 ) r+1
Therefore, the greatest coefficient in the ⇒ k2 − 3 =
1 n
expansion of (1 + x )7 is 7 C3 or 7 C 4 = (21 C1 + 21 C2 + ... + 21 C20 ) − (210 − 1)
2 ⇒ 0 < k2 − 3 ≤ 1
because both are equal to 35. 1 Qn ≥ r ⇒ r + 1 ≤ 1and n, r > 0
= (21 C1 + 21C2 + ... + 21C21 − 1) − (210 − 1)
22 In the expansion of (1 + x )2 n , the general 2  n 
1
term =2 n C k x k , 0 ≤ k ≤ 2n = (221 − 2) − (210 − 1) = 220 − 1 − 210 + 1 ⇒ 3 < k2 ≤ 4
2
As given for r > 1, n > 2, Hence, k ∈ [−2, − 3] ∪ ( 3,2)
= 220 − 210
2n
C3 r =2 n C r +2
27 We know that, 33 82 n − (62)2 n +1 = (1 + 63)n − (63 − 1)2 n +1
⇒ Either 3r = r + 2
(1 + x )20 =20 C 0 + 20 C1 x + ... = (1 + 63)n + (1 − 63)2 n +1
or 3r = 2n − (r + 2)
= [1 + C1 ⋅ 63 + n C2 ⋅ ( 63)2 + ... + ( 63)n ]
n
(Q n C x = nC y ⇒ x + y = n or x = y ) + 20C10 x10 + ... + 20 C20 x20
On putting x = −1 in the above + [1 −2 n −1 C1 ⋅ 63 + (2 n +1 ) C2 ⋅ (63)2 − ...
⇒ r = 1 or n = 2r + 1
expansion, we get + (−1)(63)(2 n +1 )] ]
We take the relation only
0 = 20C 0 − 20C1 + ... − 20C a + 20C10 = 2 + 63[ C1 + C2 ( 63) + ...
n n
n = 2r + 1 (Q r > 1)
− 20C11 + ... + 20C20 + (63)n −1 −(2 n +1 ) C1
n
r n
n − (n − r )
23 Let b = ∑ n
= ∑ n ⇒ 0 = C 0 − C1 + ... − C 9 + C10
20 20 20 20
+ (2 n +1 )
C2 (63) − ...+ (−1)(63)(2 n )]
r=0 C r r=0 Cr
n
1 n
n−r − C 9 + ... + C 0
20 20
Hence, remainder is 2.
= n∑ − ∑
r=0
n
Cr r=0
n
Cr ⇒ 0 = 2( C 0 − C1 + ... − C 9 ) + C10
20 20 20 20 34 Since, (r + 1)th term in the expansion of
n
n−r ⇒ C10 = 2(20C 0 − 20C1 + ... + 20C10 )
20 (1 + x )27 / 5
= nan − ∑ nC (Q C r = C n − r )
n n
27  27
− 1 ...  − r + 1
27
r=0 n− r 1
⇒ 20C 0 − 20C1 + ... + 20C10 = 20C10 
5 5   5  r
n 2 = x
= nan − b ⇒ 2b = nan ⇒ b = an r!
2 28 Since, x(1 + x )n = xC 0 + C1 x2 Now, this term will be negative, if the
24 Let E = ∑ (−1) C r  1 + rx 
n
r n + C2 x3 + ... + C n x n +1 last factor in numerator is the only one
r=0
 1 + nx  negative factor.
On differentiating w.r.t. x, we get
1  n 27 32
=  (1 + x )n + nx(1 + x )n −1 ⇒ − r + 1< 0 ⇒ <r
 1 + nx  r∑
 (−1) C r (1 + rx )
r n
5 5
=0 = C 0 + 2C1 x + 3C2 x2 ⇒ 6. 4 < r ⇒ least value of r is 7.
1  n
= 
 1 + nx   r∑
  (−1) ⋅ C r + ... + (n + 1)C n x n
r n
Thus, first negative term will be 8th.
=0 Put x = 1, we get 35 Given, P (n ) : n2 + n + 1
n
 C 0 + 2C1 + 3C2 + ... + (n + 1)C n
+ x ∑ r (−1)r nC r  At n = 1, P (1) : 3, which is not an even
r=0  = 2n + n2n −1 = 2n −1 (n + 2) integer.
= 
1  29 Let f = (8 − 3 7 )10, here 0 < f < 1 Thus, P(1) is not true.
 (0 + 0) = 0
 1 + nx  Also, n(n + 1) + 1 is always an odd
∴ (8 + 3 7 )10 + (8 − 3 7 )10 is an integer,
[Q n C 0− nC1 + nC2 − nC3 + ...(−1)n nC n = 0] integer.
hence this is the value of n.
36 Let the statement P (n ) be defined as
25 Let 30 We have,
 30  30  30  30 P ( n ) : 1 × 1! + 2 × 2! + 3 × 3! K
A =    −   49 + 16n − 1 = (1 + 48) + 16n − 1
n n

 0   10  1   11 + n × n ! = (n + 1) ! − 1
= 1 + C1 (48) + C2 (48) + ...
n n 2
for all natural numbers n.
 30  30  30  30 + n C n (48)n + 16n − 1 Note that P(1) is true, since
+     − ... +    
  
2 12  20  30 = ( 48n + 16n ) + C2 ( 48)2 + n C3 ( 48)3 +
n
P (1) : 1 × 1! = 1 = 2 − 1 = 2! − 1
or A = 30
C 0 . C10 − C1 . C11
30 30 30
... + n C n (48)n Assume that P (n ) is true for some
natural number k, i.e.
+ 30C2 . 30C12 − ... + 30C20 . 30C30 = 64n + 8 [ C2 ⋅ 6 + C3 ⋅ 6 ⋅ 8
2 n 2 n 3
P (k ) : 1 × 1! + 2 × 2! + 3 × 3! + ....
= Coefficient of x 20
in + n C 4 ⋅ 64 82 + ... + n C n ⋅ 6n ⋅ 8n −2 ] + k × k ! = (k + 1)! − 1 …(i)
(1 + x )30(1 − x )30 Hence, 49n + 16n − 1 is divisible by 64. To prove P (k + 1) is true, we have
= Coefficient of x 20
in (1 − x ) 2 30 n P (k + 1) : 1 × 1! + 2 × 2!
31 Since,  1 + 1  < 3 for ∀n ∈ N + 3 × 3! + ... + k × k !
= Coefficient of x 20
in  n
30 + (k + 1) × (k + 1)!
∑ (−1) C r ( x )
r 30 2 r 1000
(1001)999 1  1001  = (k + 1) ! − 1 + (k + 1)! × (k + 1)
Now, =  
r=0
(1000)1000 1001  1000  [by Eq. (i)]
= (−1)10 30C10 1000
= (k + 1 + 1)(k + 1)! – 1
1  1  1
(for coefficient of x20, let r = 10) = 1 +  < ⋅3 < 1
1001  1000  1001 = (k + 2)(k + 1)! – 1 = (k + 2)! – 1
=30 C10
76 40

Thus, P (k + 1) is true, whenever P (k ) is 1


x + x2 + ...+ x n − 1 ) dx 1 + 350
true. Therefore, by the principle of
= ∫ 0(1 + ⇒ = C 0 + C2 (2)2 + K + C 50(2)50
1 2
mathematical induction,P (n ) is true for  x2 xn  1 1 1 10
= x + +…+ = 1 + + + ...+
all natural numbers n.  2 n  0 2 3 n 5  2 /3 x +11/3 −
( x − 1) 
 x − x + 1 x − x1 /2 
37 Now, 23 n − 1 = (23 )n − 1 = (1 + 7)n − 1 3 General term is  ( x1 /3 )3 + 13
10
r
{( x )2 − 1} 
C r ⋅ (a ⋅ x2 )10 − r ⋅  
1 =  2 /3 −
Tr = 10
= 1 + nC1 ⋅ 7 + nC2 ⋅ 72 + K x ( x − 1) 
+1
 bx  x − x +1
1 /3
r
+ nC n ⋅ 7n − 1
= 10C r ⋅ (a)10 − r   ( x )20 − 3 r
1  ( x1 /3 + 1)( x2 /3 + 1 − x1 /3 )
b =
= 7[ C1 + C2 7 + K + nC n ⋅ 7n − 1 ]
n n
x2 /3 − x1 /3 + 1
 10
Since, x 5 occurs in T r + 1.
{( x )2 − 1} 
Hence, 7 divides 23n − 1 for all n ∈ N . − 
∴ 20 − 3 r = 5 x ( x − 1) 
38 S (k ) = 1 + 3 + 5 + ... + (2k − 1) = 3 + k 2 ⇒ 3 r = 15 ⇒ r = 5  ( x + 1)
10

= ( x1 /3 + 1) − = ( x1 /3 − x −1 /2 )10
Put k = 1 in both sides, we get So, the coefficient of x 5 is 10C 5(a)5(b )−5.
 x 
LHS = 1 and RHS = 3 + 1 = 4
Again, let x −5 occurs in T r +1 of ∴The genreal term is
⇒ LHS ≠ RHS
10 T r +1 =10 C r ( x1 /3 )10− r (− x −1 /2 )r
Put (k + 1) in both sides in the place of  1  10− r  1 
r

 a ⋅ x − 2 2  is C r (ax ) − 2 2 
10
10− r r
k, we get −
 b ⋅ x   b x  =10 C r (−1)r x 3 2
LHS = 1 + 3 + 5 + ... + (2k − 1) + (2k + 1) r
C r (a)10 − r  − 2  ( x )10 − 3 r
1 For independent for x, put
RHS = 3 + (k + 1)2 = 3 + k 2 + 2k + 1 = 10
 b  10 − r r
Let LHS = RHS − = 0 ⇒ 20 − 2r − 3r = 0
10 − 3 r = − 5 ⇒ 15 = 3 r ⇒ r = 5 3 2
1 + 3 + 5 + ... + (2k − 1) + (2k + 1) ⇒ 20 = 5r ⇒ r = 4
= 3 + k 2 + 2k + 1 a5
So, the coefficient of x −5 is − 10C 5 10 . 10 × 9 × 8 × 7
∴ T 5 =10 C 4 = = 210
⇒ 1 + 3 + 5+ ... + (2k − 1) = 3 + k 2 b 4×3×2×1
If S (k ) is true, then S (k + 1) is also true. According to the given condition,
a5 a5
6 We have,
Hence, S (k ) ⇒ S (k + 1) 10
C 5 5 = − a 10C 5 10 (1 + x )n = C 0 + C1 x + C2 x2 + ... + C n x n
b b
... (i)
SESSION 2 ⇒ − b 5 = a ⇒ − b 6 = ab n 2
 1 1
and  1 +  = C 0 + C1 + C2    1
1 Multiplying the numerator and 4 Let T r +1 be the general term in the  x x  x
denominator by 1 − x, we have expansion of (1 − 2 x )50. n
+ ... + C n   ... (ii)
1
1− x
E = ∴ Tr = 50 50− r
C r (1) (− 2 x 1 /2 r
)  x
+1
(1 − x )(1 + x )(1 + x2 )(1 + x 4 )
m = 50
Cr ⋅ 2 ⋅ x
r r /2
( − 1) r On multiplying Eqs. (i) and (ii) and taking
...(1 + x2 ) coefficient of constant terms in right hand
For the integral power of x and r should
1− x side = C 20 + C12 + C22 + ... + C 2n
= m
be even integer.
n
(1 − x2 )(1 + x2 )(1 + x 4 )...(1 + x2 )
In right hand side (1 + x )n  1 +  or in
25 1
∴Sum of coefficients = ∑
50
1− x C2 r (2)2 r
=  x
m
r =0
(1 − x 4 )(1 + x 4 )...(1 + x2 ) 1
1 1
= [(1 + 2)50 + (1 − 2)50] = [350 + 1] (1 + x ) or term containing x n in
2n

1− x m +1 2 2 xn
= = (1 − x )(1 − x2 )−1 (1 + x )2 n . Clearly, the coefficient of x n in
2 m +1 Alternate Method
(1 − x ) (2n )!
m +1 m +2 We have, (1 + x )2 n is equal to 2 n C n = .
= (1 − x )(1 + x2 + x2 + ... ) (1 − 2 x )50 = C 0 − C1 ⋅ 2 x n !n !
m +1
∴Coefficient of x 2
is 1. + C2 (2 x )2 + K + C 50(2 x )50 …(i) 7 We can write,
aC 0 − (a + d )C1 + ( a + 2d )C2 − ...
2 Since, (1 − x ) = C 0 − C1 ⋅ x + C2 ⋅ x
n 2
(1 + 2 x ) 50
= C 0 + C1 ⋅ 2 x
upto (n + 1) terms
− C3 ⋅ x3 + ... + C2 (2 x )2 + K + C 50(2 x )50 …(ii) = a(C 0 − C1 + C2 − ... )
⇒ 1 − (1 − x ) = C1 ⋅ x − C2 ⋅ x
n 2
On adding Eqs. (i) and (ii), we get + d (−C1 + 2C2 − 3C3 + ... ) ... (i)
+ C3 ⋅ x3 – ... (1 − 2 x )50 + (1 + 2 x )50 We know,
1 − (1 − x ) n
= 2[C 0 + C2 (2 x )2 + ... + C 50(2 x )50] (1 − x )n = C 0 − C1 x + C2 x2
⇒ = C1 − C2 ⋅ x
x (1 − 2 x )50 + (1 + 2 x )50 −...+ (−1)n C n x n ... (ii)

+ C3 ⋅ x2 − K 2 On differentiating Eq. (ii) w.r.t. x,
1 = C 0 + C2 (2 x )2 + K + C 50 (2 x )50 we get − n(1 − x )n −1 = −C1 + 2C2 x
⇒ ∫ 0(C1 − C2 ⋅ x + C3 ⋅ x2 − K ) dx
− ... + (−1)n C n nx n −1 ... (iii)
On putting x = 1, we get
11 − (1 − x )n
= ∫0 1 − (1 − x )
dx (1 − 2 1 )50 + (1 + 2 1 )50 On putting x = 1 in Eqs. (ii) and (iii), we
get
2
C1 C2 C 11 − x n C 0 − C1 + C2 − ... + (−1)n C n = 0 ... (iv)
⇒ − + 3 −K = ∫ dx = C 0 + C2 (2)2 + K + C 50 (2)50
0 1 − x
1 2 3
(− 1)50 + (3)50 and −C1 + 2C2 − ... + (−1)n C n = 0 ... (v)
Q 1
f ( x ) dx = ∫ f (1 − x ) dx 
1 ⇒
 ∫ 0
From Eq. (i),
0  2
= C 0 + C2 (2)2 + K + C 50(2)50
DAY 77

aC 0 − (a + d )C1 + (a + 2d )C2 − ... + upto 32x  3x  Thus, the greatest possible value of β is
⇒ +7= 4 + 1
(n + 1) terms 10
C10   .
9  3  20 1
= a⋅ 0 + d ⋅ 0 = 0  2
⇒ 32x − 12(3x ) + 27 = 0
[from Eqs. (iv) and (v)] ⇒ y 2 − 12 y + 27 = 0 (put y = 3x ) 14 Let P (n ) = (n )7 − n
By mathematical induction
8 Since,     =
n m n! ⇒ ( y − 3)( y − 9) = 0
(n − m )! p ! (m − p )! For n = 1,
 m  p  ⇒ y = 3, 9
P(1) = 0, which is divisible by 7.
 n  n − p  ⇒ 3x = 3, 9
=   For n = k
 ⇒ x = 1,2
 p  m − p P (k ) = k 7 − k
n
∴Given series can be rewritten as
11 Last term of  21 /3 − 1  is
Let P (k ) be divisible by 7.
 2 ∴ k 7 − k = 7λ, for some λ ∈ N ... (i)
n n
 n  n − p 
∑ ∑   
 p  m − p

 1 
n For n = k + 1,
p=1 m = p T n +1 = nC n (21 /3 )n − n  −  P (k + 1) = (k + 1)7 − (k + 1)
n n   n  n − p  2
= ∑   ∑
 p m = p
 
 m − p 1 (−1)n
= (7 C 0k 7 + 7 C1 k 6 + 7 C2 k 5 + ...+ 7 C 6 ⋅ k
p=1 = nC n (−1)n n /2
= + 7 C7 ) − (k + 1)
2 2n /2
n
 n n− p
 n − p = (k − k ) + 7{k + 3k + ... + k}
7 6 5

= ∑  
 p
∑ 
 t 

Also, we have
log3 8 = 7λ + 7{k 6 + 3k 5 + ... + k} [Using Eq. (i)]
p=1 t=0  1  = 3−( 5/3 )log3 2 = 2−5
3
 5/3  ⇒ Divisible by 7.
n
 n n − 3 
= ∑ [ put m – p = t ]
p
 2 So, both statements are true and
 p (−1) n
(−1) (−1) n 10
p=1 Thus, = 2−5 ⇒ n /2 = Statement II is correct explanation of
 n  2n /2 2 25
n
 n 1 n  1 Statement I.
= 2n ∑   ⋅ p = 2   1 +  − 1
 p 2  2 ⇒
n
= 5 ⇒ n = 10
p=1   2 15 We know that, in the expansion of
4
(a + b )n , pth term from the end is
Now, T 5 = T 4 +1 = 10C 4 (21 /3 )10− 4  −
=3 −2 n n 1 
 (n − p + 2)th term from the beginning.
n  2
∑ (−1)
r
9 10! 1 /3 6 So, 5th term from the end is
4 −1 /2 4
r=0 = (2 ) (−1) (2 )
4! 6! = (n − 5 + 2) th term from the beginning
1 3r 7r 
n
C r  r + 2 r + 3 r + ... upto m terms  = 210(2)2 (1)(2−2 ) = 210 = (n − 3) th term from the beginning
2 2 2  = (n − 4 + 1) th term from the beginning
n
1 n
3r 12 Key idea = (a + b )n + (a − b )n …(i)
= ∑ (−1) C r ⋅ r + ∑ (−1) ⋅ C r 2 r
r n r n

r=0 2 r=0 2 = 2( n C 0an + n C2 an −2b 2 + n C 4 an − 4b 4 ... ) ∴ We have,


n n
n
7 r We have 4 1   1/4 + 1 
+ ∑ (−1)r n C r + ...  2 + 4  = 2 
23 r (x + x3 − 1 )5 + ( x − x3 − 1 )5, x > 1  3  31 / 4 
r=0
n n n Now, 5th term from the beginning is
= 2( 5C 0 x 5 + 5C2 x3 ( x3 − 1 )2
=  1 −  +  1 −  +  1 −  + ...
1 3 7
T 4 + 1 = nC 4 (21 / 4 )n − 4 (3−1 / 4 )4
 2  4  8
+ 5C 4 x( x3 − 1 )4 ) n− 4
upto m terms
= 2( x + 10 x ( x − 1) + 5x( x3 − 1)2 )
5 3 3 = nC 4 2 4 3 −1 …(ii)
1 1 1
= n + 2 n + 3 n ... upto m terms
2 2 2 = 2( x + 10 x − 10 x +5x − 10 x + 5x)
5 6 3 7 4 And 5th term from the end is
T( n − 4 ) + 1 = nC n − 4 (21 / 4 )n − n + 4 (31 / 4 )n − 4
1  1 
m Sum of coefficients of all odd degree

1−  n 
n  terms is n +4
  −
2  2  = 2 −1
mn
= 2(1 − 10 + 5 + 5) = 2 = nC 4 2 ⋅ 3 4

1 − 1  2 (2n − 1)
mn
[Q C r = n C n − r ] …(iii)
n
  13 The general term in the expansion of
 2 
n
20 So, from the given condition, we have
 x cos α + sin α 
10 We have,   is Fifth term from the beginning 6
 x  =
7 r Fifth term from the end 1
 log2  sin α 
9x −1 +7
+
1 20
C r ( x cos α )20− r   n−4
2 x −1
+1 )   x  n
C4 ⋅2 4 ⋅ 3 −1 6
 (1 / 5)log2 (3
2  ⇒ =
−n + 4
 1 
7
= 20C r x20−2 r (cos α )20− r (sin α )r 1
=  9x −1 + 7 + x −1
n
C 4. ⋅ 2 ⋅ 3 4

 (3 + 1)1 / 5  For this term to be independent of x, we n−4  4 −n 


−1 −1 −  
get  4 
7−5 1 
5
⇒ 2 4 ⋅3 = 6
∴T 6 = C 5( 9 7 x −1
+ 7) 20 − 2r = 0
 (3x −1 + 1)1 / 5 
n−8 n−8
 ⇒ r = 10 ⇒ 2 4 3 4 = 61 /2
= C 5(9
7 x −1
+ 7)
1 Let β = Term indeoendent of x n−8
(3x −1 + 1) = 20C10(cos α )10(sin α)10 ⇒ (2 × 3) 4 = (2 ⋅ 3)1 /2
(9x −1 + 7) = 20C10(cos α sin α)10 n−8
⇒ 84 = 7C 5 ⇒ = 1/2
(3x −1 + 1) 10 4
sin 2α 
⇒ 9 x −1
+ 7 = 4(3 x −1
+ 1) = C10 
20
 ⇒ n = 2 + 8 ∴ n = 10
 2 
DAY EIGHT

Permutations and
Combinations
Learning & Revision for the Day
u Fundamental Principle of u Circular Permutations u Prime Factors
Counting u Combinations u Division of Objects into
u Factorial Notation u Applications of Permutations Groups
u Permutations and Combinations

Fundamental Principle of Counting


The fundamental principle of counting is a way to figure out the total number of ways in
which different events can occur. If a certain work A can be done in m ways and another
work B in n ways, then
(i) the number of ways of doing the work C, which is done only when either A or B is
done, is m + n. (addition principle)
(ii) the number of ways of doing the work C, which is done only when both A and B are
done, is mn. (multiplication principle)

Factorial Notation
The product of first n natural numbers is denoted by n! and read as ‘factorial n’.
Thus, n ! = n (n − 1) (n − 2) … 3 ⋅ 2 ⋅1 PRED
Properties of Factorial Notation MIRROR
Your Personal Preparation Indicator
1. 0 ! = 1 ! = 1
2. Factorials of negative integers and fractions are not defined. u No. of Questions in Exercises (x)—
3. n ! = n (n − 1)! = n (n − 1) (n − 2)! u No. of Questions Attempted (y)—
n! u No. of Correct Questions (z)—
4. = n (n − 1) (n − 2)… (r + 1) (Without referring Explanations)
r!
u Accuracy Level (z / y × 100)—
Permutations u Prep Level (z / x × 100)—

Permutation means arrangement of things. The number of permutations of n
different things taken r at a time is n Pr . In order to expect good rank in JEE,
your Accuracy Level should be
n! above 85 & Prep Level should be

n
Pr = n (n − 1) (n − 2)...(n − r + 1) = ,0 ≤ r ≤ n above 75.
(n − r )!
DAY 79

Properties of n Pr Number of circular permutations of n different things, taken


n +1
r at a time, when clockwise and anti-clockwise orders are
(i) n P0 = 1, n P1 = n , n Pn = n ! (ii) n
Pr + r ⋅n Pr − 1 = Pr n
P
n −1
not different, is r .
(iii) Pr = n
n
Pr − 1 (iv) Pr = (n − r + 1) Pr − 1
n n
2r
n −1 n −1
(v) Pr = (n − r ) Pr − 1
Important Results on
Important Results on Permutations Circular Permutations
(i) Number of permutations of n different things taken r at a (i) The number of ways in which m different things and n
time when a particular thing is to be always included in different things (where, m ≥ n) can be arranged in a circle,
each arrangement is r ⋅ n −1 Pr −1 . so that no two things of second kind come together is
(ii) Number of permutations of n different things taken r at a (m − 1)! m Pn .
time, when a particular thing is never taken in each
(ii) The number of ways in which m different things and n
arrangement is n −1 Pr .
different things can be arranged in a circle, so that all the
(iii) The number of permutations of n different things taken r
second type of things come together is m ! n !.
at a time, allowing repetitions is nr .
(iv) The permutations of n things of which p are identical of (iii) The number of ways in which m different things and n
one sort, q are identical of second sort, r are identical of different things (where, m ≥ n) can be arranged in the form
n! of garland, so that no two things of second kind come
third sort, is , where p + q + r = n.
p!q ! r ! together is (m − 1)! m Pn / 2.
(v) Arrangements (iv) The number of ways in which m different things and n
(a) The number of ways in which m different things and n different things can be arranged in the form of garland, so
different things (m + 1 ≥ n) can be arranged in a row, so that all the second type of things come together is m ! n !/ 2 .
that no two things of second kind come together is
m ! ( m +1 )Pn . Combinations
(b) The number of ways in which m different things and n
Combination means selection of things. The number of
different things (m ≥ n) can be arranged in a row so that combinations of n different things taken r at a time is
all the second type of things come together is  n
n
(m + 1)! n !. Cr or   .
 r
(vi) Dearrangement The number of dearrangements
(No object goes to its scheduled place) of n objects, is  n n (n − 1) (n − 2)...(n − r + 1) n! n
P
n
Cr =   = = = r
1 1 1 1 r r! r !(n − r )! r!
n! − + − ... (− 1)n .
2 ! 3 ! 4 ! n !
(vii) Sum of Digits Properties of n Cr
(a) Sum of numbers formed by taking all the given n digits
(i) n
Cr = nCn − r (ii) n Cx = nC y ⇒ x = y or x + y = n
(excluding 0) is (sum of all the n digits)
n +1
× (n − 1)! × (111 ... n times). (iii) n
Cr + Cr − 1 =
n
Cr
(b) Sum of the numbers formed by taking all the given n
digits (including 0) is (sum of all the n digits) Important Results on Combinations
× [(n − 1)! × (111 ... n times) − (n − 2) ×
{111 ...(n − 1) times}]. (i) The number of combinations of n different things, taken r
at a time, when p particular things always occur is
n− p
Cr − p.
Circular Permutations
(ii) The number of combinations of n different things, taken r

If different objects are arranged along a closed curve, then
permutation is known as circular permutation. at a time, when p particular things never occur is n − pCr .

The number of circular permutations of n different things (iii) The number of selections of zero or more things out of n
taken all at a time is (n − 1)!. If clockwise and anti-clockwise different things is n C0 + nC1 + … + nCn = 2 n .
orders are taken as different. (iv) The number of selections of one or more things out of

If clockwise and anti-clockwise circular permutations are n different things is
(n − 1)! n
C1 + nC2 + … + nCn = 2 n − 1.
considered to be same, then it is .
2 (v) The number of selections of zero or more things out of n

Number of circular permutations of n different things, identical things = n + 1.
taken r at a time, when clockwise and anti-clockwise (vi) The number of selections of one or more things out of n
n
P identical things = n.
orders are taken as different, is r .
r
80 40

(vii) The number of selections of one or more things from ( p1α 1 + 1 − 1) ( p2α 2 + 1 − 1) ( prα r + 1 − 1)
(ii) Sum of divisors of nis ...
p + q + r things, where p are alike of one kind, q are alike ( p1 – 1) ( p2 − 1) ( pr − 1)
of second kind and rest are alike of third kind, is
[( p + 1) (q + 1) (r + 1)] − 1. (iii) If p is a prime such that pr divides n! but pr +1 does not
divide n ! .
(viii) The number of selections of one or more things from p
 n  n 
identical things of one kind, q identical things of second Then, r =   +  2  + K
kind, r identical things of third kind and n different  p  p 
things, is ( p + 1) (q + 1) (r + 1) 2 n − 1
(ix) If there are m items of one kind, n items of another kind
and so on. Then, the number of ways of choosing r items
Division of Objects into Groups
out of these items = coefficient of x r in
(1 + x + x2 + K + x m ) (1 + x + x2 + K + x n )K Objects are Different
(x) If there are m items of one kind, n items of another kind (i) The number of ways of dividing n different objects into 3
and so on. Then, the number of ways of choosing r items groups of size p, q and r ( p + q + r = n) is
out of these items such that atleast one item of each kind n!
is included in every selection = coefficient of x r in (a) ; p, q and r are unequal.
p!q ! r !
( x + x2 + K + x m )( x + x2 + K + x n )K
n! n!
(b) ;q = r (c) ; p =q = r
p ! 2 !(q !)2 3 !( p !)3
Applications of Permutations
(ii) The number of ways in which n different things can be
and Combinations distributed into r different groups, if empty groups are
allowed, is r n .
Functional Applications (iii) The number of ways in which n different things can be
If the set A has m elements and B has n elements, then distributed into r different groups, if empty groups are not
allowed, is
(i) the number of functions from A to B is nm .
 r  r
(ii) the number of one-one functions from A to B is n Pm , m ≤ n. r n −   (r − 1)n +   (r − 2)n − … + (−1)r − 1 rCr − 1 ⋅ 1
1 2

(iii) the number of onto functions from A to B is


 n  n Objects are Identical
nm −   (n − 1)m +   (n − 2)m − ..., m ≤ n.
1 2 (i) The number of ways of dividing n identical objects among
r persons such that each one may get atmost n objects, is
(iv) the number of bijections from A to B is n!, if m = n.
 n + r − 1
  . In other words, the total number of ways of
 r −1 
Geometrical Applications dividing n identical objects into r groups, if blank groups
(i) Number of triangles formed from n points, when no three are allowed, is n + r − 1 Cr − 1 .
points are collinear, is n C3 .
(ii) The total number of ways of dividing n identical objects
(ii) Out of n non-concurrent and non-parallel straight lines,
among r persons, each one of whom, receives atleast one
the points of intersection are n C2 .
item, is n − 1 Cr − 1 . In other words, the number of ways in
(iii) The number of diagonals in a polygon of nsides is n C2 − n. which n identical things can be divided into r groups
(iv) The number of total triangles formed by the n points on a such that blank groups are not allowed, is n − 1 Cr − 1 .
plane of which m are collinear, is n C3 − mC3 .
(iii) Number of non-negative integral solutions of the equation
(v) The number of total different straight lines, formed by the x1 + x2 + … + x r = n is equivalent to number of ways of
n points on a plane, of which m are collinear, is distributing n identical objects into r groups if empty
n
C2 − mC2 + 1 . groups are allowed, which is n + r −1 Cr −1 .
(iv) Number of positive integral solutions of the equation
Prime Factors x1 + x2 + …+ x r = n is equivalent to the number of ways of
Let n = p1α 1 ⋅ p2α 2 ⋅ p3α 3 K prα r , where pi , i = 1, 2, K , r are distinct distributing n identical objects into r groups such that no
primes and α i , i = 1, 2 , K , r are positive integers. group empty, which is n −1 Cr −1 .
(i) Number of divisor of n is (v) Number of integral solutions of the equation
(α 1 + 1) (α 2 + 1) (α 3 + 1)K (α r + 1). x1 + x2 + … + x r = n, where a ≤ x i ≤ b , ∀ i = ,1, 2,… , r , is
given by coefficient of x n in ( x a + x a + 1 + x a + 2 + …+ xb )r .
DAY 81

DAY PRACTICE SESSION 1

FOUNDATION QUESTIONS EXERCISE


1 The value of 2n [1 ⋅ 3 ⋅ 5 K ( 2 n − 3) ( 2 n − 1)] is 11 The total number of permutations of n ( > 1) different things
(2n)! (2 n)! n! taken not more than r at a time, when each thing may be
(a) (b) (c) (d) None of these
n! 2n (2 n)! repeated any number of times is
n (n n − 1) nr − 1
2 The value of1⋅1! + 2 ⋅ 2! + 3 ⋅ 3! + … + n ⋅ n !, is (a) (b)
n −1 n −1
(a) (n + 1) ! (b) (n + 1)! + 1
n (n r − 1)
(c) (n + 1) ! − 1 (d) None of these (c) (d) None of these
n −1
3 How many different nine-digit numbers can be formed n n
n
from the digits of the number 223355888 by Pr −1 Pr Pr + 1
12 If = = , then
rearrangement of the digits so that the odd digits occupy a b c
even places? (a) b 2 = a (b + c) (b) c 2 = a (b + c)
(a) 16 (b) 36 (c) 60 (d) 180 (c) ab = a 2 + bc (d) bc = a 3 + b 2
7 − x
4 A library has a copies of one book, b copies of each of 13 The range of the function f ( x ) = Px − 3 is j AIEEE 2004
two books, c copies of each of three books and single (a) {1, 2, 3 } (b) {1, 2, 3, 4, 5, 6}
copies of d books. The total number of ways in which (c) {1, 2, 3, 4} (d) {1, 2, 3, 4, 5}
these books can be arranged, is 14 Find the number of different words that can be formed
(a + b + c + d)! (a + 2b + 3c + d)! from the letters of the word TRIANGLE, so that no vowels
(a) (b)
a! b ! c! a !(b !)2 (c !)3 are together. j NCERT Exemplar

(a + 2b + 3c + d)
(c) (d) None of these (a) 14000 (b) 14500 (c) 14400 (d) 14402
a! b ! c!
15 In a class of 10 students, there are 3 girls. The number of
5 The number of words which can be formed out of the ways they can be arranged in a row, so that no 2 girls are
letters of the word ARTICLE, so that vowels occupy the consecutive is k ⋅ 8!, where k is equal to
even place is j
NCERT Exemplar
(a) 12 (b) 24 (c) 36 (d) 42
(a) 1440 (b) 144 (c) 7! (d) 4
C4 × 3 C3
16 The sum of all the 4-digit numbers that can be formed
6 In how many ways the letters of the word ‘ARRANGE’ can using the digits 1, 2, 3, 4 and 5 without repetition of the
be arranged without altering the relative positions of digits is
vowels and consonants?
(a) 399960 (b) 288860 (c) 301250 (d) 420210
(a) 36 (b) 26
(c) 62 (d) None of these 17 If eleven members of a committee sit at a round table so
that the President and Secretary always sit together, then
7 If the letters of the word ‘SACHIN’ are arranged in all
the number of arrangements is
possible ways and these words are written in dictionary
order, then the word ‘SACHIN’ appears at serial number (a) 10! × 2 (b) 10!
(c) 9! × 2 (d) None of these
(a) 600 (b) 601 (c) 602 (d) 603
18 Let f : {1, 2, 3, 4, 5} → {1, 2, 3, 4, 5} that are onto and
8 The number of integers greater than 6000 that can be
f ( x ) ≠ x , is equal to
formed, using the digits 3, 5, 6, 7 and 8 without
(a) 9 (b) 44
repetition, is j
JEE Mains 2015
(c) 16 (d) None of these
(a) 216 (b) 192 (c) 120 (d) 72
19 There are 4 balls of different colours and 4 boxes of same
9 The number of 5-digits telephone numbers having atleast
colours as those of the balls. The number of ways in
one of their digits repeated, is j
NCERT Exemplar
which the balls, one in each box, could be placed such
(a) 90000 (b) 100000 (c) 30240 (d) 69760 that a ball does not go to box of its own colour is
10 If all the words (with or without meaning) having five (a) 8 (b) 9 (c) 7 (d) 1
letters, formed using the letters of the word SMALL and
20 How many different words can be formed by jumbling the
arranged as in dictionary, then the position of the word
letters in the word ‘MISSISSIPPI’ in which no two S are
SMALL is j
JEE Mains 2016
adjacent? j
AIEEE 2008
(a) 46th (b) 59th
(a) 7 ⋅ 6 C4 ⋅ 8 C4 (b) 8 ⋅ 6 C4 ⋅ 7 C4
(c) 52nd (d) 58th
(c) 6 ⋅ 7 ⋅ 8 C4 (d) 6 ⋅ 8 ⋅ 7 C4
82 40

21 The number of ways in which seven persons can be 31 If the total number of m elements subsets of the set
arranged at a round table, if two particular persons may A = {a1, a 2 , a 3 , ..., a n } is λ times the number of m elements
not sit together is subsets containing a 4 , then n is
(a) 480 (b) 120 (a) (m − 1) λ (b) mλ (c) (m + 1) λ (d) 0
(c) 80 (d) None of these 32 A guard of 12 men is formed from a group of n soldiers in
22 The number of ways in which 6 men and 5 women can sit all possible ways. If the number of times two particular
at a round table, if no two women are to sit together, is soldiers A and B are together on guard is thrice the
given by j AIEEE 2003 number of times three particular soldiers C, D, E are
(a) 6! × 5 ! (b) 30 (c) 5 ! × 4 ! (d) 7 ! × 5 ! together on guard, then n is equal to
5 (a) 18 (b) 24 (c) 32 (d) 36
23 The value of 47
C4 + ∑
r =1
52 − r
C3 is equal to
33 In a steamer, there are stalls for 12 animals and there are
47
horses, cows and calves (not less than 12 each) ready to
(a) C6 (b) 52 C5
52
be shipped. In how many ways, can the ship load be
(c) C4 (d) None of these
made?
n +1
24 If n C3 + nC4 > C3 , then (a) 312 − 1 (b) 312 (c) (12)3 − 1 (d) (12)3
(a) n > 6 (b) n > 7 34 There are 10 points in a plane, out of these 6 are
(c) n < 6 (d) None of these
collinear. If N is the number of triangles formed by joining
25 The number of ways in which we can choose a these points, then j AIEEE 2011

committee from four men and six women so that the (a) N > 190 (b) N ≤ 100
committee includes atleast two men and exactly twice as (c) 100 < N ≤ 140 (d) 140 < N ≤ 190
many women as men is j NCERT Exemplar
35 Let Tn be the number of all possible triangles formed by
(a) 94 (b) 126 joining vertices of an n-sided regular polygon. If
(c) 128 (d) None of these
Tn +1 − Tn = 10, then the value of n is j JEE Mains 2013

26 A box contains 2 white balls, 3 black balls and 4 red (a) 7 (b) 5
balls. The number of ways of drawing 3 balls from the (c) 10 (d) 8
box, if atleast one black ball is included, is 36 On the sides AB, BC, CA of a ∆ABC, 3, 4, 5 distinct points
(a) 36 (b) 42 (c) 56 (d) 64 (excluding vertices A , B , C) are respectively chosen. The
27 A student is allowed to select atmost n books from a number of triangles that can be constructed using these
collection of ( 2n + 1) books. If the number of ways in chosen points as vertices are j
JEE Mains 2013
which he can select atleast one book is 63, then n is (a) 210 (b) 205
equal to (c) 215 (d) 220
(a) 3 (b) 4 (c) 6 (d) 5 37 The number of divisors of the number of 38808
28 Let A and B two sets containing 2 elements and 4 (excluding 1 and the number itself) is
elements, respectively. The number of subsets of A × B (a) 70 (b) 72
having 3 or more elements is j
JEE Mains 2013 (c) 71 (d) None of these
(a) 256 (b) 220 (c) 219 (d) 211 38 If a, b, c, d , e are prime integers, then the number of
29 In an examination of 9 papers a candidate has to pass in divisors of ab 2c 2de excluding 1 as a factor is
more papers than the number of papers in which he fails, (a) 94 (b) 72 (c) 36 (d) 71
in order to be successful. The number of ways in which 39 The number of ways of distributing 8 identical balls in
he can be unsuccessful is 3 distinct boxes, so that no box is empty, is
(b)  
(a) 255 (b) 256 (c) 193 (d) 319 8
(a) 5
 3
30 There are two urns. Urn A has 3 distinct red balls and
urn B has 9 distinct blue balls. From each urn, two balls (c) 3 8 (d) 21
are taken out at random and then transferred to the 40 If 4 dice are rolled, then the number of ways of getting
other. The number of ways in which this can be done, is the sum 10 is
j
AIEEE 2010 (a) 56 (b) 64
(a) 3 (b) 36 (c) 66 (d) 108 (c) 72 (d) 80
DAY 83

DAY PRACTICE SESSION 2

PROGRESSIVE QUESTIONS EXERCISE


1 From 6 different novels and 3 different dictionaries, 4 novels Y together can throw a party inviting 3 ladies and 3 men,
and 1 dictionary are to be selected and arranged in a row so that 3 friends of each of X and Y are in this party, is
on a shelf, so that the dictionary is always in the middle. j JEE Mains 2017
The number of such arrangements is j JEE Mains 2018
(a) 485 (b) 468 (c) 469 (d) 484
(a) atleast 1000 (b) less than 500 8 The number of ways in which we can select four numbers
(c) atleast 500 but less than 750 from 1 to 30 so as to exclude every selection of four
(d) atleast 750 but less than 1000 consecutive numbers is
2 Sixteen men compete with one another in running, (a) 27378 (b) 27405 (c) 27399 (d) None of these
swimming and riding. How many prize lists could be 9 The number of numbers divisible by 3 that can be formed
made, if there were altogether 6 prizes for different by four different even digits is
values, one for running, 2 for swimming and 3 for riding? (a) 36 (b) 18 (c) 0 (d) None of these
(a) 16 × 15 × 14 (b) 163 × 15 2 × 14 10 The total number of integral solutions ( x , y , z ) such that
(c) 163 × 15 × 14 2 (d) 162 × 15 × 14
xyz = 24 is
3 The number of ways of dividing 52 cards amongst four (a) 36 (b) 90 (c) 120 (d) None of these
players so that three players have 17 cards each and the 11 Assuming the balls to be identical except for difference in
fourth players just one card, is colours, the number of ways in which one or more balls
52 ! 52 !
(a) (b) 52! (c) (d) None of these can be selected from 10 white, 9 green and 7 black balls
(17 !)3 17 ! is j AIEEE 2012

4 If the letters of the word MOTHER are written in all (a) 880 (b) 629 (c) 630 (d) 879
possible orders and these words are written out as in a 12 The number of ways in which an examiner can assign
dictionary, then the rank of the word MOTHER is 30 marks to 8 questions, giving not less than 2 marks to
(a) 240 (b) 261 (c) 308 (d) 309 any question, is j
JEE Mains 2013
 a11 a12   (a) 30
C7 (b) 21
C8 (c) 21
C7 (d) 30
C8
5 Let S =   : aij ∈ {0, 1, 2 }, a11 = a22 . Then, the number
 a
 21 22a   13 The number of divisors of the form 4n + 1, n ≥ 0 of the
of non-singular matrices in the set S, is j
JEE Mains 2013 number 1010 1111 1313 is
(a) 27 (b) 24 (c) 10 (d) 20 (a) 750 (b) 840 (c) 924 (d) 1024
6 A group of 6 is chosen from 10 men and 7 women so as 14 Out of 8 sailors on a boat, 3 can work only one particular
to contain atleast 3 men and 2 women. The number of side and 2 only the other side. Then, number of ways in
ways this can be done, if two particular women refuse to which the sailors can be arranged on the boat is
serve on the same group, is (a) 2718 (b) 1728 (c) 7218 (d) None of these
(a) 8000 (b) 7800 (c) 7600 (d) 7200 15 In a cricket match between two teams X and Y , the team
7 A man X has 7 friends, 4 of them are ladies and 3 are X requires 10 runs to win in the last 3 balls. If the possible
men. His wife Y also has 7 friends, 3 of them are ladies runs that can be made from a ball be 0, 1, 2, 3, 4, 5 and 6.
and 4 are men. Assume X and Y have no common The number of sequence of runs made by the batsman is
friends. Then, the total number of ways in which X and Y (a) 12 (b) 18 (c) 21 (d) 36

ANSWERS
SESSION 1 1. (a) 2. (c) 3. (c) 4. (b) 5. (b) 6. (a) 7. (b) 8. (b) 9. (d) 10. (d)
11. (c) 12. (a) 13. (a) 14. (c) 15. (d) 16. (a) 17. (c) 18. (b) 19. (b) 20. (a)
21. (a) 22. (a) 23. (c) 24. (a) 25. (a) 26. (d) 27. (a) 28. (c) 29. (b) 30. (d)
31. (b) 32. (c) 33. (b) 34. (b) 35. (b) 36. (b) 37. (a) 38. (d) 39. (d) 40. (d)

SESSION 2 1. (a) 2. (b) 3. (a) 4. (d) 5. (d) 6. (b) 7. (a) 8. (a) 9. (a) 10. (c)
11. (d) 12. (c) 13. (c) 14. (b) 15. (d)
84 40

Hints and Explanations


SESSION 1 Four digit number can start from 6,7 or 8. 14 In a word TRIANGLE, vowels are (A, E,
1 Clearly, [1 ⋅ 3 ⋅ 5 ...( 2n − 3)(2 n − 1 )]2 n 6, 7 or 8 I) and consonants are (G, L, N, R, T).
First, we fix the 5 consonants in
1 ⋅ 2 ⋅ 3 ⋅ 4 ⋅ 5⋅ 6...(2 n − 1 ) (2 n ) 2 n
= alternate position in 5! ways.
2 ⋅ 4 ⋅ 6... 2 n _G_L_N_R_T_
3 4 3 2
(2 n ) ! 2n (2 n )! In rest of the six blank position, three
= = Thus, total number of 4-digit number,
2 (1 ⋅ 2 ⋅ 3... n )
n
n! vowels can be arranged in 6 P3 ways.
which are greater than
n ∴ Total number of different words
6000 = 3 × 4 × 3 × 2 = 72
2 Clearly, given expression = ∑ r ⋅r! = 5! × 6 P3 = 120 × 6 × 5 × 4 = 14400
r =1 Case II When number is of 5 digit.
n n
Total number of 5-digit number, which 15 The 7 boys can be arranged in row in 7!
= ∑
r =1
((r + 1) − 1) r ! = ∑
r =1
((r + 1)! − r !)
are greater than 6000 = 5! = 120 ways. There will be 6 gaps between
them and one place before them and one
= (2! − 1!) + (3!− 2!) + …+ ((n + 1)! − n !) ∴ Total number of integers
place after them. The 3 girls can be
= (n + 1)! − 1 = 72 + 120 = 192 arranged in a row in 8 P3 = 8 ⋅ 7 ⋅ 6 ways
3 In a nine digits number, there are four 9 Using the digits 0, 1, 2, ..., 9 the number ∴ Required number of ways
even places for the four odd digits 3, 3, of five digits telephone numbers which = 7! × 8 ⋅ 7 ⋅ 6 = 42 ⋅ 8!
5, 5. can be formed is 10 5 (since, repetition is
allowed). The number of five digits
16 Required sum
4! 5!
∴ Required number of ways = ⋅ telephone numbers, which have none of = (Sum of all the n-digits)
2! 2! 2! 3!
the digits repeated = 10 P5 = 30240 × n −1 Pr −1 × (111 ... r times)
= 60
∴ The required number of telephone = (1 + 2 + 3 + 4 + 5) 4 P3 × (1111)
4 Total number of books = a + 2b + 3c + d number = 105 − 30240 = 69760 = 15 × 24 × 1111 = 399960
∴ The total number of arrangements 10 Clearly, number of words start with
(a + 2b + 3c + d )! 17 Since, out of eleven members two
= 4!
A=
= 12 members sit together, the number of
a!(b !) 2 (c !) 3 2! arrangements = 9! × 2
5 In a word ARTICLE, vowels are A, E, I Number of words start with L = 4! = 24 (Q two members can be sit in two ways.)
4!
and consonants are C, L, R, T. Number of words start with M = = 12 18 Total number of functions
In a seven letter word, there are three 2! = Number of dearrangement of 5 objects
3!
even places in which three vowels are =3
= 5!  − −  = 44
Number of words start with SA= 1 1 1 1
placed in 3! ways. In rest of the four 2! +
 2! 3! 4! 5!
places, four consonants are placed in 4! Number of words start with SL = 3! = 6
ways. Note that, next word will be ‘‘SMALL’’. 19 Use the number of dearrangements
∴ Required number of ways i.e. n ! 1 − + − + ... + (− 1)n 
Hence, position of word ‘‘SMALL’’ is 58th. 1 1 1 1
= 3! × 4! = 6 × 24 = 144 11 When we arrange things one at a time,  1! 2! 3! n !
6 Clearly, the consonants in their the number of possible permutations is Here, n = 4
4! n. When we arrange them two at a time So, the required number of ways
positions can be arranged in = 12
= 4!  − +  = 12 − 4 + 1 = 9
2! the number of possible permutations are 1 1 1
ways and the vowels in their positions n × n = n2 and so on. Thus, the total  2! 3 ! 4!
3! number of permutations are
can be arranged in = 3 ways 20 Given word is MISSISSIPPI.
2! n (n r − 1)
n + n2 + K + n r = [Q n > 1] Here, I = 4 times, S = 4 times,
∴ Total number of arrangements n−1
P = 2 times, M = 1 time
= 12 × 3 = 36 n
Pr − 1 n n
Pr + 1
Pr _M_I_I_I_I_P_P_
12 Q = =
7 The letters of given word are A, C, H, I, a b c ∴ Required number of words
N, S. b 7! 7 × 6!
From first two terms =n−r +1 = 8C 4 × = 8C 4 × = 7 ⋅ 8C 4 ⋅ 6C 4
Now, the number of words starting with a 4!2! 4!2!
A = 5! c
From last two terms =n−r 21 Clearly, remaining 5 persons can be
the number of words starting with C = 5! b
the number of words starting withH = 5! b c seated in 4! ways. Now, on five cross
Hence, = + 1 ⇒ b 2 = a (b + c ) marked places person can sit in 5 P2 ways.
the number of words starting with I = 5! a b P4
and the number of words starting with 13 Given that, f ( x ) = 7−x
Px − 3 . The above
N = 5!. P3
Then, next word is SACHIN. function is defined, if 7 − x ≥ 0 and P5
x − 3 ≥ 0 and 7 − x ≥ x − 3.
So, the required serial number is
⇒ x ≤ 7, x ≥ 3 and x ≤ 5
= (5⋅ 5!) + 1 = 601.
∴ D f = {3, 4, 5} P2
8 The integer greater than 6000 may be of Now, f (3) = 4 P0 = 1 P1
4 digits or 5 digits. So,here two cases arise. f (4) = 3 P1 = 3 and f (5) = 2 P2 = 2 So, number of arrangements
Case I When number is of 4 digit. 5!
∴ R f = {1, 2, 3} = 4! × = 24 × 20 = 480 ways
3!
DAY 85

22 First, we fix the position of men, number 29 Clearly, the candidate is unsuccessful, if n (n − 1)
⇒ × 3 = 10
of ways in which men can sit = 5!. he fails in 9 or 8 or 7 or 6 or 5 papers. 3!
∴ Numbers of ways to be unsuccessful ⇒ n − n − 20 = 0 ⇒ n = 5
2
Now, the number of ways in which
women can sit = 6 P5 = 9C 9 + 9C 8 + 9C7 + 9C 6 + 9C 5 36 Required number of triangles that can be
= 9C 0 + 9C1 + 9C2 + 9C3 + 9C 4 constructed using these chosen points as
M
vertices = 12C3 − 3C3 − 4C3 − 5C3
1
M = ( 9 C 0 + 9C1 + K + 9C 9 ) Here, we subtract those cases in which
M 2 points are collinear
1 = 220 − 1 − 4 − 10 = 220 − 15 = 205
= (29 ) = 28 = 256
2
M M 37 Since, 38808 = 23 × 32 × 72 × 111
30 ∴ Number of divisors
M
= 4 × 3 × 3 × 2 − 2 = 72 − 2 = 70
∴ Total number of ways 38 The number of divisors of ab2c 2de
3 distinct 9 distinct
= 5! × 6 P5 = 5! × 6! = (1 + 1 ) (2 + 1 ) (2 + 1 ) (1 + 1 ) (1 + 1 ) − 1
5
red balls blue balls
∑ = 2 ⋅ 3 ⋅ 3 ⋅ 2 ⋅ 2 − 1 = 71
52 − r
23 47
C4 + C3 = C4 +
47 51
C3
r =1 Urn A Urn B
+ 50
C3 + C3 +
49 48
C3 + 47
C3 39 Required number of ways is equal to the
The number of ways in which 2 balls number of positive integer solutions of
= 51
C3 + 50
C3 + 49
C3 + 48
C3 + ( C3 +
47 47
C4) from urn A and 2 balls from urn B can the equation x + y + z = 8 which equal
= 52
C4 be selected = 3C2 × 9C2 = 3 × 36 = 108
 8 − 1  7
24 n
C3 + nC 4 > n +1
C3 31 Total number of m elements subsets of to   =   = 21
 3 − 1   2
⇒ n +1
C4 > n +1
C3 ( nC r + n C r + 1 = n +1 C r +1 ) A = nC m …(i)
n +1
C n−2 and number of m elements subsets of A 40 Coefficient of x10 in ( x + x2 + ...+ x 6 )4
⇒ n +1 4 > 1 ⇒ > 1 ⇒n> 6
C3 4 each containing element a4 = n − 1 C m − 1 = Coefficient of x 6 in (1 + x + ...+ x 5 )4
According to the question, = (1 − x 6 )4 (1 − x ) − 4 in (1 − 4 x 6 + ... )
25 The number of ways in which we can
choose a committee = λ ⋅ n −1 C m −1   4 
= Choose two men and four women ⇒
n n −1
⋅ C m −1 = λ ⋅ n −1C m −1 1 +  1  x + ...
 
+ Choose three men and six women m
n  9
= 4C2 × 6C 4 + 4C3 × 6C 6 ⇒ λ= ⇒ n = mλ Hence, coefficient of x is   − 4 = 80.
6

m  6
= 6 × 15 + 4 × 1 = 90 + 4 = 94
32 Number of times A and B are together
26 The number of ways of drawing SESSION 2
 n − 2
1 black and 2 non-black balls is on guard is  .
3
C1 ⋅ 6C2 = 3 ⋅ 15 = 45  10  1 Given, 6 different novels and 3 different
The number of ways of drawing Number of times C , D and E are together dictionaries.
2 black and 1 non-black ball is  n − 3 Number of ways of selecting 4 novels
on guard is  . 6!
3
C2 ⋅ 6C1 = 3 ⋅ 6 = 18  9  from 6 novels is 6 C 4 = = 15
The number of ways of drawing 2! 4!
According to the question,
3 black balls is 3 C3 = 1 Number of ways of selecting
 n − 2  n − 3
∴ Number of ways = 45 + 18 + 1 = 64   = 3  1 dictionary from 3 dictionaries is
 10   9  3!
27 He can select 1, 2, ... or n books.
3
C1 = =3
⇒ n − 2 = 30 ⇒ n = 32 1!2!
The number of ways to select atleast one Number of arrangement of
book is 33 First stall can be filled in 3 ways, second
stall can be filled in 3 ways and so on. 4 novels and 1 dictionary where
2n + 1
C1 + 2 n + 1 C2 + K + 2 n + 1 C n dictionary is always in the middle, is 4!
∴ Number of ways of loading steamer
1
= (2 n + 1 C1 + 2 n + 1 C2 + K + 2 n + 1 C n = 3C1 × 3C1 × K × 3C1 (12 times) Required number of arrangement
2
= 3 × 3 × K × 3 (12 times) = 312 15 × 3 × 4! = 45 × 24 = 1080
+ 2 n + 1 C n + 1 + K + 2 n + 1 C2 n)
1 2n + 1 2n + 1
= (2 − C 0 − 2 n + 1 C2 n + 1 ) 34 If out of n points, m are collinear, then 2 Number of ways of giving one prize for
2 running = 16
Number of triangles = C3 − C3
n m

= 22n − 1 = 63 [given]
∴ Required number of triangles
Number of ways of giving two prizes for
⇒ 22 n = 64 = 26 ⇒ n = 3 swimming = 16 × 15
= 10C3 − 6C3 = 120 − 20
Number of ways of giving three prizes
28 Given, n( A ) = 2, n(B ) = 4. ⇒ N = 100 for riding = 16 × 15 × 14
∴ n (A × B) = 8 35 T n = nC3 , hence T n+1 = n +1
C3 ∴ Required ways of giving prizes
The number of subsets of A × B having 3 = 16 × 16 × 15 × 16 × 15 × 14
Now, T n +1 − T n = 10
or more elements = 8C 3 + 8C 4 +…+ 8C 8 n +1 = 163 × 152 × 14
⇒ C3 − nC3 = 10 [given]
= ( 8 C 0 + 8C1 + 8C2 + 8C3 +…+ 8C 8 ) (n + 1) n (n − 1)
⇒ 3 For the first player, cards can be
− ( 8 C 0 + 8C1 + 8C2 ) 3! distributed in the 52 C17 ways. Now, out
= 2 − C 0 − C1 − 8C2
8 8 8
n (n − 1) (n − 2) of 35 cards left 17 cards can be
− = 10
= 256 − 1 − 8 − 28 = 219 3! distributed for second player in 52 C17
[Q 2n = nC 0 + nC1 +…+ nC n ] n (n − 1) ways.
⇒ (n + 1 − n + 2) = 10
3!
86 40

Similarly, for third player in 18 C17 ways. where, 5C2 is the number of ways without [Qall white balls are mutually identical]
One card for the last player can be W1 and W2 and 5C1 is the number of ways Number of ways to choose zero or more
distributed in 1 C1 way. with W1 and without W2 or with W2 and from green balls = (9 + 1)
Therefore, the required number of ways without W1 .
[Qall green balls are mutually identical]
for the proper distribution. The number of ways of forming 3M , 3W
Number of ways to choose zero or more
group is 10 C3 ( 5C3 + 2C1 C2 ) = 3600
5
= 52C17 × 35C17 × 18C17 × 1 C1 from black balls = (7 + 1)
52! 35! 18! 52! where, 5C3 is the number of ways without
= × × × 1! = [Qall black balls are mutually identical]
35! 17! 18!17! 17!1! (17!) 3 W1 and W2 and 5C2 is the number of ways
Hence, number of ways to choose zero
with W1 or W2 but not both.
4 The number of words starting from or more balls of any colour
∴ Number of ways = 4200 + 3600 = 7800
E = 5! = 120 = (10 + 1) (9 + 1) (7 + 1)
The number of words starting from H 7 Given, X has 7 friends, 4 of them are ladies Also, number of ways to choose zero
= 5! = 120 and 3 are men while Y has 7 friends, 3 of balls from the total = 1
The number of words starting from ME them are ladies and 4 are men. Hence, the number of ways to choose
= 4! = 24 ∴ Total number of required ways atleast one ball
The number of words starting from MH = 3C3 × 4C 0 × 4C 0 × 3C3 [irrespective of any colour]
= 4! = 24 + 3C2 × 4C1 × 4C1 × 3C2 = (10 + 1) (9 + 1) (7 + 1) − 1
The number of words starting from MOE + 3C1 × 4C2 × 4C2 × 3C1 = 880 − 1 = 879
= 3! = 6 + 3C 0 × 4C3 × 4C3 × 3C 0 12 Let x1 , x2 , ..., x 8 denote the marks assign
The number of words starting from = 1 + 144 + 324 + 16 = 485
MOH = 3! = 6 to 8 questions.
The number of words starting from
8 The number of ways of selecting four ∴ x1 + x2 + ...+ x 8 = 30
MOR = 3! = 6
numbers from 1 to 30 without any Also, x1 , x2 , ...., x 8 ≥ 2
restriction is 30 C 4 . The number of ways
The number of words starting from
of selecting four consecutive numbers
MOTE = 2! = 2 Let, u1 = x1 − 2, u2 = x2 − 2 ... u 8
[i.e. (1, 2, 3, 4), (2, 3, 4, 5), ...,
The number of words starting from (27, 28, 29, 30)] is 27. = x8 − 2
MOTHER = 1! = 1 Then, (u1 + 2 + u2 + 2 + ...+ u 8 + 2) = 30
Hence, the number of ways of selecting
Hence, rank of the word MOTHER four integers which excludes selection ⇒ u1 + u2 + ...+ u 8 = 14
= 2 (120) + 2 (24) + 3 (6) + 2 + 1 of consecutive four numbers is
= 309 30 × 29 × 28 × 27 ∴ Total number of solutions
30
C 4 − 27 = − 27 14 + 8 − 1
5 A matrix whose determinant is non-zero 24 = C 8 −1 = 21
c7
is called a non-singular matrix. = 27378 13 210 510 1111 1313 has a divisor of the form
Here, we have 9 Possible even digits are 2,4,6,8,0. 2α ⋅ 5β ⋅ 11γ ⋅ 13δ , where
  a11 a 12   Case I Number has digits 4,6,8,0. α = 0, 1, 2,...,10; β = 0, 1, 2,...,10;
S =   ; aij ∈ {0, 1, 2}, a11 = a22 

 21a a22   (Here, sum of digits is 18, divisible by 3)
γ = 0, 1, 2,...,11; δ = 0, 1, 2,...,13
Clearly, n(s ) = 27 It is of the form 4 n + 1, if
[Qfor a11 = a22 , we have 3 choices, α = 0 ; β = 0, 1, 2,...,10;
Number of arrangements = 3 × 3!
for a12 , we have 3 choices and for a21 , we γ = 0, 2, 4,...,10;
have 3 choices] [Ist place can be filled using 4, 6, 8]
= 3 × 6 = 18 δ = 0, 1, 2,...,13.
a11 a12
Now, =0 Case II Number has digits 2, 4, 6, 0 ∴ Number of divisors
a21 a22
(Here, sum of digits is 12, divisible by 3) = 11 × 6 × 14 = 924
⇒ a11 ⋅ a22 − a12 ⋅ a21 = 0
⇒ (a11 ) 2 = a12 a21 = 0 [Q a11 = a22 ] 14 Let the particular side on which 3
particular sailors can work be named A
⇒ a12 a21 = 0, 1, 4 [Q a11 ∈ {0, 1, 2}] 1st place cannot be filled by 0. and on the other side by B on which 2
Consider a12 a21 = 0, this is possible in 5 Number of arrangements = 3 × 3! = 18 particular sailors can work. Thus, we are
cases ∴ Number of numbers = 18 + 18 = 36 left with 3 sailors only. Selection of one
a12 a21 = 1, this is possible in only 1 case sailor for side A = 3C1 = 3 and, then we
a12 a21 = 4, this is possible in only 1 case
10 24 = 2 ⋅ 3 ⋅ 4, 2 ⋅ 2 ⋅ 6, 1 ⋅ 6 ⋅ 4, 1 ⋅ 3 ⋅ 8,
1 ⋅ 2 ⋅ 12, 1 ⋅ 1 ⋅ 24 are left with 2 sailors for the other side.
Thus, number of singular matrices in Now, on each side, 4 sailors can be
[as product of three positive integers]
S are 7. arranged in 4! ways.
∴ The total number of positive integral
Hence, number of non-singular matrices ∴ Total number of arrangements
solutions of xyz = 24 is
in S are 27 − 7 = 20. = 3 × 24 × 24 = 1728
3! 3!
6 Let the men be M1 , M2 , ..., M10 and equal to3! + + 3! + 3! + 3! + i.e. 30.
2! 2! 15 Required number is the coefficient of x10
women be W1 , W2 , ..., W7 . Any two of the numbers in each in (1 + x + x2 + ... + x 6 ) 3
Let W1 and W2 do not want to be on the factorisation may be negative. So, the
same group. The six members group can = (1 − x7 )3 (1 − x )−3 = (1 − 3 x7 + ... )
number of ways to associate negative sign
contain 4 men and 2 women or 3 men   3  4 2 
1 +  1 x +  2  x + K
in each case is 3 C2 i.e. 3.
and 3 women. ∴ Total number of integral solutions  
The number of ways of forming 4M , 2W = 30 + 3 × 30 = 120 Hence, coefficient of x10 is
group is  12  5
11 The number of ways to choose zero or   − 3   = 36.
10
C 4 ( 5C2 + 2C1 ⋅ 5C1 ) = 4200
more from white balls = (10 + 1)  10  3
DAY 87

DAY NINE

Unit Test 1
(Algebra)
cos x x 1
f ′ (x ) 7 From 50 students taking examinations in Mathematics,
1 If f ( x ) = 2 sin x x2 2x , then lim is equal to Physics and Chemistry, 37 passed Mathematics,
x→ 0 x
tan x x 1 24 Physics and 43 Chemistry. Atmost 19 passed
(a) 1 (b) −1 (c) 2 (d) − 2 Mathematics and Physics, atmost 29 passed
Mathematics and Chemistry and atmost 20 passed
2 If log0. 5 ( x − 1) < log0. 25 ( x − 1), then x lies in the interval Physics and Chemistry. The largest possible number that
(a) (2 , ∞) (b) (3 , ∞) (c) (− ∞, 0) (d) (0, 3) could have passed all three examinations is
3 Sum of n terms of series 12 + 16 + 24 + 40 + … will be (a) 11 (b) 12 (c) 13 (d) 14
(a) 2 (2 n − 1) + 8n 8 The inequality | z − 4 | < | z − 2 | represents the region
(b) 2 (2 n − 1) + 6n given by
(c) 3 (2 n − 1) + 8n (a) Re(z) > 0 (b) Re(z) < 0
(d) 4 (2 n − 1) + 8n
(c) Re(z) > 3 (d) None of these
4 Let a, b and c ∈ R and a ≠ 0. If α is a root of
a 2 x 2 + bx + c = 0, β is a root of a 2 x 2 − bx − c = 0 and 9 If 1, ω and ω 2 be the three cube roots of unity, then
0 < α < β, then the equation of a 2 x 2 + 2bx + 2c = 0 has a (1 + ω )(1 + ω 2 )(1 + ω 4 ) K 2n factors is equal to
root γ, that always satisfies (a) 1 (b) −1
(a) γ = α (b) γ = β (c) 0 (d) None of these
(c) γ = (α + β) / 2 (d) α < γ < β
10 If a < 0, then the positive root of the equation
5 Between two numbers whose sum is 2
1
an even number x 2 − 2a | x − a | − 3 a 2 = 0 is
6 (a) a (− 1 − 6) (b) a (1 − 2)
of arithmetic means are inserted. If the sum of these (c) a (1 − 6) (d) a (1 + 2)
means exceeds their number by unity, then the number
11 The common roots of the equations z 3 + 2z 2 + 2z +
of means are
1 = 0 and z 1985 + z 100 + 1 = 0 are
(a) 12 (b) 10
(a) −1, ω (b) −1, ω2
(c) 8 (d) None of these
(c) ω, ω 2
(d) None of these
1 0
6 If A =  , then which of the following is not true? 12 Let z1, z 2 and z 3 be three points on | z | = 1. If θ1, θ 2 and θ 3
1 1 
are the arguments of z1, z 2 and z 3 respectively, then
1 − n  0 0 1 − n  0 0 cos (θ1 − θ 2 ) + cos (θ 2 − θ 3 ) + cos (θ 3 − θ1 )
(a) lim A =  (b) lim A = 
n→ ∞
n2  −1 0 n→ ∞
n  −1 0 3 3
(a) ≥ (b) ≥ −
 1 0 2 2
(c) A − n =  , ∀ n ∈ N (d) None of these −3
 −n 1 (c) ≤ (d) None of these
2
88 40

13 If the roots of the equation 22 If the roots of the equation x 2 − 2ax + a 2 + a − 3 = 0 are
(a + b )x + 2 (bc + ad )x + (c + d ) = 0
2 2 2 2 2
real and less than 3, then
are real, then a 2 , bd and c 2 are in (a) a < 2 (b) 2 ≤ a ≤ 3 (c) 3 ≤ a ≤ 4 (d) a > 4
(a) AP (b) GP
23 The integer k for which the inequality
(c) HP (d) None of these
x 2 − 2 ( 4 k − 1)x + 15 k 2 − 2k − 7 > 0 is valid for any x, is
14 150 workers were engaged to finish a piece of work in a (a) 2 (b) 3
certain number of days. Four workers dropped the (c) 4 (d) None of these
second day, four more workers dropped the third day
and so on. It takes 8 more days to finish the work now. 24 The maximum sum of the series
1 2
Then, the number of days in which the work was 20 + 19 + 18 + 18 + K is
completed is 3 3
(a) 29 days (b) 24 days (c) 25 days (d) 26 days (a) 310 (b) 290
(c) 320 (d) None of these
15 Let R be a relation defined by R = {( x , x ) : x is a prime
3

number < 10 }, then which of the following is true? 25 The number of common terms to the two sequences
17, 21, 25, ..., 417 and 16, 21, 26, ..., 466 is
(a) R = {(1, 1), (2 , 8), (3 , 27), (4 , 64), (5 , 125),(6, 216), (7, 343),
(a) 21 (b) 19 (c) 20 (d) 91
(8, 512), (9, 729)}
(b) R = {(2 , 8), (3 , 27), (5 , 125), (7, 343)} 26 If the sum of the first three terms of a GP is 21 and the
sum of the next three terms is 168, then the first term and
(c) R = {(2 , 8), (4 , 64), (6, 216), (8, 512)} the common ratio is
(d) None of the above (a) 3 , 4 (b) 2 , 4
16 If b > a , then the equation ( x − a )( x − b ) − 1 = 0 , has (c) 3 , 2 (d) None of these
(a) both the roots in [a , b] 27 The sum to n terms of the series
(b) both the roots in (−∞, a ] 1
+
2
+
3
+ ..., is
(c) both the roots in (b, ∞) 1 + 12 + 14 1 + 22 + 24 1 + 32 + 34
(d) one root in (−∞, a) and other in (b, ∞) n2 + n n2 − n
(a) (b)
17 The value of x satisfying log2 ( 3x − 2) = log1/ 2 x is 2 (n 2 + n + 1) 2 (n 2 + n + 1)
(a) −
1
(b) 2 n2 + n
(c) (d) None of these
3 2 (n 2 − n + 1)
1
(c) (d) None of these
2 28 If C is a skew-symmetric matrix of order n and X is n × 1
column matrix, then X ′ C X is a
r
n
18 If f ( x , n ) = ∑
r =1
logx   , then the value of x satisfying the
x
(a) scalar matrix (b) unit matrix
(c) null matrix (d) None of these
equation f ( x , 11) = f ( x , 12) is
29 Which of the following is correct?
(a) 10 (b) 11
(c) 12 (d) None of these (a) Skew-symmetric matrix of an even order is always
singular
19 The three numbers a , b and c between 2 and 18 are (b) Skew-symmetric matrix of an odd order is non-singular
such that their sum is 25, the numbers 2 ,a and b are (c) Skew-symmetric matrix of an odd order is singular
consecutive terms of an AP and the numbers b, c and18
(d) None of the above
are consecutive terms of a GP. The three numbers are
(a) 3, 8, 14 (b) 2, 9, 14 a1 b1 c1
(c) 5, 8, 12 (d) None of these 30 If a 2 b2 c 2 = 5 , then the value of
20 If X is the set of all complex numbers z such that | z | = 1, a3 b3 c3
then the relation R defined on X by b2c3 − b3c2 a3c2 − a2c3 a2b3 − a3b2

| arg z1 − arg z 2 | = , is ∆ = b3c1 − b1c3 a1c3 − a3c1 a3b1 − a1b3 is
3
b1c2 − b2c1 a2c1 − a1c2 a1b2 − a2b1
(a) reflexive (b) symmetric
(c) transitive (d) anti-symmetric (a) 5 (b) 25 (c) 125 (d) 0
21 If α and β are the roots of the equation ax − 2 bx + c = 0,2
31 The number of seven letter words that can be formed by
then α 3β 3 + α 2β 3 + α 3β 2 is equal to using the letters of the word ‘SUCCESS’ so that the two C
c2 c2 are together but no two S are together, is
(a) 3 (c − 2 b) (b) 3 (c + 2 b)
a a (a) 24 (b) 36
bc 2 (c) 54 (d) None of these
(c) 3 (d) None of these
a
DAY 89

32 The greatest integer less than or equal to ( 2 + 1)6 is


12
 2 
42 The 8th term of  3x +  when expanded in
(a) 196 (b) 197 3x 2 
(c) 198 (d) 199 ascending power of x, is
33 If A and B are square matrices such that B = − A −1BA , then 228096 228096
(a) (b)
(a) AB + BA = O x3 x9
(b) (A + B)2 = A 2 − B 2 328179
(c) (d) None of these
(c) (A + B)2 = A 2 + 2 AB + B 2 x9
(d) (A + B)2 = A + B
43 The greatest term in the expansion of ( 3 − 5x )11 when
xp + y x y 1
34 The determinant yp + z y z = 0 , if x = , is
5
0 xp + y yp + z
(a) 55 × 3 9 (b) 46 × 3 9
(a) x, y, z are in AP (b) x, y, z are in GP (c) 55 × 3 6 (d) None of these
(c) x, y, z are in HP (d) xy, yz, zx are in AP
44 In an examination a candidate has to pass in each of the
35 The value of k, for which the system of equations papers to be successful. If the total number of ways to
x + ky + 3z = 0, 3x + ky − 2z = 0 and 2x + 3y – 4z = 0 fail is 63, how many papers are there in the examination?
possess a non-trivial solution over the set of rationals, is
(a) 6 (b) 8 (c) 10 (d) 12
33 33
(a) − (b)
2 2 45 A is a set containing n elements. A subset P of A is
(c) 11 (d) None of these chosen. The set A is reconstructed by replacing the
elements of P. A subset Q of A is again chosen. The
36 The number of groups that can be made from 5 different
number of ways of choosing P and Q, so that P ∩ Q
green balls, 4 different blue balls and 3 different red balls,
contains exactly two elements is
if atleast 1 green and 1 blue ball is to be included is
(a) 9 ⋅ n C2 (b) 3 n − n C2
(a) 3700 (b) 3720
(c) 4340 (d) None of these (c) 2 ⋅ n Cn (d) None of these
46 If the sets A and B are defined as
37 If n is an integer greater than 1, then 1
a − nC1(a − 1) + nC2 (a − 2) + ...+ ( − 1)n (a − n ) is equal to A = {( x , y ) : y = ,0 ≠ x ∈ R }
x
(a) a (b) 0 (c) a 2 (d) 2 n and B = {( x , y ): y = − x , x ∈ R }, then
38 If α , β and γ are the roots of the equation (a) A ∩ B = A (b) A ∩ B = B
x ( x + e ) = e ( x + 1). Then, the value of the determinant
2 (c) A ∩ B = φ (d) None of these
1+ α 1 1 47 There are 16 points in a plane no three of which are in a
1 1+ β 1 is straight line except 8 which are all in a straight line. The
1 1 1+ γ number of triangles can be formed by joining them equals
(a) 1120 (b) 560 (c) 552 (d) 504
(a) − 1 (b) 1
48 The value of the natural numbers n such that inequality
1 1 1
(c) 0 (d) 1 + + + 2n > 2n + 1 is valid, is
α β γ
(a) for n ≥ 3 (b) for n < 3 (c) for mn (d) for any n
39 For all natural number n > 1, 24 n − 15 n − 1 is divisible by
 2π   2π 
(a) 225 (b) 125 49 Let w = cos   + i sin   and α = w + w 2 + w 4 and
 7  7
(c) 325 (d) None of these
β = w 3 + w 5 + w 6 , then α + β is equal to
40 If x is so small that its two and higher power can be
(a) 0 (b) −1 (c) −2 (d) 1
neglected and if (1 − 2x )−1/ 2 (1 − 4x )−5 / 2 = 1 + kx, then k is
f (x )
equal to 50 Let H( x ) = , where f ( x ) = 1 − 2 sin2 x and
(a) − 2 (b) 1 g( x )
(c) 10 (d) 11 g ( x ) = cos 2x , ∀ f : R → [ −1,1] and g : R → [ −1, 1].
2x + 1 4 8 Domain and range of H( x ) are respectively
41 If x = − 5 is a root of 2 2x 2 = 0 , then the (a) R and {1}
7 6 2x (b) R and {0, 1}
other two roots are π
(c) R ~ {(2n + 1) } and {1},n ∈ I
(a) 3, 3.5 (b) 1, 3.5 4
π
(c) 3 , 6 (d) 2 , 6 (d) R ~ (2n + 1)  and {0, 1},n ∈ I
 2
90 40

51 Let Tn denote the number of triangles which can be 55 The general term in the expansion of (a + x )n is
formed using the vertices of a regular polygon of n sides. n
Cr a n − r x r .
If Tn +1 − Tn = 21, then n equals Statement I The third term in the expansion of
m
(a) 4 (b) 6 2 x + 1  does not contain x . The value of x for
(c) 7 (d) None of these  
 x 2
52 If r is a real number such that | r | < 1 and if a = 5 (1 − r ), which that term equal to the second term in the
then expansion of (1 + x 3 ) 30 is 2.
(a) 0 < a < 5 (b) − 5 < a < 5 n
(c) 0 < a < 10 (d) 0 ≤ a < 10 Statement II (a + x ) n = ∑
r = 0
n
Cr a n − r x r .
Directions (Q. Nos. 53-57) Each of these questions
contains two statements : Statement I (Assertion) and 56 Sets A and B have four and eight elements, respectively.
Statement II (Reason). Each of these questions also has Statement I The minimum number of elements in
four alternative choices, only one of which is the
A ∪ B is 8.
correct answer. You have to select one of the codes
( a), (b), (c) and (d ) given below. Statement II A ∩ B = 5
(a) Statement I is true, Statement II is true; Statement II is a a b c
correct explanation for Statement I 57 Let a ≠ 0, p ≠ 0 and ∆ = 0 p q
(b) Statement I is true, Statement II is true; Statement II is p q 0
not a correct explanation for Statement I
(c) Statement I is true; Statement II is false Statement I If the equations ax 2 + bx + c = 0 and
(d) Statement I is false; Statement II is true px + q = 0 have a common root, then ∆ = 0.
Statement II If ∆ = 0, then the equations
53 Statement I The number of natural numbers which
ax 2 + bx + c = 0 and px + q = 0 have a common root.
divide 102009 but not 102008 is 4019.
58 Assume X , Y , Z , W and P are matrices of order
Statement II If p is a prime, then number of divisors of 2 × n, 3 × k , 2 × p, n × 3 and p × k , respectively.
p n is p n + 1 − 1. Now, consider the following statements
 1 0 2 0 I. PY + WY will be defined for k = 3 and p = n.
54 Suppose A =   and B =   . Let X be a 2 × 2
0 −1 0 2 II. The order of the matrix 7 X − 5 Z is n × 2 (if p = n).
matrices such that X ′ AX = B.
Choose the correct option.
Statement I X is non-singular and det ( X ) = ± 2 .
(a) Only I is true (b) Only II is true
Statement II X is a singular matrix. (c) Both I and II are true (d) Neither I nor II is true

ANSWERS
1. (d) 2. (a) 3. (d) 4. (d) 5. (a) 6. (a) 7. (d) 8. (c) 9. (a) 10. (b)
11. (c) 12. (b) 13. (b) 14. (c) 15. (b) 16. (d) 17. (d) 18. (c) 19. (c) 20. (b)
21. (b) 22. (a) 23. (b) 24. (a) 25. (c) 26. (c) 27. (a) 28. (c) 29. (c) 30. (b)
31. (a) 32. (b) 33. (a) 34. (b) 35. (b) 36. (b) 37. (b) 38. (c) 39. (a) 40. (d)
41. (b) 42. (a) 43. (a) 44. (a) 45. (d) 46. (c) 47. (d) 48. (a) 49. (b) 50. (c)
51. (c) 52. (c) 53. (c) 54. (c) 55. (b) 56. (c) 57. (c) 58. (a)
DAY 91

Hints and Explanations


1 Applying R 1 → R 1 − R 3 Given, A 1 + A 2 + K + A 2 n = 2 n + 1 = (1 + ω + ω2 + ω3 )n
cos x − tan x 0 0 ⇒ 2n + 1 =
13 n = (0 + ω3 )n = ω3 n = 1
6 10 If x ≥ a, then x2 − 2 a( x − a) − 3 a2 = 0
f ( x) = 2 sin x x2 2x
⇒ 12 n + 6 = 13 n
tan x x 1 ⇒ x 2 − 2 ax − a2 = 0
∴ n=6
= (cos x − tan x)( x − 2 x )
2 2
Hence, the number of means 2a ± 4 a2 + 4 a2
∴ x= = a(1 ± 2 )
= − x 2 (cos x − tan x) = 2 × 6 = 12 2
∴ f ′ ( x) = − 2 x(cos x − tan x) Since, x ≥ a
 1 0
− x 2 (− sin x − sec2 x) 6 A −1 =   ∴ x = a(1 + 2 ), it is impossible

 1 1
f ′ ( x)
∴ lim = lim [−2 (cos x − tan x)  1 0  1 0  1 0 because a < 0
x→ 0 x x→ 0
⇒ ( A −1 )2 =   =  ∴ x = a (1 − 2 )
+ lim x(sin x + sec2 x)]  −1 1   −1 1   −2 1 
x→ 0
 1 0 If x < a, then x2 + 2 a ( x − a) − 3 a2 = 0
= −2 ×1= −2 Similarly, ( A −1 )n =   ⇒ x 2 + 2 ax − 5 a2 = 0
2 Given, log 0.5( x − 1) < log 0.25( x − 1) −n 1
∴ x = (− 1 ± 6 ) a
⇒ log 0.5( x − 1) < log( 0.5 )2 ( x − 1) 1 1 / n 0 
Now, lim A − n = lim   [impossible x < a and a < 0]

1
log 0. 5 ( x − 1) < log 0.5( x − 1)
n→ ∞ n n→ ∞
 − 1 1 / n
2 11 z3 + 2 z2 + 2 z + 1 = 0
 0 0
⇒ log 0.5 ( x − 1) < 0 ⇒ x − 1 > 1 =  ⇒ (z + 1)(z2 + z + 1) = 0
 −1 0 
∴ x >2 ⇒ z = – 1, ω, ω2
1 / n2 0 
3 Let, S n = 12 + 16 + 24 + … + Tn 1 −n But z = − 1 does not satisfy the
and lim A = lim  1 
n→ ∞ n2 n → ∞ − 2
 n 1 / n  second equation.
Again, S n = 12 + 16 + K + Tn
Hence, common roots are ω and ω2 .
0 = (12 + 4 + 8 + 16 + K 0 0 
upto n terms) −Tn =  12 We have,| z1| = | z2| = | z3| = 1
0 0 
4 (2 n − 1 − 1) Now,| z1 + z2 + z3| ≥ 0
∴ Tn = 12 + = 2n + 1 + 8 7 Given, n (M ∪ P ∪ C) = 50,
2 −1 ⇒ | z1|2 + | z2|2 + | z3|2 + 2 Re
On putting n = 1, 2, 3,K , we get n (M ) = 37, n (P) = 24 , n (C) = 43 (z1 z2 + z2 z3 + z3 z1 ) ≥ 0
T1 = 22 + 8, T2 = 23 + 8, T3 = 2 4 + 8 … n (M ∩ P) ≤ 19, n (M ∩ C) ≤ 29, ⇒ 3 + 2 [cos(θ1 − θ2 ) + cos(θ2 − θ3 )
∴ S n = T1 + T2 + K + Tn n (P ∩ C) ≤ 20 + cos(θ3 − θ1 )] ≥ 0
= (22 + 23 + … upto n terms) ∴ n (M ∪ P ∪ C) = n (M ) + n (P) + n (C) ⇒ cos(θ1 − θ2 ) + cos(θ2 − θ3 )
+ (8 + 8 + … upto n terms) − n ( M ∩ P ) − n ( M ∩ C) 3
− n (P ∩ C) + n (M ∩ P ∩ C) + cos(θ3 − θ1 ) ≥ −
22 (2 n − 1) 2
= + 8n ⇒ 50 = 37 + 24 + 43 − n(M ∩ P)
2 −1 13 Here, D ≥ 0
− n (M ∩ C) − n (P ∩ C)
= 4 (2 n − 1) + 8 n
+ n (M ∩ P ∩ C) ∴4 (bc + ad )2 − 4 (a2 + b 2 )(c 2 + d 2 ) ≥ 0
4 Let f ( x) = a2 x2 + 2bx + 2c ⇒ n (M ∩ P ∩ C) = n (M ∩ P ) ⇒ b 2c 2 + a2d 2 + 2 abcd − a2c 2
Q We have, a2α 2 + bα + c = 0 + n (M ∩ C) + n (P ∩ C) − 54 − a2d 2 − b 2c 2 − b 2d 2 ≥ 0
and a2β2 − bβ − c = 0 ∴ n (M ∩ P ∩ C) ⇒ (ac − bd )2 ≤ 0
∴ f (α ) = a2α 2 + 2bα + 2c = bα + c ≤ 19 + 29 + 20 − 54 = 14 ⇒ ac − bd = 0
= − a2α 2 8 |( x − 4) + iy|2 < |( x − 2) + iy|2 [since, square of any expression
f (β) = a β + 2bβ + 2c = 3 (bβ + c )
2 2
[let z = x + iy] cannot be negative]
= 3 a2β2 ⇒ ( x − 4)2 + y2 < ( x − 2)2 + y2 ∴ b 2d 2 = a2c 2
But 0 < α < β ⇒ α , β are real number. ⇒ − 4 x < − 12 ⇒ x > 3 Hence, a2 , bd and c 2 are in GP.
∴ f (α ) < 0, f (β) > 0 ∴ Re(z) > 3
14 Here, a = 150 and d = − 4
Hence, α < γ < β. 9 (1 + ω)(1 + ω2 )(1 + ω 4 )(1 + ω 8) … to n
Sn = [2 × 150 + (n − 1) (− 4)]
5 Let 2n arithmetic means be 2n factors 2
A 1 , A 2 ,K , A 2 n between a and b . = (1 + ω)(1 + ω2 )(1 + ω)(1 + ω2 ) … to = n (152 − 2 n)
a+b 2n factors
Then, A 1 + A 2 +K + A 2 n = × 2n Had the workers not dropped, then
2 = [(1 + ω)(1 + ω) … to n factors] the work would have finished is
13 / 6 13 n [(1 + ω2 )(1 + ω2 ) … to n factors] (n − 8) days with 150 workers
= × 2n =
2 6 = (1 + ω)n (1 + ω2 )n working on each day.
92 40

∴ n(152 − 2 n) = 150 (n − 8) From Eqs. (i) and (ii), we get ∴ Maximum sum,
⇒ n2 − n − 600 = 0 3b = 48 − 2 c 31   2 
S31 = 2 × 20 + (31 − 1)  −  
⇒ (n − 25) (n + 24) = 0 From Eq. (iii), we get 2   3 
∴ n = 25 c 2 = 6 (48 − 2 c ) = 288 − 12 c 31
[since, n cannot be negative] ⇒ c 2 + 12 c − 288 = 0 = (40 − 20) = 310
2
15 Given, x is a prime < 10 ⇒ (c + 24)(c − 12) = 0
25 First series has common difference
⇒ c = 12 as c ≠ – 24
∴ x = {2 , 3 , 5 , 7} 4 and second series has common
∴ b = 8 and a = 5
Now, from R = {( x, x 3 ) : x = 2, 3, 5 , 7} difference 5.
= {(2 , 8), (3, 27), (5 , 125), (7, 343)} 20 Q | z| = 1 ⇒ z = cos θ + i sin θ Hence, the series with common
16 Let f ( x) = ( x − a)( x − b ) − 1 ∴ arg (z) = θ terms has common difference is
2 Let, arg (z1 ) = θ1 and arg (z2 ) = θ2 equal to the LCM of 4 and 5 i.e. 20.
We observe that the coefficient of x
2π Since, the first common term is 21.
in f ( x) is positive and Then, z1 Rz2 ⇒|arg z1 − arg z2 | = So, the series will be 21, 41, 61, …,
f (a) = f (b ) = − 1. Thus, the graph of 3
2π 401 which has 20 terms.
f ( x) is as shown in figure given ⇒ z1 Rz2 ⇒|θ1 − θ2 | =
below 3 26 Given, a1 + a2 + a3 = 21
2π ⇒ a(1 + r + r 2 ) = 21
Y ⇒ |θ2 − θ1 | =
y = f(x) 3 and a4 + a5 + a6 = 168
⇒ z2 Rz1 ⇒ ar 3 (1 + r + r 2 ) = 168
(a, 0) (b, 0) Hence, it is symmetric. ∴ r3 = 8 ⇒ r = 2
X' X 2b c
O 21 Here, α + β = and αβ = and a (1 + 2 + 4) = 21
a a ∴ a=3
(a, –1)
(b, –1) Now, (αβ)3 + α 2β2 ( β + α ) r
3 27 Tr = , r = 1, 2, 3, ..., n
Y' c c2  2 b  1 + r2 + r 4
=  + 2  
 a a  a r
It is evident from the graph that one = 2
of the roots of f ( x) = 0 lies in (− ∞, a) c 2 (c + 2 b ) (r + r + 1) (r 2 − r + 1)
=
and the other root lies in (b , ∞). a3
1 1 1 
= −
17 Given, log2 (3 x − 2) = log1 /2 x 22 According to the question, 2  r 2 − r + 1 r 2 + r + 1 
D ≥ 0 and f (3) > 0
⇒ log2 (3 x − 2) = − log 2 x n
1 n
⇒ 4 a2 − 4 (a2 + a − 3) ≥ 0 ∴ ∑ Tr = ∑
⇒ log2 (3 x − 2) = log2 x −1 r =1 2 r =1
and 32 − 2 a (3) + a2 + a − 3 > 0
⇒ 3 x − 2 = x −1  1 1 
⇒ − a + 3 ≥ 0 and a2 − 5 a + 6 > 0
⇒ 3 x2 − 2 x − 1 = 0  r2 − r + 1 − r2 + r + 1 
⇒ a ≤ 3 and a < 2 or a > 3  
⇒ (3 x + 1) ( x − 1) = 0
1 ∴ a<2  1  1 1 
⇒ x = 1 or x = −  1 −  +  −  
1  3   3 7 
3 23 Let f ( x) = x2 − 2 (4 k − 1) x + 15 k 2 =  
∴ x =1 − 2 k − 7, then f ( x) > 0 2  1 1 
 + ...+  2 − 
[since, negative of x cannot satisfy ∴ D<0   n − n + 1 n2 + n + 1  
the given equation]
⇒ 4 (4 k – 1)2 – 4 (15 k 2 – 2 k – 7) < 0 1 1  n2 + n
n
= 1− 2 =
18 f ( x, n) = ∑ (log x r − log x x) ⇒ k2 − 6 k + 8 < 0 ⇒ 2 < k < 4 2  n + n+ 
1  2(n + n + 1)
2

r =1 Hence, required integer value of k


n
is 3 . 28 Here, X is n × 1, C is n × n and X ′ is
= ∑ (log x r − 1) = log x (1⋅ 2 ... n) − n
2 1 × n order matrix. Therefore, X ′ C X
r =1
24 Here, a = 20,d = −
= log x n ! − n 3 is 1 × 1 order matrix. Let X ′ C X = K
Given, f ( x, 11) = f ( x, 12) As the common difference is Then, ( X ′ C X )′ = X ′ C′ X ′ ′
⇒ log x (11 !) − 11 = log x (12 !) − 12 negative, the terms will become = X ′ (− C) X = − K
 12 !
negative after some stage. So, the ⇒ 2K =O
⇒ log x   = 1 ⇒ log x (12) = 1 sum is maximum, if only positive ∴ K =O
 11 !
terms are added.
∴ x = 12 29 Since, the determinant of a
 2
Now, Tn = 20 + (n − 1)  −  ≥ 0 skew-symmetric matrix of an odd
19 Given, a + b + c = 25 …(i)  3
order is zero. Therefore, the matrix
Since, 2, a, b are in AP, therefore ⇒ 60 − 2(n − 1) ≥ 0 is singular.
2a = 2 + b …(ii) ⇒ 62 ≥ 2 n ⇒ 31 ≥ n 30 We know that, if A is a square
Since, b , c , 18 are in GP, therefore Thus, the first 31 terms are matrix of order n and B is the matrix
c 2 = 18 b …(iii) non-negative. of cofactors of elements of A. Then,
DAY 93

12 − 7
| B| = | A |n −1 Hence, the required number of  2 
ways = 31 × 15 × 8 = 3720 = 12C 7  2  (3 x) 7
∴ ∆ = | A |3 − 1 = 53 − 1 = 25 3x 
31 Considering CC as single letter, 37 LHS = a [C0 − C1 + C2 − C3 12 !  2 
5

U,CC, E can be arranged in 3 ! ways + ...+ (− 1) Cn ] n =   (3 x)


7

7 !5 !  3 x 2 
Here, × U × CC × E × + [C1 − 2C2 + 3 C3 −... + (−1)n − 1 ⋅ nCn ] 12 × 11 × 10 × 9 × 8 2 5 × 32
Hence, the required number of ways = ×
= a⋅0 + 0 = 0 5 × 4×3 ×2 x3
= 3 !⋅ 4C3 = 24
38 Given, equation is 228096
32 Let ( 2 + 1)6 = I + F , where I is an =
x 3 + ex2 − ex − e = 0 x3
integer and 0 ≤ F < 1
Applying R 2 → R 2 − R 1  5 x
11
Let f = ( 2 − 1)6 43 (3 − 5 x)11 = 311  1 − 
and R 3 → R 3 − R 2 ,  3
1
Now, 2 − 1 = (1 + α ) 1 1 11
2+1  1  1
= 3 11  1 −  Qx =
∆ = −α β 0  3  5 
⇒ 0 < 2 −1 <1
0 −β γ | x|(n + 1)
Also, I + F + f = ( 2 + 1)6 + ( 2 − 1)6 ∴ Greatest term =
= (1 + α ) (βγ − 0) + α (γ) + αβ (| x| + 1)
= 2 [6C0 ⋅ 23 + 6C2 ⋅ 22 + 6C4 ⋅ 2 + 6C6]
= αβ + βγ + γα + αβγ 1
= 2 (8 + 60 + 30 + 1) = 198 − (11 + 1)
= − e+e =0 3
Hence, F + f = 198 − I is an integer. = =3
39 Now, 2 4 n = (1 + 15)n 1
But 0 < F + f <2 − +1
= 1 + nC1 ⋅ 15 + nC2 ⋅ 152 3
∴ F + f = 1 and I = 197 2
+ nC3 ⋅ 153 + …  1
33 Given, B = − A −1 BA Now, T3 = 311 ⋅ 11C2  − 
∴ 2 4 n − 1 − 15 n = 152  3
⇒ AB = − A ( A −1 BA )
⇒ AB = − I (BA ) [n C2 + nC3 ⋅ 15 + K ]  11⋅ 10 1
= 311  ×  = 55 × 3 9
= 225 k  1⋅ 2 9
∴ AB + BA = O
Hence, it is divisible by 225. 44 Let the number of papers be n.
34 Applying R 3 → R 3 − pR 1 − R 2
xp + y x y
40 (1 − 2 x)−1 /2 (1 − 4 x)−5/2 ∴ Total number of ways to fail or pass
= (1 + x) (1 + 10 x) = nC0 + nC1 + K + nCn = 2 n
⇒ yp + z y z =0
[neglecting higher power] ∴Total number of ways to fail
− ( xp + 2 yp + z) 0
2
0 = 2n − 1
= 1 + 11 x
⇒ − ( xp + 2 yp + z)( xz − y2 ) = 0
2
[neglecting higher power] [since, there is only one way to pass]
Hence, x, y and z are in GP. =1 + kx According to the question,
35 For non-trivial solution, we must ∴ k = 11 2 n − 1 = 63 ⇒ 2 n = 2 6 ⇒ n = 6
have
41 Applying R 1 → R 1 + R 2 + R 3 , we get 45 Let A = {a1 , a2 , ..., an }, 1 ≤ i ≤ n
1 k 3
2 x + 10 2 x + 10 2 x + 10 (i) ai ∈ P, ai ∈ Q (ii) ai ∉ P, ai ∉ Q
3 k −2 = 0
2 2x 2 =0 (iii) ai ∉ P, ai ∈ Q (iv) ai ∈ P, ai ∉ Q
2 3 −4
7 6 2x So, P ∩ Q contains exactly two
Applying elements, taking 2 elements in (i) and
R2 → R2 − 3 R1 , R3 → R3 − 2R1 , Taking 2 x + 10 common from R 1 (n − 2) elements in (ii), (iii) and (iv).
and applying ∴ Required number of ways
1 k 3
C2 → C2 − C1 , C3 → C3 − C1 , we get = nC2 × 3 n − 2
0 − 2k − 11 = 0
1 0 0 46 It is clear from the graph that two
0 3 − 2k − 10 curves do not intersect anywhere.
2 ( x + 5) 2 2 x − 2 0 =0
⇒ 20 k + 11 (3 − 2 k ) = 0 ∴ A∩B=φ
7 −1 2x − 7
33
⇒ k = ⇒ 2 ( x + 5) (2 x − 2) (2 x − 7) = 0
2 Y
∴ x = − 5 , 1, 3.5
36 Atleast one green ball can be 1
selected out of 5 green balls in 42 For ascending power of x, we take y=
x
2 5 − 1, i.e. 31 ways. Similarly, the expression X' X
12
atleast one blue ball can be  2 
 2 + 3 x
selected from 4 blue balls in 3x 
2 4 − 1 = 15 ways and atleast one 12
 2  y = –x
red or not red can be selected in Y'
∴ T8 in  2 + 3 x
23 = 8 ways. 3x 
94 40

2
47 ∴ Required number of ways 102009 but not 10 2008 is  q  q
(2010)2 − (2009)2 = 4019 or a  −  + b  −  + c = 0
= C3 − C3 = 560 − 56 = 504
16 8
 p  p
48 Check through options, the condition 54 If X = O, then X ′ AX = O ⇒ B = O, a Thus, ax2 + bx + c = 0 and
2 n > 2 n + 1 is valid for n ≥ 3. contradiction. px + q = 0 have a common root.
6
w (1 − w 6) Let det ( X ) = a, then det ( X ′ ) = a
49 Here, α + β = ∑ wk =
1 −w ∴ det ( X ′ AX ) = det (B)
58 I. Matrices P and Y are of the
orders p × k and 3 × k,
k =1

= −1 [Qw 7 = 1 ] ⇒ a (−1) a = − 4 respectively.


[Qdet ( X ′AX) = det ( X ′ ) det (A) det (X)] Therefore, matrix PY will be
f ( x) 1 − 2 sin x 2
50 Q H ( x) = = ∴ a=±2 defined, if k = 3.
g ( x) cos 2 x
As det ( X ) ≠ 0, X cannot be a Consequently PY will be of the
cos 2 x singular matrix.
= =1 order p × k . Matrices W and Y
cos 2 x 2
 1 are of the orders n × 3 and 3 × k ,
But cos 2 x ≠ 0 55 T3 = mC2 (2 x)m − 2  
 x2  respectively.
π
⇒ 2 x ≠ nπ + , n ∈ I = mC2 (2)m − 2 ⋅ x m − 6 Since, the number of columns in
2
For independent of x, put W is equal to the number of rows
 π 
∴ x ∈ R ~ (2 n + 1) , n ∈ I  m −6 =0 ⇒ m =6 in Y , matrix WY is well-defined
 4  and is of the order n × k .
∴ T3 = 6C2 (2)6 − 2 = 15 × 16 = 240
and range = {1} Matrices PY and WY can be
According to the question,
added only when their orders are
30
C1 x 3 = 240 same.
51 Tn = nC3 and Tn +1 − Tn = 21
⇒ x3 = 8 However, PY is of the order p × k
n +1
⇒ C3 − nC3 = 21 ⇒ x =2 and WY is of the order n × k ,
⇒ n
C2 + C3 − nC3 = 21
n
therefore we must have p = n.
56 n ( A ∪ B) = n ( A ) + n (B) − n ( A ∩ B)
⇒ n
C2 = 21 Thus, k = 3 and p = n are the
= 4 + 8 − n( A ∩ B)
n(n − 1) restrictions on n, k and p, so that
⇒ = 21 = 12 − n ( A ∩ B)
2 PY + WY will be defined.
Since, maximum number of element
⇒ n2 − n − 42 = 0 in n ( A ∩ B) = 4. II. Matrix X is of the order 2 × n.
⇒ (n − 7) (n + 6) = 0 ∴ Minimum number of element is Therefore, matrix 7 X is also of
∴ n=7 [Q n ≠ − 6] n ( A ∪ B) = 12 − 4 = 8 the same order.
57 If λ is a common root of Matrix Z is of the order 2 × p,
52 Since,| r | < 1 ⇒ − 1 < r < 1 i.e. 2 × n. [Q n = p]
ax + bx + c = 0
2
Also, a = 5 (1 − r ) ⇒ 0 < a < 10
and px + q = 0, then aλ2 + bλ + c = 0, Therefore, matrix 5Z is also of
[Qat r = − 1, a = 10 the same order.
pλ + q = 0 and pλ2 + qλ = 0
and at r = 1 , a = 0]
Eliminating λ, we obtained ∆ = 0. Now, both the matrices 7 X and
53 The number of divisor of 10 m = 2 m5 m For Statement II, expanding ∆ along 5Z are of the order 2 × n.
is (m + 1)2 . C1 , we obtain Thus, matrix 7 X − 5 Z is well-
∴Number of divisors which divide aq 2 + p (bq − cp) = 0 defined and is of the order 2 × n.
DAY TEN

Real Function
Learning & Revision for the Day
u Real Valued Function and u Domain and Range of real u Inverse Function
Real Function Function u Basic Functions
u Algebra of Real Functions u Nature of a Functions

Real Valued Function and Real Function


A Function f : A → B is said to be a real valued function if B ⊆ R (the set of real numbers), if
both A and B are subset of R (the set of real numbers) then f is called a real function.

NOTE Every real function is a real valued function but converse need not be true.

Domain and Range of Real Function


The domain of y = f ( x) is the set of all real x for which f ( x) is defined (real).
Range of y = f ( x) is collection of all distinct images corresponding to each real number in
the domain.

NOTE If f : A → B, then A will be domain of f and B will be codomain of f.

To find range PRED


(i) First of all find the domain of y = f ( x) .
(ii) If domain has finite number of points, then range is the set of f − images of these points.
MIRROR
Your Personal Preparation Indicator
(iii) If domain is R or R − {some finite points}, express x in terms of y and find the values u No. of Questions in Exercises (x)—
of y for which the values of x lie in the domain.
u No. of Questions Attempted (y)—
(iv) If domain is a finite interval, find the least and the greatest values for range using u No. of Correct Questions (z)—
monotonicity. (Without referring Explanations)

Accuracy Level (z / y × 100)—


Algebra of Real Functions u

u Prep Level (z / x × 100)—


Let f : X → R and g : X → R be two real functions. Then,

The sum f + g : X → R defined as In order to expect good rank in JEE,
your Accuracy Level should be
( f + g) ( x) = f ( x) + g( x). above 85 & Prep Level should be

The difference f − g: X − R, defined as above 75.
( f − g)( x) = f ( x) − g( x)
96 TEN


The product fg : X → R, Some algebraic functions are given below
defined as ( fg) ( x) = f ( x) g( x) (i) Polynomial Function

f + g and fg are defined only, if f and g have the same (a) The function
domain. In case the domain of f and g are different, f ( x) = a0 x n + a1 x n − 1 + a2 x n − 2 + K + an − 1 x + an
domain of f + g or fg = Domain of f ∩ Domain of g. where, a0, a1 , a2 ,K , an are real numbers and n ∈ N is known

The product cf : X→ R, defined as as polynomial function. If a0 ≠ 0, then n is the degree of
(cf ) ( x) = cf ( x), where c is a real number. polynomial function.
f f f ( x) (b) Domain of polynomial function is R.

The quotient is a function defined as ( x) = ,
g g g ( x) (c) A polynomial of odd degree has its range (−∞, ∞) but a
provided g( x) ≠ 0, x ∈ X polynomial of even degree has a range which is always
subset of R.

If domain of y = f ( x) and y = g( x) are D1 and D2
respectively, then the domain of f ( x) ± g( x) or (ii) Constant Function The function f ( x) = k , where k is constant,
f ( x) is known as constant function. Its domain is R and range is {k },
f ( x) ⋅ g( x) is D1 ∩ D2 , while domain of is
g ( x) (iii) Identity Function The function f ( x) = x, is known as identity
D1 ∩ D2 − { x : g( x) = 0}. function. Its domain is R and range is R.
p( x)
Equal or Identical Functions (iv) Rational Function The function f ( x) = , where p( x) and
q ( x)
Two functions f and g are said to be equal, if q ( x) are polynomial functions and q ( x) ≠ 0 , is called rational
(i) the domain of f = the domain of g function.
(ii) the range of f = the range of g Its domain is R − { x | q ( x) = 0}.
(iii) f ( x) = g( x), ∀ x ∈ domain (v) Irrational Function The algebraic functions containing one or
more terms having non-integral rational power of x are called
irrational functions.
Inverse Functions e.g., y = f ( x) = 2 x − 3 x + 6

If f : A → B is a bijective function, then the mapping
f −1 : B → A which associate each element b ∈ B to a 1
(vi) Reciprocal Function The function f ( x) = is called the
unique element a ∈ A such that f (a) = b , is called the x
inverse function of f . reciprocal function of x. Its domain is R − {0} and range is
R − {0}.
f −1 (b ) = a ⇔ f (a) = b

The curves y = f ( x) and y = f −1 ( x) are mirror images of
each other in the line mirror y = x. 2. Piecewise Functions

f is invertible iff f is one-one and onto. Piecewise functions are special type of algebraic functions.

Inverse of bijective function is unique and bijective. (i) Absolute Valued Function ( Modulus Function) The function
−1
The solution of f ( x) = f ( x) are same as the solution  x, x ≥ 0

f ( x) = | x | =  is called modulus function.
of f ( x) = x. − x , x < 0

If fo g = I = gof , then f and g are inverse of each other. Y
−1 −1 −1 −1

fof = I B , f of = I A and ( f ) = f.
y = |x|

If f and g are two bijections such that (gof) exists, then
gof is also bijective function and (gof )−1 = f −1og −1 .
X¢ X
O

Basic Functions
Basic functions can be categorised into the following Y¢
categories. Its domain is R and range is [0, ∞).
Properties of Modulus Function
1. Algebraic Functions (a) | x | ≤ a ⇒ − a ≤ x ≤ a (a > 0)
A function, say f ( x), is called an algebraic function, if it (b) | x | ≥ a ⇒ x ≤ − a or x ≥ a (a > 0)
consists finite number of terms involving powers and (c) | x ± y | ≤ | x | + | y |
roots of the independent variable x and the four (d) | x ± y | ≥ || x | − | y ||
algebraic operations + ,−, × and ÷ .
DAY 97

(ii) Signum Function The function f ( x) Y


| x| x  1, if x > 0 4
 or , if x ≠ 0 
= sgn ( x) =  x | x| = − 1, if x < 0 3
0, if x = 0  0, if x = 0 2

called signum function. 1
Y X¢ X
–4 –3 –2 –1 0 1 2 3 4
y=1 –1
–2 f (x) = (x)
X¢ X –3
O
–4
y=–1

Y¢ Its domain ∈ R and range ∈ I.
Its domain is R and range is {− 1, 0, 1}. (vi) Transcendental function The function which is not
(iii) Greatest Integer Function The symbol [ x] indicates the algebraic is called transcendental function.
integral part of x which is nearest and smaller than to x. (vii) Exponential Function The function f ( x) = ax , a > 0, a ≠ 1,
It is also known as floor of x. is called an exponential function.
x ∀ x ∈ I Y Y
The function f ( x) = [ x] =  is called
n, n ≤ x < n + 1, n ∈ I 0<a<1
greatest integer function. 1 1
Y X¢ X
X¢ O X
O
If a > 1
2

1
Y¢ Y¢
X¢ X Its domain is R and range is (0, ∞).
–2 –1 0 1 2
It is a one-one into function.
–1 f (x) = [x]
(viii) Logarithmic Function The function f ( x) = log a x, ( x, a > 0)
–2 and a ≠ 1 is called logarithmic function.
Y Y

a>1
Its domain is R and range is I. 0<a<1

(iv) Fractional Part Function The symbol { x} indicates the 1


X¢ X X¢ X
fractional part of x. i.e. { x} = x − [ x], x ∈ R 1

∴ y = { x} = x − [ x]
0, ∀ x ∈ I
The function f ( x) = { x} =  Y¢ Y¢
 x − n, n ≤ x < n + 1, n ∈ I Its domain is (0, ∞) and range is R.
is called the fractional part function.
It is a one-one into function.
Y (ix) Trigonometric Functions Some standard trigonometric
1 functions with their domain and range, are given below
(a) Sine Function f ( x) = sin x,
X¢ X Y
–3 –2 –1 0 1 2 3

(–3p/2, 1) (p/2, 1)

Its domain is R and range is (0, 1).
X¢ –p O p X
(v) Least Integer Function The symbol ( x) indicates the
integer part of x which is nearest and greater than x.
(3p/2, –1)
 x, ∀ x ∈ I (–p/2, –1)
The function f ( x) = ( x) = 
n + 1, n < x ≤ n + 1 n ∈ I Y¢
is called least integer function.
Its domain is R and the range is [− 1, 1] .
98 TEN

(b) Cosine Function f ( x) = cos x, (c) y = f ( x) = tan −1 x


Y Y
y = tan x
(0, 1) y=x
π/2
- 2p p y = tan –1 x
-p
X¢ X
-3p 3p 2p
2 p/2 2
-p/2 –π/2
(–p, –1) (p, –1) X¢ X
O π/2

Its domain is R and the range is [− 1, 1].


(c) Tangent Function f ( x) = tan x, –π/2
Y

 π π
Its domain is R and range is  − ,  .
 2 2
–π π
X¢ X
–3π/2 –π/2 O π/2 3π/2
Nature of a Function
A function f ( x) is said to be an odd function, if

f (− x) = − f ( x), ∀ x.
(2 n + 1) π 
Its domain is R −  , n ∈ I  and range is R. A function f ( x) is said to be an even function,
 2 
if f (− x) = f ( x), ∀ x.
(x) Inverse Trigonometric Function Some standard inverse
Different Conditions for Even and Odd Functions
trigonometric functions with their domain and range, are
given below. f( x ) g ( x) f ( x) + g ( x) f ( x) − g ( x)
(a) y = f ( x) = sin −1 x
Odd Odd Odd Odd
Y y=sin–1 x
π/2 Even Even Even Even
y=x
Neither odd Neither odd
1 Odd Even
nor even nor even
y=sin x Neither odd Neither odd
−π/2 −1 Even Odd
X¢ X nor even nor even
O 1 π/2
f ( x )g ( x ) f ( x) / g ( x) (gof ) ( x ) ( fog )( x )
−1 Even Even Odd Odd

−π/2 Even Even Even Even


Y′ Odd Odd Even Even
Odd Odd Even Even
 π π
Its domain is [− 1, 1] and range is − , .
 2 2 
(b) y = f ( x) = cos −1 x NOTE
Y • Every function can be expressed as the sum of an even and
π an odd function.
–1
y= cos x
• Zero function f ( x ) = 0 is the only function which is both
even and odd.
π/2 y=x
• Graph of odd function is symmetrical about origin.
1 • Graph of even function is always symmetrical aboutY-axis.

π/2 π 3. Periodic Function


X¢ X
–1 O 1 y=cos x ●
A function f ( x) is said to be periodic function, if there exists
a positive real number T, such that f ( x + T ) = f ( x), ∀ x ∈ R.
Y′

The smallest value of T is called the Fundamental period of
Its, domain is [− 1, 1] and range is [0, π ]. f ( x).
DAY 99

Properties of Periodic Function Periods of Some Important Functions


(i) If f ( x) is periodic with period T, then cf ( x), f ( x + c) and Function Periods
f ( x) ± c is periodic with period T. sin x, cos x, sec x, cosec x, (sin x )2 n + 1 ,

(ii) If f ( x) is periodic with period T, then kf (cx + d) has (cos x )2 n + 1 , (sec x )2 n + 1 , (cosec x )2 n + 1
T tan x, cot x, tann x, cotn x, (sin x )2 n , (cos x )2 n ,
period .
|c| (sec x )2 n , (cosec x )2 n ,|sin x | ,|cos x | ,| tan x | , π
(iv) If f (x) is periodic with period T1 and g ( x) is periodic |cot x |,|sec x | ,|cosec x |
with period T2 , then f ( x) + g ( x) is periodic with period |sin x + cos x |, sin4 x + cos 4 x,
equal to LCM of T1 and T2 , provided there is no positive π /2
|sec x | + |cosec x | ,| tan x | + |cot x |
k, such that f (k + x) = g( x) and g (k + x) = f ( x).
x − [ x] 1
(iv) If f ( x) is a periodic function with period T and g ( x) is
x, x , x + 5,c ,... etc.
2 2 Period does
any function, such that range of f ⊆ domain of g, then Algebraic functions like
notexist
gof is also periodic with period T.

DAY PRACTICE SESSION 1

FOUNDATION QUESTIONS EXERCISE


1 Two sets A and B are defined as follows 6 The period of the function f ( x ) = sin3 x + cos 3 x is
A = {( x , y ) : y = e 2 x, x ∈ R } and (a) 2 π (b) π

B = {( x , y ) : y = x 2, x ∈ R }, then (c) (d) None of these
3
(a) A ⊂ B (b) B ⊂ A
(c) A ∪ B (d) A ∩ B = φ 7 Let f : R → R be defined by f ( x ) = x 2 + 1.
Then, pre-images of 17 and −3, respectively are
π
2 Domain of definition of the function f ( x ) = sin−1( 2x ) + (a) φ, {4,−4 } (b) {3,−3 }, φ j NCERT Exemplar
6
(c) {4, − 4 }, φ (d) {4,−4 },{2,−2 }
for real valued x , is
8 Suppose f ( x ) = ( x + 1)2 for x ≥ −1. If g ( x ) is the function,
(a)  − ,  (b)  − ,
1 1 1 1
 4 2   2 2  whose graph is reflection of the graph of f ( x ) w.r.t. the
(c)  − ,  (d)  − ,
1 1 1 1 line y = x , then g ( x ) is equal to
 2 9  4 4  (a) − x − 1, x ≥ 0
1 1
3 The domain of the function f ( x ) = is (b) , x > −1
|x|−x (x + 1)2
(a) (0, ∞) (b) (− ∞, 0) (c) x + 1, x ≥ − 1
(c) (− ∞, ∞) − (0) (d) (− ∞, ∞)
(d) x − 1, x ≥ 0
4 Domain of definition of the function  1 1 x
3 9 The function f : R → − , defined as f ( x ) = is
f (x ) = + log10 ( x 3 − x ), is  2 2  1+ x2
4 − x2 j
AIEEE 2003
(a) invertible j
JEE Main 2017
(a) (1, 2)
(b) injective but not surjective
(b) (−1, 0) ∪ (1, 2)
(c) surjective but not injective
(c) (1, 2) ∪ (2 , ∞)
(d) neither injective nor surjective
(d) (−1, 0) ∪ (1, 2) ∪ (2 , ∞)
ex − e−x
5. If f : R → R is a function satisfying the property 10 The inverse of the function f ( x ) = + 2 is given by
f ( x + 1) + f ( x + 3) = 2 for all x ∈ R , then f is ex + e−x
1/ 2 1/ 2
(a) periodic with period 3  x − 2  x − 1
(a) loge   (b) loge  
(b) periodic with period 4  x − 1 3 − x
(c) non- periodic 1/ 2 −2
 x   x − 1
(d) periodic with period 5 (c) loge   (d) loge  
2 − x  x + 1
100 TEN

11 If f ( x ) is an invertible function, and g ( x ) = 2f ( x ) + 5, then 18. If domain of f ( x ) and g ( x ) are D1 and D2 respectively,
the value of g −1 is then domain of f ( x ) + g ( x ) is D1 ∩ D2 , then
1
(a) 2f −1 (x) − 5 (b) Statement I The domain of the function
2f −1 (x) + 5
1
(c) f −1 (x) + 5 (d) f −1 
x −5 f ( x ) = sin−1 x + cos −1 x + tan−1 x is [ −1, 1].

2  2  Statement II sin−1 x and cos−1 x is defined in | x | ≤ 1and
tan−1 x is defined for all x.
12 Let f : ( 2 , 3) → ( 0, 1) be defined by f ( x ) = x − [ x ], then
(a) Statement I is true, Statement II is true; Statement II is a
f −1 ( x ) is equal to
correct explanation for Statement I
(a) x − 2 (b) x + 1 (c) x −1 (d) x + 2
(b) Statement I is true, Statement II is true; Statement II is
13 For a real number x , [ x ] denotes the integral part of x. not a correct explanation for Statement I
(c) Statement I is true; Statement II is false
1  1 1  1 2 
The value of + + + + +K + (d)Statement I is false; Statement II is true
 2   2 100   2 100 
1 99  19. Statement I The period of
+ is
 2 100  1 1
f ( x ) = 2 cos ( x − π ) + 4 sin ( x − π ) is 3π.
3 3
(a) 49 (b) 50 (c) 48 (d) 51
1 − x  Statement II IfT is the period of f ( x ), then the period of
14 If f 2 ( x )⋅ f   = x , [ where, x ≠ −1,1 and f ( x ) ≠ 0], then
3

1 + x  T
f (ax + b) is .
find | [f ( − 2)]| (where [.] is the greatest integer function) |a |
(a) 1/ x (b) 1 − x (c) 1 (d) 2 (a) Statement I is true, Statement II is true; Statement II is a
− 1 , x < 0 correct explanation for Statement I

15 If g ( x ) = 1 + x − [ x ] and f ( x ) =  0 , x = 0 , ∀ x, then (b) Statement I is true, Statement II is true; Statement II is
1, x > 0 not a correct explanation for Statement I
f {g ( x )} is equal to 
(c) Statement I is true; Statement II is false
(a) x (b) 1 (c) f (x) (d) g (x) (d) Statement I is false; Statement II is true
16. The function f ( x ) = log ( x + x 2 + 1), is
(a) an even function (b) an odd function 20. If the range of f ( x ) is collection of all outputs f ( x )
(c) a periodic function corresponding to each real number in the domain, then
(d) neither an even nor an odd function  1 
Statement I The range of log  2
is ( − ∞, ∞).
17. Statement I f ( x ) = | x − 2 | + | x − 3 | + | x − 5 | is an odd 1 + x 
function for all values of x lie between 3 and 5.
Statement II When 0 < x ≤ 1, log x ∈ ( − ∞, 0].
Statement II For odd function f ( − x ) = − f ( x )
(a) Statement I is true, Statement II is true; Statement II is a (a) Statement I is true, Statement II is true; Statement II is a
correct explanation for Statement I correct explanation for Statement I
(b) Statement I is true, Statement II is true; Statement II is (b) Statement I is true, Statement II is true; Statement II is
not a correct explanation for Statement I not a correct explanation for Statement I
(c) Statement I is true; Statement II is false (c) Statement I is true; Statement II is false
(d) Statement I is false; Statement II is true (d) Statement I is false; Statement II is true

DAY PRACTICE SESSION 2

PROGRESSIVE QUESTIONS EXERCISE


x −1 3 If [ x 2 ] + x − a = 0 has a solution, where a ∈ N and a ≤ 20,
1 Domain of f ( x ) = , where {}⋅ denotes the
x − 2 {x } then total number of different values of a can be
fractional part of x, is (a) 2 (b) 3 (c) 4 (d) 6
(a) (− ∞, 0) ∪ (0, 2] (b) [1, 2)
4 Total number of solutions of [ x ] = x + 2 {x }, where [⋅]
2
(c) (− ∞, ∞) ~ [0, 2) (d) (− ∞, 0) ∪ (0, 1] ∪ [2, ∞)
and {}⋅ denotes the greatest integer function and
2 Range of f ( x ) = [| sin x | + | cos x | ], where [⋅] denotes the fractional part respectively, is equal to
greatest integer function, is (a) 2 (b) 4
(a) {0} (b) {0, 1} (c) {1} (d) None of these (c) 6 (d) None of these
DAY 101

5 If f ( x ) = sin x + cos x , g ( x ) = x 2 − 1, then g { f ( x )} is 8 If f ( x ) and g ( x ) are two functions such that


invertible in the domain f ( x ) = [ x ] + [ − x ] and g ( x ) = {x } ∀x ∈ R and h( x ) = f (g ( x ));
π π π
(a)  0,  (b)  − , 
then which of the following is incorrect ?
 2   4 4  ([⋅] denotes greatest integer function and {}⋅ denotes
π π
(c)  − ,  (d) [0, π] fractional part function).
 2 2  (a) f (x) and h (x) are inertial functions
(b) f (x) = g (x) has no solution
6 Let f ( x ) = x 10 + a ⋅ x 8 + b ⋅ x 6 + cx 4 + dx 2 be a polynomial (c) f (x) + h (x) > 0 has no solution
function with real corfficient. If f (1) = 1 and f ( 2) = −5, then (b) f (x) − h (x) is a periodic function
the minimum number of distinct real zeroes of f ( x ) is
9 The period of the function f ( x ) = [ 6x + 7] + cos πx − 6x ,
(a) 5 (b) 6
(c) 7 (d) 8 where [⋅] denotes the greatest integer function, is
(a) 3 (b) 2 π (c) 2 (d) None of these
7 If f : R → R , f ( x ) = x 3 + 3, and g : R → R , g ( x ) = 2x + 1, then
−1 −1 10 The number of real solutions of the equation
f og ( 23) equals
(a) 2 (b) 3 log0. 5 | x | = 2| x | is.
(c) 4 (d) 5 (a) 1 (b) 2 (c) 0 (d) None of these

ANSWERS
SESSION 1 1 (d) 2 (a) 3 (b) 4 (d) 5 (b) 6 (a) 7 (c) 8 (d) 9 (c) 10 (b)
11 (d) 12 (d) 13 (b) 14 (d) 15 (b) 16 (b) 17 (b) 18 (a) 19 (d) 20 (d)

SESSION 2 1 (d) 2 (c) 3 (c) 4 (b) 5 (b) 6 (a) 7 (a) 8 (b) 9 (c) 10 (b)

Hints and Explanations


SESSION 1 4 Given, f ( x) =
3
+ log10 ( x3 − x)  3 sin x − sin 3 x
6 f ( x) =
4 − x2  4
1 Set A represents the set of points
lying on the graph of an For domain of f ( x), 3 cos x + cos 3 x 
x3 − x > 0 +
exponential function and set B 4 
represents the set of points lying on ⇒ x( x − 1)( x + 1) > 0
∴ Period of f ( x) = LCM of period of
the graph of the polynomial.
Take e2 x = x 2 , then the two curves – + – + {sin x, cos x, sin 3 x, cos 3 x}
does not intersect. Hence, there is –1 0 1 LCM of {2 π , 2 π }
no point common between them. = = 2π
⇒ x ∈ (−1, 0) ∪ (1, ∞) HCF of {1, 3}
2 For f ( x) to be defined, and 4 − x2 ≠ 0
π 7 Let y = x2 + 1
−1
sin (2 x) + ≥0 ⇒ x≠±2
6 ⇒ x ∈ (−∞, − 2) ∪ (−2 , 2) ∪ (2 , ∞) ⇒ x = ± y −1
π π So, common region is ∴ −1
( x) = ± x −1
⇒ − ≤ sin −1 (2 x) ≤ f
6 2 (−1, 0) ∪ (1, 2) ∪ (2 , ∞). −1
∴ f (17) = ± 17 − 1 = ± 4
 π  π
⇒ sin  −  ≤ 2 x ≤ sin   6 We have,
 6  2 and f −1 (−3) = ± −3 − 1
f ( x + 1) + f ( x + 3) = 2 ...(i)
1 1 = ± −4 ∉ R
⇒ − ≤ x≤ On replacing x by x + 2, we get
4 2 f ( x + 3) + f ( x + 5) = 2 ...(ii) ∴ −1
f (−3) = φ
 1 1 On subtracting Eq. (ii) from Eq. (i),
⇒ x∈ − , 8 Let y = ( x + 1)2 for x ≥ − 1
 4 2  we get
1 f ( x + 1) − f ( x + 5) = 0 ⇒ ± y = x +1⇒ y= x+1
3 y= ⇒ f ( x + 1) = f ( x + 5)
| x| − x ⇒ y ≥ 0, x + 1 ≥ 0
Now, on replacing x by x − 1, we
For domain,| x | − x > 0 get ⇒ x = y −1
⇒ | x| > x f ( x) = f ( x + 4) ⇒ f −1 ( y) = y −1
i.e. only possible, if x < 0. Hence, f is periodic with period 4. −1
∴ x ∈ (− ∞, 0) ⇒ f ( x) = x – 1, x ≥ 0
102 TEN

x 13 Q[ x] denotes the integral part of x. 1


9 We have, f ( x) = 20 Range of is (0, 1) and
1 + x2 1 50  1 + x2
Hence, after term + each
1  2 100  domain R
 1 x  1 
∴ f  = term will be one. Hence, the sum of
 x 1 given series will be 50. ∴ log   ∈ (− ∞, 0]
1+ 2  1 + x2 
x 1 − x
=
x
= f ( x) 14 f 2 ( x) ⋅ f   = x
3
…(i)
1 + x SESSION 2
1 + x2
1− x x −1
 1 On replacing x by , we get 1 We have, ≥ 0, here two
∴ f   = f (2) 1+ x
 2 x − 2 { x}
3
 1 1 − x 1 − x cases arise
or f   = f (3) f2   f ( x) =   …(ii)
Case I x ≥ 1 and x > 2 { x}
 3 1 + x 1 + x
and so on. From Eqs. (i) and (ii), ⇒ x ≥2
So, f ( x) is many-one function. 1 + x
3
∴ x ∈ [2, ∞).
Again, let y = f ( x) f 3 ( x) = x 6   Case II x ≤ 1 and x < 2 { x}
x 1 − x
⇒ y= ⇒ x < 1 and x ≠ 0.
1 + x2 1 + x
⇒ f ( x) = x 2   ∴ x ∈ ( − ∞, 0) ∪ (0, 1).
⇒ y + x2 y = x 1 − x
Finally, x = 1 is also a point of the
⇒ yx2 − x + y = 0 −4
As, x ∈R ⇒ f (− 2) = ⇒ [ f (− 2)] = − 2 domain.
3
∴ (− 1)2 − 4 ( y)( y) ≥ 0 ⇒ | [ f (− 2)]| = 2 2 y = | sin x | + | cos x |
⇒ 1 − 4 y2 ≥ 0
− 1 1 15 Q g ( x) = 1 + x − [ x] [put x = n ∈ Z ] ⇒ y2 = 1 + | sin 2 x |
⇒ y∈ , ⇒ 1 ≤ y2 ≤ 2 ⇒ y ∈[1, 2 ]
 2 2  ∴ g ( x) = 1 + x − x = 1
− 1 1 and g( x) = 1 + n + k − n = 1 + k ∴ f ( x) = [ y] = 1, ∀ x ∈ R
∴Range = Codomain = ,
 2 2  [put x = n + k ] 3 Since, [ x2 ] + x − a = 0
So, f ( x) is surjective. [where, n ∈ Z , 0 < k < 1 ] ∴ x has to be an integer.
Hence, f ( x) is surjective but not  −1, g( x) < 0 ⇒ a = x 2 + x = x ( x + 1)
injective. 
Now, f { g( x)} =  0, g( x) = 0 Thus, a can be 2, 6, 12, 20.
ex − e−x  1, g ( x) > 0
10 Given, y = x +2  4 [ x]2 = x + 2{ x}
e + e−x
Clearly, g ( x) > 0 , ∀ x ⇒ [ x] 2 = [ x] + 3 { x}
e2 x − 1 So, f { g( x)} = 1 , ∀ x [ x] 2 − [ x]
⇒ y = 2x +2 ⇒ { x} =
e +1 16 Given that, f ( x) = log ( x + x2 + 1 ) 3
1− y y –1
⇒ e2 x = = Now, f (− x) = log (− x + x2 + 1 ) [ x] 2 − [ x]
y −3 3 − y ⇒ 0≤ <1
∴ f ( x) + f (− x) = log ( x + x + 1)
2 3
1  y − 1
⇒ x = loge   + log (− x + x 2 + 1 )  1 − 13   1 + 13 
2 3 − y ⇒ [ x] ∈  , 0  ∪ 1, 
1 /2 = log (1) = 0  2   2 
 y − 1
⇒ f −1 ( y) = loge   Hence, f ( x) is an odd function. ⇒ [ x] = − 1, 0, 1, 2
3 − y
 −3 x + 10 , ∀ x ≤ 2 2 2
 x − 1
1 /2
 − x + 6, ∀ 2 < x ≤ 3 ⇒ { x} = , 0, 0,
⇒ f −1 ( x) = loge   17 Here, f ( x) =  3 3
3 − x
 x, ∀ 3 < x ≤ 5 ∴
1
x = − , 0, 1,
8
11 We have, g( x) = 2 f ( x) + 5 3 x − 10, ∀ x >5 3 3
∴ f ( x) = x, ∀ 3 < x < 5 5 g{ f ( x)} = (sin x + cos x)2 − 1 is
Now, on replacing x by g −1 ( x), we get
⇒ f (− x ) = − x = − f ( x )
g(g −1 ( x)) = 2 f (g −1 ( x)) + 5 invertible.
18 Since, sin −1 x is defined in [−1, 1] , ⇒ g{ f ( x)} = sin 2 x
⇒ x = 2 f (g −1 ( x)) + 5
cos −1 x is defined in [−1, 1] and We know that, sin x is bijective only
x −5
⇒ f (g −1 ( x)) = tan −1 x is defined in R.  π π
2 Hence, f ( x) is defined in [−1, 1]. when x ∈ − , .
 2 2 
−1 −1  x − 5  1
⇒ g ( x) = f   19 Period of 2 cos ( x − π ) and Thus, g{ f ( x)} is bijective, if
 2 
3 π π
− ≤ 2x ≤ .
12 f : (2 , 3) → (0, 1) and f ( x) = x − [ x] 1 2π 2π 2 2
4 sin ( x − π ) are , or 6 π , 6 π
∴ f ( x) = y = x − 2 ⇒ x = y + 2 3 1/3 1/3 π π
∴ − ≤ x≤
⇒ f −1 ( x) = x + 2 ∴ Period of their sum = 6π 4 4
DAY 103

6 Since, f ( x) is an even function. Now, let y = (gof )−1 (23) Now, as {6 x} has period
1
and
∴Its graph is symmetrical about ⇒ (gof )( y) = 23 6
Y -axis ⇒ 2 y3 + 7 = 23 cos πx has the period 2, therefore the
Also, we have, ⇒ 2 y3 = 16  1
⇒ y3 = 8 period of f ( x) = LCM  2,  which is
f (1) = 1 and f (2) = −5  6
⇒ f (−1) = 1 and f (−2) = −5 ⇒ y =2
2.
According to these information, we Hence, f −1og −1 (23) = 2
Hence, the period is 2.
have the following graph 8 We have, 10 For the solution of given equation,
 0, if x ∈ I
Y 1 f ( x ) = [ x ] + [− x ] =  let us draw the graph of y = log 0. 5| x|
−1, if x ∉ I and y = 2| x|
 0, if x ∈ I y=2|x|
g ( x) = { x} = 
( x), if x ∉ I
Y
and h( x) = f (g( x))
X1 • –2 • • 2 • X
–1 1 = f ({ x})
 f (0), x ∈ I • •
=
 f ({ x}), x ∉ I
 0 , x ∈I X1 X
=
Y1
 −1 , x ∉ I
Thus, minimum number of zeroes Clearly, option (b) is incorrect.
is 5.
y=log0.5 (–x) y=log0.5 (x)
7 Clearly, f −1og −1 (23) = (gof )−1 (23) 9 We have, f ( x) = [6 x + 7] + cos πx − 6 x Y
1

Here, gof ( x) = 2( x 3 + 3) + 1 = [6 x] + 7 + cos πx − 6 x


From the graph it is clear that there
= 2 x3 + 7 = 7 + cos πx − {6 x}
are two solution.
[Q{ x} = x − [ x]]
DAY ELEVEN

Limits, Continuity
and Differentiability
Learning & Revision for the Day

u Limits u Methods to Evaluate Limits u Differentiability


u Important Results on Limit u Continuity

Limits
Let y = f ( x) be a function of x. If the value of f ( x) tend to a definite number as x tends to a, then
the number so obtained is called the limit of f ( x) at x = a and we write it as lim f ( x).
x→ a


If f ( x) approaches to l1 as x approaches to ‘a’ from left, then l1 is called the left hand limit of
f ( x) at x = a and symbolically we write it as f (a − 0) or lim− f ( x) or lim f (a − h)
x→ a h→ 0

Similarly, right hand limit can be expressed as
f (a + 0) or lim+ f ( x) or lim f (a + h)
x→ a h→ 0

lim f ( x) exists iff lim− f ( x) and lim+ f ( x) exist and equal.
x→ a x→ a x→ a

Fundamental Theorems on Limits


If lim f ( x) = l and lim g( x) = m (where, l and m are real numbers), then
x→ a x→ a
PRED
(i) lim { f ( x) + g( x)} = l + m
x→ a
[sum rule] MIRROR
Your Personal Preparation Indicator
(ii) lim { f ( x) − g( x)} = l − m [difference rule]
x→ a

(iii) lim { f ( x) ⋅ g( x)} = l ⋅ m [product rule] u No. of Questions in Exercises (x)—


x→ a u No. of Questions Attempted (y)—
(iv) lim k ⋅ f ( x) = k ⋅ l [constant multiple rule] u No. of Correct Questions (z)—
x→ a
f ( x) l (Without referring Explanations)
(v) lim = ,m ≠0 [quotient rule]
x → a g ( x) m
u Accuracy Level (z / y × 100)—
1 Prep Level (z / x × 100)—
(vi) If lim f ( x) = + ∞ or − ∞, then lim =0 u

x→ a x→ a f ( x)
In order to expect good rank in JEE,
(vii) lim f ( x) = lim f ( x) your Accuracy Level should be
x→ a x→ a
above 85 & Prep Level should be
above 75.
(viii) lim log{ f ( x)} = log {lim f ( x)}, provided lim f ( x) > 0
x→ a x→ a x→ a
DAY 105

(ix) If f ( x) ≤ g( x), ∀x, then lim f ( x) ≤ lim g( x)


x→ a x→ a 3. Logarithmic Limits
(x) lim[ f ( x)] g (x )
= {lim f ( x)}
lim g ( x )
x →a log a (1 + x)
x→ a x→ a (i) lim = log a e; a > 0, ≠ 1
x→ 0 x
(xi) lim f {g( x)} = f {lim g( x)} = f (m) provided f is loge (1 + x) loge (1 − x)
x→ a x→ a
(ii) In particular, lim = 1 and lim = −1
continuous at lim g( x) = m. x→ 0 x x→ 0 x
x→ a

(xii) Sandwich Theorem If f ( x) ≤ g( x) ≤ h( x)∀x ∈ (α , β) − {a}


4. Expotential Limits
and lim f ( x) = lim h( x) = l , then lim g( x) = l where a ∈(α , β)
x→ a x→ a x→ a ax − 1
(i) lim = loge a, a > 0
x→ 0 x
Important Results on Limit ex −1 e λx − 1
(ii) In particular, lim = 1 and lim =λ
Some important results on limits are given below x→ 0 x x→ 0 x
 0, 0 ≤ a <1
1. Algebraic Limits  1, a =1
(iii) lim ax = 
x n − an x→ ∞
 ∞ , a >1
(i) lim = nan −1 , n ∈ Q, a > 0
x→ a x −a Does not exist, a < 0
1
(ii) lim n = 0, n ∈ N
5. 1∞ Form Limits
x→ ∞ x

(iii) If m, n are positive integers and a0, b 0 are non-zero real


numbers, then (i) If lim f ( x) = lim g( x) = 0, then
x→ a x→ a f (x )
a x m + a x m −1 + ... + am −1 x + am lim
lim{1 + f ( x)}1 / g ( x ) = e
x → a g (x )
lim 0 n 1 n −1
x→ ∞ b x + b x + ... + b n −1 x + b n x→ a
0 1
 a0 (ii) If lim f ( x) = 1 and lim g( x) = ∞, then
if m = n x→ a x→ a
b lim { f ( x ) − 1 } g ( x )
 00 lim{ f ( x)}g ( x ) = e x → a
= if m<n x→ a

 ∞ if m > n, a0 b 0 > 0 In General Cases



−∞ if m > n, a0 b 0 < 0
x
 1
(i) lim(1 + x)1 / x = e (ii) lim 1 +  = e
x→ ∞  x
(1 + x)n − 1 (1 + x)m − 1 m x→ 0
(iv) lim =n (v) lim = x
x→ 0 x x → 0 (1 + x)n − 1 n  λ
(iii) lim(1 + λx)1 / x = e λ (iv) lim 1 +  = e λ
x→ 0 x→ ∞  x
bx
2. Trigonometric Limits  a
(v) lim (1 + ax) b / x = lim 1 +  = e ab
x→ 0 x→ ∞  x
sin x x
(i) lim = 1 = lim
x→ 0 x x → 0 sin x

(ii) lim
tan x
= 1 = lim
x Methods To Evaluate Limits
x→ 0 x → 0 tan x
x To find lim f ( x), we substitute x = a in the function.
x→ a
sin −1 x x
(iii) lim = 1 = lim If f (a) is finite, then lim f ( x) = f (a).
x→ 0 x x → 0 sin −1 x x→ a
0 ∞
tan −1 x x If f (a) leads to one of the following form ; ; ∞ − ∞; 0 × ∞; 1 ∞ , 0
(iv) lim = 1 = lim 0 ∞
x→ 0 x x → 0 tan −1 x
and ∞ 0 (called indeterminate forms), then lim f ( x) can be
sin x ° π sin x cos x x→ a
(v) lim = (vi) lim = lim =0 evaluated by using following methods
x→ 0 x 180 x→ ∞ x x→ ∞ x
(i) Factorization Method This method is particularly used
(vii) lim sin x or lim cos x oscillates between −1 to 1.
x→ ∞ x→ ∞ when on substituting the value of x, the expression take
sin P mx  m 
P the form 0/0.
(viii) lim = 
x → 0 sin P nx  n (ii) Rationalization Method This method is particularly used
P
when either the numerator or the denominator or both
tan P mx  m  involved square roots and on substituting the value of x,
(ix) lim = 
x→ 0 tan P nx  n  0 ∞
the expression take the form , .
1 − cos m x m2 cos ax − cos bx a2 – b 2 0 ∞
(x) lim = 2 ; lim = NOTE To evaluate x → ∞ type limits write the given expression in the
x → 0 1 − cos n x n x → 0 cos cx − cos dx c2 – d2
form N /D and then divide both N and D by highest power of x
cos mx − cos nx n2 − m2
(xi) lim = occurring in both N and D to get a meaningful form.
x→ 0 x2 2
106 TEN

L’Hospital’s Rule Continuity of a Function in an Interval


If f ( x) and g( x) be two functions of x such that A function f ( x) is said to be continuous in (a, b ) if it is
(i) lim f ( x) = lim g( x) = 0. continuous at every point of the interval (a, b ). A function f ( x)
x→ a x→ a
is said to be continuous in [a, b ], if f ( x) is continuous in (a, b ).
(ii) both are continuous at x = a. Also, in addition f ( x) is continuous at x = a from right and
(iii) both are differentiable at x = a. continuous at x = b from left.
(iv) f ′ ( x) and g′ ( x) are continuous at the point x = a, then
f ( x) f ′ ( x) Results on Continuous Functions
lim = lim provided that g(a) ≠ 0.
x → a g ( x) x → a g ′ ( x)
(i) Sum, difference product and quotient of two
Above rule is also applicable, if lim f ( x) = ∞ and lim g( x) = ∞. continuous functions are always a continuous
x→ a x→ a f ( x)
function. However, r ( x) = is continuous at x = a
If f ′ ( x), g′ ( x) satisfy all the conditions embeded in L’Hospital’s g ( x)
rule, then we can repeat the application of this rule on only if g(a) ≠ 0.
(ii) Every polynomial is continuous at each point of real
f ′ ( x) f ′ ( x) f ′ ′ ( x)
to get lim = lim . line.
g ′ ( x) x → a g ′ ( x) x → a g ′ ′ ( x)
(iii) Every rational function is continuous at each point
where its denominator is different from zero.
Sometimes, following expansions are useful in evaluating
limits. (iv) Logarithmic functions, exponential functions,
trigonometric functions, inverse circular functions and
x2 x3 x 4 x 5 modulus function are continuous in their domain.

log(1 + x) = x − + − + + ... + (−1 < x ≤ 1)
2 3 4 5 (v) [ x] is discontinuous when x is an integer.
x2 x3 x 4 x 5

log(1 − x) = − x − − − − − ...(−1 < x < 1) (vi) If g( x) is continuous at x = a and f is continuous at g(a),
2 3 4 5 then fog is continuous at x = a.
2 3 4
x x x x (vii) f ( x) is a continuous function defined on [a, b ] such that

ex = 1 + + + + + ...
1 ! 2 ! 3 ! 4! f (a) and f (b ) are of opposite signs, then there is atleast
x 2
one value of x for which f ( x) vanishes, i.e. f (a) > 0,

ax = 1 + x (loge a) + (loge a)2 + ... f (b ) < 0 ⇒ ∃ c ∈ (a, b ) such that f (c) = 0.
2!
x3 x 5 x7

sin x = x − + − + ...
3! 5! 7! Differentiability
x2 x 4 x 6

cos x = 1 − + − − ... The function f ( x) is differentiable at a point P iff there exists a
2 ! 4! 6 !
unique tangent at point P. In other words, f ( x) is differentiable
at a point P iff the curve does not have P as a corner point, i.e.
Continuity the function is not differentiable at those points where graph
If the graph of a function has no break (or gap), then it is of the function has holes or sharp edges. Let us consider the
continuous. A function which is not continuous is called a function f ( x) = | x − 1|. It is not differentiable at x = 1. Since,
1 f ( x) has sharp edge at x = 1.
discontinuous function. e.g. x2 and e x are continuous while
x Y
f (x) = x – 1
and [ x], where [ ⋅ ] denotes the greatest integer function, are f (x) = – x + 1 Rf ' (x) = 1
discontinuous. Lf ' (x) = – 1

Continuity of a Function at a Point


X' X
A function f ( x) is said to be continuous at a point x = a of its O 1 2 3
domain if and only if it satisfies the following conditions Y'
(i) f (a) exists, where (‘a’ lies in the domain of f (graph of f ( x) describe differentiability)

(ii) lim f ( x) exist, i.e. lim− f ( x) = lim+ f ( x)


x→ a x→ a x→ a Differentiability of a Function at a Point
or LHL = RHL A function f is said to be differentiable at x = c, if left hand
(iii) lim f ( x) = f (a), f ( x) is said to be and right hand derivatives at c exist and are equal.
x→ a

left continuous at x = a, if lim− f ( x) = f (a) ●


Right hand derivative of f ( x) at x = a denoted by
x→ a
f (a + h) − f (a)
right continuous at x = a, if lim+ f ( x) = f (a) f ′ (a + 0) or f ′ (a + ) is lim ⋅
x→ a h→ 0 h
DAY 107


Left hand derivative of f ( x) at x = a denoted by f ′ (a − 0) Results on Differentiability
f (a − h) − f (a)
or f ′ (a− ) is lim . (i) Every polynomial, constant and exponential function is
h→ 0 −h
differentiable at each x ∈ R.

Thus, f is said to be differentiable at x = a, if (ii) The logarithmic, trigonometric and inverse trigonometric
f ′ (a + 0) = f ′ (a − 0) = finite. function are differentiable in their domain.

The common limit is called the derivative of f ( x) at x = a (iii) The sum, difference, product and quotient of two
f ( x) − f (a) differentiable functions is differentiable.
denoted by f ′ (a). i.e. f ′ (a) = lim . (iv) Every differentiable function is continuous but
x→ a x−a
converse may or may not be true.

DAY PRACTICE SESSION 1

FOUNDATION QUESTIONS EXERCISE


1 lim | x | [cos x ] , where [. ] is the greatest integer function, is (1 − cos 2x )( 3 + cos x )
x→ 0 9 lim is equal to
x→ 0 x tan 4x
(a) 1 (b) 0 j JEE Mains 2015, 13

(c) Does not exist (d) None of these 1


(a) 4 (b) 3 (c) 2 (d)
2 Let f : R → [ 0, ∞ ) be such that lim f ( x ) exists and 2
x→ 5
[(a − n ) nx − tan x ] sin nx
[f ( x )]2 − 9 10 If lim = 0, where n is non-zero
lim = 0. Then, lim f ( x ) is equal to x→ 0 x2
x→ 5 | x − 5| x→ 5
real number, then a is equal to
n+1 1
(a) 3 (b) 0 (c) 1 (d) 2 (a) 0 (b) (c) n (d) n +
n n
2 x  m
3 If lim = (where [⋅] denotes greatest integer sin( π cos 2 x )
x  5  n
x→ ∞ 11 lim is equal to
x→ 0 x2 j
JEE Mains 2014
function), then m + n (where m, n are relatively prime) is
π
(a) 2 (b) 7 (c) 5 (d) 6 (a) (b) 1 (c) − π (d) π
2
4 The value of the constant α and β such that sin x cos x tan x
x2 +1  f (x )
lim  − α x − β = 0 are respectively 12 If f ( x ) = x 3 x2 x , then lim is equal to
x→ ∞  x + 1  x→ 0 x 2
2x 1 1
(a) (1, 1) (b) (−1, 1) (c) (1, − 1) (d) (0, 1)
(a) 3 (b) −1 (c) 0 (d) 1
3 ⋅ 2 n +1 − 4 ⋅ 5 n +1
5 lim is equal to 13 The limit of the following is
n→ ∞ 5 ⋅ 2n + 7 ⋅ 5n
3 4 20 1 − cos( x 2 − 10x + 21)
(a) 0 (b) (c) − (d) − lim
5 7 7 x→ 3 ( x − 3)
(a) − (2)3 / 2 (c) (2)− 3 / 2
6 lim  x + x + x − x  is equal to
(b) (2)1 / 2 (d) 3
 
1  −1  x + 1  π 
x→ ∞

14 The value of lim tan   − is


(a) 0 (b)
1
(c) log2 (d) e 4 x→ 0 x   2x + 1 4 
2
x x x  x
j
JEE Mains 2013
7 The value of lim cos   cos   cos   … cos   is 1
n→ ∞  2  4  8  2n  (a) 1 (b) −
2
sinx x
(a) 1 (b) (c) (d) None of these (c) 2 (d) 0
x sin x
cot x − cos x
8 If f is periodic with period T and f ( x ) > 0∀x ∈ R , then 15 lim equals
x → π /2 ( π − 2 x )3 j
JEE Mains 2017
 f ( x + T ) + 2f ( x + 2T )+ ...+nf ( x + nT ) 
lim n   is equal to 1 1
n→ ∞  f ( x + T ) + 4f ( x + 4T )+ ...+ n 2f ( x + n 2T ) (a) (b)
24 16
2 3 1 1
(a) 2 (b) (c) (d) None of these (c) (d)
3 2 8 4
108 TEN

16 If m and n are positive integers, then (a) f is discontinuous everywhere


(cos x )1/ m − (cos x )1/ n (b) f is continuous only at x = 0
lim equals to (c) f is continuous for all irrational x and discontinuous for all
x→ 0 x2
1 1 m −n rational x
(a) m − n (b) − (c) (d) None of these (d) f is continuous for all rational x and discontinuous for all
n m 2mn
irrational x
cot −1( x + 1 − x ) 27 Let f ( x ) = 1 + | x − 2 | and g ( x ) = 1 − | x | , then the set of all
17 lim is equal to
x→ ∞ 
−1  2 x + 1
x
 points, where fog is discontinuous, is j JEE Mains 2013
sec   
 
 x −1  (a) {0, 2 } (b) {0, 1, 2 } (c) {0} (d) an empty set

(a) 1 (b) 0 (c)


π
(d) Does not exists
− 1 , x < 0

2 28 If f ( x ) =  0 , x = 0 and g ( x ) = sin x + cos x , then the
ln(cos 2x ) sin2 2x 1 , x > 0
18 Let p = lim , q = lim and 
x→ 0 3x 2 x → 0 x (1 − e x )
points of discontinuity of f {g ( x )} in ( 0 , 2π ) is
x −x π 3π 3π 7π 2π 5π 5π 7π
r = lim
x →1
. Then p, q , r satisfy (a)  ,  (b)  ,  (c)  ,  (d)  , 
ln x 2 4   4 4   3 3   4 3 
(a) p < q < r (b) q < r < p (c) p < r < q (d) q < p < r 1
29 If f ( x ) is differentiable at x = 1 and lim f (1 + h ) = 5, then
19 Let p = lim (1 + tan2 x )1/ 2 x , then log p is equal to h→ 0 h
x → 0+
j JEE Mains 2016 f ′ (1) is equal to
1 1 (a) 6 (b) 5 (c) 4 (d) 3
(a) 2 (b) 1 (c) (d)
2 4 30 If f ( x ) = 3x− 7x + 5x − 21x + 3x − 7, then the
10 8 6 3 2
x +1
 3x − 4  3 f (1 − h ) − f (1)
20 The value of lim   is value of lim is
x → ∞  3x + 2 h→ 0 h3 + 3h
53 22 22
(a) e −1 / 3 (b) e −2 / 3 (c) e −1 (d) e −2 (a) (b) (c) 13 (d)
2x
3 3 13
 a b
21 If lim 1 + +  = e 2 , then the values of a and b are 31 Let f ( 2) = 4 and f ′ ( 2) = 4. Then,
x→ ∞  x x2
x f ( 2) − 2f ( x )
(a) a ∈ R, b ∈ R (b) a = 1, b ∈ R lim is given by
x→ 2 x −2
(c) a ∈ R, b = 2 (d) a = 1, b = 2
f ( 2h + 2 + h 2 ) − f ( 2) (a) 2 (b) –2 (c) – 4 (d) 3
22 If f ′ ( 2) = 6 and f ′ (1) = 4 , then lim is
h→ 0 f (h − h 2 + 1) − f (1) 32 If f is a real-valued differentiable function satisfying
equal to | f ( x ) − f ( y )| ≤ ( x − y )2 ; x , y ∈ R and
(a) 3 (b) –3/2 (c) 3/2 (d) Does not exist f ( 0) = 0, then f (1) is equal to
23 Let f (a ) = g (a ) = k and their nth derivatives f n (a ), g n (a ) (a) 1 (b) 2 (c) 0 (d) –1
exist and are not equal for some n. Further, if log(a + x ) − log a log x − 1
33 If lim + k lim = 1, then
f (a )g ( x ) − f (a ) − g (a )f ( x ) + g (a )
x→ 0 x x → 0 x −e
lim = 4,
g( x ) − f ( x ) (a) k = e  1 − 
x→ a 1
(b) k = e (1 + a)
 a
then the value of k is
(c) k = e (2 − a) (d) Equality is not possible
(a) 4 (b) 2 (c) 1 (d) 0
34 The left hand derivative of f ( x ) = [ x ] sin ( πx ) at x = k , k is
24 If f : R → R be such that f (1) = 3 and f ′ (1) = 6. Then,
1/ x an integer, is
 f (1 + x )
lim   is equal to (a) (− 1)k (k − 1) π (b) (− 1)k − 1 (k − 1) π
x→ 0  f (1)  (c) (− 1)k kπ (d) (− 1)k − 1 kπ
(a) 1 (b) e1 / 2 (c) e 2 (d) e 3  ex, x ≤0
 sin x , x is rational
2 35 If f ( x ) =  , then
25 Let f ( x ) =  , then set of points, |1 − x | , x > 0
− sin x , x is irrational
2

(a) f (x) is differentiable at x = 0


where f ( x ) is continuous, is (b) f (x) is continuous at x = 0, 1
π
(a) (2n + 1) ,n ∈ I  (b) a null set (c) f (x) is differentiable at x = 1
 2  (d) None of the above
(c) {nπ,n ∈ I } (d) set of all rational numbers
  1
−
1
+ 
26 Let f :[a, b ] → R be any function which is such that f ( x ) is  | x | x
, x ≠ 0 , then f ( x ) is
36 If f ( x ) = xe
rational for irrational x and f ( x ) is irrational for rational x.  0 ,x = 0
Then, in [a, b ]
DAY 109

(a) continuous as well as differentiable for all x (a) f (x) is continuous and differentiable
(b) continuous for all x but not differentiable at x = 0 (b) f (x) is continuous but not differentiable
(c) neither differentiable nor continuous at x = 0 (c) f is not continuous but differentiable
(d) discontinuous everywhere (d) f is neither continuous nor differentiable
x 41 If f ( x ) = [sin x ] + [cos x ], x ∈ [ 0, 2π ], where [. ] denotes
37 The set of points, where f ( x ) = is differentiable, is
1+ |x| the greatest integer function. Then, the total number of
points, where f ( x ) is non-differentiable, is
(a) (− ∞, − 1) ∪ (−1, ∞) (b) (− ∞, ∞)
(c) (0, ∞) (d) (− ∞, 0) ∪ (0, ∞) (a) 2 (b) 3 (c) 5 (d) 4

38 Let f ( x ) = cos x and 42 If f ( x ) = | sin x | , then


(a) f is everywhere differentiable
min{f (t ): 0 ≤ t ≤ x } , x ∈ [ 0, π ]
g( x ) =  then (b) f is everywhere continuous but not differentiable at
 (sin x ) − 1 , x >π
x = nπ,n ∈Z
(a) g (x) is discontinuous at x = π (c) f is everywhere continuous but not differentiable at
(b) g (x) is continuous for x ∈ [0, ∞] π
x = (2n + 1) ,n ∈ Z
(c) g (x) is differentiable at x = π 2
(d) g (x) is differentiable for x ∈ [0, ∞] (d) None of the above
k x + 1 , 0 ≤ x ≤ 3 43 Statement I f ( x ) = | log x | is differentiable at x = 1 .
39 If the function g ( x ) =  is differentiable,
 mx + 2 , 3 < x ≤ 5 Statement II Both log x and – log x are differentiable at
then the value of k + m is x = 1.
16 10 (a) Statement I is false, Statement II is true
(a) 2 (b) (c) (d) 4
5 5 (b) Statemnt I is true, Statement II is true; Statement II is a
sin(cos −1 x ) + cos(sin−1 x ), x ≤ 0 correct explanation of Statement I
40 If f ( x ) =  (c) Statement I is true, Statement II is true; Statement II is
sin(cos x ) − cos(sin x ), x > 0
−1 −1
not a correct explanation of Statement I
then at x = 0 (d) Statement I is true, Statement II is false

DAY PRACTICE SESSION 2

PROGRESSIVE QUESTIONS EXERCISE


1 For each t ∈ R , let [t ] be the greatest integer less than or (a) 0 (b) f (x)
1   2  15  (c) g (x) (d) None of these
equal to t. Then, lim+ x  + +…+
  x   x   x  2/x
x→ 0
 a x + bx + c x 
6 The value of lim   ( where a , b , c > 0) is
j
JEE Mains 2018 x→ 0  3 
(a) is equal to 0 (b) is equal to 15
(c) is equal to 120 (d) does not exist (in R) (a) (abc)3 (b) abc
x (c) (abc)1 / 3 (d) None of these
 x 2 + 5x + 3
2 lim   is equal to  3x − x 3  1 − x 2 
x→ ∞  x2 + x + 2  7 If f ( x ) = cot −1  2 
and g ( x ) = cos −1   , then
 1 − 3x  1 + x 2 
(a) e 4 (b) e 2 (c) e 3 (d) e
f ( x ) − f (a ) 1
3 If α and β are the distinct roots of ax 2 + bx + c = 0, then lim , where 0 < a < , is equal to
x → a g ( x ) − g (a ) 2
1 − cos (ax 2 + bx + c ) 3 3 3 3
lim is equal to (a) (b) (c) (d) −
x→ α ( x − α )2 2 (1 + a 2 ) 2 (1 + x 2 ) 2 2
1 a2 a2
(a) (α − β)2 (b) − (α − β)2 (c) 0 (d) (α − β)2 8 If f : R → R is a function defined by
2 2 2
 2x − 1
4 lim sin[ π n + 1] is equal to
2 f ( x ) = [ x ] cos   π, where [ x ] denotes the greatest
n→ ∞  2 
(a) ∞ (b) 0 integer function, then f is
(c) Does not exist (d) None of these (a) continuous for every real x
5 If x > 0 and g is a bounded function, then (b) discontinuous only at x = 0
f ( x ) ⋅ e nx + g ( x ) (c) discontinuous only at non-zero integral values of x
lim is equal to (d) continuous only at x = 0
n→ ∞ e nx + 1
110 TEN

9 Let f be a composite function of x defined by 18 The function f ( x ) is discontinuous only at x = 0 such that
f (u ) = 2
1
, u( x ) =
1
⋅ f 2 ( x ) = 1 ∀x ∈ R. The total number of such function is
u +u−2 x −1 (a) 2 (b) 3
(c) 6 (d) None of these
Then, the number of points x, where f is discontinuous, is
j JEE Mains 2013 19 Let f ( x ) = x | x | and g ( x ) = sin x
(a) 4 (b) 3 (c) 2 (d) 1 Statement I gof is differentiable at x = 0 and its
10 If f : R → R be a positive increasing function with derivative is continuous at that point.

lim
f ( 3x )
= 1 . Then, lim
f ( 2x )
is equal to Statement II gof is twice differentiable at x = 0.
x→ ∞ f (x ) x → ∞ f (x )
(a) Statement I is false, Statement II is true
(a) 1 (b) 2/3 (c) 3/2 (d) 3 (b) Statement I is true, Statement II is true; Statement II is a
correct explanation of Statement I
 π 3π 
11 Let f ( x ) = max {tan x , sin x , cos x }, where x ∈ − , . (c) Statement I is true, Statement II is true; Statement II is
 2 2 
not a correct explanation of Statement I
Then, the number of points of non-differentiability is (d) Statement I is true, Statement II is false
(a) 1 (b) 3 (c) 0 (d) 2
20 Statement I The function
12 Let S = {t ∈ R : f ( x ) = | x − π|⋅(e| x | − 1)} sin| x | is not f ( x ) = ( 3x − 1) | 4x 2 − 12x + 5 | cos πx is differentiable at
differentiable at t}. Then, the set S is equal to 1 5
x = and .
(a) φ (an empty set) (b) {0} j JEE Mains 2018 2 2
(c) { π } (d) {0, π } π
n Statement II cos(2n + 1) = 0, ∀ n ∈ I .
 π 2
x + 
n
 3 (a) Statement I is true, Statement II is true; Statement II is a
13 If f ( x ) = lim n −1
, where n is an even integer,
n→ ∞
 π correct explanation for Statement I.
x n −1 +  
 3 (b) Statement I is true, Statement II is true; Statement II is
not a correct explanation for Statement I.
Then which of the following is incorrect?
(c) Statement I is true; Statement II is false.
π π
(a) If f:  , ∞  →  , ∞  , then f is both one-one and onto (d) Statement I is false; Statement II is true.
 3   3 
(b) f (x) = f (− x) has infinitely many solutions 21 Define f ( x ) as the product of two real functions f1( x ) = x ,
(c) f (x) is one-one for all x ∈ R (d) None of these x ∈ IR
n −1 x  sin 1 , if x ≠ 0
14 Let f ( x ) = lim ∑ . Then, and f2 ( x ) =  x as follows
n→ ∞ k = 0 (kx + 1){(k + 1)x + 1}
 0 , if x = 0
(a) f is continuous but not differentiable at x = 0
(b) f is differentiable at x = 0  f ( x ) ⋅ f2 ( x ), if x ≠ 0
f (x ) =  1
(c) f is neither continuous nor differentiable at x = 0  0 , if x = 0
(d) None of the above
Statement I f ( x ) is continuous on IR.

15 If x1, x 2 , x 3 ,..., x 4 are the roots of x n + ax + b = 0, then the Statement II f1( x ) and f2 ( x ) are continuous on IR.
value of ( x1 − x 2 )( x1 − x 3 )...( x1 − x n ) is equal to (a) Statement I is false, Statement II is true
(a) nx1 + b (b) nx1n − 1 + a (b) Statement I is true, Statement II is true; Statement II is
(c) nx1n − 1 (d) nx1n correct explanation of Statement I.
(c) Statement I is true, Statement II is true; Statement II is
 1 
 −b c  not a correct explanation of Statement I
 x  (d) Statement I is true, Statement II is false
1
16 If *lim x ln  0 −1  = −4, where a, b, c are real
x→ ∞  x  22 Consider the function f ( x ) = | x − 2 | + | x − 5 | , x ∈ R .
 1 0 a / x  Statement I f ′ ( 4) = 0

 
Statement II f is continuous in [ 2, 5 ] and differentiable in
numbers, then ( 2, 5) and f ( 2) = f ( 5 ).
(a) a = 1,b ∈ R , c = −1 (b) a ∈ R,b = 2,c = 4 (a) Statement I is true, Statement II is true; Statement II
(c) a = 1,b = 1,c ∈ R (d) a ∈ R,b = 1,c = 4 is a correct explanation for Statement I
1/ x 1/ x
 f (x )  f (x ) (b) Statement I is true, Statement II is true; Statement II
17 If lim 1 + x + = e 3 , then lim 1 + is equal to

x→ 0
 x  x→ 0 
 x  is not correct explanation for Statement I
(c) Statement I is true; Statement II is false
(a) e (b) e 2 (c) e 3 (d) None of these
(d) Statement I is false; Statement II is true
DAY 111

ANSWERS
SESSION 1 1 (a) 2 (a) 3 (b) 4 (c) 5 (d) 6 (b) 7 (b) 8 (c) 9 (c) 10 (d)
11 (d) 12 (d) 13 (a) 14 (b) 15 (b) 16 (c) 17 (a) 18 (d) 19 (c) 20 (b)
21 (b) 22 (a) 23 (a) 24 (c) 25 (c) 26 (a) 27 (d) 28 (b) 29 (b) 30 (a)
31 (c) 32 (c) 33 (a) 34 (a) 35 (b) 36 (b) 37 (b) 38 (b) 39 (a) 40 (d)
41 (c) 42 (b) 43 (a)

SESSION 2 1 (c) 2 (a) 3 (d) 4 (b) 5 (b) 6 (d) 7 (d) 8 (a) 9 (b) 10 (a)
11 (b) 12 (d) 13 (c) 14 (c) 15 (b) 16 (d) 17 (b) 18 (c) 19 (b) 20 (a)
21 (d) 22 (b)

Hints and Explanations


SESSION 1 n

6 ⋅   − 20
2 = lim
sin x

x / 2n
1 RHL = lim ( x )0 = 1  5 0 − 20 20 n→ ∞ x sin( x / 2n )
x → 0+
= lim = =−
n→ ∞ n
0+ 7 7 sin x
5⋅   + 7
2 =
LHL = lim− (− x )0 = lim− 1 = 1  5 x
x→ 0 x→ 0

Q RHL = LHL   8 Clearly, f ( x + T ) = f ( x + 2T ) = ...


6 xlim x+ x+ x − x
∴ lim | x |[cos x ] = 1 →∞  
x→ 0 = f ( x + nT ) = f ( x )
2 Given, lim f ( x ) exists and x+ x+ x − x  f ( x + T ) + 2 f ( x + 2T ) + ... 
x→ 5 = lim  
x→ ∞
+ nf ( x + nT ) 
[ f ( x )]2 − 9
=0
x+ x+ x + x ∴lim n 
lim n→ ∞  f ( x + T ) + 4 f ( x + 4T ) 
x→ 5
| x − 5| x+ x  
= lim
⇒ lim[ f ( x )]2 − 9 = 0
x→ ∞  + ...+ n f ( x + x T )
2 2

x→ 5
x+ x+ x + x
nf ( x )(1 + 2 + 3+ ...+ n )
2
 lim [ f ( x )] = 9 1 + x −1 / 2 = lim
⇒ = lim =
1 n→ ∞ f ( x )(1 + 22 + 32 + ...+ n2 )
 x→ 5  x→ ∞
1+ −1
+ x −3 / 2
+1 2
x n(n + 1)
∴ lim f ( x ) = 3, − 3 n  
x→ 5
 2  3
But f : R → [0, ∞ ) 7 We know that, = lim =
n→ ∞ n( n + 1)(2n + 1) 2
cos A ⋅ cos 2 A ⋅ cos 4 A K cos 2n −1
∴ Range of f ( x ) ≥ 0 6
sin 2n A
⇒ lim f ( x ) = 3 A= (1 − cos 2 x )(3 + cos x )
x→ 5 2n sin A 9 We have, lim
2  x 2x x  x→ 0
= lim  −    x x tan 4 x
3 Clearly, lim
x→ ∞ x  5 
Take A = n ,
  x→ ∞ x  5  5   2 2sin 2 x(3 + cos x )
= lim
then cos  n  ⋅ cos  n − 1  ...
2 2 x x x→ 0 tan 4 x
= −0= x× × 4x
5 5 2  2  4x
∴ m+ n =7 2
(3 + cos x )
cos   cos  
x x 2sin x
= lim × lim
 x2 + 1   4  2 x→ 0 x2 x→ 0 4
4 xlim  − α x − β = 0 sin x
→∞
 x + 1  = ×
1
2 sin  n 
x tan 4 x
x (1 − α ) − x (α + β ) + 1 − β
2 n
lim
⇒ lim =0 2  x→ 0 4x
x→ ∞ x+1
∴ limcos   cos   ...
x x 4
∴ 1 − α = 0, α + β = 0 = 2× ×1 = 2
n→ ∞  2  4 4
⇒ α = 1, β = − 1 Q lim sin θ = 1 and lim tan θ = 1 
cos  n − 1  ⋅ cos  x 
x
  n  x → 2 θ x→ 0 θ 
5 Clearly, 2  2 
3 ⋅ 2n +1 − 4 ⋅ 5n +1 6 ⋅ 2n − 20 ⋅ 5n sin x 10 Since,
lim = lim = lim
5⋅ 2 + 7 ⋅ 5 5⋅ 2n + 7 ⋅ 5n 2n sin  n 
n→ ∞ n n n→ ∞ n→ ∞ x
lim (a − n ) n −
tan x  sin nx
2  ⋅ =0
x→ 0  x 
 x
112 TEN

⇒ [(a − n ) n − 1] n = 0 (+2) 2
= lim − =−
⇒ (a − n ) n = 1 x2 + (4 x + 2)2
x→ 0 3
∴ a= n+
1 (+2) −2 −1 sin2 2 x 4 x2
=− = = q = lim ⋅ = −4
n 0 + (0 + 2)2 4 2 x→ 0 4x 2
x (1 − e x )
sin( π cos 2 x ) sin( π − π sin2 x )
11 lim = lim cot x − cos x x−x
x→ 0 x2 x→ 0 x2 15 xlim and r = lim
→ π /2 ( π − 2 x )3 x→1 ln(1 + x − 1)
sin( π sin x )
2
= lim 1 cos x(1 − sin x ) x (1 − x )
x→ 0 x2 = lim ⋅ = lim
x → π /2 8
π
3 x→1
 1 + x − 1
[Q sin( π − θ) = sin θ ] sin x  − x  ln   ⋅ ( x − 1)
2   x −1 
sin( π sin 2 x )  sin2 x 
= lim × ( π )  π  π  x (1 − x )
π sin x  x2  cos  − h  1 − sin  − h 
  = lim
x→ 0 2

1 2   2   x→1
 1 + ( x − 1)
sin θ = lim ⋅ ln   ⋅ ( x − 1)(1 + x)
= π Qlim = 1 h→ 0 8 3  x −1 
π π π
 θ → 0 θ  sin  − h   − + h  1
2  2 2  =−
12 Q f ( x ) = x( x − 1)sin x − ( x3 − 2 x2 ) 1 sin h (1 − cos h ) 2
= lim Hence, q < p < r .
cos x − x3 tan x 8 h→ 0 cos h ⋅ h3
= x sin x − x cos x − x tan x +
2 3 3 1

sin h  2sin2 
h 19 Given, p = lim (1 + tan2 x ) 2x
2 x2 cos x − x sin x 1  2 x→ 0 +

f ( x) = lim
∴ lim 2 = lim 8 h → 0 cos h ⋅ h3 (1∞ form)
x→ 0 x x→ 0

sin h ⋅ sin2  
2
h  tan x 
 sin x − x cos x − x tan x 
2
tan x 1
lim lim  
 x 
  1  2 = ex→0
+ 2x
= e2 x→0
+

 sin x  = lim
+ 2cos x − 4 h→ 0 h3 cos h 1
 x 
2 = e2
=2−1=1  sin h 
sin h   
1
1
lim 
1 2 ⋅ 1 ⋅ 1 ∴ log p = log e 2 =
1 − cos( x − 10 x + 21)
2 =  
13 lim 4 h → 0  h   h  cos h 4 2
x→3 ( x − 3)  2  x +1

( x − 3) ( x − 7) 1 1 1  3x − 4 3
2 sin = × = 20 xlim  
2 4 4 16
→∞
 3x + 2
= lim
x→3 ( x − 3) x +1
(cos x ) 1/m
− (cos x )1 / n  3x + 2 − 6 3
( x − 3) ( x − 7) 16 lim = lim  
2 sin x→ 0 x2 x→ ∞
 3x + 2 
2 ( x − 7)
= lim ⋅ 1

1
x→3 ( x − 3) ⋅ ( x − 7) 2 (cos x )m n
−1 1 x +1
= lim ⋅ lim  6  3
2 x→ 0 x2 x→ 0 (cos x )1 / n = lim  1 − 
= lim ( x − 7) 1 1
x→ ∞
 3x + 2

x→3
( x − 3) ( x − 7)  1 − 2sin2 x  m n
−1
−6 x +1
sin    
3x + 2 3x + 2

1  2 3
⋅ lim 2 × = lim   6  −6 
x→3 ( x − 3) ( x − 7) 2 x→ 0 x2 = lim  1 − 
x→ ∞  3x + 2 
x 
2 sin2  
= − (2)3 /2 = lim − 2 − 
1 1 2 = m−n −2 ( x + 1 )
x→ 0  m n x2 2mn = lim e 3x + 2
= e −2 / 3
1 
−1  x+1 π x→ ∞
14 lim  tan   − 
x→ 0 x  2x + 1 4 17 Clearly,  −2( x + 1) − 2 
Q xlim =
  x + 1  
= lim( x + 1 − x)

→∞ 3x + 2 3 
= lim  tan −1  −1
 − tan (1)
x→ ∞
x→ 0
  2x + 1  1
= lim =0
2x
1 + a + b 
 x + 1 − 1
x→ ∞
x+1+ x 21 Now, xlim 
→∞  2
 x x
1  2x + 1  and  b 
= lim ⋅ tan −1  a

x+1
x  + 
x→ 0 x  2+ 1  2x 
x x2 
1 +    = lim  1 + + 2 
x a b  b 
2x + 1
a
 2x + 1   x  =∞  + 
lim   = lim
x→ ∞  x − 1 

x→ ∞ 1 − 1 / x
x→ ∞  x x 
 x x2 

1  x + 1 − 2x − 1    a b 
= lim ⋅ tan −1     lim 2 x  + 
 x x2 
=e
x →∞
x→ 0 x
 2x + 1 + x + 1  −1
cot ( x + 1 − x) −1
cot (0)
∴ lim =
1  −x   form 0  x→ ∞
2x + 1
x
sec −1 (∞ ) Q lim (1 + x )1 / x = e  = e 2 a
= lim ⋅ tan −1  
 sec −1
  x→ ∞ 
x→ 0 x
 4x + 2 0   x −1 
2x
π /2
But lim  1 + + 2 
[using L’ Hospital rule] a b
= =1 = e2
1 π /2 x→ ∞  x x 
= lim
x→ 0 x2 ln(1 + cos 2 x − 1) ⇒ e 2a = e 2
1+ 18 Clearly, p = lim
(4 x + 2)2 x→ 0 2
3x ⇒ a=1
  4x + 2 − 4x   ln(1 + cos 2 x − 1) cos 2 x − 1 and b ∈R
× −   = lim ⋅
x→ 0 (cos 2 x − 1) 3 x2
  (4 x + 2)  
2
DAY 113

f (2h + 2 + h2 ) − f (2)  1, 0 < x < 3 π /4 34 If x is just less than k, then [ x] = k − 1


22 lim
h→ 0 f (h − h2 + 1) − f (1) 28 f { g ( x )} =  0, x = 3 π / 4 , 7 π /4 ∴ f ( x ) = (k − 1) sin π x
f ′ (2h + 2 + h2 )(2 + 2h )  − 1 , 3 π / 4 < x < 7 π /4
= lim  LHD of f ( x ) = lim
h → 0 f ′ ( h − h2 + 1)(1 − 2h )
or 7 π /4 < x < 2 x→ k

f ′ (2) × 2 (k − 1) sin πx − k sin πk


= Clearly, [ f {g ( x )}] is not continuous at
f ′ (1) × 1 3 π 7π x−k
x= , .
4 4 (k − 1) sin πx
6×2 = lim ,
= =3 x→ k x−k
4×1 f (1 + h ) − f (1)
29 f ′(1) = lim
h→ 0 h where x = k − h
f (a) g ( x ) − f ( a) − g ( a) f ( x ) + g ( a) (k − 1)sin π (k − h )
23 lim =4 f (1 + h ) f (1) = lim
x→ a g( x) − f ( x) = lim − lim
h→ 0 h h→ 0 h
h→ 0 −h
Applying L’Hospital rule, we get f (1 + h ) f (1) = (k − 1)(−1)k π
Since, lim = 5, so lim must
f (a) g ′ ( x ) − g (a)f ′ ( x ) h→ 0 h h→ 0 h  e ,
x
x≤ 0
lim =4
35 f ( x ) = 1 − x, 0 < x ≤ 1
x→ a g ′( x ) − f ′( x ) f (1)
be finite as f ′(1) exists and lim can
h→ 0 h
k g ′( x ) − k f ′( x )  x − 1, x > 1
⇒ lim =4 be finite only, if f (1) = 0 and 
x→ a g ′( x ) − f ′( x ) f (1) f (0 + h ) − f (0)
lim = 0. Rf ′ (0) = lim
∴ k =4 h→ 0 h h→ 0 h
1/x f (1 + h ) 1− h−1
 f (1 + x ) ∴ f ′ (1) = lim =5 = lim = −1
24 Let y =   h→ 0 h h→ 0 h
 f (1)  f (0 − h ) − f (0)
f (1 − h ) − f (1) Lf ′ (0) = lim
1 30 lim h→ 0 −h
⇒ log y = [log f (1 + x ) − log f (1)] h→ 0 h3 + 3h
x e−h − 1
= lim =1
f (1 − h ) − f (1) −1 h→ 0 −h
 1  = lim ⋅ 2
⇒ lim log y = lim  f ′ (1 + x ) h→ 0 −h h +3 So, it is not differentiable at x = 0.
 f (1 + x )
x→ 0 x→ 0

−1
= f ′ (1) ⋅   =
53 Similarly, it is not differentiable at x = 1
f ′ (1) 6  3  but it is continuous at x = 0 and 1.
⇒ lim log y = = 3
x→ 0 f (1) 3
x f (2) − 2 f ( x ) 36 RHL = lim h
(0 + h ) e −2 / h = lim 2 / h = 0
∴ lim y = e 2 31 lim h→ 0 h→ 0 e
x→ 0
x→2 x−2
1 1 
25 Clearly, f ( x ) is continuous only when x f (2) − 2 f (2) + 2 f (2) − 2 f ( x ) − − 
 h h
= lim LHL = lim (0 − h ) e =0
sin2 x = − sin2 x ⇒ 2sin2 x = 0 x→2 x−2 h→ 0

⇒ x = nπ f (2)( x − 2) − 2 { f ( x ) − f (2)} Hence, f ( x ) is continuous at x = 0.


= lim 1 1 
− + 
26 We have,
x→2 x−2 (0 + h ) e  h h
−0
f ( x ) − f (2) Now, Rf ′ ( x ) = lim
rational, if x ∉ Q in[a,b]
f ( x ) = 
h→ 0
= f (2) − 2 lim h
 irrational , if x ∈ Q in[a,b] x→2 x−2 = lim e − 2 / h =∞
h→ 0 1 1 
Let C ∈ [a,b] and c ∈ Q . Then,  f ( x ) − f (a) − − 

Q f ′ ( x ) = lim (0 − h ) e  h h
−0
f (c ) = irrational 
x→ a x − a  and Lf ′ ( x ) = lim
h→ 0 −h
and lim f ( x ) = lim f (c + h ) = rational or = f (2) − 2 f ′ (2)
x→c h→ 0 = lim e − 0 = 1
irrational = 4−2× 4= −4 h→ 0

Thus, f is discontinuous everywhere. ∴ Lf ′ ( x ) ≠ Rf ′ ( x )


32 Q | f ( x ) − f ( y )| ≤ ( x − y )
2

1 + x, x < 0 Hence, f ( x ) is not differentiable at x = 0.


27 g ( x ) =  ∴ lim
| f ( x ) − f ( y )|
≤ lim | x − y | g( x)
1 − x, x≥ 0 x→ y |x − y| x→ y 37 Since, f ( x ) = x = [say]
1 + | x| h( x )
1 + | x − 1|, x< 0
∴ f {g ( x )} =  ⇒ | f ′ ( y )| ≤ 0 ⇒ f ′( y ) = 0 It is clear that g ( x ) and h( x ) are
 + |− x − 1|,
1 x≥ 0
⇒ f ( y ) = Constant differentiable on (− ∞, ∞ ) and
1+ 1− x, x < 0
=  (− ∞, 0) ∪ (0, ∞ ).
1 + x + 1, x ≥ 0 ⇒ f( y) = 0 [Q f (0) = 0, given]
2 − x, x< 0 Now,
⇒ f (1) = 0
=  x
−0
2 + x, x≥ 0 33 Let f ( x ) = log x f ( x ) − f (0) 1 + | x|
lim = lim =1
It is a polynomial function, so it is 1 x→ 0 x−0 x→ 0 x
continuous in everywhere except at ⇒ f ′( x ) =
x Hence, f ( x ) is differentiable on (− ∞, ∞ ).
x = 0. Therefore, given function
x ∈ [0, π]
38 Clearly, g ( x ) = 
Now, LHL = lim 2 − x = 2, cos x ,
x→ 0 = f ′ (a) + k f ′ (e ) = 1
RHL = lim 2 + x = 2 1 k sin x − 1, x> π
x→ 0 ⇒ + =1
a e Also, g ( π − ) = g ( π ) = g ( π + ) = −1
Also, f (0) = 2 + 0 = 2
a − 1 and g ′( π − ) ≠ g ′( π + )
Hence, it is continuous everywhere. ⇒ k = e  
 a  ∴ g is continuous at x = π
but not differentiable at x = π
114 TEN

− log x, x < 1 a2 Q lim sin x = 1


39 Since, g ( x ) is differentiable ⇒ g ( x ) must 43 f ( x ) =  = lim ( x − β )2
 x → 0 x 
 log x, x ≥ 1
x→ α 2
be continuous.
a2
 k x + 1, 0 ≤ x ≤ 3  − 1 / x, x < 1 = (α − β )2
g ( x) =  f ′( x ) =  1 2
, x>1
 mx + 2, 3 < x ≤ 5  x 
1 /2

sin  nπ  1 + 2 
1
4 nlim 
At, x = 3, RHL = 3 m + 2 ∴ f ′ (1− ) = − 1 and f ′ (1+ ) = 1 →∞  n 
 
and at x = 3, LHL = 2k Hence, f ( x ) is not differentiable.
  1 1 
= limsin  nπ  1 + − + K∞ 
∴ 2k = 3 m + 2 n→ ∞

2 4

SESSION 2  2n 8 n
 k
, 0≤ x < 3  π π 
Also, g ′( x ) = 2 x + 1 1 We have, = lim sin  nπ + − + K∞
 1 15  n→ ∞ 2n 8 n3
lim x    +   + …+     
1
 m , 3< x ≤ 5
x→ 0 +  x   x   x    1 1 
k = lim (−1)n sin π  − +K ∞
∴ L(g ′ (3)) = We know, [ x] = x − { x} n→ ∞
 2n 8 n
3

4
∴ 1 = 1 − 1 =0
 
 x  x  x 
k
and R {g ′ (3)} = m ⇒ =m
4 5 Given, x > 0 and g is a bounded
Similarly,   = −  
n n n
i.e. k = 4m  x  x  x  function.
On solving Eqs (i) and (ii), we get ∴Given limit f ( x ) ⋅ e nx + g ( x )
Then, lim
1 e nx + 1
= lim+ x  −  
1 n→ ∞
8 2
k = ,m =
5 5
x→ 0  x  x  
2 2 15  15   f ( x) g( x) 
⇒ k+ m=2 + − + … − = lim  + nx 
 
x  x x  x  n→ ∞
 1 +  1  e + 1

40 Clearly, f ( x ) = 2 1 − x , x ≤ 0
2

= lim( 1 + 2 + 3+ ...+15) − x  e 
nx

 0, x> 0 x→ 0 +
f ( x) Finite
1 2  15  = + = f ( x)
∴ f ( x ) is discontinuous and hence    +   + ... +    1+ 0 ∞
 x  x  x 
non-differentiable at x = 0.
= 120 − 0 = 120  ax + b x + c x 
2/ x

41 [sin x] is non-differentiable at   6 Let y = lim  


Q 0 ≤   < 1, therefore
n
π  
x→ 0
 3 
x= , π, 2 π and [cos x] is  x
2   2  ax + b x + c x 
 0 ≤ x   < x ⇒ lim x   = 0
n n ⇒ log y = lim log  
non-differentiable at     x→ 0 x
 3 
  x x→ 0 +
 x 
π 3π
x = 0, , and 2 π. log(ax + b x + c x ) − log 3
2 2  x2 + 5x + 3 
x
= 2 lim
x→ 0 x
Thus, f ( x ) is definitely
2 Now, xlim  2 
→∞
 x + x+2 Apply L’Hospital’s rule,

non-differentiable at x = π, , 0. x ax log a + b x log b + c x log c
2  4x + 1 
= lim  1 + 2  ax + b x + c x
Also, x→ ∞
 x + x+ 2 = 2 lim
x→ 0
π π 1
f   = 1, f  − 0 = 0, ( 4 x +1 )x

 2 2   1
( 4x + 1 )
 x2 + x +2 log y = log (abc ) 2 /3
4x + 1 
= lim   1 + 2
x2 + x + 2
  ⇒ y = (abc ) 2 /3
f (2 π ) = 1, f (2 π − 0) = − 1 x→ ∞  x + x + 2
 
Thus, f ( x ) is also non-differentiable at    3 x − x3 
π  1 7 f ( x ) = cot −1  2 
x= and 2π. lim
 4 + 
x  1 − 3x 
2 x →∞
1 +
1
+
2

=e x x2
= e4  1 − x2 
42 Let u( x ) = sin x and g ( x ) = cos −1  2
 1 + 1  = e 
x
1 + x 
v( x) = |x | Q lim 
x→ 0 
 
 x  On putting x = tan θ in both equations,
we get
∴ f ( x ) = vou ( x ) = v (u( x )) = v (sin x ) =|sin x| 1 − cos (ax + bx + c )
2
3 Now, xlim  3 tan θ − tan3 θ 
→α ( x − α )2 f (θ) = cot −1  
Q u( x ) = sin x is a continuous function  1 − 3 tan θ 
2

and v ( x ) = | x| is a continuous function.  ax2 + bx + c 


2sin2   ⇒ f (θ) = cot −1 (tan 3 θ)
∴ f ( x ) = vou ( x ) is also continuous  2 
= lim π
everywhere but v ( x ) is not differentiable x→ α (x − α)2 ⇒ f (θ) = cot −1 cot − 3 θ 
at x = 0  2 
2sin2  ( x − α )( x − β )
a
⇒ f ( x ) is not differentiable π
2  = − 3θ
where sin x = 0 = lim 2 2
x→ α
 a  ( x − α )2 ( x − β )2 ∴ f ′ (θ) = − 3 … (i)
⇒ x = nπ , n ∈ Z  
 2  1 − tan2 θ 
Hence, f ( x ) is continuous everywhere and g(θ) = cos −1  2 
2
but not differentiable at  a  ( x − β )2  1 + tan θ 
 
x = nπ, n ∈ Z .  2
= cos −1 (cos 2θ) = 2θ
DAY 115

∴ g ′ (θ) = 2 Hence, the composite function y = f ( x ) and lim f ′( x ) = 0


x→ π −
Now, is discontinuous at three points,
1 = lim+ f ′( x )
 f ( x ) − f (a)  f ( x ) − f (a) x = , x = 1 and x = 2 . x→ π
lim 
x → a g ( x ) − g ( a)
 = lim   2 ∴ f is differentiable at x = 0 and x = π
  x→ a
 x−a 
1 10 Since, f ( x ) is a positive increasing Hence, f is equal to the set {0, π}.
× n
 g ( x ) − g (a) π
x n +  
function.
lim  
x→ a
 x−a  ∴ 0 < f ( x ) < f (2 x ) < f (3 x )  3
13 We have, f ( x ) = lim
n→ ∞ n −1
π
x n −1 +  
1 1 3 f (2 x ) f (3 x )
= f ′ (a) ⋅ = –3× = – ⇒ 0< 1< <
g ′ (a) 2 2 f ( x) f ( x)  3
 π 
n
8 Now, cos x is continuous, ∀x ∈ R ⇒ lim 1 ≤ lim
f (2 x ) x  1 +   
x→ ∞ x→ ∞ f ( x)   3x   π
⇒ cos π  x −  is also continuous,
1 = lim = x, if x >
n→ ∞ n −1
 2 π 3
≤ lim
f (3 x ) 1 +  
∀ x ∈ R. x → ∞ f ( x)  3x 
Hence, the continuity of f depends upon   3x  n 
the continuity of [ x] , which is By Sandwich theorem,    + 1
π   π   π
discontinuous, ∀ x ∈ I. lim
f (2 x )
=1 Also, f ( x ) = lim = ,
So, we should check the continuity of f x→ ∞ f ( x)
n→ ∞ 3
  3 x  n −1  3
at x = n, ∀ n ∈ I   π  + 1
 
LHL at x = n is given by 11 We have, f ( x ) = max{tan x,sin x,cos x},
π 3π if x < π / 3
f(n− ) = lim− f ( x ) where x ∈  − ,  π π
x→ n
 2 2  Note that f   =
= lim− [ x] cos π  x − 
1
 3 3
x→ n  2 Let us draw the graph of y = tan x,
(2n − 1) π  x, if x ≥ π
π 3π
= (n − 1) cos =0 y = sin x and y = cos x in  − ,  ∴

f ( x) =  π 3
2  2 2  , if π
RHL at x = n is given by 3 x <
 3
f (n + ) = lim+ f ( x ) y = tanx y = tanx
x→ n From the given options, it is clear that
= lim+ [ x] cos π  x − 
1 y = f(x) option (c ) is incorrect.
x→ n  2 y = f(x)
y = f(x) x
(2n − 1) π 14 Let ak +1 =
= (n ) cos =0 (kx + 1){(k + 1)x + 1}
2 y = cos(x)
=  
1 1
-π O a π π
X − 
Also, value of the function at x = n is 2 2
3π  kx + 1 ( k + 1 )x + 1 
2
f (n ) = [n] cos π  n − 
1
∴ f ( x ) = lim ∑ 
n −1
–1 1

1 
 2 y = sinx 
k = 0  kx + 1 (k + 1)x + 1 
n→ ∞
(2n − 1)π
= (n ) cos =0
= lim 1 −
1 
2 n→ ∞ 
 nx + 1 
∴ f (n + ) = f (n − ) = f (n ) From the graph, it clear that f ( x ) is
1 , if x ≠ 0
Hence, f is continuous at π
non-differentiable at x = a, and π. = 
x = n, ∀ n ∈ I . 2  0 , if x = 0
Clearly, f ( x ) is neither continuous nor
9 The function u ( x ) = 1 is 12 We have,
differentiable at x = 0.
x−1 f ( x ) =| x − π|(
⋅ e − 1)sin| x|
|x|

−x
discontinuous at the point x = 1.  ( x − π )(e − 1)sin x, x < 0 15 Clearly, x n + xa + b = ( x − x1 )

The function y = f (u ) f ( x ) =  −( x − π )(e x − 1)sin x, 0 ≤ x < π ( x − x2 )...( x − x n )
= 2
1  ( x − π )(e x − 1)sin x, x ≥ π x n + xa + b
 ⇒ = ( x − x2 )
u + u−2 x − x1
We check the differentiability at x = 0
1 ( x − x3 )...( x − x n )
= and π.
(u + 2) (u − 1 ) x n + ax + b
We have, ⇒ lim
is discontinuous at u = − 2 and u = 1. f ′( x ) = x→ x 1 x − x1
When u = −2 −x
 ( x − π )(e − 1)cos x + (e − 1)sin x
x
= ( x1 − x2 )( x1 − x3 )...( x1 − x n )
1 
⇒ = −2 −x
+ ( x − π )sin xe (−1), x < 0 nx n −1 + a
x−1  ⇒ lim
 −[( x − π )(e − 1)cos x + (e − 1)sin x
x x x→ x 1 1
1
⇒ x =  = ( x1 − x2 )( x1 − x3 )...( x1 − x n )
2  + ( x − π )sin xe ], 0 < x < π
x

When u = 1  ( x − π )(e x − 1)cos x + (e x − 1)sin x [using L’Hospital rule]


1  ⇒ nx1n −1 + a = ( x1 − x2 )
⇒ =1  + ( x − π )sin xe x , x > π
x−1 ( x1 − x3 )...( x1 − x n ).
Clearly, lim− f ′( x ) = 0 = lim+ f ′( x )
⇒ x=2 x→ 0 x→ 0
116 TEN

 1   1, x >0 Hence, Statement I is correct.


 −b c  
 x  (v) f ( x) =  1, x <0 sin  1  , x ≠ 0

1 f2 ( x ) =   
16 Let L = lim x ln  0 −1   −1 x =0  x
x→ ∞
 x    0 , x=0

 1 0
a
  −1, x > 0

 x  Here, lim f2 ( x ) = lim sin  
(vi) f ( x ) =  −1, x < 0 1
 1, x = 0
x→ 0 x→ 0  x
= lim x ln  3 + b − 
a c

x→ ∞ x x which does not exist.
Clearly, for limit to be exist, b = 1. 19 f ( x ) = x | x | and g ( x ) = sin x So, f2 ( x ) is not continuous at x = 0.
 − sin x , x < 0
2
Thus, L = lim x ln  1 + 3 − 
a c Hence, Statement II is false.
gof ( x ) = sin ( x | x |) =  ∴
x→ ∞  x
x  sin x , x ≥ 0
2
22 ∴ f ( x ) = | x − 2 | + | x − 5|
= lim x  3 − 
a c  − 2 x cos x , x < 0
2

x→ ∞ x x (gof )′ ( x ) =   (2 − x ) + (5 − x ), x<2

 2 x cos x , x ≥ 0
2
= ( x − 2) + (5 − x ), 2≤ x ≤ 5
 x2 x3 x 4 
Q ln(1 + x ) = x − + − + ... Clearly, L ( gof )′ (0) = 0 = R ( gof )′ (0) ( x − 2) + ( x − 5), x> 5
  
 2 3 4 
So, gof is differentiable at x = 0 and also
= −c its derivative is continuous at x = 0. 7 − 2 x, x<2
L = −4 
=  3, 2≤ x≤ 5
Q Now,
∴ c =4 2 x − 7,
 − 2 cos x + 4 x sin x , x> 5
2 2 2
Hence, a ∈ R,b = 1 and c = 4. 
 x< 0

1 + x + f ( x )
1/x
( gof )′ ′ ( x ) =  Now, we can draw the graph of f very
17 We have, lim = e3  2 cos x2
− 4 x2
sin x2 ,
x→ 0  x  easily.
 x≥ 0
lim 1 + x +
f( x )  1
−1 
⇒ e x →0  x x
= e3 ∴ L ( gof )′ ′ (0) = − 2 and R ( gof )′′ (0) = 2 Y
 f( x ) 
lim 1+
 x2 
⇒ = e3 ∴ L ( gof )′′ (0) ≠ R ( gof )′′ (0)
x →0
e y = 7 – 2x y = 2x – 7
f ( x)
⇒ lim 2 = 2 Hence, gof ( x ) is not twice differentiable
x→ 0 x
at x = 0. y=3
Now, Therefore, Statement I is true, Statement
lim  1 + −1 
1/x f( x ) 1

lim 1 +
f ( x )   x II is false.
= e x →0  x
x→ 0  x 
 20 Statement I is correct as though X′ X
lim
f( x )
|4 x2 − 12 x + 5| is non-differentiable at 0 2 5
= e x →0 x = e 2
2

1 5
x = and but cos πx = 0 those points. Y′
18 We have, f 2 ( x ) = 1 ∀x ∈ R 2 2
So, f ′   and f ′   exists.
∴ f can take values +1 or −1 1 5 Statement I f ′ (4) = 0
Since f is discontinuous only at x = 0  2  2
It is obviously clear that, f is constant
∴ f may be one of the followings   1 around x = 4, hence f ′ (4) = 0. Hence,
 1, x ≤ 0 21 Here, f ( x ) =  x ⋅ sin  x  , x≠0
Statement I is correct.
(i) f ( x) = 
−1, x > 0  0 , x=0
Statement II It can be clearly seen that
To check continuity at x = 0,
 1, x < 0 (i) f is continuous, ∀ x ∈ [2, 5 ]
(ii) f ( x) =  LHL = lim (− h ) sin  −   = 0
1
−1, x ≥ 0 h→ 0
  h (ii) f is differentiable, ∀ x ∈ (2, 5)
−1, x ≤ 0 RHL = lim  h sin    = 0
1 (iii) f (2) = f (5) = 3
(iii) f ( x) = 
 h
 1, x > 0
h→ 0
 Hence, Statement II is also correct but
−1, x < 0 f (0) = 0 obviously not a correct explanation of
(iv) f ( x) =  So, f ( x ) is continuous at x = 0. Statement I.
 1, x ≥ 0
DAY TWELVE

Differentiation
Learning & Revision for the Day
u Derivative (Differential u Geometrical Meaning of u Second Order Derivative
Coefficient) Derivative at a point u Differentiation of a Determinant
u Methods of Differentiation

Derivative (Differential Coefficient)


The rate of change of a quantity y with respect to another quantity x is called the derivative or
differential coefficient of y with respect to x. The process of finding derivative of a function
called differentiation.

Geometrical Meaning of Derivative at a Point


Geometrically derivative of a function at a point x = c is the slope of the tangent to the curve
y = f ( x) at the point P {c, f (c)}.
f ( x) − f (c) df ( x) 
Slope of tangent at P = lim =  or f ′ (c).
x →c x −c  dx x = c

Derivative of Some Standard Functions



d
(constant) = 0 ●
d
x n = nx n −1 PRED
dx dx
d 1 d  1 n
MIRROR

( x) = ●
 n  = − n +1 Your Personal Preparation Indicator
dx 2 x dx x  x
u No. of Questions in Exercises (x)—
d d

(sin x) = cos x ●
(cos x) = − sin x u No. of Questions Attempted (y)—
dx dx u No. of Correct Questions (z)—
d d (Without referring Explanations)

(tan x) = sec2 x ●
(sec x) = sec x tan x
dx dx
d d u Accuracy Level (z / y × 100)—

(cot x) = − cosec2 x ●
(cosec x) = − cosec x cot x
dx dx u Prep Level (z / x × 100)—
d 1 d x

(log x) = , for x > 0 ●
(e ) = e x In order to expect good rank in JEE,
dx x dx your Accuracy Level should be
d x d 1 above 85 & Prep Level should be

(a ) = ax log a, for a > 0 ●
(log a x) = , for x > 0, a > 0, a ≠ 1 above 75.
dx dx x log a
118 TEN


d
(sin −1 x) =
1
, for −1 < x < 1 (x) If u = f ( x) and v = g( x), then the differentiation of u
dx 1 − x2  du 
 
d 1 du  dx 

(cos −1 x) = − , for −1 < x < 1 with respect to v is = .
dx 1 − x2 dv  dv 
 
 dx 
d 1

(sec −1 x) = , for| x| > 1 Differentiation of a function 
dx | x| x 2 − 1 w. r. t another function 
d 1  

(cosec −1 x) = − , for| x| > 1
dx | x| x 2 − 1

d
(tan −1 x) =
1
, for x ∈ R
Differentiation Using Substitution
dx 1 + x2 In order to find differential coefficients of complicated
d 1 expressions, some substitution are very helpful, which are

(cot −1 x) = − , for x ∈ R
dx 1 + x2 listed below

S. No. Function Substitution


Methods of Differentiation
dy d (i) 2
a –x 2
x = a sin θ or a cos θ
(i) If y = f ( x) ± g( x), then = { f ( x) ± g( x)}
dx dx (ii) x 2 – a2 x = a sec θ or a cosec θ
= f ′ ( x) ± g ′ ( x)
(iii) x 2 + a2 x = a tan θ or a cot θ
(ii) If y = c ⋅ f ( x), where c is any constant, then
dy d a+ x a– x
= (c ⋅ f ( x)) = c ⋅ f ′( x). [Scalar multiple rule] (iv) or x = a cos 2θ
dx dx a− x a+ x
dy d
(iii) If y = f ( x) ⋅ g( x), then = { f ( x) ⋅ g( x)} a2 + x2 a2 – x 2
dx dx (v) or x2 = a2 cos 2θ
a2 − x2 a2 + x 2
= f ( x) ⋅ g ′ ( x) + g ( x) ⋅ f ′ ( x) [Product rule]
f ( x) dy d  f ( x)  g( x) ⋅ f ′ ( x) − f ( x) ⋅ g′ ( x) x
(iv) If y = , then =  = (vi) x = a tan2 θ
g ( x) dx dx  g( x)  {g( x)}2 a+ x
g ( x) ≠ 0 (vii) ( x – a)( x – b ) x = a sec2 θ – b tan2 θ
(v) If y = f (u) and u = g( x), then
dy df du (viii) ax – x 2 x = a sin2 θ
= ⋅ = f ′(u) ⋅ g′( x) [Chain rule]
dx du dx x
(ix) x = a sin2 θ
This rule can be extended as follows. If y = f (u),u = g(v) a– x
dy df du dv
and v = h ( x), then = ⋅ ⋅ , (x) ( x – a) (b – x) x = a cos2 θ + b sin2 θ
dx du dv dx
d
(vi) ( f {g( x)}) = f ′ (g( x)) ⋅ g′( x) Usually this is done in case of inverse trigonometric functions.
dx
(vii) If given function cannot be expressed in the form
y = f ( x) but can be expressed in the form f ( x, y) = 0, then Second Order Derivative
to find derivatives of each term of f ( x, y) = 0 w.r.t x. d  dy 
If y = f ( x), then   is called the second order derivative
[differentiation of implicit function] dx  dx 
(viii) If y is the product or the quotient of a number of d2 y
of y w.r.t x. It is denoted by 2 or f ′′( x) or y′′ or y2 .
complicated functions or if it is of the form ( f ( x))g ( x ), dx
then the derivative of y can be found by first taking log
on both sides and then differentiating it.
[logarithmic differentiation rule]
Differentiation of a Determinant
dy dp dq dr
When y = ( f ( x))g ( x ), then = ( f ( x))g ( x ) p q r
dx dy dx dx dx
If y = u v w , then = u v w
 g ( x)  dx
l m n l m n
 f ( x) ⋅ f ′( x) + log f ( x) ⋅ g′( x)
 
p q r p q r
(ix) If x = φ (t ) and y = Ψ (t ), where t is parameter, then du dv dw
dy dy/dt + + u v w
= [Parametric differentiation rule] dx dx dx dl dm dn
dx dx/dt l m n
dx dx dx
DAY 119

DAY PRACTICE SESSION 1

FOUNDATION QUESTIONS EXERCISE


 3π  11 If y = (1 − x ) (1 + x 2 ) (1 + x 4 )...(1 + x 2 n ), then
dy
at x = 0
1 If f ( x ) = | cos x |, then f ′   is equal to
 4 j NCERT Exemplar dx
1 1
is equal to
(a) (b) 2 (c) (d) 2 2 1 x x
2 2 (a) −1 (b) (c) (d)
(1+ x)2 (1 + x 2 ) (1 − x)2
2 If f ( x ) = | x − 1 | and g ( x ) = f [f {f ( x )}], then for x > 2, g ′ ( x )
12 If f : ( −1, 1) → R be a differentiable function with f ( 0) = 1
is equal to
and f ′ ( 0) = 1. Let g ( x ) = [f ( 2f ( x ) + 2 )]2 . The, g′ ( 0) is
(a) − 1, if 2 ≤ x < 3 (b) 1, if 2 ≤ x < 3 equal to
(c) 1, if x > 2 (d) None of these
(a) 4 (b) −4 (c) 0 (d) −2
3 The derivative of y = (1 − x )( 2 − x )K (n − x ) at x = 1 is 13 Let f ( x ) be a polynomial function of second degree. If
(a) 0 (b) (−1)(n − 1)! (c) n! − 1 (d) (−1)n − 1 (n − 1)! f (1) = f ( −1) and a, b, c are in AP, then f ′ (a ), f ′ (b ) and f ′ (c )
4 If f ( x ) = x , then the value of
n are in.
f ′ (1) f ′′ (1) f ′ ′′ (1) ( −1)n f n (1) (a) AP
f (1) − + − + ...+ is
1! 2! 3! n! (b) GP
(c) Arithmetic-Geometric progression
(a) 2 n (b) 0 (c) 2 n − 1 (d) None of these
(d) None of the above
x −x 2
d −1
5 If f ( x ) = 2 , where x ≠ 0, − 2, then [f ( x )] 14 If y = f ( x ) is an odd differentiable function defined on
x + 2x dx
( − ∞, ∞ ) such that f ′ ( 3) = − 2, then f ′ ( − 3) is equal to
(whenever it is defined) is equal to j JEE Mains 2013
(a) 4 (b) 2 (c) −2 (d) 0
−1 3 1 −3
(a) (b) (c) (d)
(1 − x)2 (1 − x)2 (1 − x)2 (1 − x)2 15 If f and g are differentiable function satisfying
g ′ (a ) = 2, g (a ) = b and fog = I (identity function). Then,
6 If f ( x ) = 2+| x |−| x − 1|−| x + 1| , then f ′ (b ) is equal to
 1  1  3  5
f ′  −  + f ′   + f ′   + f ′   is equal to (a)
1
(b) 2 (c)
2
(d) None of these
 2  2  2  2 2 3
(a) 1 (b) −1 (c) 2 (d) −2 16 If y is an implicit function of x defined by
7 If f ( x ) = loge | x |, then f ′( x ) equals x 2 x − 2x x cot y − 1 = 0. Then, y ′ (1) is equal to
(a)
1
, where x ≠ 0
1
(b) for | x| > 1 (a) − 1 (b) 1 (c) log 2 (d) − log 2
| x| x dy
+n
1 17 If x m y n = ( x + y ) m , then is equal to
(c) − for | x| > 1 dx
x x+ y x y
1 1 (a) (b) xy (c) (d)
(d) for x > 0 and − for x < 0 xy y x
x x
 π d 2y dy
8 If f ( x ) = | cos x − sin x |, then f ′   is equal to 18 If y = ( x + 1 + x 2 )n , then (1 + x 2 ) 2
+x is equal to
 2 dx dx
(a) n 2 y (b) −n 2 y (c) −y (d) 2 x 2 y
(a) 1 (b) −1 (c) 0 (d) None of these
2t 1−t 2
dy
9 If f ( x ) = cos x ⋅ cos 2x ⋅ cos 4x ⋅ cos 8x ⋅ cos 16x , then 19 If x = and y = , then is equal to
 π 1+ t 2
1+ t 2
dx
f ′   is equal to
 4 (a)
2t
(b)
2t
1 3 t2 + 1 t2 − 1
(a) 2 (b) (c) 1 (d) 2t
2 2 (c) (d) None of these
1− t2
dy
10 If sin y = x sin(a + y ), then is equal to  π
dx 20 For a > 0, t ∈  0,  , let x = a sin−1 t and y = a cos −1 t .
sina sin (a + y)
2  2
(a) (b) 2
sin2 (a + y) sina  dy 
Then, 1 +   equals
sin2 (a − y)  dx  j
JEE Mains 2013
(c) sina sin2 (a + y) (d)
sina x 2
y 2
x + y 2 2
x2 + y2
(a) 2 (b) 2 (c) (d)
y x y2 x2
120 TEN

 x + 1  x − 1 dy  log(e / x 2 ) −1  3 + 2 log x  d 2y
21 If y = sec−1  −1
 + sin   , then dx is equal to 30 If y = tan−1   + tan   , then is
 x − 1  x + 1  log(ex 2 )   1 − 6 log x  dx 2
1 (a) 2 (b) 1
(a) 0 (b)
x +1 (c) 0 (d) −1
(c) 1 (d) None of these 31 If graph of y = f ( x ) is symmetrical about the Y-axis and
 1  6x x  that of y = g ( x ) is symmetrical about the origin and if
22 For x ∈  0,  , if the derivative of tan−1   is d 2h( x )
 4 1 − 9x 3  h( x ) = f ( x ) ⋅ g ( x ), then at x = 0 is
dx 2
x ⋅ g ( x ), then g ( x ) equals j JEE Mains 2017
(a) f (0)g (0)
9 3x x 3x 3
(a) (b) (c) (d) (b) 0
1 + 9x 3 1 − 9x 3 1 − 9x 3 1 + 9x 3 (c) can’t be determined
dy (d) None of the above
23 If y = sec(tan−1 x ), then at x = 1 is equal to
dx f ′( x ) f ( x )
j JEE Mains 2013 32 If = 0, where f ( x ) is continuously
1 1 f ′′( x ) f ′( x )
(a) (b) (c) 1 (d) 2
2 2 differentiable function with f ′( x ) ≠ 0 and satisfies f ( 0) = 1
 1+ x 2 − 1− x 2  f ( x ) −1
and f ′( 0) = 2, then lim is
24 If y = tan−1   , then dy is equal to x→ 0 x
 1+ x 2 + 1− x 2 
  dx
(a) 1 (b) 2
x x 2x 1
(a) (b) − (c) (d) None of these (c) (d) 0
1− x4 1− x2 1− x4 2
2
3x cos x sin x
dy
25 If 1 − x + 1 − y = a ( x − y ), then
2 2
is equal to 33 If f ( x ) = 6 −1 0 , where P is a constant.
dx
P P2 P3
1− x2 1− y2 x2 − 1 y2 − 1 d2
(a) (b) (c) (d)
1− y2 1− x2 1− y2 1− x2 Then, { f ( x )} at x = 0 is equal to
dx 2
dy (a) P (b) P + P 2
26 If y = sin−1( x 1 − x + x 1 − x 2 ), then is equal to
dx (c) P + P 3 (d) independent of P
−2 x 1 −1 1
(a) + (b) − 34 Which of the following statements is/are true?
1− x2 2 x − x2 1− x2 2 x − x2 Statement I If y = (log x )log x , then
1 1 dy  1 log (log x ) 
(c) + (d) None of these = (log x )log x + .
1− x2 2 x − x2 dx  x x 

a cos
−1
x
−1 dy Statement II If y = cos(a cos x + b sin x ) for some
27 If y = cos −1 x
and z = a cos x
, then is equal to constants a and b, then
1+ a dz
y ′ = (a sin x − b cos x ) sin(a cos x + b sin x )
1 1
(a) − −1
(b) −1 (a) Only I is true
1 + a cos x
1 + a cos x

1 (b) Only II is true


(c) −1
(d) None of these (c) Both I and II are true
(1 + a cos x 2
) (d) Neither I nor II is true
1
28 Let g ( x ) be the inverse of f ( x ) such that f ′( x ) = , 35 Statement I If u = f (tan x ), v = g (sec x ) and f ′ (1) = 2,
2
1+ x 5  du  1
d (g ( x )) g′ ( 2 ) = 4 , then   = .
then is equal to  dv  x = π / 4 2
dx 2
1 g ′(x) Statement II If u = f ( x ), v = g ( x ), then the derivative of f
(a) (b)
1 + (g (x))5 1 + (g (x))5 du du /dx
(c) 5 (g (x))4 (1 + (g (x))5 (d) 1 + (g (x))5 with respect to g is = .
dv dv /dx
d 2x
29 is equal to j
AIEEE 2011 (a) Statement I is true, Statement II is true; Statement II is a
dy 2 correct explanation for Statement I
−1 −3
 d2y   dy   d 2 y  dy −2 (b) Statement I is true, Statement II is true; Statement II is
(a) −  2    (b)  2    not a correct explanation for Statement I
 dx   dx   dx   dx 
−1 (c) Statement I is true; Statement II is false
 d 2 y  dy −3  d2y 
(c) −  2    (d)  2  (d)Statement I is false; Statement II is true
 dx   dx   dx 
DAY 121

DAY PRACTICE SESSION 2

PROGRESSIVE QUESTIONS EXERCISE


1 For x ∈R ,f ( x ) = | log 2 − sin x | and g ( x ) = f (f ( x )), then 9 If f ( 2) = 4, f ′ ( 2) = 3, f ′′ ( 2) = 1, then (f −1 )′′ (4) is equal to
(a) g is not differentiable at x = 0 −1 −1
(a) (b)
(b) g′ (0) = cos(log 2) 9 81
(c) g′ (0) = − cos (log 2) −1 −1
(c) (d)
(d) g is differentiable at x = 0 and g′ (0) = − sin(log 2) 27 3
2 If y = sin x ⋅ sin 2x ⋅ sin 3x ... sin nx , then y ′ is 10 If f ( x ) = sin (sin x ) and f ′ ′ ( x ) + tan x f ′ ( x ) + g ( x ) = 0, then
n n g ( x ) is equal to j JEE Mains 2013
(a) ∑ k ⋅ tankx (b) y ⋅ ∑ k cot kx
k =1 k =1
(a) cos2 x cos (sin x) (b) sin2 x cos (cos x)
n n (c) sin2 x sin(cos x) (d) cos2 x sin(sin x)
(c) y ⋅ ∑ k ⋅ tankx (d) ∑ cotkx
k =1 k =1 11 If x = a cos t cos 2 t and y = a sin t cos 2 t
3
3 If 3f ( x ) − 2f (1 /x ) = x , then f ′ ( 2) is equal to   dy   2
2

2 1 7 1 +   
 dx 
(a)
7
(b)
2
(c) 2 (d)
2 (where, a > 0), then   at π is given by
d 2y 6
4 If f ( x ) = (cos x + i sin x ) ⋅ (cos 2x + i sin 2x ) 2
dx
(cos 3x + i sin 3x ) ... (cos nx + i sin nx ) and f (1) = 1, then
f ′ ′ (1) is equal to
2 a
n (n + 1) n (n + 1) 
(b) 
(a) (b) a 2
(a) 3
2  2 
2 2a
n (n + 1) 
2 (c) (d)
(c) −  (d) None of these 3a 3
 2 

and g ( x + 1) = x + g ( x ) ∀ x ∈ R. If n ∈ I + ,
g (x )

y
12 Let f ( x ) = elne
tan−1  
5 If x 2 + y 2 = ae x
, a>0 assuming y > 0, then y′ ′ ( 0)  1  1
f ′ n +  f′  
 3  3
is equal to then − is equal to
 1   1
(a)
2 −π / 2
e (b) −
2 π /2
e f n +  f 
 3  3
a a
2  1 
(c) − e − π / 2 (d) None of these (a) 3  1 + + + K + 
1 1 1 1 1
(b) 3  1 + + + ... +
a 
 2 3 n  3 5 2n − 1
dy π
6 If y = | sin x || x| , then the value of at x = − is (c) n (d) 1
dx 6 1− x
−π π x2 x3
13 If the function f ( x ) = − 4e 2 +1+ x + + and
2 6 2 6
2 3
(a) [6log 2 − 3 π] (b) [6log 2 + 3 π]
6 6  −7 
π g ( x ) = f −1( x ), then the value of g′   is equal to
−  6
2 6
(c) [6log 2 + 3 π] (d) None of these 1 1
6 (a) (b) −
5 5
1
7 The solution set of f ′ ( x ) > g ′ ( x ), where f ( x ) = ( 5) 2 x + 1 (c)
6
(d) −
6
2 7 7
and g ( x ) = 5x + 4x loge 5 is
14. If f ( x ) = ( x − 1)100 ( x − 2)2 ( 99 ) ( x − 3)3 ( 98 ) ...( x − 100)100 ,
(a) (1, ∞) (b) (0, 1) (c) (∞, 0) (d) (0, ∞) f ′ (101)
then the value of is
8 Let f ′ ′ ( x ) = − f ( x ), where f ( x ) is a continuous double f (101)
differentiable function and g ( x ) = f ′ ( x ).
2 2
(a) 5050 (b) 2575
 x  x (c) 3030 (d) 1250
If F ( x ) = f    + g    and F ( 5) = 5, then F (10) is
  2    2  15 The derivative of the function represented parametrically
equal to as x = 2t − | t | , y = t 3 + t 2 | t | at t = 0 is
(a) 0 (b) 5 (a) −1 (b) 0
(c) 10 (d) 25 (c) 1 (d) does not exist.
122 TEN

ANSWERS

SESSION 1 1 (a) 2 (a) 3 (b) 4 (b) 5 (b) 6 (d) 7 (b) 8 (a) 9 (a) 10 (b)
11 (a) 12 (b) 13 (a) 14 (c) 15 (a) 16 (a) 17 (d) 18 (a) 19 (b) 20 (d)
21 (a) 22 (a) 23 (a) 24 (a) 25 (b) 26 (c) 27 (c) 28 (c) 29 (c) 30 (c)
31 (b) 32 (b) 33 (d) 34 (c) 35 (a)

SESSION 2 1 (b) 2 (b) 3 (b) 4 (c) 5 (c) 6 (a) 7 (d) 8 (b) 9 (c) 10 (d)
11 (d) 12 (c) 13 (a) 14 (a) 15 (b)

Hints and Explanations


SESSION 1 f ′ (1) f ′ ′ (1) f ′ ′ ′ (1)
Hence, f ′ −  + f ′  + f ′  + f ′ 
1 1 3 5
∴ f (1) − + −
 2  2  2  2
1 When π < x < π, cos x < 0, so that
1! 2! 3!
2 (− 1)n f n (1) = (−1) + 1 + (−1) + (−1) = −2
+ ... +
|cos x |= − cos x, n!
= C 0 − C 1 + C 2 − nC 3 +
n n n 7 We have, f ( x ) = log e | x|
i.e. f ( x ) = − cos x, f ′( x ) = sin x
3π  ... + (−1)n nC n  log(− x ) , x < −1
Hence, f ′  3π  = 1
 = sin   = (1 − 1)n = 0  − log(− x ) , −1 < x < 0
 4  4 2 ∴ f ( x) = 
5 Let y =
x2 − x  − log x , 0< x < 1
2 We have, f ( x ) = | x − 1| [Q x > 2]
x2 + 2 x  log x , x >1
f [ f ( x )] = f ( x − 1) = |( x − 1) − 1 | 2y + 1  1
⇒ x= ;x≠ 0 , x< −1
−y + 1  x
=| x − 2|  1
g ( x ) = f [ f { f ( x )}] = f ( x − 2) 2x + 1 − , − 1< x < 0
⇒ f −1 ( x ) = ⇒ f ′( x ) =  x
=|( x − 2) − 1 |= | x − 3 | −x + 1 1
x − 3, if x ≥ 3 − , 0< x < 1
=  d −1 (− x + 1 ) ⋅ 2 − (2 x + 1 ) (−1 )  x
 − x + 3, if 2 ≤ x < 3 ∴ [ f ( x )] =  1
dx (− x + 1 )2 , x>1
1, if x ≥ 3  x
∴ g ′ ( x ) =  3
= 1
 −1, if 2 ≤ x < 3 (− x + 1 )2 Clearly, f ′( x ) = for| x|> 1
x
dy
3 = − [(2 − x )(3 − x )K (n − x ) + 6 We have, f ( x ) = 2+| x|−| x − 1|−| x + 1| π
dx 8 When 0 < x < , cos x > sin x
4
(1 − x )(3 − x )K(n − x ) 2 − x + ( x − 1) + ( x + 1),
2 − ∴ cos x − sin x > 0
+ K + (1 − x )(2 − x )K (n − 1 − x )] x + ( x − 1) − ( x + 1),
∴ f ( x) =  π
Also, when < x < π,cos x < sin x
⇒   2 + x + ( x − 1) − ( x + 1),
dy
= − [(n − 1)! + 0 + K + 0] 4
 dx  x = 1 2 + x − ( x − 1) − ( x + 1), ∴ cos x − sin x < 0
= (−1)(n − 1)!  if x < −1 ∴ |cos x − sin x|= − (cos x − sin x ), when
 if − 1 ≤ x < 0 π
4 We have, f ( x ) = x n < x< π
 4
⇒ f (1) = 1 = nC 0  if 0 ≤ x < 1 ⇒ f ′ ( x ) = sin x + cos x
f ′ (1) n n  if x ≥ 1 π π π
= = C1 ⇒ f ′   = sin + cos = 1 + 0 = 1
1! 1!  x + 2, if x < −1  2 2 2
f ′ ′ (1) n (n − 1) n  − x, if − 1 ≤ x < 0
⇒ = = C2 = 2sin x ⋅ cos x ⋅ cos 2 x ⋅ cos 4 x
2! 2!
 x, if 0 ≤ x < 1 ⋅ cos 8 x ⋅ cos 16 x
f ′ ′ ′ (1) n (n − 1) (n − 2) n 2 − x, if x ≥ 1 9 f ( x) =
= = C3 2sin x
3! 3! if x < −1 sin 2 x cos 2 x cos 4 x ⋅ cos 8 x ⋅ cos 16 x
 1, =
M M  −1, if −1 ≤ x < 0 2 sin x
M M ⇒ f ′( x ) = 
 1, if 0 ≤ x < 1 = 5
sin 32 x
f n (1) n ! n  −1, x≥1 2 sin x
= = Cn if
n! n!
DAY 123

1
∴ f ′( x ) = ⋅ ⇒ f ′ (− x )(−1) = − f ′ ( x ) d2 y dy
(1 + x2 ) + ⋅x
32 ⇒ f ′(− x ) = f ′( x ∴
) ⇒ dx 2
dx
32cos 32 x ⋅ sin x − cos x ⋅ sin 32 x f ′ (−3) = f ′ (3) = −2 1 + x2
sin2 x
15 Since, fog = I ⇒ fog ( x ) = x for all x n2 ( x + 1 + x2 )n
1 1 =
32 × − ×0 ⇒ f ′ ( g ( x )) g ′ ( x ) = 1 for all x
 π 2 2 1 + x2
⇒ f ′  = = 2 1 1
 4  1 
2
⇒ f ′(g (a)) = = d y2
dy
32 g ′ (a) 2 ⇒ (1 + x2 ) + x
 2  dx2 dx
1
⇒ f ′( b ) = [Q g (a) = b] = n2 ( x + 1 + x2 )n
10 ∴ sin y = x sin( a + y ) 2
2
sin y d y dy
⇒ x= 16 x2 x − 2 x x cot y − 1 = 0 …(i) ⇒ (1 + x2 ) + x = n2 y
sin(a + y ) dx2 dx
Now, x = 1,
On differentiating w.r.t. y, we get 1 − 2 cot y − 1 = 0 2t 1 − t2
dx sin(a + y )cos y − sin y cos( a + y )
19 We have, x = ,y =
= ⇒ cot y = 0 1+ t 2
1 + t2
dy sin2 (a + y ) π
⇒ y = Put t = tan θ
dx sin a 2
⇒ = 2 tan θ
dy sin2 (a + y ) On differentiating Eq. (i) w.r.t. x, we get ∴ x= = sin 2θ and
1 + tan2 θ
dy sin2 (a + y ) dy
⇒ = 2 x2 x (1 + log x ) − 2 [ x x (− cosec2 y ) 1 − tan2 θ
dx sin a dx y = = cos 2θ
+ cot y x x (1 + log x )] = 0 1 + tan2 θ
11 Given, y = (1 − x ) (1 + x2 ) π dy dy / dθ − 2sin 2θ
At  1,  , 2 (1 + log 1) ∴ = = = − tan 2θ
(1 + x 4 )...(1 + x2 n )  2 dx dx / dθ 2cos 2θ
(1 − x ) (1 + x2 )...(1 + x2 n )
2
  − 2 tan θ
or y =   =
− 2 1 (− 1)  
dy 1 − tan2 θ
(1 + x ) + 0 = 0
  dx   1 , π 

1 − ( x )4 n 
 2  − 2t 2t
= = = 2
(1 + x ) 1 − t2 t –1
⇒ 2 + 2  
dy
=0
(1 + x ) ⋅ (0 − 4n ⋅ x 4 n − 1 )  dx  1, π   
1 1
20 Q dx =  asin t × 
−1
 2 
− (1 − x 4 n ) ⋅ 1  1 − t 
dy −1
∴ =  dy  dt 2 a sin t

2

dx (1 + x )2 ∴   = −1
 dx  1, π   
 2  dy 1  acos t ×
−1 −1 
 dy  and =
∴ = −1 
 
 dx  x = 0 17 Given that, x m y n = ( x + y )m + n
dt 2 acos −1 t
 1 − t 2 
 acos t 
−1 −1

12 We have, f :(−1,1) → R Taking log on both sides, we get dy asin t


∴ =−  × 1
m log x + n log y = (m + n )log( x + y ) dx −1
a sin −1 t

f (0) = −1, f ′(0) = 1 acos t

On differentiating w.r.t. x, we get −1


g ( x ) = [ f (2 f ( x ) + 2)]2 m n dy (m + n )  dy  a cos t

+ = =−
⇒ g ′( x ) = 2[ f (2 f ( x ) + 2)] 1 +  sin −1
t
x y dx ( x + y )  dx  a
× f ′(2 f ( x ) + 2) × 2 f ′( x ) 2 −1
y2
∴ 1 +   = 1 +
dy  m + n n  m m + n dy acos t
⇒ g ′(0) = 2[ f {2 f (0) + 2}] ⇒  −  = − =1+ 2
dx  x + y y x x+ y  dx  asin −1
t x
× f ′{2 f (0) + 2} × 2 f ′(0)
= 2[ f (0)] × f ′( 0) × 2 f ′( 0) x2 + y 2
dy  my + ny − nx − ny  =
⇒   x2
= 2 × (−1) × 1 × 2 × 1 = −4 dx  y( x + y ) 
mx + my − mx − nx  x + 1  x − 1
13 Let f ( x ) = Ax2 + Bx + C 21 Q y = sec −1  −1
=  + sin  x + 1 
∴ f (1) = A + B + C x( x + y )  x − 1  
and f (−1) = A − B + C ∴
dy
=
y  x − 1 −1  x − 1 π
= cos −1   + sin  x + 1  = 2
dx x + 1
Q f (1) = f (−1) [given]  x   
⇒ A+ B+C = A−B+C 18 d ( y ) = n ( x + 1 + x2 )n − 1 ⇒
dy
= 0 Qsin −1 x + cos −1 x = 
π
dx  2 
⇒ 2B = 0 ⇒ B = 0 dx
 x 
∴ f ( x ) = Ax2 + C 1 +   
⇒ f ′( x ) = 2 Ax

 1 + x 2 22 Let y = tan −1  6 x x1 
 1 − 9x 
∴ f ′(a) = 2 Aa dy
 2 ⋅ (3 x3 /2 ) 
⇒ ( 1 + x2 ) = n ( x + 1 + x2 )n
f ′(b ) = 2 Ab and f ′(c ) = 2 Ac dx = tan −1  3 /2 2 
 1 − (3 x ) 
Also, a, b, c are in AP. d2 y dy  x 

−1
= 2 tan (3 x ) 3 /2
⇒ ( 1 + x2 ) +
So, 2 Aa, 2 Ab and 2 Ac are in AP. dx 2
dx  1 + x 
 2 Q2 tan −1 x = tan −1 2 x 
Hence, f ′(a), f ′(b )and f ′(c )are also in AP.  1 − x2 
 x 
14 Since, f ( x ) is odd. = n2 ( x + 1 + x2 )n − 1  1 + 

dy
= 2⋅
1 3 1 /2
⋅ 3 × ( x)
 + x2 
 1  dx 1 + (3 x3 /2 )2 2
∴ f (− x ) = − f ( x )
124 TEN

−1
9 acos x
= ⋅ x 27 y = , z = acos
−1
x
1 + 9 x3 1+ a cos −1 x f ′( x ) f ( x )
9 32 Since, =0
∴g ( x ) = z f ′′( x ) f ′( x )
1 + 9 x3 ⇒ y =
1+ z ∴ ( f ′( x ))2 − f ′′( x ) ⋅ f ( x ) = 0
−1
23 Given, y = sec (tan x ) dy (1 + z )1 − z (1)
⇒ = ( f ′( x ))2 − f ′′( x ) ⋅ f ( x )
Let tan −1 x = θ dz (1 + z ) 2 ⇒ =0
( f ′( x ))2
⇒ x = tanθ 1
= d  f ( x) 
∴ y = secθ = 1 + x2 (1 + z ) 2 ⇒ =0
On differentiating both sides w.r.t. x, we 1 dx  f ′( x )
=
get cos −1 x 2
(1 + a ) ⇒
f ( x)
= c , (constant)
dy 1 f ′( x )
= ⋅ 2x
dx 2 1 + x2 28 Since g ( x ) is the inverse of f ( x )
On putting x = 0, we get
At x = 1 ∴ f (g ( x )) = x
1
dy 1 ⇒ f ′(g ( x )) ⋅ g ′( x ) = 1 =c
= 2
dx 2 1
⇒ g ′( x ) = = 1 + (g ( x ))5 f ( x) 1
f ′(g ( x )) ⇒ =
 1 + x2 − 1 − x2 
24 Given, y = tan  −1 f ′( x ) 2
. ⇒ g ′′( x ) = 5 (g ( x ))4 ⋅ g ′ ( x ).
 1 + x + 1 − x 
2 2
= 5 (g ( x ))4 (1 + (g ( x ))5 ) f ′( x )
⇒ =2
Put x = cos 2θ
2 −1 f ( x)
29 Since, dx =  
dy
 cos θ − sin θ   dx  d
∴ y = tan −1  dy ⇒ (log f ( x )) = 2

 cos θ + sin θ  −2 2 dx
= −  
d2 x dy d y dx
⇒ ⋅ ⇒ log( f ( x )) = 2 x + k
 1 − tan θ 
−1 dy 2  dx  dx2 dy
= tan   On putting x = 0, we get 0 = k
 1 + tan θ   d 2 y  dy
−3
= −  2    ⇒ log( f ( x )) = 2 x
 π
= tan −1 tan  − θ
  dx   dx 
 4   ⇒ f ( x) = e2 x
 log(e / x2 ) f ( x) − 1
π π 1 30 Given, y = tan  −1
 e2 x − 1
= −θ= − cos −1 x2 Now, lim = lim ⋅ 2 = 2.
 log(ex ) 
2
x→ 0 x x → 0 2x
4 4 2
 3 + 2 log x 
dy x x + tan −1   33 f ′′( x )
∴ = 0+ =  1 − 6 log x 
dx 1 − x4 1 − x4
 log e − log x2  d2 d2 d2
∴ y = tan −1  (3 x2 ) (cos x ) (sin x )
2 
2 2
25 On putting x = sinθ and y = sin φ , we get dx dx dx2
 log e + log x  = 6 −1 0
Given equation becomes  3 + 2 log x 
+ tan −1  P P2 P3
cos θ + cos φ = a(sin θ − sin φ) 
 1 − 6 log x 
θ + φ  θ − φ
⇒ 2cos   cos    1 − 2 log x  6 − cos x − sin x
 2   2  = tan −1  
 1 + 2 log x  = 6 −1 0
θ + φ  θ − φ 
= a 2cos   sin   P P2 P3
  2   2   3 + 2 log x
+ tan −1  
θ−φ  1 − 6 log x 6 −1 0
⇒ = cot −1 a
2 −1
= tan (1) − tan (2 log x ) −1 ∴ f ′′(0) = 6 −1 0 = 0, which is
⇒ θ − φ = 2cot −1 a + tan −1 (3) + tan −1 (2 log x ) P P2 P3
⇒ sin −1 x − sin −1 y = 2cot −1 a = tan (1) + tan −1 (3)
−1
independent of P.
1 1 dy dy d2 y
⇒ − =0 Now, = 0 and 2 = 0 34 I. Let y = (log x ) log x
1 − x2 1 − y 2 dx dx dx
On taking log both sides, we get
dy 1 − y2 31 Since, y = f ( x ) is symmetrical about the log y = log ( log x ) log x
∴ =
dx 1 − x2 Y-axis ⇒ log y = log x log [log x]
∴ f ( x ) is an even function. [Q log m n = n log m]
26 On putting x = sin A and x = sin B Also, as y = g ( x ) is symmetrical about On differentiating both sides w.r.t.
y = sin −1 (sin A 1 − sin2 B the origin x, we get
∴ g ( x ) is an odd function. 1 dy
= (log x )
d
+ sin B 1 − sin2 A ) {log (log x )}
−1
Thus, h( x ) = f ( x ) ⋅ g ( x ) is an odd y dx dx
= sin (sin A cos B + sin B cos A ) function. d
= sin −1 [sin( A + B )] + log (log x ) log x
or h( x ) = − h(− x ) dx
= A + B = sin −1 x + sin −1 x
Now, h ′( x ) = h ′ (− x ) 1 1 1
dy 1 1 = (log x ) + log (log x )
⇒ = + and h "( x ) = − h ′′(− x ) log x x x
dx 1− x 2
2 x − x2 ⇒ h ′′(0) = − h ′′(0) 1
= {1 + log (log x )}
⇒ h ′′(0) = 0 x
DAY 125


dy y
= {1 + log (log x )} 3 3 f ( x ) − 2 f (1 / x ) = x …(i) When x = 0, we get from Eq. (i),
dx x y ′ = −1
Let 1 / x = y , then
(log x ) log x 2 −2 − π /2
= {1 + log (log x )} 3 f (1 / y ) − 2 f ( y ) = 1 /y ⇒ y ′′ (0) = = e
x − ae π /2 a
⇒ −2 f ( y ) + 3 f (1 / y ) = 1 /y
= (log x ) log x  +
1 log(log x )
⇒ −2 f ( x ) + 3 f (1 / x ) = 1 /x …(ii) 6 Given, y = |sin x || x |
 x x 
On multiplying Eq. (i) by 3 and Eq. (ii) In the neighbourhood of
II. Let y = cos (a cos x + b sin x ). by 2 and adding, we get π
On differentiating w.r.t. x, we get − , | x |and|sin x | both are negative
2 6
d 5 f ( x) = 3 x +
{cos(acos x + b sin x} x i.e. y = (− sin x )( − x )
dx
f ( x ) =  3 x + 
1 2 On taking log both sides, we get
= − sin(a cos x + b sin x ) ⇒
5 x log y = (− x ) ⋅ log (− sin x )
d
(acos x + b sin x ) 1 2 On differentiating both sides, we get
dx ⇒ f ′( x ) =3 − 
= (− x ) 
5 x2  1 dy 1 
= − sin(acos x + b sin x ) ⋅  ⋅ (− cos x )
y dx  − sin x 
f ′(2) =  3 −
[− asin x + b cos x] 1 2 1
⇒ =
5 4 2 + log(− sin x ) ⋅ (−1)
= (asin x − b cos x )
= − x cot x − log (− sin x )
sin (acos x + b sin x ) 4 f ( x ) = (cos x + i sin x ) = − [ x cot x + log ( − sin x )]
35 Given, u = f (tan x ) (cos 2 x + i sin 2 x )(cos 3 x + i sin 3 x ) ⇒
dy
= − y [ x cot x + log (− sin x )]
du … (cos nx + i sin nx ) dx
⇒ = f ′(tan x )sec2 x − π
dx = cos( x + 2 x + 3 x + ...+ nx ) + i sin
∴  
dy (2) 6
and v = g (sec x ) ( x + 2 x + 3 x + ...+ nx ) = [6 log 2 − 3 π]
 dx  at x=−
π 6

dv
= g ′(sec x )sec x tan x n (n + 1) n (n + 1)
= cos x + i sin x 6
dx 2 2
du (du / dx ) f ′(tan x ) 1 7 Since, f ′( x ) > g ′( x )
n (n + 1)
∴ = = ⋅ ⇒ f ′( x ) = 
⇒   52 x
dv (dv / dx ) g ′(sec x ) sin x 1 +1
 2  log e 5 × 2 >
 2
f ′(1)
∴  
du
= ⋅ 2  − sin n (n + 1) x + i cos n (n + 1) x 
 dv  x = π 4 g ′( 2 ) 5x log e 5 + 4 log e 5
 2 2  ⇒ 52 x ⋅ 5 > 5 x + 4
2 1 2
= ⋅ 2= n (n + 1)
4 2 f ′′ ( x ) = −  ⇒ 5⋅ 52 x – 5 x – 4 > 0
 2  ⇒ (5 – 1) (5⋅ 5 x + 4) > 0
x

cos n ( n + 1 ) n (n + 1)  ∴ 5x > 1
SESSION 2 
x + i sin x
 ⇒ x> 0
2 2
1 We have, f ( x ) = |log 2 − sin x| and d
{ f ′( x )} = − f ( x )
2
n (n + 1)
g ( x ) = f ( f ( x )), x ∈ R = − ⋅ f ( x) 8 Given,
 2  dx
Note that, for x→ 0, log 2 > sin x 2
⇒ g ′( x ) = − f ( x )
 n (n + 1)
∴ f ( x ) = log 2 − sin x ∴ f ′ ′ (1) = − f (1) [Q g ( x ) = f ′ ( x ), given]
⇒ g ( x ) = log 2 − sin( f ( x ))  2 
n (n + 1)
2 Also, given F ( x )
= log 2 − sin(log 2 − sin x )
= − 2 2
Clearly, g ( x ) is differentiable at x = 0 as  2  =  f  x   + g  x  
    
sin x is differentiable.   2   2
Now, 5 When x = 0, y > 0 ⇒ y = ae π /2
g ′ ( x ) = − cos (log 2 − sin x ) ( − cos x ) On taking log both sides of the given ⇒ F ′( x ) = 2  f  x f ′  x ⋅ 1
   
  2  2 2
= cos x.cos(log 2 − sin x ) equation, we get
⇒ g ′ (0) = 1 ⋅ cos (log 2) log ( x2 + y 2 ) = log a + tan −1  
1 y
+ 2  g    ⋅ g ′   ⋅ = 0
x x 1
2  x
2 We have,   2  2 2
y = sin x ⋅ sin 2 x ⋅ sin 3 x⋅....sin nx On differentiating both sides w.r.t. x, we
get Hence, f ( x ) is constant. Therefore,
∴ y ′ = cos x ⋅ sin 2 x ⋅ sin 3 x...⋅ sin nx F(10) = 5 .
1 2 x + 2 yy ′ 1 xy ′− y
× 2 = ×
+ sin x ⋅ (2cos 2 x ) sin 3x...sin nx 2 x + y2 2
x2
1 +  
y 9 Let y = f ( x ), then x = f −1 ( y ).
+ sin x ⋅ sin 2 x(3cos 3 x )...sin nx  x d2 x
+ ...+ sin x sin 2 x sin 3 x...(cos nx ) Now, = ( f −1 )′′ ( y )
⇒ x + yy ′ = xy ′− y …(i) d y2
(by product rule) Again, on differentiating both sides −1
dx  dy 
⇒ y ′ = cot x ⋅ y + 2 ⋅ cot 2 x ⋅ y w.r.t. x, we get Q = 
1 + ( y ′ )2 + yy ′′ = xy ′ ′+ y ′− y ′ dy  dx 
+3 ⋅ cot 3 x ⋅ y + ...+ n ⋅ cot nx ⋅ y
−1
⇒ y ′ = y [cot x + 2cot 2 x ⇒ 1 + ( y ′ )2 = ( x − y )y ′′ d2 x d d y 
∴ =  
+3cot 3 x + ...+ n cot nx] 1 + ( y ′ )2 d y2 dy  d x 
n ⇒ y ′′=
x− y
∑ k cot kx
−1
⇒ y′ = y ⋅ d  dy  dx
k =1 =   ⋅
dx  dx  dy
126 TEN

−2 3 /2
= −  
dy

d 2 y dx
⋅   dy  
2
d2 y 13 Since, g ( x ) = f −1 ( x )
 dx  ∴ 1 +   
dx2 dy  dx  dx2 ∴ f (g ( x )) = x ⇒ f ′ (g ( x )) ⋅ g ′ ( x ) = 1
 
−d 2 y 1
−3 ⇒ g ′( x ) =
2 = (1 + cot2 3 t )3 /2   cosec3 3t cos 2t f ′ (g ( x ))
= dx 3  a
 dy  −7
g ′   =
1
  
3 /2

 dy  
2
 dx  d2 y π  6   7 
⇒ 1 +    at t = is f ′g− 
Since, y = 4 when x = 2  dx  dx2 6   6 
 
d2 y 1
a
=
2a =
dx2 x = 2 −1 π  −1  − 7  
∴ ( f −1 )′′ (4) = − = 3 cos
3 f ′ f  
3
27   6 
 dy  3
   Q f (1) = − 4 + 1 + 1 + 1 + 1 = − 7
 dx  12 Clearly, f ( x ) = e g ( x )
x =2  2 3 6
Now, as g ( x + 1) = x + g ( x ) 
∴ f −1   = 1
10 f ( x ) = sin (sin x ) 7
∴ e g (x + 1 ) = e x + g (x ) = e x ⋅ e g (x )  6 
⇒ f ′ ( x ) = cos x ⋅ cos (sin x ) ⇒ f ( x + 1) = e x f ( x ) 1
⇒ f ′ ′ ( x ) = − sin x ⋅ cos (sin x ) =
On taking log both sides, we get 5
− cos 2 x ⋅ sin(sin x ) ln f ( x + 1) = ln (e x ⋅ f ( x ))  1−x

2  − 1  + 1 + x + x2
Now, g ( x ) = − [ f ′ ′ ( x ) + f ′ ( x ) ⋅ tan x]

1
. f ′ ( x + 1) Q f ′ ( x ) = − 4e 
 2
 
= sin x ⋅ cos (sin x ) + cos 2 x ⋅sin(sin x ) f ( x + 1)  
− tan x ⋅ cos x ⋅ cos (sin x )
= sin x ⋅ cos (sin x ) + cos2 x ⋅ sin(sin x ) =1+
1
. f ′( x ) 14 We have, f ( x ) = 100 ( x − i )i (101 − i )
f ( x) 11
− sin x ⋅ cos (sin x ) i =1

= cos 2 x ⋅ sin (sin x ) f ′ ( x + 1) f ′ ( x ) 100


⇒ − =1 ⇒ log f ( x ) = Σ i (101 − i ) log ( x − i )
f ( x + 1) f ( x) i =1
11 We have, 1 100
1
dx  cos t ⋅ sin 2t  f '  1 +  f '  
1 1 ⋅ f '( x ) = ∑ i (101 − i )⋅
= a  − sin t cos 2t −   3  3 f ( x) i =1 x−i
dt  cos 2t  ⇒ − =1
f ′ (101) 100 (101 − i )
f  1 +  f  
1 1 ⇒ = Σ i
−asin 3 t  3  3 f (101) i = 1 (101 − i )
=
cos 2 t 100 100(101)
f ′  2 +
1
f ′  1 +
1 = Σ i= = 5050
  i =1 2
dy  sin t ⋅ sin 2t   3  3
and = a cos t cos 2t −  − =1
dt  cos 2t  
f 2 +
1 
f 1 +
1 15 Given, x = 2t − |t|and y = t 3 + t 2 |t|
 
 3  3 Clearly,x = t , y = 2t 3 when t ≥ 0
acos 3 t 1
f ′  n +
1
=
cos 2 t  f ′  (n − 1) +  and x = 3t , y = 0 when t < 0
 3  3 On eliminating the parameter t, we get
− =1
dy dy / dt  1  1 2 x3 , when x ≥ 0
∴ = = − cot3t f n +  f  (n − 1) +  y =
dx dx / dt  3  3
2  0, when x < 0
d y dt
⇒ = 3cosec 2 3t ⋅ on adding columnwise, we get dy  6 x2 , when x > 0
dx2 dx Now, =
f ′  n +  f ′  
1 1 dx  0, when x < 0
−3cosec2 3t ⋅ cos 2t 
= 3  3
asin 3t − =n Q (LHD) at x = 0 = (RHD) at x = 0 = 0
f  n +  f  
1 1
∴ Its derivative at x = 0
 3
= −   cosec3 3t ⋅ cos 2t  3  3
 a (i.e. at t = 0 is 0)
DAY THIRTEEN

Applications of
Derivatives
Learning & Revision for the Day
u Derivatives as the Rate of u Tangent and Normal to a Curve u Rolle’s Theorem
Change u Angle of Intersection of Two u Lagrange’s Mean Value
u Increasing and Decreasing Curves Theorem
Function

Derivatives as the Rate of Change


dy
is nothing but the rate of change of y, relative to x. If a variable quantity y is some
dx
function of time t i.e. y = f (t ), then small change in time ∆t have a corresponding change ∆y
∆y
in y. Thus, the average rate of change =
∆t
When limit ∆t → 0 is applied, the rate of change becomes instantaneous and we get the rate
of change with respect to at the instant t , i.e.
∆y dy
lim =
∆t → 0 ∆ t dt
dy
is positive if y increases as t increase and it is negative if y decrease as t increase.
PRED
dt MIRROR
Your Personal Preparation Indicator
Increasing and Decreasing Function u No. of Questions in Exercises (x)—

A function f is said to be an increasing function in ]a, b [, if x1 < x2 ⇒ f ( x1 ) ≤ f ( x2 ), u No. of Questions Attempted (y)—
∀ x1 , x2 ∈] a, b [. u No. of Correct Questions (z)—

A function f is said to be a decreasing function in ] a, b [, if x1 < x2 ⇒ f ( x1 ) ≥ f ( x2 ), (Without referring Explanations)
∀ x1 , x2 ∈] a, b [.
u Accuracy Level (z / y × 100)—

f ( x) is known as increasing, if f ′ ( x) ≥ 0 and decreasing, if f ′ ( x) ≤ 0.
u Prep Level (z / x × 100)—

f ( x) is known as strictly increasing, if f ′ ( x) > 0 and strictly decreasing, if f ′ ( x) < 0.

Let f ( x) be a function that is continuous in [a, b ] and differentiable in (a, b ). Then, In order to expect good rank in JEE,
your Accuracy Level should be
(i) f ( x) is an increasing function in [a, b ], if f ′( x) > 0 in (a, b ). above 85 & Prep Level should be
above 75.
(ii) f ( x) is strictly increasing function in (a, b ), if f ′( x) > 0 in (a, b ).
128

(iii) f ( x) is a decreasing function in [a, b ], if f ′( x) < 0 in (a, b ). 1


(b) Equation of normal is y − y1 = − ( x − x1 )
(iv) f ( x) is a strictly decreasing function in [a, b ], if f ′( x) < 0  dy 
 
in (a, b ). dx ( x 1, y1 )

2
  dy  
Monotonic Function (c) Length of normal is PB = y1 sec ψ = y1 1 +    
  dx  ( x 1 , y1 )
A function f is said to be monotonic in an interval, if it is
either increasing or decreasing in that interval.
 dy 
(d) Length of subnormal is BC = y1 tan ψ = y1  
Results on Monotonic Function  dx  ( x
1, y1 )

(i) If f ( x) is a strictly increasing function on an interval


[a, b ], then f −1 exists and it is also a strictly increasing Angle of Intersection of Two Curves
function. The angle of intersection of two curves is defined to be the angle
(ii) If f ( x) is strictly increasing function on an interval [a, b ] between their tangents, to the two curves at their point of
such that it is continuous, then f −1 is continuous on intersection.
[ f (a), f (b )]. The angle between the tangents of the two curves y = f1 ( x) and
(iii) If f ( x) is continuous on [a, b ] such that f ′ (c) ≥ 0 [ f ′ (c) > 0] y = f2 ( x) is given by
for each c ∈(a, b ), then f ( x) is monotonically increasing  dy   dy 
  − 
on [a, b ]. dx I ( x 1 , y1 )  dx  II ( x 1 , y1 ) m1 − m2
tan φ = or
(iv) If f ( x) is continuous on [a, b ] such that f ′ (c) ≤ 0 ( f ′ (c) < 0)  dy   dy  1 + m1 m2
1+ 
for each c ∈[a, b ], then f ( x) is monotonically decreasing  dx  I ( x , y )  dx  II ( x , y )
1 1 1 1
function on [a, b ].
(v) Monotonic function have atmost one root.
Orthogonal Curves
Tangent and Normal to If the angle of intersection of two curves is a right angle, the two
curves are said to intersect orthogonally and the curves are
a Curve called orthogonal curves.
(i) If a tangent is drawn to Y π  dy   dy 
Normal Tangent If φ = , m1 m2 = − 1 ⇒     = − 1
the curve y = f ( x) at a 2  dx  I  dx  II
point P( x1 , y1 ) and this P (x1, y1)
tangent makes an angle NOTE • Two curves touch each other, if m1 = m2 .
ψ with positive ψ
ψ
X -direction, then
O A C B
X Rolle’s Theorem
(a) The slope of the Let f be a real-valued function defined in the closed interval
tangent is graph desenber tangents
[a, b ], such that
 dy  and normal
(i) f ( x) is continuous in the closed interval [a, b ].
  = tan ψ
dx  ( x , y )
1 1
(ii) f ( x) is differentiable in the open interval (a, b ).
(iii) f (a) = f (b ), then there is some point c in the open interval
 dy  (a, b ), such that f ′ (c) = 0.
(b) Equation of tangent is y − y1 =   ( x − x1 )
 dx  ( x
1, y1 )
Geometrically,
(c) Length of tangent is Y Y
2
  dy  
1 +   
  dx  ( x 1 , y1 
PA = y1 cosec ψ = y1
 dy  f(c)
 
dx ( x 1 , y1 ) X X
O a c b O a b

y1 graph of a differentiable function, satisfying the hypothesis of


(d) Length of subtangent AC = y1 cot ψ =
(dy / dx)( x 1 , y1 ) Rolle’s theorem.
There is atleast one point c between a and b, such that the
(ii) The normal to a curve at a point P( x1 , y1 ) is a line tangent to the graph at (c, f (c)) is parallel to the X -axis.
perpendicular to tangent at P and passing through P,
then Algebraic Interpretation of Rolle’s Theorem
1
(a) The slope of the normal is − Between any two roots of a polynomial f ( x), there is always a
(dy / dx)( x 1 , y1 ) root of its derivative f ′ ( x).
129

f (b ) − f (a)
Lagrange’s Mean Value Theorem f ′ (c) =
b −a
.
Let f be a real function, continuous on the closed interval [a, b ]
Geometrically For any chord of the curve y = f ( x), there is a
and differentiable in the open interval (a, b ). Then, there is
atleast one point c in the open interval (a, b ), such that point on the graph, where the tangent is parallel to this chord.
Y Remarks In the particular case, where f (a) = f (b ).
(b, f(b))
f (b ) − f (a)
The expression becomes zero.
(a, f(a)) (c, f(c))
b −a
Thus, when f (a) = f (b ), f ′ (c) = 0 for some c in (a, b ).
X Thus, Rolle’s theorem becomes a particular case of the mean
O a c b
value theorem.
graph of a continuous functions explain Lagrange’s mean
value theorem.

DAY PRACTICE SESSION 1

FOUNDATION QUESTIONS EXERCISE


1 If the volume of a sphere is increasing at a constant rate, 6 A spherical balloon is being inflated at the rate of
then the rate at which its radius is increasing, is 35 cc/min. The rate of increase in the surface area
(a) a constant (in cm 2 /min) of the balloon when its diameter is 14 cm, is
(b) proportional to the radius j JEE Mains 2013
(c) inversely proportional to the radius (a) 10 (b) 10 (c) 100 (d) 10 10
(d) inversely proportional to the surface area
7 Oil is leaking at the rate of 16 cm 3 /s from a vertically kept
2 Moving along the X -axis there are two points with cylindrical drum containing oil. If the radius of the drum is
x = 10 + 6 t , x = 3 + t 2 . The speed with which they are 7 cm and its height is 60 cm. Then, the rate at which the
reaching from each other at the time of encounter is (x is level of the oil is changing when oil level is 18 cm, is
in centimetre and t is in seconds) −16 −16 16 −16
(a) (b) (c) (d)
(a) 16 cm/s (b) 20 cm/s (c) 8 cm/s (d) 12 cm/s 49π 48 π 49π 47 π
3 An object is moving in the clockwise direction around the 8 Two men A and B start with velocities v at the same time
unit circle x 2 + y 2 = 1. As it passes through the point from the junction of two roads inclined at 45° to each
 1 3 other. If they travel by different roads, the rate at which
 ,  , its y-coordinate is decreasing at the rate of 3
2 2  they are being separated. j NCERT Exemplar

units per second. The rate at which the x-coordinate (a) 2− 2 ⋅v (b) 2+ 2 ⋅v
changes at this point is (in unit per second) (c) 2 − 1⋅ v (d) 2+ 2 ⋅v
(a) 2 (b) 3 3 (c) 3 (d) 2 3
9 The interval in which the function f ( x ) = x 1 / x is
4 The position of a point in time ‘ t ’ is given by
increasing, is
x = a + bt − ct 2 , y = at + bt 2 . Its acceleration at time ‘ t ’ is
(a) (−∞, e) (b) (e, ∞)
(a) b − c (b) b + c (c) 2 b − 2 c (d) 2 b 2 + c 2
(c) (−∞, ∞) (d) None of these
5 Water is dripping out from a conical funnel of x
π 10 The function f ( x ) = is
semi-vertical angle at the uniform rate of 2 cm 2 /s in the 1+ | x |
4
(a) strictly increasing
surface area, through a tiny hole at the vertex of the
(b) strictly decreasing
bottom. When the slant height of cone is 4 cm, the rate of
(c) neither increasing nor decreasing
decrease of the slant height of water, is j NCERT Exemplar (d) not differential at x = 0
2 1
(a) cm/s (b) cm/s 11 The length of the longest interval, in which the function
4π 4π
1 3 sin x − 4 sin3 x is increasing, is
(c) cm/s (d) None of these π π 3π
π 2 (a) (b) (c) (d) π
3 2 2
130

π 23 The tangent drawn at the point (0, 1) on the curve


12 An angle θ, 0 < θ < , which increases twice as fast as its
2 y = e 2 x , meets X -axis at the point
sine, is
(a)  , 0 (b)  − , 0
j NCERT Exemplar 1 1
(c) (2, 0) (d) (0, 0)
π 3π π π 2   2 
(a) (b) (c) (d)
2 2 4 3
24 If the normal to the curve y 2 = 5x − 1 at the point (1,−2) is
13 The value of x for which the polynomial of the form ax − 5y + b = 0 , then a and b are
2 x 3 − 9x 2 + 12 x + 4 is a decreasing function of x, is (a) 4 ,− 14 (b) 4 ,14 (c) −4 ,14 (d) 4 , 2
(a) −1 < x < 1 (b) 0 < x < 2
25 The curve y = ax + bx + cx + 5 touches the X -axis at
3 2
(c) x > 3 (d) 1 < x < 2
P( −2, 0) and cuts the Y -axis at a point Q, where its
1
14 If f ( x ) = − log(1 + x ), x > 0, then f is gradient is 3. Then,
x +1 1 3
(a) a = − , b = − and c = 3
(a) an increasing function 2 4
(b) a decreasing function 1 3
(b) a = , b = − and c = − 3
(c) both increasing and decreasing function 2 4
(d) None of the above 1 1
(c) a = , b = − and c = 3
2 4
15 If f ( x ) = sin x − cos x , the interval in which function is (d) None of the above
decreasing in 0 ≤ x ≤ 2 π, is
5π 3π π π 26 The product of the lengths of subtangent and subnormal
(a)  , (b)  ,  at any point of a curve is
 6 4   4 2 
3π 5π (a) square of the abscissa (b) square of the ordinate
(c)  , (d) None of these (c) constant (d) None of these
 2 2 
27 The tangent at (1, 7) to the curve x 2 = y − 6 touches the
16 If f ( x ) = − 2x 3 + 21x 2 − 60x + 41, then circle x 2 + y 2 + 16x + 12y + c = 0 at
(a) f (x) is decreasing in (−∞, 1) (a) (6, 7) (b) (−6, 7) (c) (6, − 7) (d) (−6, − 7)
(b) f (x) is decreasing in (−∞, 2)
(c) f (x) is increasing in (−∞, 1)
28 If the line ax + by + c = 0 is normal to curve xy + 5 = 0,
(d) f (x) is increasing in (−∞, 2) then
λ sin x + 6 cos x (a) a + b = 0 (b) a > 0 (c) a < 0, b < 0 (d) a = − 2b
17 Function f ( x ) = is monotonic increasing, if x
2 sin x + 3 cos x 29 The length of subnormal to the curve y = at the
(a) λ > 1 (b) λ < 1 (c) λ < 4 (d) λ > 4
1− x2
point having abscissa 2 is
18 The sum of intercepts on coordinate axes made by
tangent to the curve x + y = a , is (a) 5 2 (b) 3 3 (c) 3 (d) 3 2
(a) a (b) 2a 30 If m is the slope of a tangent to the curve e = 1 + x 2 , then
y

(c) 2 a (d) None of these (a) m ≤1 (b) m > − 1 (c) m >1 (d) m >1
19 Line joining the points (0, 3) and (5, −2) is a tangent to 31 If the curves y = a and y = b intersects at angle α , then
x x

ax tan α is equal to
the curve y = , then
1+ x (a)
a −b
(b)
loga − logb
(a) a = 1 ± 3 (b) a = φ 1 + ab 1 + loga logb
(c) a = −1 ± 3 (d) a = −2 ± 2 3 a+b loga + logb
(c) (d)
4 1 − ab 1 − loga logb
20 The equation of the tangent to the curve y = x + , that
x2 32 If the curves y 2 = 6x , 9x 2 + by 2 = 16 intersect each other
is parallel to the X-axis, is j AIEEE 2010 at right angles, then the value of b is
(a) y = 0 (b) y = 1 (c) y = 2 (d) y = 3 7
(a) 6 (b)
21 The slope of the tangent to the curve 2
9
x = 3 t 2 + 1, y = t 3 − 1, at x = 1 is (c) 4 (d)
2
1
(a) 0 (b) (c) ∞ (d) − 2
2 33 Angle between the tangents to the curve y = x 2 − 5x + 6
at the points (2, 0) and (3, 0) is
22 Coordinates of a point on the curve y = x log x at which
π π
the normal is parallel to the line 2 x − 2 y = 3, are (a) (b)
2 6
(a) (0, 0) (b) (e,e) π π
(c) (d)
(c) (e 2 , 2 e 2 ) (d) (e − 2 , − 2 e − 2 ) 4 3
131

x2 y2 38 A value of C for which the conclusion of mean value


34 If the curves + = 1 and y 3 = 16x intersect at right
α 4 theorem holds for the function f ( x ) = loge x on the interval
angles, then the value of α is j JEE Mains 2013
[1, 3] is j AIEEE 2007

4 1 3 1
(a) 2 (b) (c) (d) (a) 2 log3 e (b) loge 3
3 2 4 2
35 f ( x ) satisfies the conditions of Rolle’s theorem in [1, 2] (c) log3 e (d) loge 3
2
and f ( x ) is continuous in [1, 2], then ∫ f ′ ( x ) dx is equal to 39 The abscissa of the points of the curve y = x 3 in the
1
interval [ −2, 2], where the slope of the tangents can be
(a) 3 (b) 0 (c) 1 (d) 2
obtained by mean value theorem for the interval [ −2, 2],
36 If the function f ( x ) = x 3 − 6x 2 + ax + b satisfies Rolle’s are
 2 3 + 1 2
theorem in the interval [1, 3] and f ′   = 0, then (a) ± (b) + 3
 3  3
(a) a = −11 (b) a = − 6 (c) a = 6 (d) a = 11 (c) ±
3
(d) 0
37 If f ( x ) satisfies the conditions for Rolle’s theorem in [3, 5], 2
5
then ∫ f ( x ) dx is equal to 40 In the mean value theorem, f (b ) − f (a ) = (b − a )f ′ (c ), if
3
4 a = 4 , b = 9 and f ( x ) = x , then the value of c is
(a) 2 (b) −1 (c) 0 (d) −
3 (a) 8.00 (b) 5.25 (c) 4.00 (d) 6.25

DAY PRACTICE SESSION 2

PROGRESSIVE QUESTIONS EXERCISE


1 A kite is moving horizontally at a height of 151.5 m. If the 6 If f ( x ) satisfy all the conditions of mean value theorem in
speed of kite is 10 m/s, how fast is the string being let out, 1
[ 0 , 2 ]. If f ( 0) = 0 and | f ′ ( x )| ≤ for all x in [ 0, 2 ], then
when the kite is 250 m away from the boy who is flying the 2
kite? The height of boy is 1.5 m. j NCERT Exemplar
(a) f (x) < 2 (b) | f (x)| ≤ 1
(a) 8 m/s (b) 12 m/s (c) 16 m/s (d) 19 m/s (c) f (x) = 2 x (d) f (x) = 3 for atleast one x in [0, 2]
2 The normal to the curve y ( x − 2) ( x − 3) = x + 6 at the π
7 If f ′ (sin x ) < 0 and f ′ ′ (sin x ) > 0 , ∀x ∈  0 , 
point, where the curve intersects the Y -axis passes  
2
through the point and g ( x ) = f (sin x ) + f (cos x ), then g ( x ) is decreasing in
π π π π π π
(a)  − , − 
1 1
(b)  ,
1 1
 (a)  ,  (b)  0,  (c)  0,  (d)  , 
 2 2 2 2  4 2  4  2  6 2
(c)  , −  (d)  ,
1 1 1 1
 8 If f ( x ) = ( x − p )( x −q )( x −r ), where p < q < r , are real
2 3 2 3
numbers, then application of Rolle’s theorem on f leads to
3 The values of a for which the function (a) (p + q + r )(pq + qr + rp) = 3
(a + 2)x 3 − 3 ax 2 + 9 ax − 1 = 0 decreases monotonically (b) (p + q + r )2 = 3 (pq + qr + rp)
throughout for all real x, are (c) (p + q + r )2 > 3 (pq + qr + rp)
(a) a < − 2 (b) a > − 2 (d) (p + q + r )2 < 3 (pq + qr + rp)
(c) −3 < a < 0 (d) − ∞ < a ≤ − 3 9 If f ( x ) is a monotonic polynomial of 2m −1 degree, where
x x
4 If f ( x ) = and g ( x ) = , where 0 < x ≤ 1, then in m ∈ N, then the equation
sin x tan x [f ( x ) + f ( 3x ) + f ( 5x ) + ...+ f ( 2m − 1)x ] = 2m − 1 has
this interval
(a) atleast one real root (b) 2m roots
(a) both f (x) and g (x) are increasing functions
(c) exactly one real root (d) (2m + 1) roots
(b) both f (x) and g (x) are decreasing functions
(c) f (x) is an increasing function 10 A spherical balloon is filled with 4500 π cu m of helium
(d) g (x) is an increasing function gas. If a leak in the balloon causes the gas to escape at
x
5 f ( x ) = ∫ | log2 [log3 {log4 (cos t + a )}]| dt . If f ( x ) the rate of 72π cu m/min, then the rate (in m/min) at
0
which the radius of the balloon decreases 49 min after
is increasing for all real values of x, then the leakage began is j AIEEE 2012

(a) a ∈ (−11 ,) (b) a ∈ (15


, ) 9 7 2 9
(a) (b) (c) (d)
(c) a ∈ (1, ∞) (d) a ∈ (5 , ∞) 7 9 9 2
132

11 The normal to the curve x 2 + 2xy − 3y 2 = 0 at (1, 1) (a) increases with increase in (b − a)
(b) decreases with increase in (b − a)
(a) does not meet the curve again j JEE Mains 2015
(c) increases with decreases in (b − a)
(b) meets the curve again in the second quadrant (d) None of the above
(c) meets the curve again in the third quadrant
(d) meets the curve again in the fourth quadrant 15 The angle of intersection of curves,
12 If f and g are differentiable functions in ( 0, 1) satisfying y = [| sin x | + | cos x | ] and x 2 + y 2 = 5, where [⋅]
denotes greatest integral function is
f ( 0) = 2 = g (1), g ( 0) = 0 and f (1) = 6, then for some
π
(b) tan−1  
c ∈] 0, 1[ j JEE Mains 2014
1
(a)
4  2
(a) 2f ′ (c) = g ′ (c) (b) 2f ′ (c) = 3g ′ (c)
(c) f ′ (c) = g ′ (c) (d) f ′ (c) = 2g ′ (c) (c) tan−1 (2) (d) None of these

13 If y = f ( x ) is the equation of a parabola which is touched 16 In [0, 1], Lagrange’s mean value theorem is not
by the line y = x at the point where x = 1, then applicable to
(a) 2f ′ (0) = 3f ′ (1) (b) f ′ (1) = 1  1 − x, x<
1
 2  sin x , x ≠ 0
(c) f (0) + f ′ (1) + f ′ ′ (1) = 2 (d) 2f (0) = 1 + f ′ (0) (a) f (x) =  2 (b) f (x) = 
2  x
 − x  , x ≥
1 1  1,
14 Let a + b = 4, a < 2 and g ( x ) be a monotonically x=0
 2  2
increasing function of x. Then,
a b (c) f (x) = x | x | (d) f (x) = | x |
f ( x ) = ∫ g ( x ) dx + ∫ g ( x ) dx
0 0

ANSWERS
SESSION 1 1 (d) 2 (c) 3 (b) 4 (d) 5 (a) 6 (a) 7 (c) 8 (a) 9 (a) 10 (a)
11 (a) 12 (d) 13 (d) 14 (b) 15 (d) 16 (b) 17 (d) 18 (a) 19 (b) 20 (d)
21 (a) 22 (d) 23 (b) 24 (a) 25 (a) 26 (b) 27 (d) 28 (c) 29 (d) 30 (a)
31 (b) 32 (d) 33 (a) 34 (b) 35 (b) 36 (d) 37 (d) 38 (a) 39 (a) 40 (d)

SESSION 2 1 (a) 2 (b) 3 (d) 4 (c) 5 (d) 6 (b) 7 (b) 8 (c) 9 (a) 10 (c)
11 (d) 12 (d) 13 (b) 14 (a) 15 (c) 16 (a)

Hints and Explanations


SESSION 1 3 The equation of given circle is and point is y = at + bt 2 acceleration
in y direction
1 Given that, dV = k (say) x + y =1
2 2

dt d2 y
On differentiating w.r.t. t, we get = 2 = 2b
4 3 dV dR dy dt
Q V = πR ⇒ = 4 πR 2 2x
dx
+ 2y =0
3 dt dt dt dt ∴ Resultant acceleration
dR k 2 2
⇒ = dx dy
2
d x d y 
2

dt 4 πR 2 ⇒ x + y =0 =  2 + 2 
dt dt  dt   dt 
Rate of increasing radius is inversely
proportional to its surface area. 1
But we have, x = , y =
3 = (−2c )2 + (2b )2 = 2 b 2 + c 2
2 2
2 They will encounter, if 5 If S represents the surface area, then
10 + 6t = 3 + t 2 dy 1 dx 3
and = − 3, then + (− 3) = 0 dS
dt 2 dt 2 = 2 cm2 / s
⇒ t − 6t − 7 = 0 ⇒ t = 7
2
dt
At t = 7 s, moving in a first point dx
⇒ =3 3 r
d dt
v 1 = (10 + 6t ) = 6 cm/s
dt
At t = 7 s, moving in a second point
4 Given point is x = a + bt − ct 2 h l
d Acceleration in x direction and point π/4
v 2 = (3 + t 2 ) = 2t = 2 × 7 = 14 cm/s
dt d2 x
is y = at + bt 2 = 2 = − 2c
∴ Resultant velocity dt
= v 2 − v 1 = 14 − 6 = 8 cm/s
133

π π 2 From ∆LOM, 1 1
S = πrl = πl ⋅ sin l = l f ′( x ) = − −
4 2 OL2 + OM 2 − LM 2 ( x + 1)2 1 + x
cos 45° =
Therefore, 2 ⋅ OL ⋅ OM
⇒ f ′( x ) = − 
1 1 
+
dS 2 π dl dl 1 x2 + x2 − y 2 2 x2 − y 2  x + 1 ( x + 1)2 
= l⋅ = 2 πl ⋅ ⇒ = =
dt 2 dt dt 2 2⋅ x ⋅ x 2 x2 ⇒ f ′ ( x ) = − ve, when x > 0
dl 2 ⇒ 2x = 2x − y
2 2 2
∴ f ( x ) is a decreasing function.
when l = 4 cm, =
dt 2π ⋅ 4 ⇒ (2 − 2 ) x2 = y 2 15 Q f ( x ) = sin x − cos x
1 2
= = cm / s ∴ y = 2 − 2x On differentiating w.r.t. x, we get
2 2π 4π
On differentiating w.r.t. t, we get f ′ ( x ) = cos x + sin x
6 V = 4 πr 3 ⇒ dV = 4 π3r 2 dr dy
= 2− 2
dx = 2 
1
cos x +
1
sin x 
3 dt 3 dt dt dt  2 2 
dr dr 5
⇒ 35 = 4 π (7)2 ⇒ = Q dx = v  π π
dt dt 28 π = 2 − 2v   = 2  cos cos x + sin sin x 
 dt   4 4 
Surface area of balloon, S = 4 πr 2
Hence, they are being separated from  π 

dS
= 8πr
dr = 2 cos  x − 
dt dt each other at the rate 2 − 2 v.   4  
5 9 Given, f ( x ) = x1 / x For decreasing, f ′ ( x ) < 0
= 8π ×7× = 10 cm2 / min
28 π π  π 3π
1 <  x −  <
⇒ f ′( x ) = (1 − log x ) x1 / x 2  4 2
7 Let h be height of oil level at any instant x2
(within 0 ≤ x ≤ 2 π)
t and V be the volume of oil in f ′ ( x ) > 0, if 1 − log x > 0
π π  π π 3π π
cylindrical drum. ⇒ log x < 1 ⇒ x < e ⇒ + < x − +  < +
Given, h = 60 cm, r = 7 cm 2 4  4 4 2 4
∴ f ( x ) is increasing in the interval
dV (− ∞, e ). 3π 7π
and = − 16 cm3 /s ⇒ < x<
dt x 4 4
10 Given, f ( x) =
1 + | x| 16 Given,
| x| f ( x ) = − 2 x 3 + 21 x2 − 60 x + 41 …(i)
(1 + | x |) ⋅ 1 − x ⋅ On differentiating Eq. (i) w.r.t. x, we get
∴ f ′( x ) = x
(1 + | x |)2 f ′ ( x ) = − 6 x2 + 42 x − 60
H = − 6 ( x2 − 7 x + 10 )
1
= > 0, ∀ x ∈ R = − 6 ( x − 2) ( x − 5)
h (1 + | x |)2
If x < 2, f ′ ( x ) < 0 i.e. f ( x ) is decreasing.
⇒ f ( x ) is strictly increasing.
r 17 Q f ( x ) = λ sin x + 6cos x …(i)
11 Let f ( x ) = 3 sin x − 4 sin3 x = sin 3 x 2sin x + 3cos x
dV dh On differentiating w.r.t. x, we get
Q V = πr 2 h ⇒ = πr 2 Since, sin x is increasing in the interval
dt dt
− π , π  ⋅  (2sin x + 3cos x ) 
(since, r is constant all the time)  2 2   (λ cos x − 6sin x ) 
dh dh 16  − (λ sin x + 6cos x )
⇒ −16 = π(7)2 ⇒ =− π π π π
dt dt 49π ∴ − ≤ 3x ≤ ⇒ − ≤ x≤  
2 2 6 6  (2cos x − 3sin x ) 
dh
⇒   =−
16 f ′( x ) =
 dt  at h =18 49π Thus, length of interval (2sin x + 3cos x )2
π  π π The function is monotonic
So, height of oil is decreasing at the rate = − −  =
16 6  6 3 increasing, if f ′ ( x ) > 0
of cm/s.
49π
12 Given, 2 d (sinθ) = dθ ⇒
3λ (sin2 x + cos 2 x ) − 12
8 Let L and M be the positions of two men dt dt (sin2 x + cos 2 x )> 0
dθ dθ
A and B at any time t. ⇒ 2 × cos θ =
dt dt ⇒ 3λ − 12 > 0 (Q sin2 x + cos 2 x = 1)
Let OL = x and LM = y
1 π ⇒ λ> 4
Then, OM = x ⇒ 2cos θ = 1 ⇒ cos θ = ⇒θ =
dx dy 2 3
1 1 dy
Given, = v and we have to find 18 x + y = a ; (i ) + =0
dt dt 13 Let f ( x ) = 2 x3 − 9 x2 + 12 x + 4 2 x 2 y dx
M ⇒ f ′ ( x ) = 6 x2 − 18 x + 12 dy y
f ′ ( x ) < 0 for function to be decreasing ∴ =−
dx x
⇒ 6( x2 − 3 x + 2) < 0
y Hence, tangent at ( x, y ) is
⇒ ( x2 − 2 x − x + 2) < 0
x y
⇒ ( x − 2)( x − 1) < 0 ⇒ 1 < x < 2 Y − y =− ( X − x)
x
1 ⇒ X y + Y x = xy ( x + y)
14 Given curve is f ( x ) = − log(1 + x )
B 45° x+1
⇒ X y +Y x = xy a
O On differentiating w.r.t. x, we get
A x L
(using Eq. (i))
134

X Y dy dy
= 2e 2 x ⇒  
⇒ + =1 which is also a tangent to the given
= 2e 0 = 2
a x a y dx  dx  ( 0, 1 ) circle.
i.e. x2 + (2 x + 5)2 + 16 x
Clearly, its intercepts on the axes are Equation of tangent at (0, 1) with slope + 12 (2 x + 5) + c = 0
a ⋅ x and a ⋅ y . 2 is ⇒ (5x2 + 60 x + 85 + c = 0) must have
y − 1 = 2( x − 0) ⇒ y = 2 x + 1 equal roots.
Sum of intercepts = a( x + y)
Let the roots be α = β.
= a⋅ a = a This tangent meets X -axis. 60
∴ α +β=−
1 5
19 Equation of line joining the points (0, 3) ∴ y = 0 ⇒ 0 = 2x + 1 ⇒ x = −
2 ⇒ α =−6
and (5, −2) is y = 3 − x. If this line is ∴ x = − 6 and
tangent to y =
ax ∴ Coordinates of the point on X -axis is
, then y = 2x + 5 = − 7
( x + 1)  − 1 , 0 .
  28 Given curve, is xy = − 5 < 0
(3 − x ) ( x + 1) = ax should have equal  2 
roots. dy dy −y
24 We have, y 2 = 5x − 1 …(i) ⇒ x + y = 0⇒ = >0
Thus, (a − 2)2 + 12 = 0 ⇒ no value of a dx dx x
⇒ a ∈ φ. dy  5  −5 [as xy = − 5 < 0]
At (1, − 2), =  =
dx  2 y  (1 , −2 ) 4 −1 x
20 We have, y = x + 42 Slope of normal = = <0
dy y
x ∴ Equation of normal at the point (1, − 2)
On differentiating w.r.t. x, we get is dx
dy 8 −5 [as xy = −5 < 0]
= 1− 3 [ y − (− 2)]   + x − 1 = 0 Hence, slope of normal will be negative.
dx x  4
Since, the tangent is parallel to X -axis, The line ax + by + c = 0
∴ 4 x − 5y − 14 = 0 …(ii)
therefore ⇒ by = − ax − c
As the normal is of the form
dy −a c
= 0 ⇒ x3 = 8 ax − 5y + b = 0 ⇒ y = x−
dx b b
∴ x = 2 and y = 3
On comparing this with Eq. (ii), we get −a
Slope of normal is negative.
a = 4 and b = − 14 b
21 Given curve is x = 3t 2 + 1, y = t 3 − 1 −a a
25 Since, we have the curve ⇒ < 0⇒ > 0
For x = 1, 3t 2 + 1 = 1 ⇒ t = 0 y = ax3 + bx2 + cx + 5 touches X -axis at b b
dx dy ⇒ a > 0, b > 0 or a < 0, b < 0
∴ = 6t , = 3t 2 P(−2, 0), then X -axis is the tangent at
dt dt (−2, 0). The curve meets Y-axis in (0, 5).
 dy 
29 Given, y = x 2
dy 1− x
dy  dt  3t 2 t = 3ax2 + 2bx + c
Now, = = = dx At x = 2, y = − 2, therefore
dx  dx  6t 2 dy
 dt  ⇒   = 0 + 0 + c = 3 (given) point is ( 2, − 2 ).
 dx  0, 5
 dy  0 dy 1 + x2
∴   = =0 ⇒ c =3 …(i) ∴ =
 dx  ( t = 0 ) 2 dx (1 − x2 )2
dy
and   =0 dy 1+ 2
22 Given curve is y = x log x  dx  ( −2, 0 ) ⇒   = =3
 dx  ( 2, − 2 ) (1 − 2)2
⇒ 12a − 4b + c = 0 [from Eq. (i)]
On differentiating w.r.t. x, we get ∴ length of subnormal at ( 2, − 2 )
dy ⇒ 12a − 4b + 3 = 0 …(ii)
= 1 + log x and ( −2, 0) lies on the curve, then = |(− 2 )(3)| = 3 2
dx
0 = − 8a + 4b − 2c + 5
The slope of the normal 30 We have, e y = 1 + x2
1 −1 ⇒ 0 = − 8a + 4b − 1 (Q c = 3)
=− = ⇒ 8a − 4b + 1 = 0
dy
(dy /dx ) 1 + log x …(iii) ⇒ ey =2x
From Eqs. (ii) and (iii), we get dx
The slope of the given line 2 x − 2 y = 3 is 1 3 dy 2x
a = − ,b = − ⇒ =
1. 2 4 dx 1 + x2
Since, these lines are parallel.
26 Length of subtangent = y dx ⇒ m=
2x
,
−1 dy 1 + x2
∴ =1
1 + log x dy 2| x| 2| x|
and length of subnormal = y |m|= = ≤1
⇒ log x = − 2 dx |1 + x2 | 1 + | x|2
⇒ x = e −2 ∴ Product = y 2 Q | x|2 + 1 − 2| x|≥ 0 ≥
and y = − 2e − 2
⇒ Required product is the square of ⇒ (| x|− 1 )2 ≥ 0
∴ Coordinates of the point are
the ordinate. ⇒ | x|2 + 1 ≥ 2| x|
(e − 2 , − 2e − 2 ).
27 The tangent to the parabola 2 | x|
23 Given curve is y = e 2 x ⇒ 1≥
x2 = y − 6 at (1, 7) is 1 + | x|2
On differentiating w.r.t. x, we get 1
x(1) = ( y + 7) − 6 ⇒ y = 2 x + 5
2
135

31 Clearly, the point of intersection of Now, apply the condition of 40 f ( x) = x


curves is (0, 1). perpendicularity of two curves,
∴ f (a) = 4=2
Now, slope of tangent of first curve, i.e. m1 m2 = − 1
1
dy 4
and get α = with the help of equation f (b ) = 9 = 3; f ′ ( x ) =
m1 = = ax log a 3 x 2
dx
of curves. f (b ) − f (a) 3 − 2 1
dy Also, f ′ (c ) = = =
⇒   = m1 = log a 2 b −a 9− 4 5
 dx  ( 0 , 1 ) 35 ∫ 1
f ′ ( x )dx = [ f ( x )]21 = f (2) − f (1) = 0
1 1 25
∴ = ⇒C = = 6.25
Slope of tangent of second curve, [Q f ( x ) satisfies the 2 c 5 4
dy conditions of Rolle’s theorem]
m2 = = b x log b
dx ∴ f (2) = f (1) SESSION 2
dy
⇒ m2 =   = log b 36 Q f ( x )= x − 6 x + ax + b
3 2 1 Let AB be the position of boy who is
 dx  ( 0 , 1 ) flying the kite and C be the position of
m1 − m2 On differentiating w.r.t. x, we get the kite at any time t.
∴ tan α =
1 + m1 m2 f ′ ( x ) = 3 x2 − 12 x + a C
log a − log b By the definition of Rolle’s theorem
=
1 + log a log b

150 m
f ′ (c ) = 0 ⇒ f ′  2 +
1 
 =0 y
 3
32 We have, y 2 = 6 x

151.5 m
2

⇒ 3  2 +
dy 1   1  A E
⇒ 2y =6  − 12  2 +  + a= 0
dx  3  3

1.5 m
dy 3
⇒ 3  4 + +
⇒ = 1 4   1  1.5 m
dx y  − 12  2 +  + a= 0
 3 3  3
3
Slope of tangent at ( x1 , y 1 ) is m1 = ⇒ 12 + 1 + 4 3 − 24 − 4 3 + a = 0 B D
y1 x
⇒ a = 11
Also, 9 x + by = 16
2 2
Let BD = x and AC = y , then AE = x
⇒ 18 x + 2by
dy
=0
37 Since, f ( x ) satisfies all the conditions of Given, AB = 1.5 m, CD = 151.5 m
dx Rolle’s theorem in [3, 5].
dy −9 x ∴ CE = 150 m
⇒ = Let f ( x ) = ( x − 3)( x − 5) = x2 − 8 x + 15
dx
dx by 5 5 Given, = 10 m/s
−9 x1
Now, ∫ 3
f ( x )dx = ∫ 3
( x2 − 8 x + 15)dx dt
Slope of tangent at ( x1 , y 1 ) is m2 = dy
5
by 1 x
3
8 x2  Here, we have to find when
= − + 15x dt
Since, these are intersection at right  3 2 3
angle. y = 250 m
125
∴ m1 m2 = − 1 ⇒
27 x1
=1 =  − 100 + 75 − (9 − 36 + 45) Now, from ∆CAE, y 2 = x2 + 1502
by 12  3 
On differentiating, we get
27 x1 50 4
⇒ =1 [Q y 12 = 6 x1 ] = − 18 = − 2y
dy
= 2x
dx
6bx1 3 3 dt dt
9 38 Using mean value theorem,
⇒ b = ⇒
dy x dx
= ⋅
x
= ⋅ 10 …(i)
2 f (3) − f (1) dt y dt y
f ′ (c ) =
33 Q y = x − 5x + 6
2
3−1
dy  f (b ) − f (a) In ∆ACE, x = 2502 − 1502 (Q y = 250)
∴ = 2x − 5 Q f ′ (c ) =
dx  b − a  = 200 m
dy ∴ From Eq. (i), we get
Now, m1 =   = 4 − 5= −1 1 log e 3 − log e 1
 dx  (2 , 0 ) ⇒ = dy 200
c 2 = × 10 = 8 m/s
dy dt 250
and m2 =   = 6 − 5= 1 ∴ c =
2
= 2 log 3 e
 dx  (3, 0 ) log e 3 2 Given curve is
Now, m1 m2 = − 1 × 1 = − 1 y ( x − 2)( x − 3) = x + 6 …(i)
π
39 Given that, equation of curve Put x = 0 in Eq. (i), we get
Hence, angle between the tangents is . y = x = f ( x)
3

2 y(− 2) (− 3) = 6 ⇒ y = 1
So, f (2) = 8 and f (− 2) = − 8
2 2
f (2) − f (– 2) So, point of intersection is (0, 1).
34 Slope of the curve, x + y = 1 is Now, f ′( x ) = 3 x2 ⇒ f ′( x ) = x+ 6
α 4 2 − (− 2) Now, y =
−4 x ( x − 2)( x − 3)
m1 = . 8 − (− 8)
αy ⇒ = 3 x2 1 ( x − 2)( x − 3) − ( x + 6)
4 dy ( x − 3 + x − 2)
Now, slope of the curve, y = 16 x is
3
2 ⇒ =
16 ∴ x= ± dx ( x − 2)2 ( x − 3)2
m2 = . 3
3 y2
136

dy 6 + 30 36 dr
⇒  
From Eq. (i), | f ( x )| at the rate t = 49 min,
= = =1 Now, to find
 dx  ( 0, 1 ) 4 × 9 36 =
f (2)
x =
f (2) 1
| x| ≤ | x|
dt
2 2 2 dV
∴ Equation of normal at (0, 1) is given by we require the radius (r ) at that
−1 [from Eq. (ii)] dt
y −1= ( x − 0) ⇒ x + y − 1 = 0 dV
1 In interval [0, 2], for maximum x, stage. = − 72 π m3 / min
1 dt
1 1
which passes through the point  ,  . | f ( x )| ≤ ⋅ 2 ⇒ | f ( x )| ≤ 1 [Q x = 2] Also, amount of volume lost in 49 min
 2 2 2
= 72 π × 49 m3
3 Let f ( x ) = (a + 2) x3 − 3 ax2 + 9 ax − 1 7 g ′( x ) = f ′(sin x ) ⋅ cos x − f ′(cos x ) ⋅ sin x ∴ Final volume at the end of 49 min
= (4500 π − 3528 π ) m3
decreases ⇒ g ′ ′ ( x ) = − f ′ (sin x ) ⋅ sin x
monotonically for all x ∈ R, then = 972 π m3
+ cos2 x f ′ ′ (sin x ) If r is the radius at the end of 49 min,
f ′ ( x ) ≤ 0 for all x ∈ R
+ f ′ ′ (cos x ) ⋅ sin2 x − f ′ (cos x ) ⋅ cos x 4
⇒ 3(a + 2) x2 − 6 ax + 9 a ≤ 0 then πr 3 = 972 π ⇒ r 3 = 729
π
for all x ∈ R > 0, ∀ x ∈  0,  3
 2 ⇒ r =9
⇒ (a + 2) x2 − 2 ax + 3 a ≤ 0
π
for all x ∈ R ⇒ g ′( x ) is increasing in  0,  . Radius of the balloon at the end of 49
 2 min = 9 m
⇒ a + 2 < 0 and discriminant ≤ 0
π From Eq. (i),
⇒ a < − 2, − 8 a2 − 24a ≤ 0 Also, g ′   = 0
 4  dV 
⇒ a < − 2 and a (a + 3) ≥ 0  
π π dr dV / dt  dr   dt  t = 49
⇒ a < − 2, a ≤ − 3 or a ≥ 0 ⇒ a ≤ − 3 ⇒ g ′ ( x ) > 0, ∀ x ∈  ,  = ⇒  =
 4 2 dt 4 πr 2
 dt  t = 49 4π (r 2 ) t = 49
∴ − ∞ < a≤ − 3
π 72 π 2
sin x − x cos x and g ′ ( x ) < 0, ∀ x ∈  0,   dr 
  = = m/min
4 Now, f ′( x ) =  4  dt  t = 49 4 π(9 2 ) 9
sin2 x
π
=
cos x (tan x − x ) Thus, g ( x ) is decreasing in  0,  . 11 Given equation of curve is
sin2 x  4
x2 + 2 xy − 3 y 2 = 0
∴ f ′ ( x ) > 0 for 0 < x ≤ 1 8 We have, f ( x ) = ( x − p )( x − q )( x − r ) On differentiating w.r.t. x, we get
So, f ( x ) is an increasing function. 2 x + 2 xy ′ + 2 y − 6 yy ′ = 0
⇒ f ( p ) = 0 = f (q ) = f (r )
tan x − x sec2 x x+ y
Now, g ′ ( x ) = ⇒ p,q and r are three distinct real ⇒ y′ =
tan2 x roots of f ( x ) = 0 3y − x
sin x cos x − x sin 2 x − 2 x
= = dy
At x = 1, y = 1, y ′ = 1 i.e.  
So, by Roolle’s theorem, f '( x ) has one
sin2 x 2sin2 x =1
real root in the interval ( p,q ) and other  dx  (1 , 1 )
d in the interval (q, r ). Thus, f '( x ) has two
Again, (sin 2 x − 2 x ) = 2 cos 2 x − 2 Equation of normal at (1, 1) is
dx distinct real roots.
1
= 2 (cos 2 x − 1) < 0 Now, f ( x ) = ( x − p )( x − q )( x − r ) y − 1 = − ( x − 1) ⇒y − 1 = − ( x − 1)
1
So, sin 2 x − 2 x is decreasing. ⇒ f ( x ) = x3 − x2 ( p + q + r )
⇒ x+ y =2
⇒ sin 2 x − 2 x < 0 + x( pq + qr + rp ) − pqr
On solving Eqs. (i) and (ii)
∴ g ′ ( x) < 0 ⇒ f '( x ) = 3 x2 − 2( p + q + r )x
simultaneously, we get
So, g ( x ) is decreasing. + ( pq + qr + rp )
x2 + 2 x (2 − x ) − 3 (2 − x )2 = 0
As f '( x ) has distinct real roots
5 f ′( x ) = | log 2 [log 3 {log 4 (cos x + a)}] | ⇒ x2 + 4 x − 2 x2 − 3(4 + x2 − 4 x ) = 0
∴ 4( p + q + r )2 − 12( pq + qr + rp ) > 0
Clearly, f ( x ) is increasing for all values ⇒ − x2 + 4 x − 12 − 3 x2 + 12 x = 0
⇒ ( p + q + r )2 > 3( pq + qr + rp )
of x, if ⇒ − 4 x2 + 16 x − 12 = 0
9 Given that, f ( x ) is monotonic. ⇒ 4 x2 − 16 x + 12 = 0
log 2 [log 3 {log 4 (cos x + a)}] is defined
⇒ f ′( x ) = 0 or f ′( x ) > 0, ∀ x ∈ R ⇒ x2 − 4 x + 3 = 0
for all values of x.
⇒ f ′( px ) < 0 or f ′( px ) > 0, ∀ x ∈ R ⇒ ( x − 1) ( x − 3) = 0
⇒ log 3 [log 4 (cos x + a)] > 0, ∀ x ∈ R
So, f ′( px ) is also monotonic. ⇒ x = 1, 3
⇒ log 4 (cos x + a) > 1, ∀ x ∈ R
Hence, f ( x ) + f (3 x ) + ... + f [(2m − 1)x] Now, when x = 1, then y = 1
⇒ cos x + a > 4, ∀ x ∈ R
is a monotonic. and when x = 3, then y = − 1
∴ a> 5
Polynomial of odd degree (2m − 1), so it ∴ P = (1, 1) and Q = (3, − 1)
f (2) − f (0)
6 Since, = f ′( x ) will attain all real values only once. Hence, normal meets the curve again at
2− 0 (3, − 1) in fourth quadrant.
f (2) − 0 df ( x ) f (2)
10 Since, the balloon is spherical in shape,
⇒ = f ′( x ) ⇒ = Alternate Method
hence the volume of the balloon is
2 dx 2 4 Given, x2 + 2 xy − 3 y 2 = 0
f (2) V = πr 3. ⇒ ( x − y ) ( x + 3y ) = 0
⇒ f ( x) = x+C 3
2 On differentiating both the sides w.r.t. t, ⇒ x − y = 0 or x + 3 y = 0
Q f (0) = 0 ⇒ C = 0 we get Equation of normal at (1, 1)
f (2) dV 4 dr  y − 1 = − 1 ( x − 1) ⇒ x + y − 2 = 0
∴ f ( x) = x …(i) = π  3 r 2 × 
2 dt 3  dt  It intersects x + 3 y = 0 at (3, − 1) and
1 f (2) 1 hence normal meets the curve in fourth
Also, | f ′ ( x )|≤ ⇒ ≤ …(ii) ⇒
dr dV / dt
= …(i)
2 2 2 quadrant.
dt 4 πr 2
137

x+y=2 14 a + b = 4 ⇒ b = 4 − a x2 + 1 = 5 ⇒ x = ±2
Y y=x
and b − a = 4 − 2a = t (say) Now, P(2, 1) and Q(−2, 1)
x + 3y = 0
a b a On differentiating x2 + y 2 = 5 w.r.t. x,
Now, ∫ g ( x ) dx + ∫ g ( x ) dx = ∫ g( x) we get
0 0 0
4−a dy dy x
(1, 1) dx + ∫ 0
g ( x ) dx = I (a) 2x + 2y
dx
=0 ⇒
dx
=−
y
X′ X dI (a)
⇒ = g (a) − g (4 − a)  dy  = −2
O da  
 dx  (2, 1 )
As a < 2 and g ( x ) is increasing.
and  dy  =2
(3, –1) ⇒ 4 − a > a ⇒ g (a) − g (4 − a) < 0  
 dx  ( −2, 1 )
Y′ dI (a)
⇒ <0
da Clearly, the slope of line y = 1 is zero
12 Given, f (0) = 2 = g (1), g (0) = 0 and the slope of the tangents at P and Q
dI (a) dI (a) dt dI (a) are (−2) and (2), respectively.
and f (1) = 6 Now, = = − 2⋅
d (a) dt da dt Thus, the angle of intersection is
f and g are differentiable in (01 , ).
dI (a) tan −1 (2).
Let h( x ) = f ( x ) − 2g ( x ) …(i) ⇒ >0
⇒ h(0) = f (0) − 2g (0) dt 16 There is only one function in option (a),
⇒ h(0) = 2 − 0 ⇒ h(0) = 2 Thus, I (a) is an increasing function of t. 1
whose critical point ∈ (0,1) but in
and h(1) = f (1) − 2g (1) = 6 − 2(2) Hence, the given expression increasing 2
⇒ h(1) = 2, h( 0) = h(1) = 2 with (b − a). other parts critical point 0 ∉( 0,1). Then,
Hence, using Rolle’s theorem, we get we can say that functions in options (b),
15 We know that,
(c) and (d) are continuous on [0, 1] and
h ′ (c ) = 0, such that c ∈ (01 , ) 1 ≤ |sin x | + |cos x | ≤ 2 differentiable in (0, 1).
On differentiating Eq.(i) at c, we get
f ′ (c ) − 2g ′ (c ) = 0 ⇒ f ′ (c ) = 2g ′ (c )  1 − x, x<
1
√5  2 2
Now, for f ( x ) = 
13 Let y = ax2 + bx + c  1 
2
1
O P y=1  − x , x ≥
[equation of parabola]  2  2
As it touches y = x at x = 1.
1
∴ y = a+ b + c x Here, Lf ′   = − 1
–√5 O  2
and y =1 ⇒ a+ b + c =1
1 1 1
and Rf ′   = 2  −  (−1) = 0
dy
Now, = 2ax + b x 2+y 2=5
dx  2  2 2
dy
⇒   = 2a + b ⇒ 2a + b = 1 –√5 ∴
1 1
Lf ′   ≠ Rf ′  
 dx  at x = 1  2  2
[from y = x, slope = 1] ⇒ y = [|sin x | + |cos x |] = 1
1
Now, f ( x ) = ax2 + bx + c ⇒ f is non-differentiable at x = ∈ (0,1).
Let P and Q be the points of intersection 2
⇒ f ′ ( x ) = 2ax + b ⇒ f ′ ′ ( x ) = 2a of given curves.
∴ f (0 ) = c , f ′ (0 ) = b , f ′ ′ (0 ) = 2a, ∴ Lagrange mean value theorem is not
Clearly, the given curves meet at points applicable to f ( x ) in [0, 1].
f ′ (1 ) = 2a + b = 1 where y = 1, so we get
DAY FOURTEEN

Maxima and
Minima
Learning & Revision for the Day
u Maxima and Minima of a Function u Concept of Global Maximum/Minimum

Maxima and Minima of a Function


A function f ( x) is said to attain a maximum at x = a, Y Greatest value/
Local Local absolute
if there exists a neighbourhood (a − δ, a + δ), x ≠ a maximum maximum maximum
i.e. f ( x) < f (a), ∀ x ∈ (a − δ, a + δ), A C
x ≠ a ⋅ h > 0 (very small quantity) Local minimum
D
In such a case f (a) is said to be the maximum value X′ X
of f ( x) at x = a.
A function f ( x) is said to attain a minimum at B
x = a, if there exists a neighbourhood (a − δ, a + δ) Least value/
absolute minimum
such that f ( x) > f (a), ∀x ∈(a − δ, a + δ), x ≠ a. Y′
Graph of a continuous function explained local maxima (minima) and absolute maxima
(minima). In such a case f (a) is said to be the minimum value of f ( x) at x = a.
The points at which a function attains either the maximum or the minimum values are known
as the extreme points or turning points and both minimum and maximum values of f ( x) are
called extreme values. The turning points A and C are called local maximum and points B and
D are called local minimum.

Critical Point
PRED

A point c in the domain of a function f at which either f ′ (c) = 0 or f is not differentiable is MIRROR
Your Personal Preparation Indicator
called a critical point of f. Note that, if f is continuous at point c and f ′ (c) = 0, then there
exists h > 0 such that f is differentiable in the interval (c − h, c + h). u No. of Questions in Exercises (x)—

The converse of above theorem need not be true, that is a point at which the derivative u No. of Questions Attempted (y)—
vanishes need not be a point of local maxima or local minima. u No. of Correct Questions (z)—
(Without referring Explanations)
Method to Find Local Maxima or Local Minima u Accuracy Level (z / y × 100)—
First Derivative Test u Prep Level (z / x × 100)—
Let f be a function defined on an open interval I and f be continuous at a critical point c in I.
Then, In order to expect good rank in JEE,
your Accuracy Level should be
(i) If f ′ ( x) changes sign from positive to negative as x increases through c, i.e. if f ′ ( x) > 0 at above 85 & Prep Level should be
every point sufficiently close to and to the left of c and f ′ ( x) < 0 at every point above 75.
sufficiently close to and to the right of c, then c is a point of local maxima.
DAY 139

(ii) If f ′ ( x) changes sign from negative to positive as x Important Results


increases through point c, i.e. if f ′ ( x) < 0 at every point ●
If n is even and f n (a) < 0 ⇒ x = a is a point of local
sufficiently close to and to the left of c and f ′ ( x) > 0 at maximum.
every point sufficiently close to and to the right of c, ●
If n is even and f n (a) > 0 ⇒ x = a is a point of local
then c is a point of local minima. minimum.
(iii) If f ′ ( x) does not change sign as x increases through c, ●
If n is odd ⇒ x = a is a point of neither local maximum nor
then c is neither a point of local maxima nor a point of a point of local minimum.
local minima. Infact, such a point is called point of ax + b

The function f ( x) = has no local maximum or
inflection. cx + d
minimum regardless of values of a, b , c and d.
Y Point of local maxima Point of non-differentiability ●
The function f (θ) = sin m θ ⋅ cos n θ attains maximum values at
f′(c1)=0 and point at local maxima
0  m
f′ (

x)> θ = tan −1   .
f ′(
x) <

 n
Point of non-differentiability
0

Point f ′(c2)=0
>0

but it is a point of local ●


If AB is diameter of circle and C is any point on the
of local minima
x)

minima circumference, then area of the ∆ ABC will be maximum, if


f′ (

X′ X
O c1 c2 c3 c4 triangle is isosceles.
Y′

Graph of f around c explained following points.


Concept of Global
(iv) If c is a point of local maxima of f, then f (c) is a local Maximum/Minimum
maximum value of f . Similarly, if c is a point of local ●
Let y = f ( x) be a given function with domain D and
minima of f, then f (c) is a local minimum value of f . [a, b ] ⊆ D, then global maximum/minimum of f ( x) in [a, b ] is
basically the greatest / least value of f ( x) in [a, b ].
Second or Higher Order Derivative Test ●
Global maxima/minima in [a, b ] would always occur at critical
(i) Find f ′ ( x) and equate it to zero. Solve f ′ ( x) = 0 let its points of f ( x) within [a, b ] or at end points of the interval.
roots be x = a1 , a2 ,...
(ii) Find f ′ ′ ( x) and at x = a1 ,
Global Maximum/Minimum in [a, b]
In order to find the global maximum and minimum of f ( x) in
(a) if f ′ ′ (a1 ) is positive, then f ( x) is minimum at x = a1 .
[a, b ].
(b) if f ′ ′ (a1 ) is negative, then f ( x) is maximum at Step I Find out all critical points of f ( x) in [a, b ]
x = a1 .
[i.e. all points at which f ′ ( x) = 0] and let these
(iii) (a) If at x = a1 , f ′ ′ (a1 ) = 0, then find f ′ ′ ′ ( x). If points are c1 , c2 ,..., c n .
f ′ ′ ′ (a1 ) ≠ 0 , then f ( x) is neither maximum nor
Step II Find the value of f (c1 ), f (c2 ) ,..., f (c n ) and also at the
minimum at x = a.
end points of domain i.e. f (a) and f (b ).
(b) If f ′ ′ ′ (a1 ) = 0, then find f iv ( x).
iv
(c) If f ( x) is positive (minimum value) and f ( x) is iv Step III Find M1 → Global maxima or greatest value
negative (maximum value). and M2 → Global minima or least value.
(iv) If at x = a1 , f (a1 ) = 0 , then find f ( x) and proceed
iv v where, M1 = max { f (a), f (c1 ), f (c2 ),..., f (c n ), f (b )}
similarly. and M2 = min { f (a), f (c1 ), f (c2 ),..., f (c n ), f (b )}

Point of Inflection Some Important Results on Maxima


At point of inflection and Minima
(i) It is not necessary that 1st derivative is zero. (i) Maxima and minima occur alternatively i.e. between
two maxima there is one minimum and vice-versa.
(ii) 2nd derivative must be zero or 2nd derivative changes
(ii) If f ( x) → ∞ as x → a or b and f ′ ( x) = 0 only for one
sign in the neighbourhood of point of inflection.
value of x (say c) between a and b, then f (c) is
necessarily the minimum and the least value.
nth Derivative Test
(iii) If f ( x) → − ∞ as x → a or b, then f (c) is necessarily the
Let f be a differentiable function on an interval I and a be an
maximum and greatest value.
interior point of I such that
(iv) The stationary points are the points of the domain,
(i) f ′ (a) = f ′′(a) = f ′′′(a) = ... = f n −1 (a) = 0 and where f ′ ( x) = 0.
(ii) f n (a) exists and is non-zero.
140 40

DAY PRACTICE SESSION 1

FOUNDATION QUESTIONS EXERCISE


1 If f is defined as f ( x ) = x +
1
, then which of following is 10 If the function f ( x ) = 2x 3 − 9 ax 2 + 12 a 2 x + 1, where a > 0
x attains its maximum and minimum at p and q
true? j NCERT Exemplar respectively such that p 2 = q , then a is equal to
(a) Local maximum value of f (x) is − 2 1
(a) 3 (b) 1 (c) 2 (d)
(b) Local minimum value of f (x) is 2 2
(c) Local maximum value of f (x) is less than local minimum 11 If f ( x ) = x 2 + 2bx + 2c 2 and g ( x ) = − x 2 − 2cx + b 2 such
value of f (x) that minimum f ( x ) > maximum g ( x ), then the relation
(d) All the above are true between b and c is
2 If the sum of two numbers is 3, then the maximum value (a) 0 < c < b 2 (b) | c | < | b | 2
of the product of the first and the square of second is (c) | c | > | b | 2 (d) No real values of b and c
j NCERT Exemplar 12 Let f ( x ) be a polynomial of degree four having extreme
(a) 4 (b) 1 (c) 3 (d) 0  f (x )
values at x = 1 and x = 2. If lim 1 + 2 = 3, then f ( 2) is
x→ 0  x 
3 If y = a log x + bx + x has its extremum value at x = 1
2 
and x = 2, then (a, b ) is equal to equal to j JEE Mains 2015

−1 −2 −1 (a) −8 (b) −4
(a)  1,  (b)  , 2  (c)  2 ,  (d)  , 
1 1
 2 2   2  3 6 (c) 0 (d) 4

4 The function f ( x ) = a cos x + b tan x + x has extreme 13 If a differential function f ( x ) has a relative minimum at
π x = 0, then the function φ( x ) = f ( x ) + ax + b has a relative
values at x = 0 and x = , then
6 minimum at x = 0 for
(a) a = −
2
,b = −1 (b) a =
2
,b = −1 (a) all a and all b (b) all b, if a = 0
3 3 (c) all b > 0 (d) all a > 0
2 2
(c) a = − , b = 1 (d) a = , b = 1 14 The denominator of a fraction is greater than 16 of the
3 3
square of numerator, then least value of fraction is
5 The minimum radius vector of the curve (a) −1/4 (b) − 1/ 8
4 9 (c) 1/12 (d) 1/16
+ = 1 is of length
x2 y2 b
15 The function f ( x ) = ax + , b, x > 0 takes the least value
(a) 1 (b) 5 (c) 7 (d) None of these x
6 The function f ( x ) = 4x 3 − 18x 2 + 27x − 7 has at x equal to
(a) one local maxima j
NCERT Exemplar b
(a) b (b) a (c) b (d)
(b) one local minima a
(c) one local maxima and two local minima  tan x
, x ≠0
(d) neither maxima nor minima 16 Let f be a function defined by f ( x ) =  x
x2 − 2  1 , x = 0
7 The function f ( x ) = has
x2 − 4 Statement I x = 0 is point of minima of f .
(a) no point of local minima Statement II f ′ (0) = 0. j
AIEEE 2011
(b) no point of local maxima (a) Statement I is true, Statement II is true; Statement II is a
(c) exactly one point of local minima correct explanation for Statement I
(d) exactly one point of local maxima
(b) Statement I is true, Statement II is true; Statement II is
k − 2x , if x ≤ − 1 not a correct explanation for Statement I
8 Let f : R → R be defined by f ( x ) =  .
2x + 3, if x > − 1 (c) Statement I is true; Statement II is false
If f has a local minimum at x = −1, then a possible value (d) Statement I is false; Statement II is true
of k is j
AIEEE 2010 17 The absolute maximum and minimum values of the
(a) 1 (b) 0 (c) −
1
(d) −1 function f given by f ( x ) = cos 2 x + sin x , x ∈[ 0, π ]
2 j
NCERT Exemplar
9 The minimum value of 9x + 4y , where xy = 16 is (a) 2.25 and 2 (b) 1.25 and 1
(c) 1.75 and 1.5 (d) None of these
(a) 48 (b) 28 (c) 38 (d) 18
DAY 141

x 1 1
18 The maximum value of f ( x ) = on [ −1,1] is (a) ( area of ∆ABC) (b) ( area of ∆ABC)
4+ x + x2 2 4
1 1
(a) −
1
(b) −
1
(c)
1
(d)
1 (c) ( area of ∆ABC) (d) ( area of ∆ABC)
4 3 6 5 6 8

19 In interval [1, e ], the greatest value of x 2 log x is 28 If y = f ( x ) is a parametrically defined expression such
2 1 1 2
that x = 3 t 2 − 18 t + 7 and y = 2 t 3 − 15 t 2 + 24 t + 10,
(a) e (b) log (c) e log e (d) None of these
e e ∀ x ∈ [ 0, 6].
3x 2 + 9x + 17 Then, the maximum and minimum values of y = f ( x ) are
20 If x is real, the maximum value of is (a) 36, 3 (b) 46, 6 (c) 40, −6 (d) 46, −6
3x 2 + 9x + 7
AIEEE 2007
j 29 The value of a , so that the sum of the squares of the
(a) 41 (b) 1 (c)
17
(d)
1 roots of the equation x 2
− (a − 2)x − a + 1 = 0 assume
7 4 the least value is
21 The maximum and minimum values of (a) 2 (b) 1 (c) 3 (d) 0
f ( x ) = sec x + log cos 2 x , 0 < x < 2π are respectively 30 The minimum intercepts made by the axes on the
j NCERT Exemplar x2 y2
tangent to the ellipse + = 1 is
(a) (1, − 1) and {2 (1 − log 2), 2 (1 + log 2)} 16 9
(b) (1, − 1) and {2 (1 − log 2), 2 (1 − log 2)} (a) 25 (b) 7 (c) 1 (d) None of these
(c) (1, − 1) and (2, − 3)
(d) None of the above
31 The curved surface of the cone inscribed in a given
sphere is maximum, if
22 The difference between greatest and least values of the 4R R 2R
 π π (a) h = (b) h = (c) h = (d) None of these
function f ( x ) = sin 2x − x , on − , is 3 3 3
 2 2  j NCERT Exemplar
32 The volume of the largest cone that can be inscribed in a
π
(a) π (b) 2 π (c) 3 π (d) sphere of radius R is j NCERT
2 3
(a) of the volume of the sphere
23 The point of inflection for the curve y = x 5 / 2 is 8
(a) (1, 1) (b) (0, 0) (c) (1, 0) (d) (0, 1) 8
(b) of the volume of the sphere
27
24 The maximum area of a right angled triangle with 2
hypotenuse h is jJEE Main 2013 (c) of the volume of the sphere
7
3 2 2
h h h h2 (d) None of the above
(a) (b) (c) (d)
2 2 2 2 4
33 Area of the greatest rectangle that can be inscribed in
25 A straight line is drawn through the point P ( 3, 4) meeting x2 y2
the positive direction of coordinate axes at the points A the ellipse 2 + 2 = 1 is
a b
and B. If O is the origin, then minimum area of ∆OAB is a
equal to (a) ab (b) (c) 2ab (d) ab
b
(a) 12 sq units (b) 6 sq units
(c) 24 sq units (d) 48 sq units 34 The real number x when added to its inverse gives the
minimum value of the sum at x equal to j
AIEEE 2003
26 Suppose the cubic x 3 − px + q has three distinct real
(a) 2 (b) 1 (c) –1 (d) –2
roots, where p > 0 and q > 0. Then, which one of the
following holds? 35 The greatest value of

(a) The cubic has maxima at both


p
and −
p f ( x ) = ( x + 1)1/ 3 − ( x − 1)1/ 3 on [0, 1] is j
AIEEE 2002
3 3 (a) 1 (b) 2 (c) 3 (d) 1/3
p p
(b) The cubic has minima at and maxima at − 36 The coordinate of a point on the parabola y 2 = 8x whose
3 3
distance from the circle x 2 + ( y + 6) 2 = 1 is minimum, is
p p
(c) The cubic has minima at − and maxima at (a) (2,− 4) (b) (2,4) (c) (18,−12) (d) (8, 8)
3 3
p p 37 The volume of the largest cylinder that can be inscribed
(d) The cubic has minima at both and −
3 3 in a sphere of radius r cm is
4 πr 3 4 πr 3 4 πr 3 4 πr 3
27 If A ( x1, y1 ), B ( x 2 , y 2 ) and C ( x 3 , y 3 ) are the vertices of a (a) (b) (c) (d)
∆ ABC. A parallelogram AFDE is drawn with D, E and F 3 3 3 2 3 5 2
on the line segment BC, CA and AB, respectively. Then, 38 Maximum slope of the curve y = − x 3 + 3x 2 + 9x − 27 is
maximum area of such parallelogram is (a) 0 (b) 12 (c) 16 (d) 32
142 TEN

39 If ab = 2 a + 3b, a > 0, b > 0, then the minimum value of ab 40 The perimeter of a sector is p. The area of the sector is
is maximum, when its radius is
(a) 12 (b) 24 1 p p
1 (a) p (b) (c) (d)
(c) (d) None of these p 2 4
4

DAY PRACTICE SESSION 2

PROGRESSIVE QUESTIONS EXERCISE


a2 b2 ` 75 while at 40 km/h, it is ` 65. Then, the most
1 The minimum radius vector of the curve + = 1 is of
x2 y2 economical speed (in km/h) of the bus is j JEE Mains 2013
length (a) 45 (b) 50 (c) 60 (d) 40
(a) a − b (b) a + b | x 2 − 2 | , − 1 ≤ x < 3
(c) 2a + b (d) None of these  x

8 If f ( x ) =  , 3 ≤ x < 2 3, then the points,
2 f ( x ) = x 2 − 4 | x | and  3
3 − x , 2 3≤x ≤4
min {f (t ) : − 6 ≤ t ≤ x }, x ∈ [ − 6, 0]
g( x ) =  , then g ( x ) has
max {f (t ) : 0 < t ≤ x }, x ∈ ( 0, 6] where f ( x ) takes maximum and minimum values, are
(a) exactly one point of local minima (a) 1, 4 (b) 0, 4
(b) exactly one point of local maxima (c) 2, 4 (d) None of these
(c) no point to local maxima but exactly one point of local  3  1
| x + x 2 + 3x + sin x |  3 + sin  , x ≠ 0
minima 9 Let f ( x ) =   x , then
(d) neither a point of local maxima nor minima  0, x =0
4x − x 3 + log (a 2 − 3 a + 3), 0≤x <3 number of points [where, f ( x ) attains its minimum value]
3 f (x ) = 
x − 18 , x ≥3 is
(a) 1 (b) 2
Complete the set of values of a such that f ( x ) has a local
(c) 3 (d) infinite many
maxima at x = 3, is
(a) [−1, 2] (b) (−∞, 1) ∪ (2, ∞) 10 A wire of length 2 units is cut into two parts which are
(c) [1, 2] (d) (−∞, − 1) ∪ (2, ∞) bent respectively to form a square of side = x units and a
circle of radius = r units. If the sum of the areas of the
4 The point in the interval [ 0, 2π] , where f ( x ) = e x sin x has
square and the circle so formed is minimum, then
maximum slope is
(a) 2 x = (π + 4)r (b) (4 − π)x = πr
π π
(a) (b) (c) x = 2r (d) 2x = r
4 2
1 1
(c) π (d) None of these 11 Let f ( x ) = x 2 + 2
and g ( x ) = x − , x ∈ R − {−1, 0, 1}. If
x x
f (x )
5 The total number of local maxima and local minima of the h( x ) = , then the local minimum value of h( x ) is
g( x ) j
JEE Mains 2018
( 2 + x )3 , − 3 < x ≤ − 1
function f ( x ) =  2 / 3 is (a) 3 (b) −3 (c) −2 2 (d) 2 2
 x , −1 < x < 2
n2
(a) 0 (b) 1 (c) 2 (d) 3 12 The largest term in the sequence a n = 3 is given by
n + 200
6 If 20 m of wire is available for fencing off a flower-bed in 529 8
(a) (b)
the form of a circular sector, then the maximum area (in 49 89
sq m) of the flower-bed is j
JEE Mains 2017 49
(c) (d) None of these
(a) 12.5 (b) 10 (c) 25 (d) 30 543
b  13 All possible values of the parameter a so that the function
7 The cost of running a bus from A to B , is ` av +  ,
v  f ( x ) = x 3 − 3(7 − a )x 2 − 3( 9 − a 2 )x + 2 has a negative
where v km/h is the average speed of the bus. When the point of local minimum are
bus travels at 30 km/h, the cost comes out to be ` 75 (a) all real values (b) no real values
(c) (0, ∞) (d) (−∞,0)
DAY 143

14 The circle x 2 + y 2 = 1 cuts the R x2+y2=1 15 Given, P ( x ) = x 4 + ax 3 + bx 2 + cx + d such that x = 0 is


X -axis at P and Q. Another circle the only real root of P ′ ( x ) = 0. If P ( − 1) < P (1), then in the
with centre at Q and variable interval [ − 1, 1] . j AIEEE 2009

radius intersects the first circle at Q


T S
P (a) P (− 1) is the minimum and P (1) is the maximum of P
R above the X -axis and the line O
(b) P (− 1) is not minimum but P (1) is the maximum of P
segment PQ at S. Then, the
(c) P (− 1) is the minimum and P (1) is not the maximum of P
maximum area of the ∆QSR is
(d) Neither P (− 1) is the minimum nor P (1) is the maximum
(a) 4 3 sq units (b) 14 3 sq units of P
4 3
(c) sq units (d) 15 3 sq units
9

ANSWERS
SESSION 1 1 (d) 2 (a) 3 (d) 4 (a) 5 (b) 6 (d) 7 (d) 8 (d) 9 (a) 10 (c)
11 (c) 12 (c) 13 (b) 14 (b) 15 (d) 16 (b) 17 (b) 18 (c) 19 (a) 20 (a)
21 (b) 22 (a) 23 (b) 24 (d) 25 (c) 26 (b) 27 (a) 28 (d) 29 (b) 30 (b)
31 (a) 32 (b) 33 (c) 34 (b) 35 (b) 36 (a) 37 (b) 38 (b) 39 (b) 40 (d)

SESSION 2 1 (b) 2 (d) 3 (c) 4 (b) 5 (c) 6 (c) 7 (c) 8 (b) 9 (a) 10 (c)
11 (d) 12 (c) 13 (b) 14 (c) 15 (b)

Hints and Explanations


SESSION 1 At x = 3, ⇒ b = − 1, a = −
2
dy d 2P 3
1 Let y = x + 1 ⇒ 1
=1− 2
dx2
= 18 − 12 = 6 > 0 [minima]
x dx x 5 The given curve is 42 + 92 = 1
dy At x = 1, x y
Now, = 0 ⇒ x2 = 1 d 2P
dx = −6 < 0 Put x = r cos θ, y = r sin θ, we get
⇒ x=±1 dx2 r 2 = (2 sec θ)2 + (3 cosec θ)2
d2 y 2 So, P is maximum at x = 1. So, r 2 will have minimum value
⇒ = 3 , therefore
dx2 x ∴ Maximum value of P = 1 (3 − 1)2 = 4 ( 2 + 3)2 .
d2 y dy a
(at x = 1) > 0 3 Q = + 2bx + 1 or r have minimum value equal to 5.
dx2 dx x
d2 y 6 f ( x )= 4 x 3 − 18 x2 + 27 x − 7
(at x = − 1) < 0  dy 
and ⇒   = a + 2b + 1 = 0 f ′ ( x ) = 12 x2 − 36 x + 27
dx2  dx  x =1
Hence, local maximum value of y is at = 3(4 x2 − 12 x + 9) = 3(2 x − 3)2
⇒ a = − 2b − 1 3
x = − 1 and the local maximum value f ′ ( x ) = 0 ⇒ x = (critical point)
and  
dy a
= − 2. = + 4b + 1 = 0 2
 dx  x =2 2 3
Local minimum value of y is at x = 1 Since, f ′ ( x ) > 0 for all x < and for all
and local minimum value = 2. − 2b − 1 2
⇒ + 4b + 1 = 0
Therefore, local maximum value − 2 is 2 3
x>
less than local minimum value 2. 1 −1 2
⇒ – b + 4b + = 0 ⇒ 3b =
2 2 3
2 Let two numbers be x and (3 − x). −2
Hence, x = is a point of inflection i.e.,
−1 1 2
Then, product P = x(3 − x )2 ⇒ b = and a = − 1 =
dP 6 3 3 neither a point of maxima nor a point
= −2 x(3 − x ) + (3 − x )2 of minima.
dx 4 f ′( x ) = − a sin x + b sec2 x + 1 3
dP d 2P x = is the only critical point and f
= (3 − x )(3 − 3 x ) and 2 = 6 x − 12 π
dx dx Now, f ′ (0) = 0 and f ′   = 0 2
 6 has neither maxima nor minima.
dP
For maxima or minima, put =0 a 4b x2 − 2 dy − 4x
dx ⇒ b + 1 = 0 and − + + 1= 0 7 For y = ⇒ =
⇒ (3 − x )(3 − 3 x ) = 0 ⇒x = 3, 1 2 3 x2 − 4 dx ( x2 − 4)2
144 TEN


dy
> 0, for x < 0 ( 4 c 2 + 4b 2 ) ∴ f ( x ) is maximum at x = 4.
and maximum of g ( x ) = −
dx 4 (− 1) and at x = − 4, f ′ ′ ( x ) > 0, f ( x ) is
dy = b2 + c 2 minimum.
and < 0, for x > 0
dx Since, min f ( x ) > max g ( x ) ∴ Least value of
−4 1
Thus, x = 0 is the point of local maxima ⇒ 2c 2 − b 2 > b 2 + c 2 f ( x) = =−
1 16 + 16 8
for y. Now, ( y ) x = 0 = (positive). Thus, ⇒ c 2 > 2b 2
2 ⇒ |c |> 2 |b |
x = 0 is also the point of local 15 Given, f ( x ) = ax + b
x
x2 − 2 12 Central Idea Any function have
maximum for y = 2 . On differentiating w.r.t. x, we get
x −4 extreme values (maximum or b
minimum) at its critical points, where f ′( x ) = a − 2
8 If f ( x ) has a local minimum at x = −1, x
f ′( x ) = 0.
then For maxima or minima, put f ′ ( x ) = 0
Since, the function have extreme values
lim f ( x ) = lim− f ( x ) at x = 1 and x = 2. b
x→ −1 + x→ −1
⇒ x=
∴ f ′( x ) = 0 at x = 1 and x = 2 a
⇒ lim+ 2 x + 3 = lim− 1 < −2 x
x→ −1 x→ −1 ⇒ f ′(1) = 0 and f ′(2) = 0 Again, differentiating w.r.t. x, we get
⇒ −2 + 3 = k + 2 ⇒ k = −1 2b
Also it is given that f ′ ′( x ) = 3
lim 1 + 2  = 3 ⇒ 1 + lim 2 = 3
Y f ( x) f ( x) x
x→ 0  x  b
 x→ 0 x
At x = , f ′ ′ ( x ) = positive
f ( x) a
f(x)=1<–2x f(x)=2x+3 ⇒ lim 2 = 2
x→ 0 x b
(–1,1) ⇒ f ( x ) is minimum at x = .
O ⇒ f ( x ) will be of the form a
X′ X ax 4 + bx3 + 2 x2 b
(–1,0) ∴ f ( x ) has the least value at x = .
[Q f ( x ) is of four degree polynomial] a
Y′ Let f ( x ) = ax 4 + bx3 + 2 x2 ⇒ f ′( x )
9 Let S = 9 x + 4 y  tan x
= 4ax3 + 3bx2 + 4 x 16 f ( x ) =  x , x ≠ 0
Since, xy = 16 is given. ⇒ f ′(1) = 4a + 3b + 4 = 0 ...(i) 1, x=0
16 64 and f ′(2) = 32a + 12b + 8 = 0
∴ y = or S = 9 x + tan x
x x ⇒ 8a + 3b + 2 = 0 ...(ii) As > 1, ∀ x ≠ 0
x
On differentiating both sides, we get On solving Eqs. (i) and (ii), we get ∴ f (0 + h ) > f (0) and f (0 − h ) > f (0)
dS 64 1
= 9− 2 ...(i) a = ,b = − 2 At x = 0, f ( x ) attains minima.
dx x 2 f (h ) − f (0)
dS 64 8 x4 Now, f ′ (0) = lim
Q = 0⇒ 2 = 9⇒ x = ± ∴ f ( x) = − 2 x3 + 2 x2 h→ 0 h
dx x 3 2 tan h
−1
Again, on differentiating Eq. (i) ⇒ f (2) = 8 − 16 + 8 = 0 tan h − h
= lim h = lim
d 2S 128 13 φ′( x ) = f ′( x ) + a
h→ 0 h h→ 0 h2
w.r.t. x, we get 2 = 3
dx x [using L’ Hospital’s rule]
Q φ′ (0) = 0 ⇒ f ′ (0) + a = 0
8 sec 2 h − 1
Hence, it is minimum at x = and ⇒ a= 0 [Q f ′ (0) = 0] = lim [Q tan2 θ = sec 2 θ − 1]
h→ 0 2h
minimum value of S is 3 Also, φ′ (0) > 0 [Q f ′ ′ (0) > 0]
tan2 h 1
= lim ⋅h= ⋅0 = 0
S min = 9   + 4(6) = 48 ⇒ φ( x ) has relative minimum at
8 h→ 0 2h2 2
 3 x = 0 for all b, if a = 0
10 We have, Therefore, Statement II is true.
14 Let the number be x, then Hence, both statements are true but
f ( x ) = 2 x3 − 9 ax2 + 12a2 x + 1 x
f (x ) = Statement II is not the correct
f ′( x ) = 6 x2 − 18 ax + 12a2
x2 + 16 explanation of Statement I.
f ′ ′( x ) = 12 x − 18 a
On differentiating w.r.t. x, we get 17 Given, f ( x ) = cos2 x + sin x, x ∈ [0, π]
For maximum and minimum,
( x2 + 16) ⋅ 1 − x (2 x )
6 x2 − 18 ax + 12a2 = 0 f ′( x ) = Now,
( x2 + 16)2 f ′ ( x ) = 2 cos x (− sin x ) + cos x
⇒ x2 − 3 ax + 2a2 = 0
⇒ x = a or x = 2a x + 16 − 2 x
2 2
16 − x2 = − 2sin x cos x + cos x
= = 2 …(i)
At x = a maximum and at x = 2a ( x + 16)
2 2
( x + 16)2 For maximum or minimum put
minimum. Put f ′ ( x ) = 0 for maxima or minima f ′( x ) = 0
Q p2 = q f ′ ( x ) = 0 ⇒ 16 − x2 = 0 ⇒ −2sin x cos x + cos x = 0
∴ a2 = 2a ⇒ a = 2 or a = 0 ⇒ x = 4, − 4 ⇒ cos x (− 2sin x + 1 ) = 0
But a > 0, therefore a = 2 Again, on differentiating w.r.t. x, we 1
⇒ cos x = 0 or sin x =
get 2
11 Minimum of f ( x ) = − D ( x2 + 16)2 (−2 x ) − (16 − x2 ) π π
4a ⇒ x= ,
− ( 4b 2 − 8 c 2 ) 2( x2 + 16) 2 x 6 2
= f ′ ′( x ) = At
4 ( x2 + 16)4 For absolute maximum and absolute
= 2c 2 − b 2 x = 4, f ′ ′ ( x ) < 0 minimum, we have to evaluate
DAY 145

π π π π π
f (0), f   , f   , f ( π ) and f   = sin( π) − = −
10
=1+
 6  2  2
1  2 2 2
3   x +  +
3
   π π
At x = 0,  2 12  Clearly, is the greatest value and −
f (0) = cos 2 0 + sin 0 = 12 + 0 = 1 2 2
So, the maximum value of f ( x ) at is the least.
π π π π π π
At x = , f   = cos 2   + sin
3
x = − is Therefore, difference = + = π
6  6  6 6 2 2 2
2 23 Given, y = x 5/2
f  −
 3 3 10
=
1 5
 + = = 125
. =1+ = 1 + 40 = 41 dy 5 d 2 y 15 1 /2
 2
3  
 ∴ = x 3 /2 , 2 =
1 x
 2  2 4
π  12  dx 2 dx 4
At x = , 2
dy d y
2 21 Given, f ( x ) = sec x + log cos2 x At x = 0, = 0, =0
π π π
f   = cos 2   + sin = 02 + 1 = 1 ⇒ f ( x ) = sec x + 2 log(cos x )
dx dx2
 2  2 2
3
d y
Therefore, and 3 is not defined,
At x = π, dx
f ′ ( x ) = sec x tan x − 2 tan x
f ( π ) = cos 2 π + sin π = (−1)2 + 0 = 1 when x = 0, y = 0
= tan x (sec x − 2)
Hence, the absolute maximum value of f ′( x ) = 0 ∴(0, 0) is a point of inflection.
π
f is 1.25 occurring at x = and the ⇒ tan x = 0 or sec x = 2 ⇒ cos x =
1 24 Area of triangle, ∆ = 1 x h2 − x2
6 2 2
absolute minimum value of f is 1 Therefore, possible values of x are d∆ 1  2 x ( −2 x ) 
π π 5π =  h − x + 2
=0
occurring at x = 0, and π. x = 0, x = π and x = or x = . dx 2  2 h2 − x2 
2 3 3 
Note If close interval is given, to determine Again,f ′ ′ ( x ) = sec x (sec x − 2)
2
⇒ x=
h
global maximum (minimum), check the + tan x (sec x tan x ) 2
value at all critical points as well as end = sec3 x + sec x tan2 x − 2 sec2 x C
points of a given interval. = sec x (sec2 x + tan2 x − 2 sec x )
18 Q f ( x ) = x ⇒ f ′ ′ (0 ) = 1 (1 + 0 − 2) = − 1 < 0 h 
4 + x + x2 Therefore, x = 0 is a point of maxima. √h2 – x2
On differentiating w.r.t. x, we get f ′ ′ ( π ) = − 1 (1 + 0 + 2) = − 3 < 0
4 + x + x2 − x ( 1 + 2 x ) Therefore, x = π is a point of maxima.
f ′( x ) = x
(4 + x + x2 )2 π A B
f ′ ′   = 2 (4 + 3 − 4) = 6 > 0
For maximum, put f ′ ( x ) = 0  3 d ∆
2
h
π ⇒ < 0 at x =
4 − x2 Therefore, x = is a point of minima. dx2 2
⇒ = 0 ⇒ x = 2, − 2 3
(4 + x + x2 )2 5π  h2 h2
f ′ ′ 
1 h
 = 2 (4 + 3 − 4) = 6 > 0 ∴ ∆ = × h2 − =
Both the values of x are not in the  3  2 2 2 4
interval [−1, 1] . 5π
−1 −1 Therefore, x = is a point of minima. 25 Let the equation of drawn line be
∴ f ( −1 ) = = 3 x y
4−1+ 1 4 + = 1, where a > 3,
Maximum value of y at x = 0 is a b
1 1 1 + 0 = 1. b > 4, as the line passes through (3, 4)
f (1 ) = = (maximum)
4+ 1+1 6 Maximum value of y at x = π is and meets the positive direction of
− 1 + 0 = − 1. coordinate axes.
19 Given, f ( x ) = x2 log x π 3 4 4a
Minimum value of y at x = is We have, + = 1 ⇒ b =
On differentiating w.r.t. x, we get 1 3 a b (a − 3)
f ′ ( x ) = (2 log x + 1 ) x 2 + 2 log = 2 (1 − log 2).
2 Now, area of ∆AOB,
For a maximum, put f ′ ( x ) = 0 5π 1 2a2
Minimum value of y at x = is ∆ = ab =
⇒ (2 log x + 1 ) x = 0 1 3 2 (a − 3)
⇒ x = e −1 /2 , 0 2 + 2 log = 2 (1 − log 2).
2 d∆ 2a (a − 6)
0 < e −1 / 2 < 1 =
Q 22 Given, f ( x ) = sin 2 x − x da (a − 3)2
None of these critical points lies in the
⇒ f ′ ( x ) = 2 cos 2 x − 1 Clearly, a= 6 is the point of minima for ∆.
interval [1, e ] .
1 2 × 36
Put f ′ ( x ) = 0 ⇒ cos 2 x = Thus, ∆ min = = 24 sq units
So, we only compute the value of f ( x ) at 2 3
the end points 1 and e. π π π π
⇒ 2x = − or ⇒ x = − or 26 Let f ( x ) = x3 − px + q
We have, f (1 ) = 0, f (e ) = e 2 3 3 6 6
Hence, greatest value of f ( x ) = e 2 π π π
Now, f  −  = sin(− π ) +
Maxima
=
10  2 2 2
20 Let f (x ) = 1 +
3  x + 3 x + 
 7 π 2π  π π
f  −  = sin  −
2 3
 3  + =− +
 6  6 6 2 6 −√p/3
π 2π  π π
f   = sin 
3 Minima
 − = −
 6  6 6 2 6 √p/3
146 TEN

Then, f ′ ( x ) = 3 x2 − p d2 y OA = AC − OC = h − R
At (t = 1), <0
Put f ′( x ) = 0 dx2 In ∆OAB, R2 = r 2 + (h − R )2
p p ⇒ t = 1 is a point of local maxima. ⇒ r = 2Rh − h2
⇒ x= ,−
3 3 d2 y ∴ S = πrl = π ( 2Rh − h2 )( h2 + r 2 )
At (t = 4), 2 > 0
Now, f ′′ ( x ) = 6 x dx
⇒ t = 4 is a point of local minima. = ( π 2Rh − h2 )( 2Rh )
p p
At x = , f ′′ ( x ) = 6 >0 dy d2 y Let S 2 = P
3 3 [minima] At (t = 3), and 2 are not defined
dx dx ∴ P = π2 2R ( 2 Rh2 − h3 )
p Since, S is maximum, if P is maximum,
and at x = − f ′′ ( x ) < 0 [maxima] and change its sign.
3 d2 y then
is unknown in the vicinity of t = 3, dP
27 We have, AF || DE and AE || FD dx2 = 2 π2 R (4Rh − 3h2 ) = 0
thus t = 3 is a point of neither maxima dh
4R
A nor minima. ∴ h = 0,
Finally, maximum and minimum values 3
y of expression y = f ( x ) are 46 and −6, dP
Again, on differentiating , we get
respectively. dh
2
d P
F E 29 Let α and β be the roots of the equation = 2 π2 R ( 4 R − 6 h )
dh2
x2 − (a − 2) x − a + 1 = 0 d 2P 4R
x < 0 at h =
Then, α + β = a − 2, αβ = − a + 1 dh2 3
Let z = α 2 + β2
B D C
= (α + β )2 − 2αβ 32 Let OC = x, CQ = r
Now, in ∆ABC and ∆EDC, = (a − 2)2 + 2 (a − 1) Now, OA = R [given]
∠DEC = ∠BAC , ∠ACB is common. = a2 − 2a + 2 Height of the cone = h = x + R
⇒ ∆ABC ≅ ∆EDC dz ∴ Volume of the cone
⇒ = 2a − 2 1
b − y x c da = V = π r 2h …(i)
Now, = ⇒ x = (b − y ) 3
b c b dz
Put = 0 , then A
Now, S = Area of parallelogram da
AFDE = 2 (area of ∆AEF ) ⇒ a=1
d 2z
S = 2  xy sin A 
1 ∴ = 2> 0
⇒ R
2  da2 h
O
c So, z has minima at a = 1.
= (b − y )y sin A x
b So, α2 + β2 has least value for a = 1.
dS  c
=  sin A  (b − 2 y )
This is because we have only one P r Q
dy b  stationary value at which we have C
dS minima.
Sign scheme of , Hence, a = 1.
dy Also, in right angled ∆OCQ,
30 Any tangent to the ellipse is OC 2 + CQ 2 = OQ 2
+ –
b/2 x y
cos t + sin t = 1, where the point of ⇒ x2 + r 2 = R2
b 4 3 ⇒ r 2 = R2 − x2 …(ii)
Hence, S is maximum when y = ⋅
2 contact is (4cos t , 3sin t ) From Eqs. (i) and (ii),
c b b x y 1
V = π (R2 − x2 )( x + R )
∴ S max =   × sin A or + = 1, …(iii)
b  2 2 4 sec t 3 cosec t 3
[Q h = x + R ]
=  bc sin A  = (area of ∆ABC )
1 1 1 It means the axes Q (4sec t , 0) and
On differentiating Eq. (iii) w.r.t. x, we
22  2 R (0, 3 cosec t ).
get
∴The distance of the line segment QR is dV 1
28 We have, QR2 = D = 16 sec2 t + 9 cosec2 t = π [(R2 − x2 ) − 2 x( x + R )]
dy dx 3
= 6 t 2 − 30 t + 24 = 6 (t − 1) (t − 4)
dt So, the minimum value of D is (4 + 3)2 dV π
or QR = 7. ⇒ = (R2 − x2 − 2 x2 − 2 xR )
dx dx 3
and = 6 t − 18 = 6 (t − 3)
dt 31 Let S be the curved surface area of a dV π
⇒ = (R2 − 2 xR − 3 x2 )
dy (t − 1) (t − 4) cone. dx 3
Thus, = C
dx (t − 3) dV π
⇒ = (R − 3 x )(R + x ) …(iv)
which indicates that t = 1, 3 and 4 are dx 3
the critical points of y = f ( x ). dV
For maxima, put =0
d2 y d  dy  dt h dx
Now, =  ⋅ O π
dx2 dt  dx  dx R ⇒ (R − 3 x )(R + x ) = 0
3
t 2 − 6 t + 11 1 ⇒ x=
R
or x = − R ⇒ x =
R
= × B
(t − 3)2 6 (t − 3) A 3 3
r
[since, x cannot be negative]
DAY 147

On differentiating Eq. (iv) w.r.t. x, we get Any point on parabola is (at 2 ,2at ), Let g ( x ) = f ′ ( x ) = − 3 x2 + 6 x + 9
d V 2
π i.e., (2t 2 , 4t ). On differentiating w.r.t. x, we get
= [(−3)(R + x ) + (R − 3 x )]
dx2 3 For its minimum distance from the
g ′( x ) = − 6 x + 6
π π circle means its distance from the centre
= (−2R − 6 x ) = − (2 R + 6 x ) For maxima or minima put g ′ ( x ) = 0
3 3 (0, − 6) of the circle.
R d 2V −π  6R  Let z = (2t 2 )2 + (4t + 6)2 ⇒ x=1
At x = , =  2R +  = 4(t 4 + 4t 2 + 12t + 9)
3 dx2 3  3  Now, g ′ ′ ( x ) = − 6 < 0 and hence, at
dz x = 1, g ( x ) (slope) will have maximum
4π ∴ = 4(4t 3 + 8t + 12)
=− R<0 dt value.
3
⇒ 16(t 3 + 2t + 3) = 0 ∴[g ( 1 )] max = − 3 × 1 + 6( 1 ) + 9 = 12
So, V has a local maxima at x = R / 3.
⇒ (t + 1 )(t 2 − t + 3) = 0
Now, on substituting the value of x in 39 Given,
⇒ t = −1
Eq. (iii), we get ab = 2 a + 3b ⇒ (a − 3) b = 2 a
d 2z
π R2   R ⇒ = 16(3t + 2)> 0, hence minimum.
2
2a
V =  R2 −  R +  dt 2 ⇒ b =
3 9  3 a−3
So, point is (2, − 4).
π 8R 4R2
8 4 3 2 a2
= ⋅ ⋅ =  πR  37 We know that, volume of cylinder, Now, let z = ab =
3 9 3 27  3  a−3
V = πR 2 h
8 On differentiating w.r.t. x, we get
⇒ V = × Volume of sphere
27 B′ A′ dz 2 [(a − 3) 2 a − a2 ] 2 [a2 − 6a]
C′ = =
33 Y da (a − 3)2 (a − 3)2
h/2
(–a cos θ, b sin θ) (a cos θ, b sin θ) dz
For a minimum, put =0
B A O da
h/2 r ⇒ a2 − 6a = 0
X′ X
⇒ a = 0, 6
C d 2z
C D B A At a = 6, 2 = positive
(–a cos θ, –b sin θ) (a cos θ, –b sin θ) R da
Y′ 2 When a = 6, b = 4
In ∆OCA, r 2 =   + R2
h
Area of rectangle ABCD ∴ (ab )min = 6 × 4 = 24
 2
= (2a cos θ) (2b sin θ) = 2ab sin 2θ 40 Q Perimeter of a
h2 A
Hence, area of greatest rectangle is ⇒ R2 = r 2 − sector = p
equal to 2ab when sin 2θ = 1. 4 r
 h2  Let AOB be
34 Let f ( x) = x +
1 ∴ V = π  r2 − h the sector with O θ s
x  4
radius r. r
1 π
f ′( x ) = 1 − 2 ⇒ V = πr 2 h − h3 …(i) If angle of the B
x 4 sector be θ radians,
For maxima and minima, put f ′ ( x ) = 0 On differentiating Eq. (i) both sides then area of sector,
1 w.r.t. h, we get 1
⇒ 1 − 2 = 0⇒ x = ± 1 A = r 2θ …(i)
x dV 3 πh2 2
= πr 2 −
2 dh 4 s
Now, f ′ ′ ( x ) = 3 and length of arc, s = r θ ⇒ θ =
x d2 V −3 πh r
⇒ =
At x = 1, f ′ ′ ( x ) = + ve [minima] dh2 2 ∴Perimeter of the sector
and at x = − 1, f ′ ′ ( x ) = −ve [maxima] For maximum or minimum value of V , p = r + s + r = 2 r + s …(ii)
Thus, f ( x ) attains minimum value at dV 3 πh2 s
= 0 ⇒ πr 2 − =0 On substituting θ = in Eq. (i), we get
x = 1. dh 4 r
2
35 Given that, f ( x ) = ( x + 1)1 /3 − ( x − 1)1 /3 4r 2
A =  r 2    = rs ⇒ s =
⇒ h2 = ⇒ h= r 1 s 1 2A
On differentiating w.r.t. x, we get
3 3 2   r  2 r
 d2 V 
1 1 1  Now,  2  = − 3 πr < 0 Now, on substituting the value of s in
f ′( x ) =  −  dh  h = 2 r
3  ( x + 1)2 /3 ( x − 1)2 /3  3
Eq. (ii), we get
p = 2 r + 
2A
( x − 1)2 /3 − ( x + 1)2 /3  ⇒ 2 A = pr − 2 r
2
2r
= Thus, V is maximum when h = , then  r 
3( x2 − 1)2 /3 3
2 On differentiating w.r.t. r, we get
Clearly, f ′( x ) does not exist at x = ± 1. h2 1  2r  2 2
R2 = r 2 − = r2 −   = r 2
dA
= p − 4r
Now, put f ′ ( x ) = 0, then 4 4 3 3 dr
( x − 1)2 /3 = ( x + 1)2 /3 ⇒ x = 0 4 πr 3
Max V = πR h =
2 For the maximum area, put
At x = 0 3 3 dA
f ( x ) = (0 + 1)1 /3 − (0 − 1)1 /3 = 2 =0
38 Let f ( x ) = − x 3 + 3 x2 + 9 x − 27 dr
Hence, the greatest value of f ( x ) is 2.
⇒ p − 4r = 0
36 Q y 2 = 8 x. But y 2 = 4ax The slope of this curve
p
f ′ ( x ) = − 3 x2 + 6 x + 9 ⇒ r =
⇒ 4a= 8 ⇒ a= 2 4
148 TEN

Clearly, f ′( x ) changes its sign at x = − 1 b


SESSION 2 from positive to negative and so f ( x ) has
So, at v = the speed is most
a
1 Let radius vector is r. local maxima at x = − 1. economical.
∴ r 2 = x2 + y 2 Also, f ′(0) does not exist but f ′ (0− ) < 0 ∴Most economical speed is
a2 y 2  a2 b2  and f ′ (0+ ) > 0. It can only be inferred
⇒ r2 = 2 + y2 Q 2 + 2 = 1 b a
y − b2  x y  that f ( x ) has a possibility of a minimum c =a +b = 2 ab
at x = 0. Hence, it has one local maxima a b
For minimum value of r, at x = − 1 and one local minima at x = 0 1
d (r 2 ) −2 yb 2 a2 c =2 × 1800 = 2 × 30
=0 ⇒ + 2y = 0 So, total number of local maxima and 2
dy ( y 2 − b 2 )2 local minima is 2.
⇒ c = 60
⇒ y 2 = b( a + b ) 6 Total length = 2r + r θ = 20
∴ x2 = a(a + b )  | x2 − 2 |, − 1 ≤ x < 3

⇒ r 2 = (a + b )2 ⇒ r = a + b 8 f ( x ) =  x , 3 ≤ x < 2 3
2 Bold line represents the graph of θ  3
y = g ( x ) , clearly g ( x ) has neither a 3 − x, 2 3 ≤ x ≤ 4
point of local maxima nor a point of r r Y
local minima.
2 y = |x2 – 2|
y= x
3
y = x2 – 4|x| rθ
1

20 − 2r 4
⇒ θ= X′ X
–2 0 2 r
–1 O √2 √3 2√3
–6 –4 4 6 Now, area of flower-bed,
y=3 – x
1
A = r 2θ Y′
2
From the above graph,
20 − 2r 
A = r 2 
1
⇒  Maximum occurs at x = 0 and
3 Clearly, f ( x ) in increasing just before 2  r 
minimum at x = 4.
x = 3 and decreasing after x = 3. For ⇒ A = 10r − r 2 | x + x + 3 x + sin x |
3 2

x = 3 to be the point of local maxima. dA  x≠0


f (3) ≥ f (3 − 0)

dr
= 10 − 2r 9 f ( x ) =   3 + sin  1   ,
   x  
⇒ − 15 ≥ 12 − 27 + log (a2 − 3 a + 3) For maxima or minima, put
dA
= 0.  0, x=0
⇒ 0 < a2 − 3 a + 3 ≤ 1 ⇒ 1 ≤ a ≤ 2 dr
⇒ 10 − 2r = 0 ⇒ r = 5 Let g ( x ) = x3 + x2 + 3 x + sin x
4 (Slope) f ′( x ) = e x cos x + sin xe x g ′ ( x ) = 3 x2 + 2 x + 3 + cos x
1 2  20 − 2 (5)
= e x 2 sin ( x + π/4) ∴ A max = (5)
= 3  x2 + + 1 + cos x
  2x
2 5
f ′ ′ ( x ) = 2e x {sin ( x + π/4)  3 
1
+ cos ( x + π/4)} = × 25 × 2 = 25sq m
 8
2
2
= 3   x +  +  + cos x > 0
1
= 2e x ⋅ sin ( x + π/ 2)
For maximum slope, put f ′ ′ ( x ) = 0 7 Let c = av + b …(i)  3 9
v
⇒ sin ( x + π/2 ) = 0  1
and 2 < 3 + sin   < 4
When v = 30 km/ h, then c = ` 75  x
⇒ cos x = 0
b
∴ x = π / 2, 3 π / 2 ∴ 75 = 30 a + …(ii) Hence, minimum value of f ( x ) is 0 at
30
f ′ ′ ′ ( x ) = 2e x cos ( x + π/ 2) x = 0.
When v = 40 km/h, then c = ` 65
f ′ ′ ′ ( π/2 ) = 2e x ⋅ cos π = − ve Hence, number of points = 1
b
∴ 65 = 40 a + …(iii)
Maximum slope is at x = π /2. 40 10 According to given information, we
3 (2 + x ) , − 3 < x ≤ − 1
2
On solving Eqs. (ii) and (iii), we get have
5 f ′( x ) =  2 −1 /3 a=
1
and b = 1800 Perimeter of square + Perimeter of circle
x , − 1< x < 2
 3 2 = 2 units
Y On differentiating w.r.t. v in Eq. (i), ⇒ 4 x + 2 πr = 2
dc
= a− 2
b 1 − 2x
⇒ r = ...(i)
A dv v π
For maximum or minimum c,3 Now, let A be the sum of the areas of
dc b the square and the circle. Then,
X′ X =0 ⇒ v =±
(–2, 0) (–1, 0) O dv a A = x2 + πr 2
d 2c 2b (1 − 2 x ) 2
⇒ = 3 at v =
b dx
, >0 = x2 + π
(–3, 0) dv 2 v a dv 2 π2
(1 − 2 x )2
Y′ ⇒ A( x ) = x2 +
π
DAY 149

Now, for minimum value of A( x ), 49


>
8 8r − 3 r 3
and =
dA 543 89 4 4 − r2
=0
dx 49
⇒ a7 = is the greatest term. dA
2 (1 − 2 x ) = 0, when r (8 − 3 r 2 ) = 0 giving
⇒ 2x + ⋅ (− 2) = 0 543 dr
π 8
2 − 4x 13 f ( x ) = x 3 − 3(7 − a)x2 − 3(9 − a2 )x + 2 r =
⇒ x= 3
π f ′ ( x ) = 3 x2 − 6(7 − a)x − 3(9 − a2 )
4 4 − r 2 (8 − 9r 2 )
⇒ πx + 4 x = 2 For real root D ≥ 0,
⇒ 49 + a2 − 14a + 9 − a2 ≥ 0 (−4r )
⇒ x=
2
...(ii) − (8r − 3 r 3 )
π+ 4 ⇒a≤
58 d2 A 4 − r2
⇒ 2 =
Now, from Eq. (i), we get 14 dr 16 (4 − r )
2

2 For local minimum 8 d2 A


1 − 2⋅ When r = , then 2 < 0
π+ 4 f ′ ′ ( x ) = 6 x − 6(7 − a) > 0 3 dr
r =
π ⇒ 7− x 8
π + 4− 4 1 Hence, A is maximum when r = .
= = ...(iii) has x must be negative 3
π(π + 4) π+ 4 ⇒ 7 − a< 0 Then, maximum area
From Eqs. (ii) and (iii), we get ⇒ a> 7 8 8 4 3
= 4− = sq unit
x = 2r Thus constradictory, i.e., for real roots 4×3 3 9
58
11 We have, a≤ and for negative point of local
14 15 Given,
1 1
f ( x ) = x2 + and g ( x ) = x − minimum a > 7. P ( x ) = x 4 + a x3 + bx2 + cx + d
x2 x ⇒ P ′ ( x ) = 4 x3 + 3 ax2 + 2 bx + c
f ( x) No possible values of a.
⇒ h( x ) = Since, x = 0 is a solution for
g( x) 14 From the given figure coordinate of Q is P ′ ( x ) = 0, then
1  x − 1 + 2
2
(− 1, 0 ). c =0
x2 +  
x 2  x The equation of circle centre at Q with ∴ P ( x ) = x 4 + ax3 + bx2 + d …(i)
∴ h( x ) = = variable radius r is
1 1 Also, we have P (− 1) < P (1)
x− x−
x x R ⇒ 1− a+ b + d <1+ a+ b + d
 1 2 ⇒ a> 0
⇒ h( x ) =  x −  +
 x 1 Since, P ′ ( x ) = 0, only when x = 0 and
x−
x P ( x ) is differentiable in (− 1, 1), we
should have the maximum and
x − > 0, x −  +
1 1 2
∈ [2 2, ∞ ) minimum at the points x = − 1, 0 and 1.
x  x 1 Q T O S P
x− Also, we have P (− 1) < P (1)
x
1 ∴ Maximum of P ( x ) = Max {P (0), P (1)}
x − < 0, and minimum of P ( x ) = Min
x
 x − 1 + 2 {P (− 1), P (0)}
  ∈ (−∞, 2 2] In the interval [0, 1],
 x 1
x− ( x + 1 )2 + y 2 = r 2 …(i)
x P ′ ( x ) = 4 x3 + 3ax2 + 2 bx
This circle meets the line segment QP at
∴ Local minimum value is 2 2. = x (4 x2 + 3ax + 2 b )
S, where QS = r
Since, P ′( x ) has only one root x = 0,
12 Consider the function It meets the circle x2 + y 2 = 1 at …(ii)
x2 then 4 x2 + 3 ax + 2 b = 0 has no real
f (x ) =  r2 − 2 r 
R , 4 − r2  roots.
( x + 200)
3
 2 2 
∴ (3a)2 − 32 b < 0
(400 − x 3 )
f ′( x ) = x 3 =0 [on solving Eqs. (i) and (ii)
9 a2
( x + 200)2 A = Area of ∆QSR ⇒ <b
1 32
when x = (400)1 /3 , (Q x ≠ 0) = × QS × RT
2 ∴ b>0
x = (400)1 /3 − h ⇒ f ′ ( x ) > 0 Thus, we have a > 0 and b > 0.
= r  . 4 − r 2 
1 r
x = (400)1 /3 + h ⇒ f ′ ( x ) < 0 2 2  ∴ P ′ ( x ) = 4 x3 + 3 ax2 + 2 bx > 0,
∴ f ( x ) has maxima at x = (400)1 /3 [since, RT is the y-coordinate of R] ∀ x ∈ (0, 1)
1 Hence, P ( x ) is increasing in [0, 1].
Since, 7 < (400) < 8, either a7 or a8 is
1 /3 = [r 2 4 − r 2 ]
4 ∴ Maximum of P ( x ) = P (1)
the greatest term of the sequence.
dA 1  r 2 (− r )  Similarly, P ( x ) is decreasing in [−1 , 0].
a7 =
49 ∴ = 2r 4 − r 2 + 
Q dr 4 4 − r 2  Therefore, minimum P ( x ) does not
543 
occur at x = − 1.
8 {2r (4 − r 2 ) − r 3 }
and a8 = =
89 4 4 − r2
DAY FIFTEEN

Indefinite
Integrals
Learning & Revision for the Day
u Integral as an u Fundamental Integration u Methods of Integration
Anti-derivative Formulae

Integral as an Anti-derivative
A function φ ( x) is called a primitive or anti-derivative of a function f ( x), if φ′ ( x) = f ( x). If f1 ( x)
and f2 ( x) are two anti-derivatives of f ( x), then f1 ( x) and f2 ( x) differ by a constant. The collection
of all its anti-derivatives is called indefinite integral of f ( x) and is denoted by ∫ f ( x) dx.
d
Thus, {φ ( x) + C} = f ( x) ⇒ ∫ f ( x) dx = φ ( x) + C
dx
where, φ ( x) is an anti-derivative of f ( x), f ( x) is the integrand and C is an arbitrary constant
known as the constant of integration. Anti-derivative of odd function is always even and of
even function is always odd.

Properties of Indefinite Integrals



∫ { f ( x) ± g( x)} dx = ∫ f ( x) dx ± ∫ g( x) dx PRED

∫ k ⋅ f ( x) dx = k ⋅ ∫ f ( x) dx, where k is any non-zero real number. MIRROR
Your Personal Preparation Indicator

∫ [k f ( x) + k f ( x) +...+ k f ( x)] dx = k ∫ f ( x) dx + k ∫ f ( x) dx + ... + k ∫ f ( x) dx,
1 1 2 2 n n 1 1 2 2 n n

where k1 , k2 ,... k n are non-zero real numbers. u No. of Questions in Exercises (x)—
u No. of Questions Attempted (y)—
Fundamental Integration Formulae u No. of Correct Questions (z)—
(Without referring Explanations)
There are some important fundamental formulae, which are given below
u Accuracy Level (z / y × 100)—
1. Algebraic Formulae u Prep Level (z / x × 100)—
xn + 1
(i) ∫ x ndx =
n+1
+ C, n ≠ − 1 In order to expect good rank in JEE,
your Accuracy Level should be
+1 above 85 & Prep Level should be
1 (ax + b )n
(ii)∫ (ax + b )n dx = ⋅ + C, n ≠ − 1 above 75.
a n+1
DAY 151

1
(iii) ∫ x
dx = log| x | + C 3. Exponential Formulae
∫ e dx = e +C
x x
(i)
1 1
(iv) ∫ ax + b
dx = (log| ax + b |) + C
a 1 ( ax
∫e
( ax + b ) + b)
(ii) dx = ⋅e +C
1 1 a+ x a
(v) ∫ a −x
dx = log
a− x
+C ax
∫a
2 2
2a (iii) x
dx = + C, a > 0 and a ≠ 1
loge a
1 1 x−a
(vi) ∫ dx = log +C 1 a (bx + c )
∫a
(bx + c )
x2 − a2 2a x+a (iv) dx = ⋅ + C, a > 0 and a ≠ 1
b loge a
1 1  x
(vii) ∫a+ x2
2
dx = tan −1   + C
a  a
−1 1  x
Methods of Integration
(viii) ∫ 2 dx = cot −1   + C
a + x2 a  a Following methods are used for integration
1
(ix) ∫ x − a2
2
dx = log| x + x2 − a2 | + C 1. Integration by Substitutions
The method of reducing a given integral into one of the
1
(x) ∫ x2 + a2
dx = log| x + x2 + a2 | + C standard integrals, by a proper substitution, is called method
of substitution.

(xi) ∫
1  x
dx = sin −1   + C
To evaluate an integral of the form ∫ f {g( x)} ⋅ g′ ( x) dx, we
a2 − x2  a
substitute g( x) = t , so that g′ ( x)dx = dt and given integral
−1  x reduces to ∫ f (t ) dt .

−1
(xii) dx = cos   + C
a −x
2 2  a
[ f ( x )]n +1
1 1  x
NOTE • ∫ [ f ( x )]n ⋅ f ′( x ) = +C
(xiii) ∫ dx = sec −1   + C n+ 1
x x −a
2 2 a  a 1
• If ∫ f ( x ) dx = φ( x ), then ∫ f ( ax + b) dx = φ( ax + b) + C
−1 1  x a
(xiv) ∫ dx = cosec −1   + C
x −a
2 2
a  a
x (i) To evaluate integrals of the form
1 1  x dx dx
(xv) ∫ a2 − x2 dx = x a2 − x2 + a2 sin −1   + C
2 2  a ∫ ax or ∫ or
2
+ bx + c ax + bx + c
2
1 1
(xvi) ∫ x2 − a2 dx = x x2 − a2 − a2 log| x + x2 − a2 | + C
2 2 ∫ ax 2 + bx + c dx
1 1
(xvii) ∫ x2 + a2 dx = x x2 + a2 + a2 log| x + x2 + a2 | + C  b c
We write, ax2 + bx + c = a  x2 + x + 
2 2  a a
2. Trigonometric Formulae  b c b2
= a x +  + −
2

 2 a a 4c
(i) ∫ sin x dx = − cos x + C This process reduces the integral to one of following forms
(ii) ∫ cos x dx = sin x + C dX dX dX
=∫ 2
X − A2 ∫ X 2 + A2
, or ∫ 2 ,
A − X2
(iii) ∫ tan x dx = − log|cos x | + C = log|sec x | + C dX dX dX
∫ A 2 − X 2 , ∫ x2 − A 2 , ∫ X 2 + A 2
(iv) ∫ cot x dx = log|sin x | + C = − log| cosec x | + C
π x
(v) ∫ sec xdx = log| sec x + tan x |+ C = log tan  +  + C
or ∫ A2 − X 2 dX , ∫ X 2 − A2 dX , ∫ A2 + X 2 dX
 4 2
x
(vi) ∫ cosec x dx = log| cosec x − cot x | + C = log tan + C (ii) To evaluate integrals of the form
2
(px + q) (px + q)
(vii) ∫ sec x dx = tan x + C
2
∫ ax 2
+ bx + c
dx or ∫ ax 2 + bx + c
dx
(viii) ∫ cosec x dx = − cot x + C
2

(ix) ∫ sec x ⋅ tan x dx = sec x + C


or ∫ (px + q) ax 2 + bx + c dx
We put px + q = A {differentiation of (ax2 + bx + c)} + B,
(x) ∫ cosec x ⋅ cot x dx = − cosec x + C where A and B can be found by comparing the coefficients of
like powers of x on the two sides.
152

2. Integration using Trigonometric ● To evaluate integrals of the form


1 1
Identities ∫ a sin x + b cos x dx or ∫ a + b sin x dx
In this method, we have to evaluate integrals of the form
1 1
or ∫ dx or ∫ dx

∫ sin mx ⋅ cos nx dx or ∫ sin mx ⋅ sinnx dx or a + b cos x a sin x + b cos x + c
∫ cos mx ⋅ cos nx dx or ∫ cos mx ⋅ sinnx dx 2 tan
x
2
x
1 − tan2
2
(i) Put sin x = and cos x =
In this method, we use the following trigonometrical identities x x
1 + tan2 1 + tan2
(i) 2 sin A ⋅ cos B = sin ( A + B) + sin ( A − B) 2 2
x x x
(ii) 2 cos A ⋅ sin B = sin ( A + B) − sin ( A − B) (ii) Replace 1 + tan2 by sec2 and put tan = t .
2 2 2
(iii) 2 cos A ⋅ cos B = cos ( A + B) + cos ( A − B)
a sin x + b cos x
(iv) 2 sin A ⋅ sin B = cos ( A − B) − cos ( A + B) ● To evaluate integral of form∫ dx,
(v) 2 sin A ⋅ cos A = sin 2 A c sin x + d cos x
 1 + cos 2 A  d
(vi) cos2 A =   we write a sin x + b cos x = A ( c sin x + d cos x)
 2  dx
 1 − cos 2 A  +B( c sin x + d cos x)
(vii) sin2 A =  
 2  Where A and B can be found by equating the coefficient of
(viii) cos2 A − sin2 A = cos 2 A sin x and cos x on both sides.

(ix) sin2 A + cos2 A = 1 a sin x + b cos x + c


To evaluate integral of the form∫ dx.
p sin x + q cos x + r
3. Integration of Different Types d
We write a sin x + b cos x + c = A (p sin x + q cos x + r)
dx
of Functions + B (p sin x + q cos x + r) + C
● To evaluate integrals of the form ∫ sin p x cos q x dx Where A, B and C can be found by equating the coefficient of
sin x, cos x and the constant term.
Where p, q ∈ Q, we use the following rules : x2 + 1
(i) If p is odd, then put cos x = t ● To evaluate integrals of the form ∫ dx
x 4 + kx 2 + 1
(ii) If q is odd, then put sin x = t
x2 − 1
(iii) If both p, q are odd, then put either sin x = t or cos x = t or ∫ dx
(iv) If both p, q are even, then use trigonometric identities x 4 + kx 2 + 1
only. We divide the numerator and denominator by x2 and make
 p + q − 2
2
 1
(v) If p, q are rational numbers and   is a negative perfect square in denominator as  x ±  and then put
 2   x
integer, then put cot x = t or tan x = t as required. 1 1
x + = t or x – = t as required.
x x
dx
● To evaluate integrals of the form ∫ or
a + b cos 2 x ● Substitution for Some Irrational Integrand
dx dx a− x a+ x
∫ a + bsin 2
x
or ∫
a sin x + b cos 2 x
2
, (i)
a+ x
,
a− x
, x = a cos 2θ

dx dx x a+ x 1
∫ ( asin x + bcos x) 2 or ∫ a + bsin2 x + c cos 2 x (ii)
a+ x
,
x
, x (a + x),
x (a + x)
, x = a tan2 θ

or x = a cot2 θ
(i) Divide both the numerator and denominator by
cos2 x. x a− x 1
(iii) , , x (a − x), , x = a sin2 θ
(ii) Replace sec2 x by 1 + tan2 x in the denominator, if any. a− x x x (a − x)
(iii) Put tan x = t , so that sec2 x dx = dt or x = a cos2 θ
DAY 153

x x−a 1 e ax
, x ( x − a) , , x = a sec2 θ ∫e cos(bx + c)dx =
ax
(iv) , (iii)
x−a x x ( x − a) a + b2 2

{a cos (bx + c) + b sin (bx + c)} + k


dx  x − α Here, c and k are integration constant.
(v) ∫ ( x − α)(β − x) , ∫ 
β − x
 dx

∫ ( x − α )(β − x) dx, put x = α cos2 θ + β sin2 θ 5. Integration by Partial Fractions


P( x )
dx
To evaluate the integral of the form ∫ Q( x) dx, where P( x), Q( x)
(vi) ∫ ( px + q ) ax + b
, put ax + b = t 2
are polynomial in x with degree of P( x) < degree of Q( x) and
Q( x) ≠ 0, we use the method of partial fraction.
dx
(vii) ∫ (ax 2
+ bx + c) px + q
, put px + q = t 2 The partial fractions depend on the nature of the factors of
Q( x).
dx 1 (i) According to nature of factors of Q( x), corresponding
(viii) ∫ , put px + q = form of partial fraction is given below:
( px + q ) ax2 + bx + c t
If Q ( x) = ( x − a1 ) ( x − a2 ) ( x − a3 ) .... ( x − an ), then
dx 1 we assume that
(ix) ∫ ( px 2
+ q ) (ax + b )
2
first put x =
t P ( x) A1 A2 A3 An
= + + + ... + ,
and then a + bt 2 = z2 Q( x) ( x − a1 ) ( x − a 2 ) ( x − a3 ) ( x − an )
where the constants A1 , A2 , K , An can be determined
by equating the coefficients of like power of x or by
4. Integration by Parts substituting x = a1 , a2 , K , an .
(i) If u and v are two functions of x, then (ii) If Q ( x) = ( x − a)k ( x − a1 ) ( x − a2 ) K ( x − ar ), then we
 du  assume that
∫ uI vII dx = u ∫ v dx − ∫  dx ⋅ ∫ v dx dx P( x ) A1 A2 Ak
= + + ... +
We use the following preference in order to select the Q( x) ( x − a) ( x − a)2 ( x − a)k
first function B1 B2 Br
+ + + ... +
I → Inverse function ( x − a1 ) ( x − a2 ) ( x − ar )
L → Logarithmic function where the constants A1 , A2 ,..., Ak , B1 , B2 ..., Br can be
A → Algebraic function obtained by equating the coefficients of like power of
T → Trigonometric function x.
E → Exponential function (iii) If some of the factors in Q( x) are quadratic and
(ii) If one of the function is not directly integrable, then we non-repeating, corresponding to each quadratic factor
take it as the first function. ax2 + bx + c (non-factorisable), we assume the partial
Ax + B
(iii) If both the functions are directly integrable, then the fraction of the type 2 , where A and B are
ax + bx + c
first function is chosen in such a way that its derivative
vanishes easily or the function obtained in integral sign constants to be determined by comparing coefficients
of like powers of x.
is easily integrable.
(iv) If some of the factors in Q( x) are quadratic and
(iv) If only one which is not directly integrable, function is
repeating, for every quadratic repeating factor of the
there e.g. ∫ log x dx, then 1 (unity) is taken as second
type (ax2 + bx + c)k where ax2 + bx + c cannot be further
function. factorise, we assume
A1 x + A2 A3 x + A4 A2 k − 1 x + A2 k
+ + ... +
Some more Special Integrals Based on ax + bx + c (ax + bx + c)
2 2 2
(ax2 + bx + c)k
Integration by Parts If degree of P ( x) > degree of Q( x), then we first divide P( x) by
∫e { f ( x) + f ′ ( x)}dx = f ( x)e x + C P( x ) P ( x)
x
(i) Q( x) so that is expressed in the form of T ( x) + 1 , where
Q( x) Q( x)
e ax
∫e sin (bx + c) dx = P ( x)
ax
(ii)
a + b2
2 T ( x) is a polynomial in x and 1 is a proper rational function
Q( x)
{a sin (bx + c) − b cos (bx + c)} + k
(i.e. degree of P1 ( x) < degree of Q( x))
DAY PRACTICE SESSION 1

FOUNDATION QUESTIONS EXERCISE


dx x6 8 If ∫ x + x 2 + 5 dx = P {x + x 2 + 5} 3 / 2
1 If ∫ = p( x ), then ∫ x + x 7 dx is equal to
x + x7 j JEE Mains 2013
Q
+ + C, then the value of 3PQ is
(a) log | x | − p (x) + C (b) log | x | + p (x) + C x x2 + 5
(c) x − p (x) + C (d) x + p (x) + C (a) −1 (b) −4 (c) −3 (d) −5
x 3 −1 dx
2 ∫ ( x 4 + 1)( x + 1)
dx is equal to 9 ∫ cos x − sin x
is equal to

1 1
log(1 + x 4) + log(1 + x 3) + C x π
log tan  −  + C (b) log cot   + C
(a) 1 1 x
4 3 (a)
1 1 2 2 8 2  2
(b) log(1 + x 4) − log(1 + x 3) + C
4 3 x 3π x 3π
log tan  − log tan  +
1 1
1 (c)  + C (d)  +C
(c) log(1 + x 4) − log(1 + x) + C 2 2 8  2 2 8 
4
1
(d) log(1 + x 4) + log(1 + x) + C sin8 x − cos 8 x
4
10 ∫ 1 − 2 sin2 x cos 2 x dx is equal to
∫ ( x + 1)( x + 2) ( x + 3) dx is equal to
7
3 1
(a) sin2x + C (b) − sin 2 x + C
2
(x + 2) 10
(x + 2) 8
1
(a) − +C (c) sin 2 x + C (d) − sin2x + C
10 8 2
(x + 1)2 (x + 2)8 (x + 3)2
(b) − − +C ( 3 x + 2 − x 2 )( 6 1 − x 2 − x 2 )
2
(x + 2) 10
8 2 11 ∫ 3
1− x 2
dx ;x ∈( 0,1) equals
(c) +C
10 (a) 21/ 6 x + C (b) 21/12 x + C (c) 21/ 3 x + C (d) None of these
(x + 1) 2
(x + 2)8 (x + 3)2
(d) + + +C  cos 6x + 6 cos 4x + 15 cos 2x + 10 
2 8 2 12 ∫ 10 cos 2  dx = f ( x ) + C,
x + 5 cos x cos 3x + cos x cos 5x 
dx
4 The integral ∫ 3
equals then f (10) is equal to
x ( x 4 + 1) 4
2
j
JEE Mains 2015 (a) 20 (b) 10 (c) 2 sin10 (d) 2 cos10
1
 x + 1 4
1 2x 12 + 5x 9
13 The integral ∫
4
(a)   +C (b) (x 4 + 1) 4 + C dx is equal to
 x4  ( x 5 + x 3 + 1)3 j
JEE Mains 2016
1
− x5 x10
 x 4 + 1 4 +C +C
1
(a) (b)
(c) − (x + 1) + C
4 4
(d) −   +C (x + x 3 + 1)2
5
2 (x + x 3 + 1)2
5
 x4 
x5 − x10
(c) +C (d) +C
sin x 2 (x + x 3 + 1)2
5
2 (x + x 3 + 1)2
5
5 If ∫ dx = Ax + B log sin ( x − α ) + C,
sin( x − α )
x 2 −1
then the value of ( A , B ) is 14 ∫x 3
2x 4 − 2x 2 + 1
dx is equal to
(a) (sinα,cosα) (b) (cosα,sinα )
(c) (− sinα,cosα) (d) (− cosα,sinα) 2x 4 − 2x 2 + 1 2x 4 − 2x 2 + 1
(a) +C (b) +C
f (x ) x2 x
6 If ∫ dx = log log sin x + C, then f ( x ) is equal to 2x − 2x + 1
4 2
2x − 2x + 1
4 2
log sin x (c) +C (d) +C
2x 2x 2
(a) sinx (b) cos x
dx
(c) logsinx
2
(d) cot x 15 ∫ (1 + x 2
) p 2 + q 2 (tan−1 x )2
is equal to
 (log x − 1) 
7 ∫ 1 + (log x )2 

dx is eqal to
(a)
1
log[q tan−1 x + p 2 + q 2 (tan−1 x)2 ] + C
q
x xe x
(a) +C (b) +C (b) log[q tan−1 x + p 2 + q 2 (tan−1 x)2 ] + C
(log x)2 + 1 1+ x 2 2
x log x (c) (p 2 + q 2 tan−1 x)3 / 2 + C
(c) 2 +C (d) +C 3q
x +1 (log x)2 + 1
(d) None of the above
DAY 155

cos 8x + 1 x 2 −1
16 In the integral ∫ dx = A cos 8x + k , where 24 ∫x dx is equal to
cot 2x − tan 2x 4
+ x2 +1
k is an arbitrary constant, then A is equal to 1  x 2 + x + 1 1  x 2 − x − 1
j JEE Mains 2013 (a) log  2  +C (b) log  2  +C
2  x − x + 1 2  x + x + 1
1 1 1 1
(a) − (b) (c) (d) −
16 16 8 8  x 2 − x + 1 1  x 2 − x + 1
(c) log  2  +C (d) log  2  +C
(sin θ + cos θ )  x + x + 1 2  x + x + 1
17 ∫ sin 2 θ
dθ is equal to
x
j JEE Mains 2017
(a) log | cosθ − sinθ + sin 2 θ |
25 ∫ a −x3
3
dx is equal to

(b) log | sinθ − cosθ + sin 2 θ| 3/ 2 3/ 2

(a) sin−1 
x 2 −1  x 
+C (b) sin   +C
(c) sin−1 (sinθ − cosθ) + C a 3 a
(d) sin−1 (sinθ + cosθ) + C 3/ 2 2/ 3
3 −1  x  3 −1  x 
 f ( x ) ⋅ g ' ( x ) − f ' ( x ) ⋅ g ( x ) (c) sin   +C (d) sin   +C
a a
18 ∫ 
 f ( x ) ⋅ g( x )
 ((log g ( x ) − log f ( x )) dx is

2 2
26 If an anti-derivative of f ( x ) is e x and that of g ( x ) is cos x,
equal to
2 then ∫ f ( x ) cos x dx + ∫ g ( x ) e dx is equal to
x

 g (x)  1  g (x) 
(a) log  +C (b)   (a) f (x) ⋅ g (x) + C (b) f (x) + g (x) + C
 f (x)  2  f (x) 
2 2 (c) e x cos x + C (d) f (x) − g (x) + C
1   g (x)    g (x) 
(c)  log  + C (d) log  +C 27. If ∫ f ( x ) dx = Ψ( x ), then ∫ x 5f ( x 3 ) dx is equal to
2   f (x)    f (x) 
1 3
19 Let I n = ∫ tann x dx (n > 1). If (a) [x Ψ (x 3)] − ∫ x 2 Ψ (x 3)dx + C
3 j JEE Mains 2013
I 4 + I 6 = a tan5 x + bx 5 + C, where C is a constant of 1
(b) [x 3 Ψ (x 3)] − 3 ∫ x 3 Ψ (x 3)dx + C
integration, then the ordered pair (a, b ) is equal to 3
(a)  − , 1 (b)  , 0 (c)  , − 1 (d)  − , 0
1 1 1 1 1
(c) [x 3 Ψ (x 3) − ∫ x 2 Ψ (x 3)dx] + C
 5  5  5   5  3
1
tan x 2 (d) [x 3 Ψ (x 3)] − ∫ x 3 Ψ (x 3)dx + C
20 If ∫ dx = x − tan−1 3
1 + tan x + tan2 x A
 2 tan x + 1 1 − 6 cos 2 x f (x )
  + C, then the value of A is
28 If ∫ 6 2
dx = + C, then f ( x ) is equal to
 A  sin x cos x (sin x )6

(a) 1 (b) 2 (a) sinx (b) cos x (c) tanx (d) cot x

∫ tan
−1
(c) 3 (d) None of these 29 xdx is equal to j
NCERT Exemplar
3 11
− −
21 ∫ cos 7
x sin 7
x dx is equal to (a) (x + 1) tan−1 x − x + C (b) x tan −1 x − x + C
4 4 (c) x − x tan−1 x + C (d) x − (x + 1) tan −1 x + C
4
(a) log | sin7 x | + C (b) tan7 x + C
7 30 If I n = ∫ (log x )n dx , then I n + n I n − 1 is equal to
4 3
−7 −
(c) tan 7 x + C (d) log| cos 7 x | + C (a) x (log x)n (b) (x log x)n (c) (log x)n −1 (d) n (log x)n
4
dx 31 If ∫ f ( x ) dx = g ( x ), then ∫ f −1( x ) dx is equal to
22 ∫ 2 + sin x + cos x
is equal to
j
NCERT Exemplar (a) g −1(x) (b) xf −1(x) − g (f −1(x))
(c) xf −1(x) − g −1(x) (d) f −1(x)
tan (x / 2) + 1 −1  tan (x / 2) + 1
(a) 2 tan−1  + C (b) tan   +C
    ( x + 3) e x
2 2 32 ∫ ( x + 4) 2
dx is equal to
(c) 2 tan−1
tan (x / 2) 
 +C (d) None of these
 2  (a)
1
+C (b)
ex
+C
(x + 4) 2 (x + 4) 2
23 If the integral
ex ex
5 tan x (c) +C (d) +C
∫ tan x − 2 dx = x + a ln | sin x − 2 cos x | + k , x+ 4 x+ 3
x 2 − x + 1 cot −1 x
33 If ∫
−1
then a is equal to e dx = A ( x ) e cot x + C, then A ( x ) is
x +1
2
(a) − 1 (b) − 2
(c) 1 (d) 2 equal to j
JEE Mains 2013
(a) − x (b) x (c) 1− x (d) 1+ x
156

34 If g ( x ) is a differentiable function satisfying dx 3


d
35 ∫x 3
( x n + 1)
is equal to
{ g ( x )} = g ( x ) and g( 0) = 1, then
dx 3  xn  1  xn 
 2 − sin 2x  (a) ln  +C (b) ln  +C
n  x n + 1 n  x n + 1
∫ g ( x ) 1 − cos 2x  dx is equal to
3  x n + 1  x n +1 
(a) g (x) cot x + C (b) − g (x) cot x + C
(c) ln  +C (d) 3n ln n  + C
n  xn   x 
g (x)
(c) +C (d) None of these
1 − cos 2 x

DAY PRACTICE SESSION 2

PROGRESSIVE QUESTIONS EXERCISE


1 If ∫ f ( x ) dx = f ( x ), then ∫ {f ( x )} 2 dx is equal to 1 −1
(a) +C (b) +C
3 (1 + tan3 x) 3 (1 + tan3 x)
1 {f (x)}3 1 −1
(a) {f (x)}2 (b) {f (x)}3 (c) (d) {f (x)}2 (c) +C (d) +C
2 3 1 + cot 3 x 1 + cot 3 x
0 x 2 − sin x cos x − 2 (where C is a constant of integration)
2 If f ( x ) = sin x − x 2 0 1 − 2x ,  cos θ + sin θ 
2 − cos x 2x − 1 0 6 ∫ cos 2 θ log  cos θ − sin θ  dθ is equal to
then∫ f ( x ) dx is equal to  cosθ + sinθ 
(a) (cosθ − sinθ) 2 log   +C
 cosθ − sinθ 
3
x
(a) − x 2 sin x + sin 2 x + C
3  cosθ + sinθ 
x3 (b) (cosθ + sinθ) 2 log   +C
(b) − x 2 sin x − cos 2 x + C  cosθ − sinθ 
3
x3 (cosθ − sin θ) 2  cosθ − sinθ 
(c) − x 2 cos x − cos 2 x + C (c) log   +C
3 2  cosθ + sinθ 
(d) None of the above π
sin 2 θ logtan  + θ − log sec 2 θ + C
1 1
(d)
1 − cos 2a x 2 4  2
∫e
2 ax
3 dx is equal to
1 + sin 2a x
x2
(a)
π
− e 2ax cos  + ax  + C
1 7 ∫ (x sin x + cos x ) 2
dx is equal to
a 4 
π sin x + cos x
(b) − e cot  + ax  + C
1 2ax  (a) +C
2a 4  x sin x + cos x
π x sin x − cos x
e cos  + ax  + C
 +C
1 2ax (b)
(c) −
2a 4  x sin x + cos x
π sin x − x cos x
− e cosec  + ax  + C

1 2ax
(d) (c) +C
a 4  x sin x + cos x
4 If x 2 ≠ nπ − 1, n ∈ N. Then, the value of (d) None of the above
2 sin ( x + 1) − sin 2 ( x + 1)
2 2
x 2 dx
∫x 2 sin ( x 2 + 1) + sin 2 ( x 2 + 1)
dx is equal to 8 If f ( x ) = ∫
(1 + x )(1 + 1 + x 2 )
2
and f ( 0) = 0, then the

1  x 2 + 1 value of f (1) is
(a) log sec (x 2 + 1) + C (b) log sec   +C π
2  2  (a) log (1 + 2) (b) log (1 + 2) −
1 4
(c) log | sec (x 2 + 1) | + C (d) None of these π
2 (c) log (1 + 2) + (d) None of these
2
5 The integral
sin2 x cos 2 x dx 1+ x
∫ (sin dx 9 If I = ∫ =k 3 + C, then k is equal to
5
x + cos x sin2 x + sin3 x cos 2 x + cos 5 x )2
3 3
( x + 1) ( x − 1)
2 4 1− x
is equal to j
JEE Mains 2018 (a) 2/3 (b) 3/2
(c) 1/3 (d) 1/2
DAY 157

dx
10 ∫ (sin x + 2)(sin x − 1) is equal to 1
 1 x+
12 The integral ∫ 1 + x −  e x
dx is equal to
  1   x
2 tan + 
x JEE Mains 2014
 
j

2 2 −1 2 2 
− +C
1 1
(a) tan  x+ x+
(a) (x − 1) e +C +C
3  tan − 1 3 3
x 3
x
(b) xe x
 
   
1 1
x+ x+
2 (c) (x + 1) e x
+C (d) − xe x
+C
2
(b) +C
 tan x + 1
∫ (sin(101 x ) ⋅ sin
99
  13 x ) dx is equal to
 2 
 2 tan x − 1
100
sin(100x) (sin x) cos(100 x) (sin x)100
(a) +C (b) +C
2 1 2 −1  2  100 100
(c) − + tan   +C
3  tan x − 1 3 3  3 
cos(100 x)(cos x) 100
cos(100 x)(cos x) 100
  (c) +C (d) +C
 2    100 100
 2 tan x − 1
2 2 2  2  ( 2018)2 x
(d) − + tan−1   +C ∫
−1 x
14 ( 2018)sin ( 2018 ) dx is equal to
3  tan x − 1 3 3  3  1 − ( 2018)2 x
 
 2    −1
( 2018 ) x
(a) (log2018 e)2 (2018)sin +C
2
x x + sin −1 ( 2018 ) x
+C

2
11 dx is equal to (b) (log2018 e) (2018)
( 2 + 3 x 2 )5 / 2 2
(c) (log2018 e) (2018) x − sin −1 ( 2018 ) x
+C
3/ 2 −1
1 x2 
x
(2018)sin ( 2018)
(a)  +C (d) +C
5  2 + 3 x 2  (log2018 e)2

1 x2 
3/ 2
 2 1
∫ (∫ e log x + − 2  ) dx is equal to
x
(b) +C 15
6  2 + 3 x 2   x x 
1
1 x2 
7/ 2
(a) e x log x + C1x + C2 (b) logx + + C1x + C2
(c) +C x
6  2 + 3 x 2  (c)
logx
+ C1x + C2 (d) None of these
x
(d) None of the above

ANSWERS
SESSION 1 1 (a) 2 (c) 3 (a) 4 (d) 5 (b) 6 (d) 7 (a) 8 (d) 9 (d) 10 (b)
11 (a) 12 (a) 13 (b) 14 (d) 15 (a) 16 (a) 17 (c) 18 (c) 19 (b) 20 (c)
21 (c) 22 (a) 23 (d) 24 (d) 25 (b) 26 (c) 27 (a) 28 (c) 29 (a) 30 (a)
31 (b) 32 (c) 33 (b) 34 (b) 35 (a)

SESSION 2 1 (a) 2 (d) 3 (b) 4 (b) 5 (b) 6 (d) 7 (c) 8 (b) 9 (b) 10 (a)
11 (b) 12 (b) 13 (a) 14 (a) 15 (a)
158

Hints and Explanations


x6 x6 6 We have, ∫
f ( x) = ∫ (sin 4 x − cos 4 x )dx
1 Let I = ∫x+ x7
dx = ∫ x (1 + 6
x )
dx
log sin x
= log(log sin x ) + C
= ∫ (sin2 x − cos 2 x )dx
d
(1 + x 6 ) − 1 ∴ f ( x) = (log sin x )
=∫
x (1 + x 6 )
dx dx = ∫ − cos 2x dx
 f '( x ) 
Q ∫ f ( x ) dx = log( f ( x )) + C 
dx dx sin2 x
⇒ I =∫ −∫ =− +C
x x + x7   2
= log| x| − p( x ) + C = cot x ( x + 2 − x2 )( 1 − x 2 − x2 )
3 6

x −1
3 2 11 Let I = ∫ dx
2 Let I = ∫ (x  (log x − 1)  1 − x2
3
dx
4
+ 1)( x + 1) 7 ∫ 1 + (log x )2  dx  1 
x + 2 − x 2  6 (2 − 2 x 2 − x 2 )
3
x3 + x 4 − x 4 − 1
= ∫ dx (log x ) + 1 − 2 log x
2
= ∫
 2 
( x 4 + 1)( x + 1) = ∫ [(log x ) 2 + 1] 2
dx 3
1 − x2
dx
x ( x + 1) − ( x + 1)
3 4
= ∫ ( x 4 + 1)( x + 1)
dx
(log x )2 + 1 − 2 x  log x ⋅ 
1 3
x + 2− x2
 x
 x3 1  =∫ dx  [ x 2 + ( 2 − x2 )2 − 2 x 2 − x 2 ] 
= ∫  x 4

+ 1 x + 1 
dx [(log x ) 2 + 1]2 6



d  x   2 
=
1
log( x 4 + 1) − log( x + 1) + C
= ∫ dx  (log x ) 2
+ 1 
dx = ∫ dx
4
3
1− x2
x 3
x + 2− x2
3
2− x2 − x
3 Let I = ∫ ( x + 1)( x + 2) 7 ( x + 3)dx = +C
(log x ) 2 + 1 = ∫ 21/6 3
1 − x2
dx
Put x + 2 = t
⇒ x = t − 2 and dx = dt 8 Let I = ∫ x+ x2 + 5 dx 3
(2 − x 2 ) − x 2
∴ I = ∫ (t − 1)t 7 (t + 1)dx
Put x + x + 5=t
2
= ∫ 21 / 6 3 1 − x 2
dx

= ∫ (t − 1)⋅ t dx = ∫ (t − t )dx
2 7 9 7
⇒ x2 + 5 = t − x
= 21 / 6 ∫ dx = 21 / 6 x + C
t 10
t ( x + 2)
8
( x + 2) 10 8
⇒ x2 + 5 = t 2 + x2 − 2 xt
= − +C= − +C
10 8 10 8 ⇒ 5 = t 2 − 2 xt  cos 6 x + 6cos 4 x 
⇒ 2 xt = t 2 − 5  
4 ∫ dx
=∫
dx + 15 cos 2 + 10
12 Let I = ∫ 
x  dx
x =  t − 
1 5
⇒  10cos x + 5cos x cos 3 x 
3 3
2
x ( x + 1) x 5  1 + 4 
2 4 4 1 4 2 t  
 x   + cos x cos 5x 
dx =  1 + 2 
1 5
and
1 2 t  (cos 6 x + cos 4 x ) + 5 (cos 4 x
Put 1 + = t4
x4 + cos 2 x ) + 10(cos 2 x + 1)
⋅  1 + 2  dt
1 5 =∫
4 Now, I = ∫t 1 /2 dx
⇒ − 5 dx = 4t 3dt 2 t  10cos 2 x + 5cos x cos 3 x
x 1 1 /2 + cos x cos 5x
2∫
= (t + 5t −3 / 2 )dt
dx 2cos 5x ⋅ cos x + 10 ⋅ cos 3 x
⇒ = − t 3dt
x5 12 10  1 3 /2 5
=  t 3 /2 − +C= t − +C ⋅ cos x + 10 ⋅ (2cos 2 x )
Hence, the integral becomes 23 t 3 t = ∫ 10cos 2 x + 5cos x ⋅ cos 3 x
dx = 2∫ dx
− t 3dt Clearly, 3PQ = −5
∫ t 3 = − ∫ dt = − t + C + cos x cos 5x
1/4 1/4 9 Let I = 1 ∫ dx
= 2x + C
 x4 + 1
= −  1 + 4  2  1 cos x − 1 sin x 
1
+ C = −  +C   Clearly, f (10) = 20
 x   x4   2 2 
2 x12 + 5x 9
5 Let I = ∫ sin x dx =
1  x + π  dx
∫  4 
sec
13 Let I = ∫ (x 5
+ x3 + 1) 3
dx
sin( x − α ) 2
2 x12 + 5x 9
Put x − α = t ⇒ dx = dt =
1 π
log tan  +
x π
+  + C
= ∫x 15
(1 + x − 2 + x − 5 ) 3
dx
sin (t + α ) 2 4 2 8
∴I =∫ dt 2 x − 3 + 5x − 6
sin t =
1 x 3π
log tan  +  +C
= ∫ (1 + x − 2 + x − 5)3
dx
cos t 2 2 8 
= ∫ cos α dt + ∫ sin α ⋅ sin t dt Now, put 1 + x − 2 + x − 5 = t
sin x − cos x
8 8
⇒ (− 2 x − 3 − 5x − 6 ) dx = dt
= cos α ⋅ t + sin α log sin t + C 10 Let I = ∫ 1 − 2 sin 2
x cos 2 x
dx
⇒ (2 x − 3 + 5x − 6 ) dx = − dt
= x cos α + sin α dt
(sin 4 x + cos 4 x ) (sin 4 x − cos 4 x ) ∴ I = − ∫ 3 = − ∫ t − 3 dt
log {sin( x − α )} + C I = ∫ (sin2
x + cos 2 x )2 − 2sin2 x cos 2 x
dx t
∴ A = cos α, B = sin α
DAY 159

t −3 +1 1 g( x) cosec2 x cosec2 x
=−
−3+ 1
+C= 2 +C
2t
Put
f ( x)
=t = ∫  cos
3 /7
x
dx = ∫ cot
3 /7
x
dx
 
x10 f ( x )⋅ g '( x ) − g ( x )⋅ f '( x )  sin3 / 7 x 
= +C ⇒ dx = dt
2( x + x3 + 1) 2
5 ( f ( x )) 2 Put cot x = t ⇒ − cosec2 x dx = dt
x2 − 1 f ( x )⋅ g '( x ) − g ( x ) ⋅ f '( x ) dt −7 4 / 7
⇒ I = − ∫ 3 /7 =
14 Let I = ∫x
2x4 − 2x 2 + 1
3
dx
f ( x )⋅ g ( x )

t 4
t +C

x2 − 1 g( x) 7
=∫ dx ⋅ ⋅ dx = dt = − tan − 4 / 7 x + C
2 1 f ( x) 4
x5 2 − 2 + 4
 f ( x )⋅ g '( x ) − g ( x )⋅ f '( x ) dt dx
x x ⇒   dx = 22 Let I = ∫
1 1  f ( x )⋅ g ( x )  t 2 + sin x + cos x

=∫ x3 x 5 (log t )2 dx
dx 1 ⇒ I =∫
2
2− 2 + 4
1 Now, I = ∫ t ⋅ log t dt = 2
+C
2 tan
x
1 − tan2
x
x x 2
2+ 2 + 2
1  g ( x )  x x
2 1
Now, putting 2 − 2 + 4 = t, we get =  log   + C 1 + tan2 1 + tan2
x x 2  f ( x )  2 2
x
4 3 − 5  dx = dt
1 1 sec2 dx
19 We have, I n = ∫ tan n x dx
x x  =∫ 2
x x x
∫ tan ∫ tan
n +2
1 dt ∴ I n + I n +2 = n
x dx + x dx 2 + 2 tan2 + 2 tan + 1 − tan2
∴I = ∫ 2 2 2
4 t = ∫ tan n
x(1 + tan x ) dx
2
x
sec2 dx
1 2x4 − 2x 2 + 1
= ⋅2 t + C = +C = ∫ tan n
x sec2 x dx I =∫ 2
4 2x 2 x x
tann +1
x tan2 + 2 tan + 3
15 Put q tan −1 x = t = +C 2 2
n+1 x 1 x
q 1 dt Put tan = t ⇒ sec2 dx = dt
⇒ dx = dt ⇒ dx = tan 5 x 2 2 2
1 + x2 1 + x2 q Put n = 4, we get I 4 + I 6 = +C
2dt
5 ∴I = ∫ 2
∴∫
dt 1
= log [ t + p2 + t 2 ] 1 t + 2t + 3
∴ a= and b = 0
q p2 + t 2 q 5 dt 2dt
= 2∫ 2 = 2∫
tan x t + 2t + 1 + 2 (t + 1)2 + ( 2 ) 2
=
1
log [q tan −1 x 20 Let I = ∫
q 1 + tan x + tan2 x 1  t + 1
= 2⋅ tan −1   +C
+ p + q (tan
2 2 −1
x) ] + C
2 sin x 2  2 
2 dx = ∫ cos x dx  tan x + 1 
2cos 4 x sin2 x  

16 LHS = sin x
dx 1+ + ⇒ I = 2 tan  −1 2  +C
cos 2 2 x − sin2 2 x cos 2 x cos x
 2 
cos 2 x sin 2 x sin 2 x  
=∫ dx
2cos 2 4 x × cos 2 x sin 2 x 2 + sin 2 x 5 tan x
= ∫ dx 23 Given, ∫ dx
cos 4 x = ∫ dx − 2 ∫
dx tan x − 2
1 2 + sin 2 x
= ∫ cos 4 x × sin 4 xdx = ∫ sin 8 xdx = x + a ln|sin x − 2 cos x | + k …(i)
2 sec2 x
−1 cos 8 x = x−2∫ dx Now, let us assume that
= + k 2 sec x + 2 tan x
2
5 tan x
2 8 I =∫ dx
−1 Let tan x = t tan x − 2
Hence, we get A =
16 ⇒ sec2 x dx = dt On multiplying by cos x in numerator
sin θ + cos θ 2
= x− ∫ 2
dt and denominator, we get
17 Let I = ∫ dθ 5 sin x
1 − (1 − 2sin θ cos θ ) 2 t +t +1 I =∫ dx
dt sin x − 2 cos x
sin θ + cos θ = x−∫
= ∫ 1 − (sin θ − cos θ) 2

t + 1 +  3 
2 2 Let 5 sin x = A (sin x − 2 cos x )
    + B (cos x + 2 sin x )
 2  2 
Put sin θ − cos θ = t ⇒ 0 ⋅ cos x + 5 sin x = ( A + 2B ) sin x
⇒ (cos θ + sin θ) dθ = dt 2  2 tan x + 1  + (B − 2 A ) cos x
⇒I = x− tan −1   +C
dt 3  3  On comparing the coefficients of sin x
∴ I =∫ = sin −1 ( t ) + C
1 − t2 Hence, we get A = 3. and cos x, we get
A + 2B = 5 and B − 2 A = 0
−1
= sin (sin θ – cos θ) + C − 3  −11 
21 Here, m + n = +   = −2 ⇒ A = 1 and B = 2
7  7 
 f ( x ) ⋅ g '( x ) − f '( x )⋅ g ( x ) ⇒ 5 sin x = (sin x − 2 cos x )
18 Let I = ∫   I = ∫ cos −3 /7 x(sin( − 2 + 3 /7 ) x )dx
 f ( x )⋅ g ( x )  + 2 (cos x + 2 sin x )
= ∫ cos −3 /7 x sin −2 x sin3 /7 xdx
 g ( x )
log   dx
 f ( x )
160

5 sin x 1 − 6cos 2 x 1
⇒I = ∫ sin x − 2 cos x dx 28 Let I = ∫ sin6
x cos 2 x
dx = ex ⋅
x+ 4
+C

(sin x − 2cos x ) + 2 (cos x + 2 sin x ) 1 dx [Q∫ e ( f ( x ) + f '( x )dx = e x ⋅ f ( x ) + C ]


x
= ∫ dx =∫ dx − 6∫
(sin x − 2 cos x ) sin 6 x cos2 x sin 6 x
= I1 − I2  x2 + 1
x 
33 LHS = ∫  2
−1
d (sin x − 2 cos x ) (say) − 2 cot x
⇒ I = ∫ 1 dx + 2 ∫ e dx
(sin x − 2 cos x ) sec2 x  x + 1 x + 1
Here, I1 = ∫ dx
sin 6 x x
= ∫ 1 ⋅ e cot ∫x
−1 −1
I = x + 2 log|(sin x − 2 cos x )| + k x
dx − e cot x dx
1 1 2
+1
…(ii) =∫ ⋅ sec2 xdx = ⋅ tan x
sin 6 x sin 6 x 
where, k is the constant of integration. 1 
− ∫ x ⋅ e cot
−1 −1
(−6) = xe cot x x
−  dx
On comparing the value of I in Eqs. (i) −∫ ⋅ cos x tan xdx  1+ x 
2

and (ii), we get a = 2 sin7 x


x
∫1+
−1 −1
tan x tan x − e cot x
dx + C = xe cot x
+C
1 − 1  = + I2 ⇒ I1 − I2 = +C
  sin 6 x sin 6 x x2
 x2 
24 ∫ 1
2
dx Thus, I =
tan x
+C ∴ A( x ) = x
+ − sin 6 x


x 
x
1
Hence, f ( x ) = tan x 34 We have, d {g ( x )} = g ( x )
dx
Put x + = t ⇒  1 − 2  dx = dt
1 1 ⇒ g ′( x ) = g ( x )
29 Let I = ∫ tan −1 x (1) dx
x  x 
= tan −1 x ∫ 1dx g ′( x )
dt 1  t − 1 ⇒ ∫ dx = ∫ 1 dx
∴ ∫ t 2 − 1 = 2 log  t + 1  + C g( x)
−  ∫ (tan −1 x )∫ (1) dx dx
d
 dx  ⇒ log e {g ( x )} = x + log C1
1  x2 − x + 1 
= log  2  +C ⇒ g ( x ) = C1 e x
2  x + x + 1 = tan −1 x⋅x−∫
1
×
1
. xdx
1+ x 2 x Now, g(0) = 1 ⇒ C1 = 1
25 Let I = ∫ 3 x 3 dx ∴ g( x) = e x
1 x
a −x = x tan −1
x− ∫ dx
2 (1 + x ) x  2 − sin 2 x 
Put x = a(sin θ) 2 / 3 ∴ ∫ g ( x )  1 − cos 2 x  dx
2 Put x = t 2 ⇒ dx = 2t ⋅ dt
⇒ dx = a(sin θ)−1 / 3 cos θ dθ = ∫ e (cosec x − cot x ) dx
x 2
3 1 t2
2 ∫ (1 + t 2 ) ⋅ t
∴ I = x tan −1 x− 2t dt
a1 / 2 (sin θ) 1 / 3 ⋅
2 = − e cot x + C
x

3 2 = − g ( x ) cot x + C
t
a(sin θ)−1 /3 ⋅ cos θ = x tan −1 x−∫ dt
∴ I =∫ dθ 1 + t2 dx3 3 x2 dx
a3 − a3 sin2 θ
35 Let I = ∫ x (x
n
+ 1)
3
dx = ∫ 3 n
x ( x + 1)
dx
= x tan −1 x − t + tan −1 t + C
2 a3 / 2 ⋅ cos θ 2 dx x n −1 dx
= ∫ 3 /2 dθ = ∫ dθ = x tan −1 x − x + tan −1 x + C = 3∫ = 3∫ n n
3 a cos θ 3 x( x + 1)
n
x ( x + 1)
2 −1  x 
3 /2 = ( x + 1)tan −1 x − x+C
2
= θ + C = sin   + C On putting x n = t , we get
3 3  a 30 I n = ∫ ( log x ) dx = x (log x ) 3 dt 3 1 1 
I = ∫
n t (t + 1) n ∫  t t + 1 
n n
= − dt
∫ f ( x ) cos x dx + ∫ g ( x ) e dx 1
x
26 − n ∫ ( log x ) n –1 ⋅ ⋅ x dx 3
e x x = [log t − log(t + 1)] + C
= (cos x + sin x ) ∴ I n + n I n − 1 = x (log x ) n n
2
= log 
3 t 
x
31 Consider, ∫ f −1 ( x )dx = ∫ f −1 ( x )⋅1dx +C

e
(sin x − cos x ) + C n  t + 1
2 d −1  xn 
= f −1 ( x )⋅ x − ∫( f ( x ))⋅ xdx =
3
ex dx
log  n +C
= (2cos x ) + C n  x + 1
−1
2 Now, let f ( x ) = t , then
= e cos x + C
x
d −1
( f ( x )) =
dt
dx dx
SESSION 2
27 Given, ∫ f ( x ) dx = Ψ( x )

d −1
( f ( x ))⋅ dx = dt 1 We have, ∫ f ( x ) dx = f ( x)
Let I = ∫ x 5 f ( x3 ) dx dx

d
{ f ( x )} = f ( x )
Put x3 = t ∴ ∫ f −1 ( x )dx = x ⋅ f −1 ( x ) − ∫ f (t )dt
dx
⇒ x dx =
2 dt
…(i) [Qf −1 ( x ) = t ⇒ x = f (t )] ⇒
1
d [ f ( x ) ] = dx
3 = x ⋅ f −1 ( x ) − g (t ) f ( x)
1 1
∴ I = ∫ t f ( t ) dt = [t Ψ (t ) − ∫ Ψ (t ) dt ] = x ⋅ f −1 ( x ) − g ( f −1 ( x )) ⇒ log { f ( x )} = x + log C
3 3
1
= [ x3 Ψ ( x3 ) − 3 ∫ x2 Ψ( x3 ) dx] + C
32 Let I = ∫ e  x + 3 2  dx
x ⇒ f ( x ) = Ce x
 ( x + 4)  ⇒ { f ( x )}2 = C 2e 2 x
3
[from Eq. (i)]  x + 4 − 1
= ∫ex  dx
∴ ∫ { f ( x )}2 dx = ∫ C 2e 2 xdx
1 3  ( x + 4) 2  C 2e 2 x 1
= x Ψ ( x ) − ∫ x Ψ ( x ) dx + C
3 2 3
= = { f ( x )}2
3  1 1  2 2
= ∫ex −  dx
 x + 4 ( x + 4) 2 
DAY 161

2 We have, 1 − cos ( x2 + 1) cos x − x (− sin x )


0 x − sin x cos x − 2
2 = ∫x 1 + cos ( x2 + 1)
dx
cos 2 x
dx

−x
f ( x ) = sin x − x2 0 1 − 2x  x2 + 1  = + tan x + C
2 − cos x 2x − 1 0
= ∫ x tan  2 
 dx ( x sin x + cos x )cos x
− x + x sin2 x + sin x ⋅ cos x
sin x − x2 2 − cos x = +C
0  x2 + 1   x2 + 1  ( x sin x + cos x ) ⋅ cos x
⇒ f ( x ) = x − sin x
2
0 2x − 1
∴ ∫ tan   d
2   2 

sin x − x cos x
= +C
cos x − 2 1 − 2x 0  x2 + 1  x sin x + cos x
= log sec   +C
 2  x 2 dx
[interchanging rows and columns]
⇒ f ( x ) = (− 1)3
8 f ( x) = ∫ (1 + x )(1 +
2
1 + x2 )
5 We have,
0 x2 − sin x cos x − 2 sin x ⋅ cos x
2 2

sin x − x 2
0 1 − 2x
I = ∫ (sin 5
x + cos 3 x ⋅ sin2 x
dx Put x = tanθ ⇒ dx = sec2 θ dθ
= (1 + x2 ) dθ
2 − cos x 2x − 1 0 + sin3 x ⋅ cos 2 x + cos 5 x )2 tan2 θ sec 2 θ
[taking (−1) common from each column] sin2 x cos 2 x
∴ f ( x) = ∫ sec θ(1 + sec θ)dθ
2

⇒ f ( x) = − f ( x) ⇒ f ( x) = 0
= ∫ {sin x(sin2 x + cos2 x ) +
3
dx
1 − cos 2 θ
∴ ∫ f ( x ) dx = ∫ 0 dx = C cos 3 x(sin2 x + cos2 x )}2 = ∫ cos θ (1 + cos θ) dθ
1 − cos 2ax sin2 x cos 2 x = ∫ sec θ dθ – ∫ dθ
3 Let I = ∫ e 2 ax dx = ∫ (sin
3
x + cos 3 x )2
dx
1 + sin 2ax
2
sin x cos x2 = log (sec θ + tan θ) – θ + C
1 2 t 1 − cos 2t =∫ dx
a∫
⇒I = e dt , [where, ax = t ] cos 6 x(1 + tan3 x )2 = log ( x + 1 + x2 ) − tan – 1 x + C
1 + sin 2t
tan2 x sec2 x ⇒ f (0) = log (0 + 1 + 0)
1 2t = ∫ (1 + dx
a∫
⇒ I = e tan3 x )2 − tan −1 (0) + C
π π Put tan x = t 3 ⇒ C=0
1 − 2 sin  + t  ⋅ cos  + t  π
4  4  ⇒ 3 tan2 x sec2 xdx = dt ∴ f (1) = log (1 + 2) – +0
dt 1 dt −1 4
2  π  ∴I = ∫ ⇒I = +C
2 sin  + t  dx
4  3 (1 + t )2 3(1 + t ) 9 Let I = ∫ 2
−1  x + 1
1 ⇒ I = +C (1 − x )2
  3
⇒ I = ∫ e2 t 1 − x
a 3(1 + tan3 x )
1+ x 2
 1 cosec2  π + t  − cot  π + t   dt 6 Since, Put =t ⇒ dx = dt
     1− x (1 − x ) 2
2 4  4   cos θ + sin θ  π 
log   = log tan  + θ 1 dt 3
π  cos θ − sin θ  4  ∴ I = ∫ 2 /3 = [ t 1 /3 ] + C
e 2 t cosec2  + t  dt
1
2a ∫ I
⇒ I = 2 t 2
II 4   π + θ 3 1+ x 
π
and ∫ sec θ dθ = log tan  4

2 =  + C
− ∫ e 2 t cot  + t  dt
1 2 1− x 
4  π
⇒ ∫ sec 2θ dθ = log tan  + θ
a 1
3
π 2 4  ∴ k =
e cot  + t  + ∫ e 2 t
1 2t 1
⇒ I =− 2
π
4  log tan  + θ

2a a d dx
2sec 2θ =
π dθ 4  10 ∫ (sin x + 2) (sin x − 1)
cot  + t  dt − ∫ e 2 t
1
4  1  π 
a ∴I = sin 2θ log tan  + θ − ∫ tan2θ dθ 1 dx 1 dx
3 ∫ (sin x − 1) 3 ∫ (sin x + 2)
2 4  = −
π
cot  + t  dt + C π
sin 2θ log tan  + θ
4  1
= 1 dx
2 4  = ∫
π 3  x 
e cot  + t  + C
1 2t
⇒ I =− 1
− log sec 2θ + C  2 tan 
2a 4   2 − 1
2
π  1 + tan2 x 
e cot  + ax  + C

1 2 ax
∴ I =− x2  2 
2a 4  7 Let I = ∫ ( x sin x + cos x ) 2
dx
1 dx
4 We have, − ∫
x cos x x 3  x 
2 sin ( x 2 + 1) − sin 2 ( x 2 + 1)
= ∫ ( x sin x + cos x ) 2

cos x
dx  2 tan


2 + 2
∫ x
2 sin ( x 2 + 1) + sin 2( x 2 + 1)
dx
 1 + tan2 x 
Q d ( x sin x + cos x ) = x cos x   2 
2 sin ( x 2 + 1) − 2 sin ( x 2 + 1)  dx  x
Put tan = t
⋅ cos ( x2 + 1) −1 x 2
∴ I =
= ∫x 2 sin ( x2 + 1) + 2 sin ( x 2 + 1)
dx .
( x sin x + cos x ) cos x 1 2 x
⇒ sec dx = dt
⋅ cos ( x 2 + 1) 1
+ ∫ 2 2
( x sin x + cos x ) 1 2dt 1 2dt
3 ∫ 2t − 1 − t 2 3 ∫ 2t + 2t 2 + 2
∴ −
162

2 dt 1 dt  1  x+x
1

3 ∫ (t − 1)2 3 ∫ 
=− − (2018) 2 x
∫  1 + ∫
−1
x−
x
2 12 e dx 14 Let I = ⋅ (2018) sin (2018 ) dx
 3 x
2
1 1 − (2018) 2 x
t +  +   1 1
 2  2  x+
 1  x+x
= ∫e x
dx + ∫ x  1 − e dx (2018)x

−1
(2018 )x
x2  = ⋅ (2018) sin
t + 1 1 − (2018) 2 x
  1 1 1
2 1 1 2  2 = ∫e
x+
dx + x e
x+ d x+
− ∫ ( x ) e x dx
= − tan −1 +C x x
Put sin −1 (2018)x = t
3 (t − 1) 3 3 3 dx
1 1 1
x+ x+ x+ 1
2 = ∫e x
dx + xe x
− ∫e x
dx ⇒
=
2 1

2
tan −1 1 − (2018x ) 2
3  x  3 3   1  x+x x+ 
1 1

 tan − 1 Q ∫  1 − x2  e dx = e x  ⋅(2018)x l n(2018)dx = dt


 2    1
ln (2018) ∫
 tan x + 1  ∴I = (2018) t dt
  = ∫ e x +1 / xdx + xe x +1 / x − ∫ e x + 1 / xdx
 2 2
+C x+
1 1 (2018)t
3 = xe x
+C = ⋅ +C
ln (2018) ln 2018
2
2 13 Let I = ∫ (sin(101 x )⋅sin 99 x )dx (2018) t
= = +C
3  tan − 1 = ∫ sin(100 x + x )sin ln2 (2018)
x 99
xdx
 2  −1
(2018 ) x
= ∫ (sin(100 x ) ⋅ cos x + cos(100 x ) = (log 2018 e ) 2 ⋅ (2018) sin +C
  x 1 
2 tan + 
2   2 2  ⋅ sin x )sin 99
xdx 15 Let I = ∫ (∫ e x (log x + 2 − 12 )dx )dx
− tan −1  +C
3 3 3 = ∫ sin 100 x ⋅ (cos x ⋅ sin 99 x )dx x x
  I II
= ∫ (∫ e x  log x + + − 2  dx )dx
  1 1 1
+ ∫ cos(100 x ) ⋅ sin 100
xdx
 x x x 
x2
11 ∫ (2 + 3 x2 )5/2 dx 
= sin(100 x )⋅
sin100 x
− ∫ cos(100 x )   log x + 1  dx + e x  1 − 1   dx
 = ∫  ∫ e ∫  x x2  
x
 100  
On substituting 2 + 3 x2 = t 2 x2  x
sin100 x 
dx + ∫ cos(100 x ) ⋅ sin100 xdx = ∫  e x logx + e x + C1  dx
2 ⋅100 ⋅ 1
⇒ x2 = 2 
( t − 3) 100   x 
2t sin(100 x ) ⋅ sin100 x [Q∫ e x ( f ( x ) + f '( x ))dx = e x ⋅ f ( x ) + C ]
∴ dx = − dt =
x ( t − 3)2
2 100
= ∫ e x  logx +  dx + ∫ C1 dx
1
− ∫ sin100 x ⋅ cos(100 x )dx
x2  − 2t  dt  x
∴∫ ⋅  dt = − 2∫ 4 + ∫ cos(100 x ) ⋅ sin100 xdx
( tx )5  x( t 2 − 3)2  4t = e x log x + C1 x + C2
3 /2 sin(100 x ) ⋅ sin100 x
1 dt 1 1 x  2 = +C
2 ∫ t 4 6t 3
=− = +C=   +C 100
6  2 + 3 x2 
DAY SIXTEEN

Definite
Integrals
Learning & Revision for the Day
u Concept of Definite Integrals u Walli’s Formula u Definite Integration as the Limit
u Leibnitz Theorem u Inequalities in Definite Integrals of a sum

Concept of Definite Integrals


d
Let φ ( x) be an anti-derivative of a function f ( x) defined on [a, b ] i.e. [φ ( x)] = f ( x). Then,
dx
b
definite integral of f ( x) over [a, b ] is denoted by ∫ a
f ( x) dx and is defined as [φ (b ) − φ (a)] i.e.
b

∫ a
f ( x) dx = φ (b ) − φ (a). The numbers a and b are called the limits of integration, where a is

called lower limit and b is upper limit.

NOTE • Every definite integral has a unique value.


• The above definition is nothing but the statement of second fundamental theorem of integral PRED
calculus.
MIRROR
Your Personal Preparation Indicator
Geometrical Interpretation of Definite Integral
b u No. of Questions in Exercises (x)—
In general, ∫ f ( x)dx represents an algebraic sum of areas of the region bounded by the curve
a
u No. of Questions Attempted (y)—
u No. of Correct Questions (z)—
y = f ( x), the X -axis, and the ordinates x = a and x = b as show in the following figure. (Without referring Explanations)

u Accuracy Level (z / y × 100)—


Y′
y=f(x) u Prep Level (z / x × 100)—

In order to expect good rank in JEE,


X′ X your Accuracy Level should be
Y a b above 85 & Prep Level should be
above 75.
164

(xi) Some important integrals, which can be obtained with


Evaluation of Definite Integrals by the help of above properties.
Substitution π /2 π /2
π  1
When the variable in a definite integral is changed due to
(a) ∫ log sin xdx = ∫ log cos xdx = 2 log  2 .3
0 0
π /4
substitution, then the limits of the integral will accordingly be π
changed. (b) ∫
0
log(1 + tan x)dx =
8
log 2.
For example, to evaluate definite integral of the form
b (xii) If a function f ( x) is discontinuous at points x1 , x2 ,..., x n
∫ a
f [ g ( x )] ⋅ g ′ ( x ) dx, we use the following steps in (a, b ), then we can define sub-intervals
(a, x1 ),( x1 , x2 ),...,( x n −1 , x n ), ( x n , b ) such that f ( x) is
Step I Substitute g( x) = t so that g ′ ( x) dx = dt
continuous in each of these sub-intervals and
b x1 x2 xn b
Step II Find the limits of integration in new system of
variable. Here, the lower limit is g(a), the upper limit
g (b )
∫a
f ( x)dx = ∫ f ( x)dx +
a

x1
f ( x)dx + ... + ∫
xn−1
f ( x)dx + ∫ f ( x)dx.
xn
is g(b ) and the integral is now ∫ f (t ) dt .
g ( a)

StepIII Evaluate the integral, so obtained by usual method.


Leibnitz Theorem
Properties of Definite Integrals If function φ( x) and ψ ( x) are defined on [α , β] and differentiable
α on [ α , β ] and f (t ) is continuous on [ψ (α ), φ (β)], then
(i) ∫ f ( x)dx = 0 d  ψ (x ) d  d 
f (t ) dt  =  ψ ( x) f (ψ ( x))−  {φ ( x)} { f (φ( x)}
dx ∫ φ( x )
α
β β
 dt  dx 
(ii) ∫ α
f ( x) dx = ∫ f (t ) dt
α

(iii) ∫
β α
f ( x) dx = − ∫ f ( x) dx Walli’s Formula
α β

β c1 c2 β
This is a special type of integral formula whose limits from 0
(iv) ∫ α
f ( x) dx = ∫
α
f ( x) dx + ∫
c1
f ( x) dx + ... + ∫ cn
f ( x) dx to π /2 and integral is either integral power of cos x or sin x or
cos x sin x.
where, α < c1 < c2 < ... < c n < β π /2 π /2

(v) (a) ∫ f ( x) dx =
β β ∫ sin n x dx = ∫ cos n x dx
α ∫ α
f (α + β − x) dx 0 0

(n − 1) (n − 3)(n − 5) K 5 ⋅ 3 ⋅ 1 π
α α × , n = 2 m (even)
 n(n − 2)(n − 4)K 6 ⋅ 4 ⋅ 2
(b) ∫ 0
f ( x) dx = ∫ 0
f (α − x) dx

2
=
(n − 1) (n − 3) (n − 5) K 6 ⋅ 4 ⋅ 2 , n = 2 m + 1 (odd)
α
(vi) ∫ −α
f ( x) dx
 n(n − 2) (n − 4) K 5 ⋅ 3 ⋅ 1

2 α f ( x) dx, if f (− x) = f ( x)
 ∫0 where, n is positive integer.
 π /2
= i.e. f ( x) is an even function
∫ sin x ⋅ cos n x dx
m
0, if f (− x) = – f ( x)
 0
 i.e. f ( x) is an odd function (m − 1)(m − 3)...(2 or 1).(n − 1)(n − 3)...(2 or 1) π
,
2α α α  (m + n)(m + n − 2)...(2 or 1) 2

(vii) ∫ f ( x)dx = ∫ f ( x)dx + ∫ f (2α − x)dx when both m and n are even positive integers
0 0 0 
=  (m − 1)(m − 3)...(2 or 1) ⋅ (n − 1)(n − 3)...(2 or 1) ,
2 α f ( x) dx, if f (2α − x) = f ( x)
f ( x) dx =  ∫ 0 

(m + n)(m + n − 2)...(2 or 1)
(viii) ∫ 0
 0, if f (2α − x) = − f ( x) 
 when either m or n or both are odd
β
1  positive integers
(ix) ∫ α
f ( x) dx = (β − α )∫ f [(β − α) x + α]dx
0

(x) If f ( x) is a periodic function with period T, then Inequalities in Definite Integrals


α + nT T
∫ f ( x) dx = n ∫ f ( x) dx, n ∈ I
β β
(a)
α 0
(i) If f ( x) ≥ g( x) on [α , β], then ∫ f ( x) dx ≥ ∫ g( x) dx
α α
βT T
(b) ∫ αT
f ( x) dx = ( β – α) ∫ f ( x) dx, α, β ∈ I
0 (ii) If f ( x) ≥ 0 in the interval [α , β], then ∫ f ( x) dx ≥ 0
α
β

β + nT β
(c) ∫ α + nT
f ( x) dx = ∫ α
f ( x) dx, n ∈ I (iii) If f ( x), g( x) and h( x) are continuous on [a, b ] such that
b b b
g( x) ≤ f ( x) ≤ h( x), then ∫ g( x) dx ≤ ∫ f ( x)dx ≤ ∫ h( x)dx
a a a
DAY 165

(iv) If f is continuous on [α , β] and l ≤ f ( x) ≤ M , ∀ where, h =


b–a
→0 as n → ∞
β n
x ∈[α , β], then l (β − α ) ≤ ∫ f ( x) dx ≤ M (β − α )
α
The converse is also true, i.e. if we have an infinite series of
(v) If f is continuous on [α , β], the above form, it can be expressed as definite integral.
β β
then ∫ α
f ( x) dx ≤ ∫ | f ( x)| dx
α
Some Particular Cases
(vi) If f is continuous on [α , β] and| f ( x)| ≤ k , ∀ x ∈ [α , β], n
1  r 1  r
n −1
1

then ∫
β
f ( x) dx ≤ k (β − α )
(i) lim
n→ ∞ ∑r =1
f   or lim ∑
n  n n→ ∞
r =1 n
f   = ∫ f ( x) dx
 n 0
α
pn
1  r β
Definite Integration as the (ii) lim
n→ ∞ ∑r =1 n  n ∫ α
f  = f ( x) dx
Limit of a Sum r
where, α = lim =0 (Qr = 1)
Let f ( x) be a continuous function defined on the closed n→ ∞ h
n –1
b
interval [a, b ], then ∫ f ( x) dx = lim h ∑ f (a + rh) and
r
β = lim = p (Qr = pn)
a n→ ∞ n→ ∞ h
r=0

DAY PRACTICE SESSION 1

FOUNDATION QUESTIONS EXERCISE


3π /4 2
dx
1 ∫ π /4 1 + cos x
is equal to 7 ∫ 0
[ x 2 ] dx is equal to
j JEE Mains 2017
(a) 2 − 2 (b) 2 + 2
(a) − 2 (b) 2 (c) 4 (d) − 1
(c) 2 − 1 (d) − 2 − 3 + 5
2 If f ( x ) is continuous function, then x
2 2
8 If ∫ | 2 t | dt = f ( x ), then for any x ≥ 0 , f ( x ) is equal to
(a) ∫ −2
f (x) dx = ∫ 0
[f (x) − f (− x)] dx −2

5 10 (a) 4 + x 2 (b) 4 − x 2
(b) ∫ −3
2 f (x) dx = ∫ 6
f (x − 1) dx 1
(c) (4 + x 2 )
1
(d) (4 − x 2 )
5 4 2 4
(c) ∫ −3
f (x) dx = ∫ −4
f (x − 1) dx
π /2 cos x
5 6 9 ∫ 1 + ex
dx is equal to
(d) ∫ −3
f (x) dx = ∫ −2
f (x − 1) dx −π / 2

π /4 (a) 1 (b) 0
3 ∫ 0
[ tan x + cot x ] dx is equal to (c) −1 (d) None of these
π π 10 Let a, b and c be non-zero real numbers such that
(a) 2π (b) (c) (d) 2 π
3 3
2 2
1  1+ x
∫ 0
( 3 ax 2 + 2bx + c ) dx = ∫ ( 3 ax 2 + 2 bx + c ) dx , then
1
4 ∫ 0
sin  2 tan−1

 dx is equal to
1− x  (a) a + b + c = 3 (b) a + b + c = 1
(c) a + b + c = 0 (d) a + b + c = 2
(a) π / 6 (b) π /4 (c) π / 2 (d) π a
π
sin( 2n − 1) sin nx
π 2 11 The value of ∫ [ x ] f ′ ( x ) dx , a > 1, where [ x ] denotes the
5 If I1( n ) = ∫ dx and I 2 ( n ) = ∫
2 2 1
dx ,
0 sin x 0 sin2 x greatest integer not exceeding x, is
n ∈ N, then (a) [a]f (a ) − {f (1) + f (2) + . . . + f ([a])}
(a) I 2 ( n + 1) − I 2 ( n ) = I1 ( n ) (b) I 2 ( n + 1) − I 2 ( n ) = I1 ( n + 1) (b) [a]f ([a]) − {f (1) + f (2) + . . . + f (a )}
(c) I 2 ( n + 1) + I1( n ) = I 2 ( n ) (d) I 2 ( n + 1) + I1 ( n + 1) = I 2 ( n ) (c) af ([a]) − {f (1) + f (2) + . . . + f (a )}
1 (d) af (a ) − {f (1) + f (2) + . . . + f ([a])}
6 ∫ −1
{x 2 + x − 3} dx, where {x } denotes the fractional part
π 
π /2 
 
of x, is equal to 12 The correct evaluation of ∫ sin  x −  dx is
0
  4 
1 1
(a) (1 + 3 5 ) (b) (1 + 3 5 ) (a) 2 + 2 (b) 2 − 2
3 6
1 1 (c) − 2 + 2 (d) 0
(c) (3 5 − 1) (d) (3 5 − 1)
3 6
166

3π /2   2x  1 1 2 3n 
13 ∫ 0
sin   dx, where [ ⋅ ] denotes the greatest
  π 
23 The value of lim
n→ ∞
 +
n n + 1 n + 2
+ ...+  is
4n 
integer function is equal to (a) 5 − 2 ln 2 (b) 4 − 2 ln 2
π π (c) 3 − 2 ln 2 (d) 2 − 2 ln 2
(a) (sin1 + cos1) (b) (sin1 − sin 2)
2 2
π π 24 If f and g are continuous functions in [0, 1] satisfying
(c) (sin1 − cos1) (d) ( sin1 + sin 2)
f ( x ) = f (a − x ) and g ( x ) + g (a − x ) = a, then
2 2 a

14 If [. ] denotes the greatest integer function, then the value ∫ 0


f ( x ) ⋅ g ( x ) d x is equal to j
NCERT Exemplar
1.5
of ∫ 2
x [ x ] dx is (a)
a a a
(b) ∫ f (x)d x
0
2 2 0
a a

(a)
5
(b) 0 (c)
3
(d)
3 (c) ∫ f (x)d x (d) a ∫ f (x)d x
0 0
4 2 4
π /2 2
8 log (1 + x ) sin x
15 The value of ∫
1
dx is 25 The value of ∫ dx is
0 1+ x2
−π / 2 1 + 2x j JEE Mains 2018, 13
π π π π
(a) log 2 (b) log 2 (c) log2 (d) π log2 (a) π (b) (c) 4 π (d)
8 2 2 4
π

16 ∫
π /2

0
1 − sin 2x is equal to j NCERT Exemplar
26 ∫ 0
[cot x ] dx , where [] denotes the greatest integer

(a) 2 2 (b) 2 ( 2 + 1) (c) 2 (d) 2 ( 2 − 1) function, is equal to


π π
e 2
loge x (a) (b) 1 (c) − 1 (d) −
17 The value of ∫ −1
dx is 2 2
e x
4 log x 2
(a) 3/2 (b) 5/2 (c) 3 (d) 5 27 The integral ∫ dx is equal to
2 log x + log( 36 − 12 x + x 2 )
2
n
1
18 The value of ∑ ∫ f (k − 1 + x ) dx is j JEE Mains 2015
0
k =1 (a) 2 (b) 4 (c) 1 (d) 6
1 2 n 1 −π /2
(a) ∫ 0
f (x) dx (b) ∫ 0
f (x) dx (c) ∫ 0
f (x) dx (d) n ∫ 0
f (x) dx 28 ∫ −3π /2
[( x + π )3 + cos 2 ( x + 3π )] dx is equal to

 π4  π π π π4
(c)   − 1(d)
19 f ( x ) is a continuous function for all real values of x and
2 (a)   + (b)
n +1 n 5
 32  2 2  4 32
satisfies ∫ f ( x ) dx = , ∀ n ∈ I, then ∫ f (| x | ) dx is
n 2 −3

equal to 29 If f : R → R and g : R → R are one to one, real valued


19 35 function, then the value of the integral
(a) (b) π
2 2 ∫ −π
[f ( x ) + f ( − x )] [g ( x ) − g ( − x )] dx is
17
(c) (d) None of these (a) 0 (b) π (c) 1 (d) None of these
2
1 1 2  n n
20 If I1 = ∫ 2 x dx ,I 2 = ∫ 2 x dx ,I 3 = ∫
2 3 2
2 x dx and 30 The value of lim  +
n → ∞ (n + 1) 2n + 1 (n + 2) 2 ( 2n + 2)
0 0 1 
2
I4 = ∫
3
2 x dx , then n 1 
1 + + ... +  is
(n + 3) 3 ( 2n + 3) 2n 3 
(a) I 3 > I 4 (b) I 3 = I 4
(c) I1 > I 2 (d) I 2 > I1 π π
(a) (b)
7π / 3 3 2
21 The value of ∫ tan2 x dx is π
7π / 4 j
JEE Mains 2013 (c) (d) None of these
3 4
(a) log 2 2 (b) log 2 3π π
2 31 If P = ∫ f (cos 2 x ) dx and Q = ∫ f ( cos 2 x ) dx , then
0 0
(c) 2 log 2 (d) log 2
π /2 sin 2x (a) P − Q = 0 (b) P − 2 Q = 0
22 ∫ 0 sin x +cos x
dx is equal to j NCERT Exemplar (c) P − 3 Q = 0 (d) P − 5 Q = 0
π 2x (1 + sin x )
(a)
1
log( 2 + 1) (b)
1
log( 2 + 1) 32 ∫ −π 1 + cos 2 x
dx is equal to
2 2
(c) − log( 2 + 1) (d) None of these π2 π
(a) (b) π 2 (c) 0 (d)
4 2
DAY 167

t2 2 5 2 2
33 If f ( x ) is differentiable and ∫ x f ( x ) dx = t , then (a) I > and J < 2 (b) I > and J > 2
0 5 3 3
 4 2 2
f   is equal to (c) I < and J < 2 (d) I < and J > 2
 25 3 3
2 4 ( 2 − 1)
(a)
2
(b) −
5
(c) 1 (d)
5 42 Statement I ∫ 0
f ( x ) dx =
3
,
5 2 2
x 2 , for 0 ≤ x < 1
1 x
where, f ( x ) = 
34 If f ( x ) =
x2 ∫ 4
[ 4 t 2 − 2f ′ ( t )] dt , then f ′ ( 4) is equal to
 x , for 1 ≤ x ≤ 2
32 Statement II f ( x ) is continuous in [0, 2].
(a) 32 (b)
3
32 (a) Statement I is true, Statement II is true; Statement II is a
(c) (d) None of these correct explanation for statement I
9
(b) Statement I is true, Statement II is true; Statement II is
x2

35 The value of lim


∫ 0
sin t dt
is
not a correct explanation for statement I
(c) Statement I is true; Statement II is false
x→ 0 x3 (d)Statement I is false; Statement II is true
(a) 0 (b) 2 / 9 π /3 dx π
(c) 1 / 3 (d) 2 / 3 43 Statement I The value of the integral ∫ is .
π /6 1 + tan x 6
b b
2 dx 2 dx Statement II ∫ f ( x ) dx = ∫ f (a + b − x ) dx
36 If I1 = ∫ and I 2 = ∫ , then a a
1
1+ x2 1 x j JEE Mains 2013

(a) I1 > I 2 (b) I 2 > I1 (c) I1 = I 2 (d) I1 > 2 I 2 (a) Statement I is true; Statement II is true; Statement II is a
y dt d y 2 correct explanation for Statement I
37 If x = ∫ , then is equal to (b) Statement I is true; Statement II is true; Statement II is
0
1 + t2 dx 2 j JEE Mains 2013 not a correct explanation for Statement I
(a) y (b) 1+ y 2
(c) Statement I is true; Statement II is false
y (d)Statement I is false; Statement II is true
(c) (d) y 2
1+ y 2 1
44 Statement I If ∫ e sin x dx = λ , then ∫
200
e sin x dx = 200λ
0 0
38 Let f : R → R be a differentiable function having f ( 2) = 6, na a

 1 f (x ) 4t3 Statement II If ∫ f ( x ) dx = n ∫ f ( x ) dx , n ∈I and


f ′ ( 2) =   . Then, lim ∫
0 0
dt is equal to
 48 x→ 2 6 x −2 f (a + x ) = f ( x )
(a) 18 (b) 12 (c) 36 (d) 24 (a) Statement I is true; Statement II is true; Statement II is a
[1 + 2 + 3 + K + n ] [1 + 2 + 3 + K + n ]
4 4 4 3 3 3 correct explanation for Statement I
39 lim − is (b) Statement I is true; Statement II is true, Statement II is
n→ ∞ n5 n5
not a correct explanation for Statement I
equal to
(c) Statement I is true; Statement II is false
1 1 1
(a) (b) 0 (c) (d) (d)Statement I is false; Statement II is true
30 4 5
1 1
45 Statement I ∫ e − x cos 2 x dx < ∫ e − x cos 2 x dx
2
π /2 dx
40 If I = ∫ dx , then 0 0
0
1 + sin3 x b b
Statement II ∫ f ( x ) dx < ∫ g ( x ) dx , ∀ f ( x ) < g ( x )
π a a
(a) 0 < I < 1 (b) I >
2 2 (a) Statement I is true; Statement II is true; Statement II is a
(c) I < 2 π (d) I < 2 π correct explanation for Statement I
sin x
1 1 cos x (b) Statement I is true; Statement II is true, Statement II is
41 If I = ∫ dx and J = ∫ dx . Then, which one not a correct explanation for Statement I
0 x 0 x
(c) Statement I is true; Statement II is false
of the following is true?
(d)Statement I is false; Statement II is true
168

DAY PRACTICE SESSION 2

PROGRESSIVE QUESTIONS EXERCISE


π
 1
1 If f ( x ) is a function satisfying f   + x 2f ( x ) = 0 for all
x
10 ∫ 0
x f (sin x ) dx is equal to
cosec θ π π π /2
(a) π ∫
2 ∫0
non-zero x, then ∫ f ( x ) dx is equal to 0
f (sin x ) dx (b) f (sin x ) dx
sin θ
π /2 π
(a) sinθ + cosec θ (b) sin2 θ (c) π ∫ 0
f (cos x ) dx (d) π ∫ f (cos x ) dx
0
(c) cosec2 θ (d) None of these
x2 + 1
11 If f ( x ) = ∫ e − t dt , then f ( x ) increases in
2

d e sin x 4 3
; x > 0. If ∫
3
2 If f (x ) = e sin x dx = f (k ) − f (1), then x2

dx x 1 x (a) (2, 2) (b) no value of x


the possible value of k, is (c) (0, ∞) (d) (− ∞, 0)
(a) 15 (b) 16 (c) 63 (d) 64 1/ n
 (n + 1)(n + 2) K 3n 
12 lim   is equal to
 
2
3 If g (1) = g ( 2), then ∫ [ f {g ( x )}]−1 f ′ {g ( x )} g ′ ( x ) dx is n→ ∞ n2n j JEE Mains 2016
1
18 27
equal to (a) 4 (b) 2
(a) 1 (b) 2 (c) 0 (d) None of these e e
9
π (c) 2 (d) 3 log 3 − 2
4 For 0 ≤ x ≤ , the value of e
2
sin2 x cos 2 x 13 The least value of the function
∫ ) dt + ∫
−1
sin ( t cos −1( t ) dt is x  5π 4π 
f (x ) = ∫ ( 3 sin u + 4 cos u ) du on the interval
0 0 j JEE Mains 2013 ,
5π /4  4 3 
π π
(a) (b) 0 (c) 1 (d) −
4 4 is
3 3 1
 1 x log t (a) 3 + (b) − 2 3 + +
5 If F ( x ) = f ( x ) + f   , where f ( x ) = ∫ dt . Then, F (e ) 2 2 2
x 1 1+ t 3 1
is equal to (c) + (d) None of these
2 2
(a) 1/2 (b) 0 (c) 1 (d) 2
n


x 1
0
[ x ] dx 14 If g ( x ) = ∫ f (t ) dt , where f is such that, ≤ f ( t ) ≤ 1, for
6 The expression n
, where [ x ] and {x } are integral 0 2
∫ 0
{x } dx 1
t ∈[ 0, 1] and 0 ≤ f ( t ) ≤ , for t ∈[1, 2]. Then, g( 2) satisfies
2
and fractional part of x and n ∈ N, is equal to
the inequality
1 1
(a) (b) (c) n (d) n − 1 3 1 1 3
n −1 n (a) − ≤ g (z) < (b) ≤ g(2) ≤
2 2 2 2
ex 3 5
7 If f ( x ) = , I1 = ∫
f (a )
x g [ x (1 − x )] dx and (c) < g(2) ≤ (d) 2 < g(2) < 4
1+ e x f (− a ) 2 2
f (a ) I 15 Let n ≥ 1,n ∈ z . The real number a ∈( 0, 1) that minimizes
I2 = ∫ g [ x (1 − x )] dx , then the value of 2 is 1
f (− a ) I1
the integral ∫ | x n − a n | dx is
(a) 2 (b) – 3 (c) –1 (d) 1 0
x
{∫ e dt }
2
t 2 1
0
(a) (b) 2
8 The value of lim x
is 2

x→ ∞ 2
e 2 t dt 1
0 (c) 1 (d)
3
(a) 1 / 3 (b) 2 / 3 (c) 1 (d) None of these
π
16 The value of
x x
9 The integral ∫
1/ n
1 + 4 sin2 − 4 sin dx is equal to   π  2π   3π   nπ  
2 2 lim tan   tan   tan  K tan    is
0 j
JEE Mains 2014 n→ ∞
  2n   2n   2n   2n  

(a) π − 4 (b) −4−4 3 (a) 1 (b) 2
3
π (c) 3 (d) Not defined
(c) 4 3 − 4 (d) 4 3 − 4 −
3
DAY 169

x −1 2 +1 (a) Statement I is true, Statement II is true; Statement II is a


17 If f ( x ) = , f ( x ) = f ( x ), ..., f k ( x ) = f {f k ( x )},
x +1 correct explanation for Statement I
1 (b) Statement I is true, Statement II is true; Statement II is
k = 1, 2, 3,K and g ( x ) = f 1998 ( x ), then ∫ g ( x ) dx is equal
1 /e not a correct explanation for Statement I
to (c) Statement I is true; Statement II is false
(a) 0 (b) 1 (c) −1 (d) e (d)Statement I is false; Statement II is true

18 If f ( x ) is a function satisfying f ′ ( x ) = f ( x ) with 20 Consider sin6 x and cos 6 x is a periodic function with π.
π /2
f ( 0) = 1 and g ( x ) is a function that satisfies
1
Statement I ∫ ( sin6 x + cos 6 x ) dx lie in the interval
f ( x ) + g ( x ) = x . Then, the value of ∫ f ( x ) g ( x ) dx , is
2 0

0 π π
 , .
e 2
5 e 3 2 8 2
(a) e − − (b) e + −
2 2 2 2 Statement II sin6 x + cos 6 x is periodic with period π / 2.
e2 3 e2 5
(c) e − − (d) e + + (a) Statement I is true, Statement II is true; Statement II is a
2 2 2 2 correct explanation for Statement I
19 If n > 1, then (b) Statement I is true, Statement II is true; Statement II is
∞ dx 1 dx not a correct explanation for Statement I
Statement I ∫
1 + x n ∫ 0 (1 − x n )1/ n
=
0 (c) Statement I is true; Statement II is false
b b (d)Statement I is false; Statement II is true
Statement II ∫ f ( x ) dx = ∫ f (a + b − x ) dx
a a

ANSWERS

SESSION 1 1 (b) 2 (d) 3 (c) 4 (b) 5 (b) 6 (b) 7 (d) 8 (a) 9 (a) 10 (c)
11 (a) 12 (b) 13 (d) 14 (d) 15 (d) 16 (d) 17 (b) 18 (c) 19 (b) 20 (c)
21 (b) 22 (a) 23 (c) 24 (b) 25 (d) 26 (d) 27 (c) 28 (b) 29 (a) 30 (a)
31 (c) 32 (b) 33 (a) 34 (c) 35 (d) 36 (b) 37 (a) 38 (a) 39 (d) 40 (b)
41 (c) 42 (d) 43 (d) 44 (d) 45 (a)

SESSION 2 1 (d) 2 (d) 3 (c) 4 (a) 5 (a) 6 (d) 7 (a) 8 (d) 9 (d) 10 (c)
11 (d) 12 (b) 13 (b) 14 (b) 15 (a) 16 (a) 17 (c) 18 (c) 19 (b) 20 (b)
170

Hints and Explanations


SESSION 1 5 I2 ( n ) − I2 ( n − 1 ) On putting x = − x in 1st integral, we get
x
dx 3π / 4 0 cos x π / 2 e cos x
1 Let I = ∫π / 4 π /2 [sin nx − sin (n − 1) x]
2 2
∫ − π /2 1 + e x dx = ∫ 0 1 + e x dx
1 + cos x = ∫ 0 sin2 x
dx
3 π / 4 1 − cos x
=∫ dx sin( 2n − 1) x ⋅ sin x π /2 e x cos x π / 2 cos x
∫ dx + ∫
π /2
1 − cos 2 x ∴ I =
∫ dx
π/4
= dx
0
sin2 x
0
1 + ex 0
1 + ex
3 π / 4 1 − cos x
=∫ dx (1 + e x ) cos x
sin (2n − 1) x π /2

π /2
π/4
sin2 x =
3π / 4
= ∫ dx = I1( n) 0
(1 + e x )
dx
∫ sin x
0
= (cosec2 x − cosec x cot x )dx
π/4 π /2

π /2
⇒ I2 ( n + 1 ) − I2 ( n ) = I1 ( n + 1 ) = cos x dx = [sin x] =1
= [− cot x + cosec x]3ππ/ /44 0
0
1 1
∫ { x + x − 3} dx = ∫ { x + x} dx
2 2
= [(1 + 2 ) − (− 1 + 2 )] = 2 6 3
−1
1
−1 10 ∫ 0
(3 ax2 + 2bx + c ) dx
2 Since, f is continuous function. = ∫ −1
( x2 + x − [ x2 + x]) dx
= ∫
3
(3 ax2 + 2bx + c ) dx
x = t −1
1
Let 1
x x  3 2 1

0
∴ dx = dt =
 3
+  −
2  −1 ∫ −1
[ x2 + x] dx ⇒
0
(3 ax2 + 2bx + c ) dx
When x tends to − 3 and 5, then 3

t tends to − 2 , 6.
5 −1 + ∫ (3 ax2 + 2bx + c ) dx

1
5 6 − 2 [ x + x] dx
2

Therefore, ∫ ∫
f ( x ) dx = f (t − 1) dt
3

0
−3 −2 1
= (3 ax2 + 2bx + c ) dx

1
6 − [ x2 + x] dx
=∫ f ( x − 1) dx ( 5 − 1 )/ 2 1
−2

⇒ ∫ (3 ax2 + 2bx + c ) dx = 0
5 − 1
0
2
π /4 = + 1 − 0 − 1 1 −   3 ax3 
1
3 Let I = ∫ 0
[ tan x + cot x ] dx 3  2  ⇒
2bx2
 3 + 2 + cx  = 0
π /4 sin x + cos x  0
5−1 1
= ∫ 0
sin x cos x
dx =
2
+ = (1 + 3 5) ∴ a+ b + c = 0
3 2 6
a 2
π /4 sin x + cos x 11 Since, ∫ 1 [ x] f ′( x ) dx = ∫ f ′ ( x ) dx

2 1
= 2
0
1 − (sin x − cos x ) 2
dx 7 ∫ 0
[ x2 ] dx = ∫ 0
[ x2 ] dx
3
1
a
2 3 + ∫ 2f ′ ( x ) dx + . . . + ∫ [a] f ′ ( x ) dx
Put sin x − cos x = t + ∫ [ x2 ] dx + ∫ [ x2 ] dx 2 [ a]
1 2
⇒ (cos x + sin x ) dx = dt 2 = [ f ( x )]21 + 2 [ f ( x )]32 + . . . + [a][ f ( x )][aa ]
∴ I= 2∫
0 dt
⇒ I = 2 [sin −1 t ] −01
+ ∫ 3
[ x2 ] dx
= f (2) − f (1) + 2 f (3) − 2 f (2) + . . .
−1
1 − t2 1 2
+ [a] f (a) − [a] f ([a])

= 2 [0 − (− π / 2)] =
π
= ∫ 0
0 dx + ∫ 1
1 dx = [a] f (a) − { f (1) + f (2) + . . . + f ([ a])}

 sin  x − π  

2 π /2
∫  dx
3 2

1  1 + x
+ ∫ 2
2 dx + ∫ 3
3 dx 12 Let I =
 0 4 
4 ∫ 0 sin 2 tan −1 dx
 1 − x  = [ x]1 2 + [2 x] 3
2
+ [3 x]2 3 =−∫
π /4 π
sin  x −  dx
0  4
Put x = cos θ, then
= 2−1+ 2 3−2 2+ 6−3 3 π
sin  x −  dx
π /2
 1 + cos θ 
sin 2 tan −1
+ ∫  4
1 − cos θ  = 5− 3 − 2 π/4

 π/4 π /2
 π   π 
= cos  x −  − cos  x − 
x
 θ 
= sin 2 tan −1  cot 
8 ∫ −2
|2t | dt = f ( x )
  4   0   4   π / 4
  2   0 x

 π θ 
= ∫ |2t|dt + ∫ 2t dt
= 1−
1
−
 1 
− 1 = 2 − 2
= sin 2 tan –1  tan  −   
−2 0

 2 2  
0 x 2  2 
  t 2  t 2 
= −2 +2
2 2
sin   dx = ∫ sin   dx
 π θ 
3 π /2 2x π /2 2x
= sin 2  − 
  2 2  
  −2  0 13 ∫ 0
 π  0
 π 
 x2 
= − 2 [0 − 2] + 2 − 0 = 4 + x2
+ ∫ sin   dx + ∫ sin   dx
2  π 2x 3π / 2 2x
= sin ( π − θ) = sin θ   π /2  π  π  π 
= 1 − cos 2 θ = 1 − x2 π /2 cos x π 3π / 2

1 1 + x
9 Let I = ∫ −π / 2
1 + ex
dx = 0 + sin 1 ∫
π /2
dx + sin 2 ∫
π
dx
∴ ∫ sin 2 tan −1 dx
0
 1 − x  0 cos x =
π
(sin 1 + sin 2)
1
= ∫ − π /2 1 + ex
dx 2
= ∫ 1 − x dx 2
1. 5
0
14 Here, ∫ 0 x [ x2 ] dx
π /2 cos x

1
π +
=  x 1 − x2  + [sin −1 x]
1 1 dx …(i)
1
= 1 + ex
1 2 1. 5
I = ∫ x ⋅ 0 dx + ∫ x ⋅ 1 dx + ∫ x ⋅ 2dx
0
 2  0 2 0
4 0 1 2
DAY 171

0 2
 x2 
2
 z2   z2  π /2 sin2 x
= 0+ + [ x2 ] 1 .25 ∴I = − +
1
= + 2=
5 22 We have, l = ∫0 dx
2  2 2 sin x + cos x
 1   −1  0 2 2
π
1 1
sin2  − x 
2
=
{2 − 1} + {(1.5) 2 − 2} 18 Let I = ∫ 0
f (k − 1 + x ) dx
=∫
π /2 2 
dx
π π
 
sin  − x  + cos  − x 

k 0
1
= + 2. 25 − 2 = + =
2
1 1 3
2 4 4
⇒ I = ∫ k −1
f ( t ) dt , where
2  2 
1 8 log (1 + x )
t = k −1+ x π /2 cos 2 x
15 Let I = ∫ 0 dx l =∫ dx
(1 + x2 ) ⇒ dt = dx 0 sin x + cos x
k

Put x = tan θ ⇒ dx = sec2 θd θ


⇒ I = ∫ k −1
f ( x ) dx
Thus, 2l =
1 π /2

dx
2 0 cos  x − π 
n
 
∑ ∫
k
When x = 0 ⇒ tan θ = 0 ∴ f ( x ) dx  4
k −1
k =1
∴ θ=0 1 π /2  π
π
1 2 n = ∫ sec  x − 4  dx
When x = 1 = tanθ ⇒ θ = = ∫ 0
f ( x ) dx + ∫ 1
f ( x ) dx + ...+ ∫
n −1
f ( x ) dx 2 0
π /2
4
1   π π 
= log sec  x −  + tan  x −   
n
8 log (1 + tan θ)
∴I = ∫
π/4
⋅ sec2 θd θ = ∫ 0
f ( x ) dx
2 
  4   4   0
0 1 + tan2 θ
1  π π
log  sec + tan 
5 3 5
∫ ∫ ∫ =
π/4
= 8∫ log (1 + tan θ) d θ 19 f (| x |) dx = f (|x|) dx + f (|x|)dx
0 −3 −3 3 2   4 4
3 5
I = 8∫
π/4
= 2∫ f ( x ) dx + ∫ π π 
− log sec  −  + tan  −   
log (1 + tan θ) d θ …(i) f ( x ) dx
0 3
0
  4  4  
= 2  ∫ f ( x )dx + f ( x ) dx + ∫ f ( x ) dx 
1 2 3
π
= 8 ⋅  ⋅ log 2  0 ∫ 1 2  1
2  = [log ( 2 + 1) − log ( 2 − 1 )]
2
+  ∫ f ( x ) dx + ∫ 4 f ( x ) dx 
4 5
Q π / 4 log (1 + tan θ)dθ = π log 2  3

 0 8  =
1
log
2 +1
=
1  ( 2 + 1)2 
log  
 1 2 
= 2  0 + +  +  +
9 16  35 2
2 2−1 2 1 
= 
= π log 2  2 2  2 2 2 2
π /2 = log ( 2 + 1 )]
16 Let l = ∫ 0
1 − sin2 xdx
20 Given that, I1 = ∫
1
2 x dx
2
2
π /4 0 1
=∫ (cos x − sin x ) 2 dx 1 ∴ l= log ( 2 + 1)

3
0
I2 = 2 x dx, 2
π /2
+∫
0
(sin x − cos x ) dx 2
2
1  1 3n 
∫ 2
π /4 2
I3 = 2 x dx 23 nlim  + + ...+ 
π
[Qcos x − sin x > 0 when x ∈  0,  and n n + 1 n+ 2 n + 3n
1 →∞
2
 4

3
and I 4 = 2 x dx 3n
1  r 

1
cos x − sin x < 0 when x ∈ ( π / 4, π / 2)] = lim  
π /4 Since, 2 x3
<2 x2
for 0 < x < 1 n→ ∞
r =1 n n + r
= ∫ (cos x − sin x ) dx 3
2 x > 2 x for x > 1
2
3 x
and = ∫
0
π /2 dx
+∫ (sin x − cos x ) dx 1 1
0 1+ x
∫ ∫
x3 x2
π /4 ∴ 2 dx < 2 dx
0 0 3  1 
= [sin x + cos x] π0 / 4 2 2 = ∫ 1 −  dx
∫ ∫ +
3
x2
+ [− cos x − sin x] ππ // 24 and 2 x
dx > 2 dx 0
 1 x
1 1
 π π  = [ x − ln (1 + x )] 30 = 3 − ln 4
=  sin + cos  − (sin 0 + cos 0) ⇒ I2 < I1 and I 4 > I3
  4 4  7 π /3
= 3 − 2 ln 2
21 I = ∫ tan2 x dx a
 π π π
+  − cos − sin  −  − cos − sin 
 
π  7π / 4 24 Ql = ∫ f ( x )⋅ g ( x ) dx
4  
0
2  2π 7π / 3

 1
2 4 = ∫ 7π / 4
|tan x |dx + ∫ 2π
|tan x |dx =∫
a
f (a − x ) g ( a − x )dx
1    1 1  0
=  + − 1 + −1 −  − − 
2π 7π / 3
 2
 2     2 2  
=− ∫ 7π / 4
tan x dx + ∫

tan x dx
=∫
a

0
f ( x ){ a − g ( x )} dx
 2  = − [log sec x] 2π
+ [log sec x] 72 ππ /3 a a
= − 1 + [−1 + 2] 7π / 4
= a ∫ f ( x )d x− ∫ f ( x ) ⋅ g ( x )dx
 2 
7π 
= −  log sec 2 π − log sec
0 0
a
= ( 2 − 1) + (−1 + 2 ) = 2( 2 − 1)  4  = a ∫ f ( x)d x − I
0

+  log sec − log sec 2 π 
a a
log e x log e x ∴ l = ∫ f ( x ) dx
2
e 1
17 ∫ e −1
x
dx = ∫ e −1 x
dx  3  2 0
π
= −  log 1 − log sec
e 2 log x  π /2 sin2 x
+ ∫
1
e

x
dx
 4 
25 Let I = ∫ − π /2
1 + 2x
dx

− log e x π
+  log sec − log 1 −π π
e log x
2

sin2  + − x  dx
1
= ∫ e −1
x
dx + ∫
1
x
e
dx
 3  π /2  2 2 
0 2 ⇒ I = ∫ dx
∫ ∫ 1 −π π
= − z dz + z dz = log 2 + log 2 = log 2 + log 2
− π /2 + −x
−1 0
2 1+ 22 2

Q b f ( x ) dx = b
f (a + b − x ) dx 
 ∫a ∫
[put log e x = z ⇒ (1 / x ) dx = dz) 3
= log 2 a 
2
172

sin2 x − π /2
π sin x

π /2 π
⇒ I = [ − ( x + π ) 3 + cos 2 x] dx ⇒ I = 4∫
⇒ I = ∫ − π /2 1 + 2− x
dx −3 π / 2 0
1 + cos 2 x
dx
− π /2
π /22x ⋅ sin2 x ⇒ 2I = ∫ 2cos 2 x dx
−4 ∫
π x sin x
⇒ I = ∫ − π /2 1 + 2x dx −3 π / 2

− π /2
0
1 + cos 2 x
dx
= ∫ −3 π / 2
(1 + cos 2 x ) dx π sin x
I =
π /22x ⋅ sin2 x
∫ − π /2 1 + 2x dx − π /2
⇒ I = 4π ∫ 0
1 + cos 2 x
dx − I
= x +
sin 2 x 
π /2 π /2  2  −3 π /2 [from Eq. (i)]
⇒ 2I = ∫ − π /2
sin2 x dx = ∫ 0
2sin2 x dx
π sin x
π sin (− π ) ∫
π /2 = − + ⇒I = 2π dx
= ∫ (1 − cos 2 x ) dx  2 1 + cos 2 x
0
0 2
π /2
π 3 π sin (−3 π )  Put cos x = t
⇒ 2I =  x − −  −
sin 2 x 
= +  = π ⇒ − sin x dx = dt
 2  0 2  2 2 
π
−1 1
∴ I = ∴ I =
π ∴ I = − 2π ∫ 1
1 + t2
dt
4 2
π π
= 2 π[tan −1 t ]1−1 = 2 π  + 
π
26 Let I = ∫ 0
[cot x] dx …(i) 29 Let φ ( x ) = [ f ( x ) + f (− x )] [g ( x ) − g (− x )]  4 4 
π
∴ φ (− x ) = [ f (− x ) + f ( x )] [g (− x )
⇒ I = ∫ 0
[cot ( π − x )] dx = π2
π
− g ( x )] = − φ ( x )
= ∫ 0
[− cot x] dx …(ii)
∴ ∫
π
φ ( x ) dx = 0 33 Using Newton-Leibnitz’s formula, we get
t 2 { f (t 2 )}  (t )2  − 0
−π d
On adding Eqs. (i) and (ii), we get
π π
(Qφ( x ) is an odd function)  dt 
2I = ∫ [cot x] dx + ∫ [− cot x] dx n
n
⋅ f (0)  (0) = 2t 4
d
→∞∑
0 0
30 S = nlim
π
(n + r ) r (2n + r )  dt 
= ∫ 0
(− 1) dx r =1

n
1 ⇒ t 2 { f (t 2 )} 2t = 2t 4
  − 1 if x ∉ Z  = lim ∑
⇒ f (t 2 ) = t
Q[ x] + [− x] =  0, if x ∈ Z 
n→ ∞
n  1 + 
r =1 r r  r
   2 + 
f   = ±  put t = ± 2 
4 2
 n n n ⇒
= [− x] π0 = − π  25 5  5
π 1 dx
∴I =− S = ∫ 0
(1 + x ) 2 x + x2
∴ 
f  =
4 2
2  25 5
log x 2 1 dx [neglecting negative sign]

4
27 l = ∫
2
log x2 + log(36 − 12 x + x2 )
dx =
0
(1 + x ) (1 + x )2 − 1 34 We have,
1 x
x2 ∫ 4
4 2 log x S = [sec (1 + x )] −1 1
f ( x) = [4t 2 − 2 f ′(t )] dt
= ∫ 2
2 log x + log(6 − x ) 2
dx 0
π
= sec −1 2 − sec −1 1 = 1
4 2 log x dx 3 ∴ f ′ ( x ) = 2 [4 x2 − 2 f ′ ( x )]
= ∫ 2[log x + log (6 − x )]

x
31 P = ∫
2
f (cos 2 x ) dx and 2 x
− 3 ∫ [4 t 2 − 2 f ′(t )] dt
0
log xdx 4 π x 4
⇒ l = ∫
[log x + log(6 − x )]
2
...(i) Q =∫
0
f (cos 2 x ) dx
⇒ f ′ (4) =
1
[64 − 2 f ′ (4)] − 0
π 16
4 log(6 − x ) Also, P = 3∫ f (cos 2 x )dx = 3Q
⇒ l =∫ dx ...(ii) 0 32
2
log (6 − x ) + log x ∴ f ′ (4) =
∴ P − 3Q = 0 9
Q b f ( x ) dx = b f ( a + b − x ) dx 
 ∫a ∫a π 2 x(1 + sin x ) x2
 32 Let I = ∫ − π
1 + cos 2 x
dx
35 lim
∫ 0
sin t dt
 form 0 
On adding Eqs.(i) and (ii),we get x→ 0 x3  0 
π 2x
2l = ∫
4 log x + log (6 − x )
dx
= ∫ −π
1 + cos 2 x
dx
= lim
sin x ⋅ 2 x
2
log x + log(6 − x ) x→ 0 3 x2
π 2 x sin x 2 sin x

4
⇒ 2l = ∫2
dx = [ x]24 +
−π 1 + cos 2 x
dx = lim
3 x→ 0 x
⇒ 2l = 2 ⇒ l = 1 2 2
− π /2 ⇒ I = 0+ 4∫
π x sin x
dx …(i) = ⋅1 =
28 Let I = ∫ −3 π / 2
[( x + π )3 + cos 2 x] dx 0 1 + cos 2 x 3 3

 π 3 π 3
 2x 36 We have, (1 + x2 ) > x2 , ∀ x
−π / 2 
⇒ I = ∫ −3π /2   − 2 − 2 − x + π  Q 1 + cos 2 x is an odd function
 ⇒ 1 + x2 > x, ∀ x ∈ (1, 2)

1 1
π 3π   ⇒ < , ∀ x ∈ (1, 2)
+ cos 2  − − x  dx
2 x sin x
− and is an even function  1 + x2 x
 2 2   1 + cos 2 x 
2 dx 2 dx
Q b f ( x ) dx = b f ( a + b − x ) dx 
 ∫a ∫a  ⇒ I = 4∫
π ( π − x ) sin ( π − x )
dx
∴ ∫ 1
1+ x 2
< ∫ 1 x
⇒ I1 < I2
0
1 + cos 2 ( π − x )
⇒ I2 > I1
DAY 173

⇒ 2I = 0 ⇒ I = 0
1
y dt


37 We have, x = 1
0
1+ t 2
= ∫ 0
x 2
dx = 2 [ x1 /2 ]10 = 2
sin x

∴ J<2 2 d f ( x) = e
By Leibnitz rule, we get dx x
2
1 dy 3 sin x3 4 4 3x
42 Since, f ( x ) is continuous in [0, 2].
∫ 1 x e dx = ∫ 1 x3 e dx
sin x3
1= ⋅ ∴
1 + y 2 dx 2
∴ ∫ f ( x ) dx =
1 2

0 ∫ 0
f ( x ) dx + ∫ 1
f ( x ) dx
Put x3 = t
dy
⇒ = 1 + y2 1 2
⇒ 3 x2 dx = dt
dx = ∫ 0
x2 dx + ∫ 1
x dx
64 e
sin t

d2 y y dy 1 2
∴ f (t ) = ∫ dt
⇒ = ⋅  x3   x3 /2  t
1

dx2 1 + y 2 dx = + = [ f (t )]1
3   64
  0 3 / 2  1
d2 y y = f (64) − f (1)
∴ = ⋅ 1 + y2 = y =
1 2 3 /2
+ (2 − 1) On comparing, we get
dx2 1 + y2 3 3
k = 64
1 4 2 2  4 2 − 1
f( x )
= + − = 
f( x ) 4t 3 ∫ 4t 3 dt
3 3 3  3  3 Let I = ∫
2
[ f {g ( x )}] −1 f ′ {g ( x )} {g ′ ( x )} dx
x→2 ∫ 6
38 lim dt = lim 6
1
x−2 x→2 ( x − 2) π /3 dx Put f {g ( x )} = z
 form 0  43 Let l = ∫ π /6
1 + tan x
...(i)
⇒ f ′ {g ( x )} g ′ ( x ) dx = dz
 0  When x = 1, then z = f {g (1)}
π /3 dx
[by Leibnitz’s rule] ∴ l = ∫ π /6
π
When x = 2, then z = f {g (2)}
1 + tan  − x 
3
4 { f ( x )} f { g(2 )} 1
= lim f ′ ( x ) = 4 { f (2)}3 f ′ (2) ∴ I =∫ dz = [log z] ff {{ gg((21 )})}
x→2 1 2  f { g(1 )}
z
1 ⇒ I = log f {g (2)} − log f {g (1)} = 0
= 4 × (6)3 × π /3 tan x dx
48 ⇒l = ∫ π /6
1 + tan x
...(ii) [Q g (2) = g (1)]
Q f (2) = 6 and f ′ (2) = 1 , given  4 Put t = sin2 z in 1st integral and
 48  On adding Eqs. (i) and (ii), we get
π /3
t = cos 2 u in 2nd integral, we get
= 18
2l = ∫ π /6
dx
dt = 2sin z cos z and
1 + 24 + 34  1 + 23 + 33  ⇒ 2l = [ x] ππ //36 dt = − 2cos u sin udu
 4  3 1 π π π
l =  −  =
x

39 nlim  + . . . + n  − lim  + . . . + n 
5 5

2  3 6  12
∴I = ∫ 0
z(2sin z cos z ) dz
→∞ n n→ ∞ n x
4 3
Hence, Statement I is false but ∫ f ( x )dx
b
+ ∫ − u (2cos u sin udu )
= lim ∑   − lim × lim r
1 n r 1 1 n π /2

r =1  n 
∑  
 n
a
x x
= ∫ z sin 2z dz − ∫
b

n→ ∞ n n→ ∞ n n→ ∞ n
r =1 = f (a + b − x ) dx is a true statement by 0 I II π /2
u sin 2u du
a
1 1 1
= ∫ x 4 dx − lim × ∫
x
x3 dx property of definite integrals.
= − z⋅
cos 2z sin 2z 
0 n→ ∞ n 0 +
1 44 Since, period of e sin x
is 2 π.  2 4  0
 x5  1
= − 0=
x
− u cos 2u sin 2u 
−
200
 5 ∴ ∫ dx ≠ 200λ
sin x
 0 5 e +
 4  π / 2
0
2

40 Since, x ∈  0, π  ⇒ 1 ≤ 1 + sin3 x ≤ 2 45 For 0 < x < 1,x > x2 − { 0 + 0} 
cos 2 x sin 2 x
= −x⋅ +
 2  2  2 4 
⇒ − x < − x2 ⇒ e − x < e − x
 cos 2 x sin 2 x  π 
−  + 0
1 1
⇒ ≤ ≤1
1
⇒ ∫ e − x cos 2 x dx<
1
− −x⋅ +
∫ e − x cos 2 xdx
2

 4  
2 1 + sin3 x 0 0 2 4
If f ( x ) ≥ g ( x ), then ∫ f ( x ) dx ≥
b b
π
π /2 1 π /2 dx ∫ g ( x ) dx =
⇒ ∫ 0
2
dx ≤ ∫ 0
1 + sin x 3
a a
4
x log t
π /2 SESSION 2 5 Since, f ( x ) = ∫ 1 dt
≤ ∫ dx 1+ t
1 We have, f  1  + x2 f ( x ) = 0
0

and f ( x ) = f ( x ) + f  
π π 1
∴ ≤I≤  x
 x
2 2 2 1  1
⇒ f ( x) = − 2 f   ∴ F (e ) = f (e ) + f  
1
x sin x x  x
1 1
e 
41 Since, I = ∫
dx < ∫ dx, cosec θ

0 0
x x I = f ( x ) dx e log t 1 /e log t
sinθ ⇒ F (e ) = ∫ dt + ∫ dt
because in x ∈ (0, 1), x > sin x 1 1 + t 1 1+ t
I<∫
1 2 cosec θ  − 1 f  1   dx

0
x dx = [ x3 /2 ]10
3
= ∫ sinθ
 2  
 x  x
Put t =
1
in second integration
t
2 e − log t
⇒ I< d  
sin θ 1 e log t 1
3
= ∫ cosec θ
f (t ) dt, where t =
x
∴ F (e ) = ∫
11 + t
dt + ∫
1 1 t 
1 cos x
1+
1 1
and J = ∫ dx < ∫ cosec θ t
0
x 0
x
⇒ I =− ∫ sin θ
f (t ) dt = − I
174

t log t  − dt  2
= e −(x + 1 )2
⋅ 2 x − e −(x
2 2 2
e log t e
 ∫ e t dt 
)
⋅ 2x
∫ ∫
x
= dt − ×
2

  0 
1
1+ t 1
(1 + t )  t 2  8 xlim = 2 xe − ( x + 2x + 1 )
4 2
(1 − e 2 x2 + 1
)
x

→∞ 2

log t log t e2 t d t
e e
For f ′ ( x ) > 0,
= ∫ 1+ t
dt + ∫ t (1 + t )
dt 0
2
+1
 2 ∫ e t dt  (e x ) then 2 x (1 − e 2 x )> 0
1 1 x 2 2

 0   form 0 
e log t e log t log t e = lim ⇒ 2x < 0
= ∫ 1 1+ t
dt + ∫ 1
t
dt − ∫
1 (1
+ t)
dt x→ ∞
e 2x
2
 0 
⇒ x< 0
x
2∫ e t2
dt
e log t  1 1 1  = lim  form 0 1
(n + 1) ⋅ (n + 2) K (3n ) n

0
= dt Qt (1 + t ) = t − t + 1   
1 t  
x→ ∞
e x2
0  12 Let l = nlim 
→∞ 

x2
n2 n 
2e 1
= lim = lim =
e 1
 (log t )2  0 (n + 1) ⋅ (n + 2) ... ( n + 2n ) n
= = lim 
2
x→ ∞ x→ ∞ x
 2 xe x 
n→ ∞  n2 n 
 2 1
9 Use the formula, 1
1  n + 1  n + 2  n + 2n   n
= [(log e)2 − (log 1)2 ] | x − a|= 
x − a, x≥ a
to break given = lim     K 
2 n→ ∞       n 
 −( x − a) x < a n n
1
= integral in two parts and then integrate On taking log on both sides, we get
2 separately.   1 + 1   1 + 2  
n
1     
n 

n   
6 We have, ∫ 0 [ x] dx
2
π  1 − 2sin x  dx = π |1 − 2sin x | dx log l = lim  log  
1 2 3
∫ 0

 2
 ∫0 2
n→ ∞ n
  ...  1 + 2n   
= ∫ 0 dx + ∫ 1 dx + ∫ 2 dx + K π
 1 − 2sin x  dx − π  1 − 2sin x  dx    n   
∫ ∫ π 
0 1 2
= 3
  
n
 2 2 1
+ ∫ (n − 1) dx
0
3 ⇒ log l = lim
n −1 n→ ∞
π /3 π n
=  x + 4cos  −  x + 4cos 
x x
= 1 (2 − 1) + 2 (3 − 2) + 3 (4 − 3) + 
log  1 +  + log  1 +  + ...
1 2
 20  2 π
  n  n
K + (n − 1) {n − (n − 1)} 3
π
= 1 + 2 + 3 + K + (n − 1) = 4 3 − 4− 2n  
3 + log  1 + 
n (n − 1)  n  
= π
2 10 Let I =∫ 0
x f (sin x ) dx …(i)
⇒ log l = lim ∑ log  1 + 
1 2n r
n π n→ ∞ n  n 
I =∫
n n
and ∫ { x} dx = ∫ ( x − [ x] ) dx = ⇒ ( π − x )f [sin ( π − x )] dx r =1
0 2
2

0 0
π ⇒ log l = log (1 + x ) dx
n ⇒ I =∫ ( π − x ) f (sin x ) dx …(ii) 0


∫ 0
[x] dx
=n −1
0

⇒ log l = log (1 + x )
On adding Eqs. (i) and (ii), we get 
n

∫ 0
{x } dx
2I = ∫
π

0
π f (sin x ) dx
1 
2

e x
π π
⋅ x− ∫ 1 + x ⋅ x dx 
7 Given that, f ( x ) =
1 + ex
⇒I =
2 ∫ 0
f (sin x ) dx …(iii) n

⇒ log l = [log (1 + x ) ⋅ x]20


 2a
 ∫0
∴ f (a) =
ea
…(i) Q f ( x ) dx = 2 x+ 1−1
1 + ea  − ∫ 0
1+ x
dx
2 f ( x ) dx, if f (2a − x ) = f ( x )
a

and f (− a) =
e −a
=
1
…(ii)  ∫0  2  1 
1 + e −a 1 + e a  0 ,if f (2a − x ) = − f ( x )  ⇒ log l = 2 ⋅ log 3 − ∫ 0
1 −

 dx
1 + x
π /2
On adding Eqs. (i) and (ii), we get ⇒ I = π ∫ 0
f (sin x ) dx
⇒ log l = 2 ⋅ log 3 − [ x − log 1 + x ] 20
f (a) + f (− a) = 1 π π
Put − x=t ⇒ x= −t ⇒ log l = 2 ⋅ log 3 − [2 − log 3]
⇒ f (a) = 1 − f (− a) 2 2
Put dx = − dt in Eq. (iii), we get ⇒ log l = 3 ⋅ log 3 − 2
Let f (− a) = t
π π /2 ⇒ log l = log 27 − 2
2 ∫ −π / 2
⇒ f (a) = 1 − t I = f (cos t ) dt
27
1 −t
π ⇒ l = e log 27 − 2 = 27 ⋅ e − 2 =

π
Now, I1 = xg [ x(1 − x )] dx / 2

2 ∫ −π / 2
…(iii) = f (cos x ) dx e2
t

Q I = b b
f ( a + b − x ) dx 
π /2
13 We have, f ′( x ) = 3 sin x + 4 cos x
 ∫ a
f ( x ) dx = ∫ a 
= π ∫ 0
f (cos x ) dx
5π 4 π 
1 −t
Since, in  , , f ′ ( x ) < 0, so assume
⇒ I1 = ∫ t
(1 − x )g [ x(1 − x )] dx …(iv) [Q f (cos x ) is an even function]
 4 3 
11 On differentiate the given interval by 4π
On adding Eqs. (iii) and (iv), we get the least value at the point x = .
1 −t
using Newton-Leibnitz formula,
3
2 I1 = ∫ t
g [ x(1 − x )] dx = I2 [given]
we get f ′( x ) = e − ( x
2
+ 1 )2
⋅  ( x 2 + 1)
d
Thus,
I  dx 
∴ 2 =2 4π 
f
4 π /3
I1
− e −(x
2 2
)
⋅  ( x ) 2 
d  3 
= ∫ 5π / 4
(3 sin u + 4cos u ) du
 dx 
DAY 175

= [– 3cos u + 4 sin u] 45 ππ //34 f ( x ) + g ( x ) = x2


1/n
rπ 
n
 Also,
16 Let P = nlim  rΠ tan   
⇒ g ( x ) = x2 − e x
=
3
−2 3+
1 →∞

=1  2n   …(ii)
1
2 2 1 rπ
ln tan  
n Now, ∫ f ( x )g ( x ) dx
x
∴ ln P = lim
n→ ∞ ∑
 2n 
0

14 We have, g ( x ) = ∫ 0
f ( t ) dt n r =1
= ∫
1
e x ( x2 − e x ) dx
πx 
= ∫ ln tan 
1 0
 dx
2
 2 
⇒ g (2) = ∫ [from Eqs. (i) and (ii)]
0
f ( t ) dt
0 1 1
2 π /2
π ∫0
= ∫ x2e dx − ∫ e 2 x dx
x
1 2 ⇒ ln P = ln tan x dx …(i)
⇒ g (2) = ∫ 0
f ( t )dt + ∫ 1
f ( t ) dt 0

1 2x 1
0

2 π /2 = [x e − ∫ 2 xe dx]10 −
2 x x

π ∫0
and ln P = [e ] 0
We know that, m ≤ f ( x ) ≤ M for x ∈ [a, b] ln cot x dx …(ii) 2
b
⇒ m (b − a) ≤ ∫ a
f ( x ) dx ≤ M (b − a) On adding Eqs. (i) and (ii), we get
1
= [ x2e x − 2 xe x + 2e x ]10 − (e 2 − 1)
2
1 2 π /2
∴ ≤ f ( t ) ≤ 1, for t ∈[0, 1]
π ∫0
2 ln P = ( ln tan x + lncot x )dx 1
= [( x2 − 2 x + 2)e x ]10 − e 2 +
1
2
2 2
1 2 π /2
and 0 ≤ f ( t ) ≤ , for t ∈ [1, 2]
π ∫0
= ln 1 dx = 0 = [(1 − 2 + 2)e 1 − (0 − 0 + 2)e 0 ]
2 1 1
1 − e2 +
⇒ ln P = 0
1
⇒ (1 − 0) ≤ ∫ f ( t ) dt ≤ 1 (1 − 0) 2 2
2 0
∴ P =1 e2 1 e2 3
2 1 =e −2− + =e − −
and 0 (2 − 1) ≤ ∫ f ( t ) dt ≤ (2 − 1) 2 2 2 2
1
2
x−1
1 1 2
⇒ ≤ ∫ f ( t ) dt ≤ 1 and 0 ≤ ∫ f ( t ) dt ≤
1 17 We have, f ( x ) = 19 In LHS, put x n = tan2 θ
2 0 1
2 x+1
⇒ nx n − 1 dx = 2 tan θ sec2 θ dθ
1 1 2 3 x − 1
2 ∫0
⇒ ≤ f ( t ) dt + ∫ f ( t ) dt ≤ ⇒ f 2 ( x ) = f [ f ( x )] = f   ∞ dx 2 π /2
x + 1 ∴∫
2 = ∫ tan1 − 2 + 2 / n θ d θ
1

1 3 x−1
0
1 + xn n 0

∴ ≤ g(2) ≤ −1
2 2 x+1 1 2 π /2
= =−
x−1 x
=
n ∫ 0
tan (2 / n) −1 θ d θ
1
+1
15 Let f (a) = ∫ | x − a |dx
n n
0 x+1
a 1 In RHS, put x n = sin2 θ
= ∫ (a − x ) dx + ∫ ( x n − an ) dx f ( x ) = f [ f ( x )] = f  − 
1
n n
⇒ 4 2 2 2
⇒ nx n − 1 dx = 2 sin θ cos θ dθ
 x
0 a
a 1
 x   x
n +1 n +1
 −1 ∴∫
1dx 2 π /2 1
= ∫
= an x − + − an ⋅ x = = x
 n + 1  0  n + 1  0
(1 – x n ) 1 / n n 0 cos 2/n θ
 a −
1
2
 an +1  x −1 2 π /2
= an +1 −

+ ∴ g ( x ) = f 1998 ( x ) = f 2of 1996 ( x )
sin n θ cos θ dθ = ∫ tan (2 / n) − 1 θ dθ
n + 1  n 0

⇒ g ( x ) = f 2 [ f 1996 ( x )]
 1 an +1  20 sin 6 x + cos 6 x = (sin2 x ) 3 + (cos 2 x ) 3
− an − + an +1 ⇒ g ( x ) = f 2 ( x ) [Q f 1996 ( x )
n + 1 n+1  = (sin2 x + cos2 x ) 3 − 3 sin2
  = {( f 4of 4of 4o K f 4 )} ( x ) = x]
14243 x cos 2 x (sin2 x + cos 2 x )
2an +1 1
= 2an +1 − − an + 499 times = 1 − 3 sin x cos x
2 2

n+1 n+1 ⇒ g( x) = −
1
3
= 1 − sin2 2 x Q period = π 
= 2an +1 
n  1 x 
− an + 4 2 
 n + 1  n+1 1 1  − 1  dx
⇒ f ′(a) = n (2a − 1) a n −1
∴ ∫ 1 /e
g ( x ) dx = ∫ 1 /e

 x
 So, the least and greatest value of
1
sin 6 x + cos 6 x are and 1.
Thus, only critical point in (0, 1) is = − [log e x] 11 /e 4
a = 1/2  π  1
g ( x ) dx = −  log e 1 − log e  Hence,  − 0 ×
1 1
⇒ ∫  e  2  4
Also, f ′(a) < 0 for a ∈  0, 
1 1 /e

 2 π
< ∫ (sin 6 x + cos 6 x )dx <  − 0 × 1
π /2
= − [0 + 1] = − 1
0 2 
and f ′(a) > 0 for a ∈  , 1 .
1 18 Given, f ′( x ) = f ( x )
2  π π /2 π
and f (0) = 1 ∴ < ∫ (sin x + cos x ) dx <
6 6

1 8 0
2
∴ f (a) is minimum for a = . Let f ( x) = e x …(i)
2

You might also like